You are on page 1of 229

Continuum: Lifelong Learning in Neurology—Stroke, Volume 20, Issue 2, April 2014

Issue Overview

Stroke, April 2014;20(2)

Continuum: Lifelong Learning in Neurology® is designed to help practicing neurologists stay

abreast of advances in the field while simultaneously developing lifelong self-directed learning

skills.

Learning Objectives

Upon completion of the Continuum: Lifelong Learning in Neurology Stroke issue, participants

will be able to:

► Provide an overview of emergent evaluation of the stroke patient with an emphasis on

practical issues regarding ischemic stroke treatment

► Summarize the recommendations for management of hypertension, dyslipidemia, diabetes

mellitus, diet, physical activity, and lifestyle choices commonly encountered in neurologic

practice

► Describe the diagnostic workup and management options for cardioembolic stroke

► Outline the primary findings of recent randomized clinical trials clarifying the treatment

options for conditions such as carotid stenosis and intracranial atherosclerosis and provide

current recommendations for treatment

► Discuss the most up-to-date literature on the epidemiology, diagnosis, management, and

prognosis of cerebral venous thrombosis

► Summarize the evaluation and management of ischemic stroke in young adults, with an

emphasis on cervical artery dissection, patent foramen ovale, and hypercoagulable states

Copyright © American Academy of Neurology. Unauthorized reproduction of this article is prohibited.


► Discuss pediatric arterial ischemic stroke, from recognition and diagnosis to the short-term

and long-term management based on current available literature

► Explain the epidemiology, screening strategies, and management options for patients with

unruptured intracranial aneurysms

► Describe the clinical and radiographic features, genetic determinants, and treatment options

for the most well-characterized monogenic disorders associated with stroke

► List the predictors of functional outcome and mortality after ischemic stroke

► Discuss the ethical issues raised by telephone consultations for the administration of tissue

plasminogen activator in acute ischemic stroke

► Estimate the costs of the stroke diagnostic evaluation and identify scenarios in which tests

have low clinical yield

Core Competencies

The Continuum Stroke issue covers the following core competencies:

► Patient Care

► Medical Knowledge

► Practice-Based Learning and Improvement

► Interpersonal and Communication Skills

► Professionalism

► Systems-Based Practice

Disclosures

CONTRIBUTORS

Copyright © American Academy of Neurology. Unauthorized reproduction of this article is prohibited.


Adam G. Kelly, MD, Guest Editor
Assistant Professor of Neurology, University of Rochester Medical Center; Chief of Neurology,
Highland Hospital, Rochester, New York
a
Dr Kelly has received research support from the Donald W. Reynolds Foundation and compensation from the
American Academy of Neurology as a question writer for Continuum.
b
Dr Kelly reports no disclosure.

Kevin M. Barrett, MD, MSc


Assistant Professor of Neurology, Mayo Clinic College of Medicine, Mayo Clinic Florida,
Jacksonville, Florida
a
Dr Barrett receives support from the National Institute of Neurological Disorders and Stroke for his role in the
Stroke Hyperglycemia Insulin Network Effort (SHINE) trial and has received a speaking honorarium for a Lee
Memorial Health System continuing medical education course.
b
Dr Barrett reports no disclosure.

Pratik D. Bhattacharya, MD, MPH


Assistant Professor of Neurology, Wayne State University; Chief of Neurology and Director of
Stroke Services, Sinai-Grace Hospital, Detroit, Michigan
a
Dr Bhattacharya has received a research grant from the Ethel & James Flinn Foundation to study poststroke
depression in stroke patients.
b
Dr Bhattacharya reports no disclosure.

James F. Burke, MD, MS


Assistant Professor, Stroke Program and Department of Neurology, University of Michigan, Ann
Arbor, Michigan
a
Dr Burke has reviewed case materials for medical malpractice defense cases and receives funding from the
National Institute of Neurological Disorders and Stroke.
b
Dr Burke reports no disclosure.

Cheryl Bushnell, MD, MHS


Associate Professor of Neurology, Wake Forest School of Medicine; Director, Wake Forest
Baptist Stroke Center, Winston-Salem, North Carolina
a
Dr Bushnell has received grants from the Hazel K. Goddess Fund for Stroke Research in Women, the North
Carolina Stroke Care Collaborative, the World Federation of Neurology, and the World Stroke Organization.
b
Dr Bushnell reports no disclosure.

Seemant Chaturvedi, MD, FAAN


Professor of Neurology, Stroke Program Director, Wayne State University, Detroit, Michigan
a
Dr Chaturvedi has received compensation for expert witness testimony and research support from AstraZeneca,
Daiichi Sankyo, and Johnson & Johnson Services, Inc. Dr Chaturvedi serves as a consultant for Abbott Vascular
and W. L. Gore & Associates, Inc; on the executive committee of the Asymptomatic Carotid Trial (ACT)–1 and
Carotid Revascularization Endarterectomy versus Stenting Trial (CREST)–2 studies; and as a contributing editor to
NEJM Journal Watch Neurology.
b
Dr Chaturvedi reports no disclosure.

Ji Y. Chong, MD
Assistant Professor of Neurology, Weill Cornell Medical College; Stroke Center Director, New
York Presbyterian Lower Manhattan Hospital, New York, New York
a,b
Dr Chong reports no disclosures.

Bart M. Demaerschalk, MD, MSc, FRCP(C)

Copyright © American Academy of Neurology. Unauthorized reproduction of this article is prohibited.


Professor of Neurology, Mayo Clinic College of Medicine, Phoenix, Arizona
a,b
Dr Demaerschalk reports no disclosures.

Gabrielle deVeber, MD
Professor of Pediatrics, University of Toronto, The Hospital for Sick Children, Toronto, Ontario,
Canada
a
Dr deVeber’s spouse holds an investment of more than $10,000 in Thornhill Research Inc.
b
Dr deVeber reports no disclosure.

Timothy J. Ingall, MB BS, PhD


Associate Professor of Neurology, Cerebrovascular Diseases Center, Mayo Clinic,
Phoenix, Arizona
a,b
Dr Ingall reports no disclosures.

Pooja Khatri, MD, MSc


Professor of Neurology, University of Cincinnati Medical Center, Cincinnati, Ohio
a
Dr Khatri has received research grants from the NIH and research support from Penumbra Inc and Genentech,
Inc.
b
Dr Khatri discusses the use of IV tissue plasminogen activator for minor stroke and of endovascular therapy for
stroke treatment, neither of which are approved by the US Food and Drug Administration.

Anthony S. Kim, MD, MAS


Assistant Professor, Department of Neurology; Medical Director, UCSF Stroke Center,
University of California, San Francisco, San Francisco, California
a
Dr Kim has received research grants from the American Heart Association, the NIH, and SanBio, Inc; has served
as an expert witness providing record review in a court case; and has received an honorarium and travel expenses
for speaking at an American Neurological Association training course.
b
Dr Kim reports no disclosure.

Jason Mackey, MD, MS


Assistant Professor of Neurology, Indiana University School of Medicine, Indianapolis, Indiana
a
Dr Mackey has served as an expert medical record reviewer in vascular neurology for a court case and has
received funding as the principal investigator for outcomes and processes of care in intracerebral hemorrhage for
the Indiana University Health Values Fund and the Indiana Clinical and Translational Sciences Institute Project
Development Team.
b
Dr Mackey reports no disclosure.

James F. Meschia, MD, FAAN


Professor and Chair, Department of Neurology, Mayo Clinic Florida, Jacksonville, Florida
a
Dr Meschia serves on the editorial board of the European Journal of Neurology and receives support from the
National Institute of Neurological Disorders and Stroke for his roles in the Carotid Revascularization
Endarterectomy versus Stenting Trial (CREST) and the Stroke Genetics Network (SiGN).
b
Dr Meschia reports no disclosure.

Mahendra Moharir, MD, MSc, FRACP


Staff Neurologist and Assistant Professor, Division of Neurology, Department of Pediatrics,
University of Toronto, The Hospital for Sick Children, Toronto, Ontario, Canada
a,b
Dr Moharir reports no disclosures.

Shyam Prabhakaran, MD, MS

Copyright © American Academy of Neurology. Unauthorized reproduction of this article is prohibited.


Associate Professor, Department of Neurology; Director of Stroke Research, Northwestern
University, Feinberg School of Medicine, Chicago, Illinois
a
Dr Prabhakaran has served as an author of UpToDate and received research support as a principal investigator
from the National Institute of Neurological Disorders and Stroke and the American Stroke Association.
b
Dr Prabhakaran reports no disclosure.

David A. Rempe, MD, PhD


Stroke Medical Director, Mercy Hospital, St Louis, Missouri
a,b
Dr Rempe reports no disclosures.

Gustavo Saposnik, MD, MSc, FAHA, FRCP(C)


Associate Professor and Clinician Scientist; Director, Stroke Outcomes Research Center, St
Michael’s Hospital, University of Toronto, Toronto, Ontario, Canada
a
Dr Saposnik is supported by the Distinguished Clinician Scientist Award from the Heart and Stroke Foundation of
Canada.
b
Dr Saposnik reports no disclosure.

Justin A. Sattin, MD
Assistant Professor and Residency Program Director, Department of Neurology; Medical
Director, University of Wisconsin Health Comprehensive Stroke Program, University of
Wisconsin–Madison, Madison, Wisconsin
a,b
Dr Sattin reports no disclosures.

MULTIPLE-CHOICE QUESTION WRITERS

Ronnie Bergen, MD
Assistant Professor of Clinical Neurology, University of Arizona College of Medicine; Staff
Neurologist, Southern Arizona VA Healthcare System, Tucson, Arizona
a,b
Dr Bergen reports no disclosures.

Douglas J. Gelb, MD, PhD, FAAN


Professor of Neurology, University of Michigan, Ann Arbor, Michigan
a
Dr Gelb has received personal compensation in an editorial capacity for Continuum.
b
Dr Gelb reports no disclosure.

a
Relationship Disclosure
b
Unlabeled Use of Products/Investigational Use Disclosure

Methods of Participation and Instructions for Use

Continuum: Lifelong Learning in Neurology® is designed to help practicing neurologists stay

abreast of advances in the field while simultaneously developing lifelong self-directed learning

skills. In Continuum, the process of absorbing, integrating, and applying the material presented is

as important as, if not more important than, the material itself.

Copyright © American Academy of Neurology. Unauthorized reproduction of this article is prohibited.


The goals of Continuum include disseminating up-to-date information to the practicing

neurologist in a lively, interactive format; fostering self-assessment and lifelong study skills;

encouraging critical thinking; and, in the final analysis, strengthening and improving patient

care.

Each Continuum issue is prepared by distinguished faculty who are acknowledged leaders in

their respective fields. Six issues are published annually and are composed of review articles,

case-based discussions on ethical and practice issues related to the issue topic, coding

information, , and comprehensive CME and self-assessment offerings, including a self-

assessment pretest, multiple-choice questions with preferred responses, and a patient

management problem. For detailed instructions regarding Continuum CME and self-assessment

activities, visit aan.com/continuum/cme.

The review articles emphasize clinical issues emerging in the field in recent years. Case reports

and vignettes are used liberally, as are tables and illustrations. Video material relating to the

issue topic accompanies issues when applicable.

The text can be reviewed and digested most effectively by establishing a regular schedule of

study in the office or at home, either alone or in an interactive group. If subscribers use such

regular and perhaps new study habits, Continuum’s goal of establishing lifelong learning patterns

can be met.

Copyright © American Academy of Neurology. Unauthorized reproduction of this article is prohibited.


LIFELONG LEARNING IN NEUROLOGY ®

Cerebrovascular Disease
Volume 20 Number 2 April 2014

CONTRIBUTORS
Adam G. Kelly, MD, Guest Editor
Assistant Professor of Neurology, University of Rochester Medical Center;
Chief of Neurology, Highland Hospital, Rochester, New York
aDr Kelly has received research support from the Donald W. Reynolds Foundation and
compensation from the American Academy of Neurology as a question writer for Continuum.
bDr Kelly reports no disclosure.

Kevin M. Barrett, MD, MSc


Assistant Professor of Neurology, Mayo Clinic College of Medicine, Mayo Clinic
Florida, Jacksonville, Florida
aDr Barrett receives support from the National Institute of Neurological Disorders and Stroke
for his role in the Stroke Hyperglycemia Insulin Network Effort (SHINE) trial and has received
a speaking honorarium for a Lee Memorial Health System continuing medical
education course.
bDr Barrett reports no disclosure.

Pratik D. Bhattacharya, MD, MPH


Assistant Professor of Neurology, Wayne State University; Chief of Neurology
and Director of Stroke Services, Sinai-Grace Hospital, Detroit, Michigan
aDr Bhattacharya has received a research grant from the Ethel & James Flinn Foundation to
study poststroke depression in stroke patients.
bDr Bhattacharya reports no disclosure.

James F. Burke, MD, MS


Assistant Professor, Stroke Program and Department of Neurology,
University of Michigan, Ann Arbor, Michigan
aDr Burke has reviewed case materials for medical malpractice defense cases and receives
funding from the National Institute of Neurological Disorders and Stroke.
bDr Burke reports no disclosure.

aRelationship Disclosure
bUnlabeled Use of Products/Investigational Use Disclosure

Continuum (Minneap Minn) 2014;20(2) www.ContinuumJournal.com

Copyright © American Academy of Neurology. Unauthorized reproduction of this article is prohibited.


LIFELONG LEARNING IN NEUROLOGY ®

CONTRIBUTORS continued
Cheryl Bushnell, MD, MHS
Associate Professor of Neurology, Wake Forest School of Medicine; Director,
Wake Forest Baptist Stroke Center, Winston-Salem, North Carolina
aDr Bushnell has received grants from the Hazel K. Goddess Fund for Stroke Research in
Women, the North Carolina Stroke Care Collaborative, the World Federation of Neurology,
and the World Stroke Organization.
bDr Bushnell reports no disclosure.

Seemant Chaturvedi, MD, FAAN


Professor of Neurology, Stroke Program Director, Wayne State University,
Detroit, Michigan
aDr Chaturvedi has received compensation for expert witness testimony and research
support from AstraZeneca, Daiichi Sankyo, and Johnson & Johnson Services, Inc.
Dr Chaturvedi serves as a consultant for Abbott Vascular and W. L. Gore & Associates, Inc;
on the executive committee of the Asymptomatic Carotid Trial (ACT)–1 and Carotid
Revascularization Endarterectomy versus Stenting Trial (CREST)–2 studies; and as a
contributing editor to NEJM Journal Watch Neurology.
bDr Chaturvedi reports no disclosure.

Ji Y. Chong, MD
Assistant Professor of Neurology, Weill Cornell Medical College; Stroke Center
Director, New York Presbyterian Lower Manhattan Hospital, New York, New York
a,bDr Chong reports no disclosures.

Bart M. Demaerschalk, MD, MSc, FRCP(C)


Professor of Neurology, Mayo Clinic College of Medicine, Phoenix, Arizona
a,bDr Demaerschalk reports no disclosures.

Gabrielle deVeber, MD
Professor of Pediatrics, University of Toronto, The Hospital for Sick Children,
Toronto, Ontario, Canada
aDr deVeber’s spouse holds an investment of more than $10,000 in Thornhill Research Inc.
bDr deVeber reports no disclosure.

aRelationship Disclosure
bUnlabeled Use of Products/Investigational Use Disclosure

www.ContinuumJournal.com April 2014

Copyright © American Academy of Neurology. Unauthorized reproduction of this article is prohibited.


LIFELONG LEARNING IN NEUROLOGY ®

CONTRIBUTORS continued
Timothy J. Ingall, MB BS, PhD
Associate Professor of Neurology, Cerebrovascular Diseases Center, Mayo Clinic,
Phoenix, Arizona
a,bDr Ingall reports no disclosures.

Pooja Khatri, MD, MSc


Professor of Neurology, University of Cincinnati Medical Center,
Cincinnati, Ohio
aDr Khatri has received research grants from the NIH and research support from Penumbra
Inc and Genentech, Inc.
bDr Khatri discusses the use of IV tissue plasminogen activator for minor stroke and of
endovascular therapy for stroke treatment, neither of which are approved by the US Food
and Drug Administration.

Anthony S. Kim, MD, MAS


Assistant Professor, Department of Neurology; Medical Director, UCSF Stroke
Center, University of California, San Francisco, San Francisco, California
aDr Kim has received research grants from the American Heart Association, the NIH,
and SanBio, Inc; has served as an expert witness providing record review in a court case;
and has received an honorarium and travel expenses for speaking at an American
Neurological Association training course.
bDr Kim reports no disclosure.

Jason Mackey, MD, MS


Assistant Professor of Neurology, Indiana University School of Medicine,
Indianapolis, Indiana
aDr Mackey has served as an expert medical record reviewer in vascular neurology for a
court case and has received funding as the principal investigator for outcomes and processes
of care in intracerebral hemorrhage for the Indiana University Health Values Fund and the
Indiana Clinical and Translational Sciences Institute Project Development Team.
bDr Mackey reports no disclosure.

James F. Meschia, MD, FAAN


Professor and Chair, Department of Neurology, Mayo Clinic Florida,
Jacksonville, Florida
aDr Meschia serves on the editorial board of the European Journal of Neurology and
receives support from the National Institute of Neurological Disorders and Stroke for his roles
in the Carotid Revascularization Endarterectomy versus Stenting Trial (CREST) and the Stroke
Genetics Network (SiGN).
bDr Meschia reports no disclosure.

aRelationship Disclosure
bUnlabeled Use of Products/Investigational Use Disclosure

Continuum (Minneap Minn) 2014;20(2) www.ContinuumJournal.com

Copyright © American Academy of Neurology. Unauthorized reproduction of this article is prohibited.


LIFELONG LEARNING IN NEUROLOGY ®

CONTRIBUTORS continued
Mahendra Moharir, MD, MSc, FRACP
Staff Neurologist and Assistant Professor, Division of Neurology, Department of
Pediatrics, University of Toronto, The Hospital for Sick Children, Toronto,
Ontario, Canada
a,bDr Moharir reports no disclosures.

Shyam Prabhakaran, MD, MS


Associate Professor, Department of Neurology; Director of Stroke Research,
Northwestern University, Feinberg School of Medicine, Chicago, Illinois
aDr Prabhakaran has served as an author of UpToDate and received research support as a
principal investigator from the National Institute of Neurological Disorders and Stroke and
the American Stroke Association.
bDr Prabhakaran reports no disclosure.

David A. Rempe, MD, PhD


Stroke Medical Director, Mercy Hospital, St Louis, Missouri
a,bDr Rempe reports no disclosures.

Gustavo Saposnik, MD, MSc, FAHA, FRCP(C)


Associate Professor and Clinician Scientist; Director, Stroke Outcomes Research
Center, St Michael’s Hospital, University of Toronto, Toronto, Ontario, Canada
aDr Saposnik is supported by the Distinguished Clinician Scientist Award from the Heart
and Stroke Foundation of Canada.
bDr Saposnik reports no disclosure.

Justin A. Sattin, MD
Assistant Professor and Residency Program Director, Department of Neurology;
Medical Director, University of Wisconsin Health Comprehensive Stroke
Program, University of Wisconsin–Madison, Madison, Wisconsin
a,bDr Sattin reports no disclosures.

aRelationship Disclosure
bUnlabeled Use of Products/Investigational Use Disclosure

www.ContinuumJournal.com April 2014

Copyright © American Academy of Neurology. Unauthorized reproduction of this article is prohibited.


LIFELONG LEARNING IN NEUROLOGY ®

SELF-ASSESSMENT AND CME TEST WRITERS


Ronnie Bergen, MD
Assistant Professor of Clinical Neurology, University of Arizona College of
Medicine; Staff Neurologist, Southern Arizona VA Healthcare System,
Tucson, Arizona
a,bDr Bergen reports no disclosures.

Douglas J. Gelb, MD, PhD, FAAN


Professor of Neurology, University of Michigan, Ann Arbor, Michigan
aDr Gelb has received personal compensation in an editorial capacity for Continuum.
bDr Gelb reports no disclosure.

aRelationship Disclosure
bUnlabeled Use of Products/Investigational Use Disclosure

Continuum (Minneap Minn) 2014;20(2) www.ContinuumJournal.com

Copyright © American Academy of Neurology. Unauthorized reproduction of this article is prohibited.


Volume 20 n Number 2 n April 2014

LIFELONG LEARNING IN NEUROLOGY ®


www.ContinuumJournal.com

Cerebrovascular Disease
Guest Editor: Adam G. Kelly, MD

Editor’s Preface . . . . . . . . . . . . . . . . . . . . . . . . . . . . . . . . . . . . . . . . . . . . . . . . . . . . . . . . 281

REVIEW ARTICLES
Evaluation and Management of Acute Ischemic Stroke . . . . . . . . . . . . . . . . . . . 283
Pooja Khatri, MD, MSc

Risk Factor Management for Stroke Prevention . . . . . . . . . . . . . . . . . . . . . . . . . . . 296


Shyam Prabhakaran, MD, MS; Ji Y. Chong, MD

Evaluation and Prevention of Cardioembolic Stroke . . . . . . . . . . . . . . . . . . . . . 309


Anthony S. Kim, MD, MAS

Large Artery Atherosclerosis: Carotid Stenosis, Vertebral Artery Disease,


and Intracranial Atherosclerosis . . . . . . . . . . . . . . . . . . . . . . . . . . . . . . . . . . . . . . . . 323
Seemant Chaturvedi, MD, FAAN; Pratik Bhattacharya, MD, MPH

Evaluation and Management of Cerebral Venous Thrombosis . . . . . . . . . . . . 335


Cheryl Bushnell, MD, MHS; Gustavo Saposnik, MD, MSc, FAHA, FRCP(C)

Evaluation and Management of Stroke in Young Adults . . . . . . . . . . . . . . . . . . 352


Jason S. Mackey, MD, MS

Pediatric Arterial Ischemic Stroke . . . . . . . . . . . . . . . . . . . . . . . . . . . . . . . . . . . . . . . 370


Mahendra Moharir, MD, MSc, FRACP; Gabrielle de Veber, MD

Unruptured Intracranial Aneurysms: Screening and Management . . . . . . . . 387


Adam G. Kelly, MD

Genetic Stroke Syndromes . . . . . . . . . . . . . . . . . . . . . . . . . . . . . . . . . . . . . . . . . . . . . 399


Kevin M. Barrett, MD, MSc; James F. Meschia, MD, FAAN

Predicting Outcomes After Transient Ischemic Attack and Stroke . . . . . . . . . 412


David A. Rempe, MD, PhD

Volume 20 n Number 2 www.ContinuumJournal.com 269

Copyright © American Academy of Neurology. Unauthorized reproduction of this article is prohibited.


LIFELONG LEARNING IN NEUROLOGY ®

ETHICAL PERSPECTIVES
Telephone Consultations for Tissue Plasminogen Activator Administration
in Acute Stroke . . . . . . . . . . . . . . . . . . . . . . . . . . . . . . . . . . . . . . . . . . . . . . . . . . . . . . . . . 429
Justin A. Sattin, MD

PRACTICE ISSUES
Cost and Utility in the Diagnostic Evaluation of Stroke . . . . . . . . . . . . . . . . . . . . 436
James F. Burke, MD

Coding for Telestroke . . . . . . . . . . . . . . . . . . . . . . . . . . . . . . . . . . . . . . . . . . . . . . . . . . . 441


Timothy Ingall, MBBS, PhD; Bart M. Demaerschalk, MD, MSc, FRCP(C)

APPENDIX
Appendix A: National Institutes of Health Stroke Scale Summary . . . . . . . . . 444

SELF-ASSESSMENT AND CME


Learning Objectives . . . . . . . . . . . . . . . . . . . . . . . . . . . . . . . . . . . . . . . . . . . . . . . . . . . . 271
Instructions for Completing the Self-Assessment Pretest and Tally Sheet . . . . . 273
Self-Assessment Pretest . . . . . . . . . . . . . . . . . . . . . . . . . . . . . . . . . . . . . . . . . . . . . . . . 275
Instructions for Completing CME and Tally Sheet . . . . . . . . . . . . . . . . . . . . . . . . . 447
Postreading CME Test . . . . . . . . . . . . . . . . . . . . . . . . . . . . . . . . . . . . . . . . . . . . . . . . . . . 449
Postreading CME Test—Preferred Responses . . . . . . . . . . . . . . . . . . . . . . . . . . . . . 459
Patient Management Problem . . . . . . . . . . . . . . . . . . . . . . . . . . . . . . . . . . . . . . . . . . . 476
Patient Management Problem—Preferred Responses . . . . . . . . . . . . . . . . . . . . . 482
Adam G. Kelly, MD

Index . . . . . . . . . . . . . . . . . . . . . . . . . . . . . . . . . . . . . . . . . . . . . . . . . . . . . . . . . . . . . . . . . . 494
List of Abbreviations . . . . . . . . . . . . . . . . . . . . . . . . . . . . . . . . . . . . . . . . . . . .Back Cover

270 www.ContinuumJournal.com April 2014

Copyright © American Academy of Neurology. Unauthorized reproduction of this article is prohibited.


Learning Objectives
Upon completion of this Continuum: Lifelong Learning in Neurology
Cerebrovascular Disease issue, participants will be able to:

Provide an overview of emergent evaluation of the stroke patient with an


emphasis on practical issues regarding ischemic stroke treatment


Summarize the recommendations for management of hypertension, dyslipidemia,

diabetes mellitus, diet, physical activity, and lifestyle choices commonly


encountered in neurologic practice
Describe the diagnostic workup and management options for cardioembolic stroke

Outline the primary findings of recent randomized clinical trials clarifying the

treatment options for conditions such as carotid stenosis and intracranial


atherosclerosis and provide current recommendations for treatment
Discuss the most up-to-date literature on the epidemiology, diagnosis, management,

and prognosis of cerebral venous thrombosis


Summarize the evaluation and management of ischemic stroke in young adults,

with an emphasis on cervical artery dissection, patent foramen ovale, and


hypercoagulable states
Discuss pediatric arterial ischemic stroke, from recognition and diagnosis to the

short-term and long-term management based on current available literature


Explain the epidemiology, screening strategies, and management options for

patients with unruptured intracranial aneurysms


Describe the clinical and radiographic features, genetic determinants, and

treatment options for the most well-characterized monogenic disorders associated


with stroke
List the predictors of functional outcome and mortality after ischemic stroke

Discuss the ethical issues raised by telephone consultations for the administration

of tissue plasminogen activator in acute ischemic stroke


Estimate the costs of the stroke diagnostic evaluation and identify scenarios in

which tests have low clinical yield

Core Competencies
This Continuum: Lifelong Learning in Neurology Cerebrovascular Disease issue
covers the following core competencies:

Patient Care

Medical Knowledge

Practice-Based Learning and Improvement


Interpersonal and Communication Skills


Professionalism

Systems-Based Practice

Continuum (Minneap Minn) 2014;20(2) www.ContinuumJournal.com

Copyright © American Academy of Neurology. Unauthorized reproduction of this article is prohibited.


Copyright © American Academy of Neurology. Unauthorized reproduction of this article is prohibited.
SELF-ASSESSMENT


INSTRUCTIONS FOR THE Cerebrovascular Disease
SELF-ASSESSMENT PRETEST
To earn American Board of Psychiatry and Volume 20 n Number 2 n April 2014
Neurology (ABPN) Maintenance of Certification
(MOC) self-assessment participation credit for this Tally Sheet
issue of , complete the Self-Assessment
Pretest in one of three ways: SELF-ASSESSMENT PRETEST RESPONSES
1. Go to www.aan.com/continuum/cme and complete After completing this tally sheet, please enter your
the test online (available to subscribers only), or
answers online at
2. Read through the questions in the issue and www.aan.com/continuum/cme.
mark your answers on the adjacent tally sheet
before entering them online at 1. a b c d e 21. a b c d e
www.aan.com/continuum/cme, or
3. Request a faxable paper scorecard if you do not 2. a b c d e 22. a b c d e
have computer access or you are a nonsubscriber
3. a b c d e 23. a b c d e
who has purchased a single issue (send an email to
ContinuumCME@aan.com); however, we encourage 4. a b c d e 24. a b c d e
all subscribers to use the online system.
Once you have completed the Self-Assessment 5. a b c d e 25. a b c d e
Pretest, use your results as a guide as you read the 6. a b c d e 26. a b c d e
issue. To obtain self-assessment CME credits, you
must complete the Postreading CME Test at the end 7. a b c d e 27. a b c d e
of the issue, along with an issue evaluation. No
self-assessment credit will be awarded for 8. a b c d e 28. a b c d e
completing the Self-Assessment Pretest alone.
9. a b c d e 29. a b c d e
Upon completion of the Self-Assessment Pretest
and Postreading CME Test, participants may earn 10. a b c d e 30. a b c d e
up to 12 AMA PRA Category 1 CreditsTM toward ABPN
MOC self-assessment (part 2). Self-assessment 11. a b c d e 31. a b c d e
credits earned will appear on AAN member CME 12. a b c d e 32. a b c d e
transcripts within 2 business days and may be
viewed at www.aan.com/cme-and-moc/track-and- 13. a b c d e 33. a b c d e
view-transcript/. AAN nonmember subscribers may
request a transcript of credits earned by contacting 14. a b c d e 34. a b c d e
AAN Member Services at memberservices@aan.com
or (800) 879-1960. 15. a b c d e 35. a b c d e
AMA PRA Category 1 Credits not designated for 16. a b c d e 36. a b c d e
self-assessment may be obtained by completing
only the Postreading CME Test (up to 10) and/or the 17. a b c d e 37. a b c d e
Patient Management Problem (up to 2) found at the
18. a b c d e 38. a b c d e
end of the issue.
Participants have up to 3 years from the date of 19. a b c d e 39. a b c d e
publication to earn CME credits. No CME will be
awarded for the Cerebrovascular Disease issue after 20. a b c d e 40. a b c d e
April 30, 2017.

The ABPN has reviewed Continuum: Lifelong


Learning in Neurology and has approved this
product as part of a comprehensive lifelong
learning and self-assessment program, which is
mandated by the American Board of Medical
Specialties (ABMS) as a necessary component of
maintenance of certification.


Continuum (Minneap Minn) 2014;20(2) www.ContinuumJournal.com 273

Copyright © American Academy of Neurology. Unauthorized reproduction of this article is prohibited.


Copyright © American Academy of Neurology. Unauthorized reproduction of this article is prohibited.
Self-Assessment

Self-Assessment Pretest 4. Which of the following disorders affecting the coagulation


cascade is most closely associated with arterial (as opposed
to venous) thrombosis?
A. antiphospholipid antibody syndrome
The Self-Assessment Pretest is designed B. antithrombin III deficiency
to help neurologists meet the American Board of Psychiatry C. factor V Leiden mutation
and Neurology (ABPN) self-assessment and lifelong learning D. protein S deficiency
component (part 2) for Maintenance of Certification (MOC). E. prothrombin gene (G20210A) mutation
To obtain self-assessment CME credits, complete the Self-
5. An 85-year-old man who has a history of atrial fibrillation but
Assessment Pretest (40 questions) before reading this issue.
is otherwise healthy is seen for an assessment of stroke risk.
Pretest results are intended to help you focus your learning He has hypertension but is not diabetic and besides his atrial
by identifying your current knowledge gaps with regard to fibrillation has no other known heart disease. He has never
the topic. After submitting your pretest responses, study the had a reported neurologic event. According to current data,
entire issue using your pretest results as a guide to direct what is the most appropriate stroke-prevention treatment in
your learning. Then complete the Postreading CME Test at this patient?
the end of the issue. Upon completion of both the A. anticoagulation
Self-Assessment Pretest and the B. anticoagulation plus 81 mg aspirin
Postreading CME Test, you may earn up to 12 AMA PRA C. aspirin 81 mg daily
Category 1 Creditsi toward self-assessment. D. aspirin 81 mg daily plus clopidogrel
AMA PRA Category 1 Credits for E. no additional therapy is needed
may be obtained by completing only the Postreading CME
6. Which of the following is the most common clinical
Test, but these credits will not qualify for self-assessment
presenting sign or symptom of cerebral venous thrombosis?
without completion of the Self-Assessment Pretest.
A. headache
B. hemiparesis
C. seizure
D. syncope
E. visual changes

1. Which of the following therapies is recommended as the 7. An 86-year-old woman with hypertension, hyperlipidemia,
initial step in treating cerebral venous thrombosis (CVT) in and diabetes mellitus was found to have 80% stenosis of
the presence of hemorrhagic transformation of a venous the proximal left internal carotid artery on a CT angiogram
infarction? (ordered by her primary physician to evaluate prominent
calcification in several intracranial vessels on a head CT scan,
A. aspirin which had been done as part of an evaluation for mild
B. balloon venoplasty problems with memory). The CT scan showed no evidence of
C. low-molecular-weight heparin ischemia or atrophy. In addition to aggressive management
D. mechanical clot retrieval of risk factors, which of the following is the most appropriate
E. tissue plasminogen activator management of this patient?
2. Which of the following levels of daily alcohol intake is A. antiplatelet therapy alone
associated with the lowest relative risk of ischemic stroke? B. left carotid angioplasty and stent plus antiplatelet therapy
C. left carotid angioplasty plus antiplatelet therapy but no stent
A. zero drinks D. left carotid endarterectomy plus antiplatelet therapy
B. one to two drinks E. warfarin alone
C. three to four drinks
D. five to six drinks 8. An 80-year-old man who had a left pontine stroke two days
E. seven or more drinks ago is found to have 90% stenosis of the proximal left
vertebral artery. In addition to aggressive management of risk
3. A 54-year-old man with no known medical problems is donating factors, which of the following is the most appropriate
blood when he suddenly develops a left hemiparesis. He is management of this patient?
immediately brought to the emergency department, where his
CT scan is normal. His weakness continues to progress. Which of A. angioplasty and stenting of the left vertebral artery plus
the following blood test results must be reviewed before antiplatelet therapy
administering IV recombinant tissue-type plasminogen activator B. angioplasty and stenting of the left vertebral artery plus
(rtPA) to this patient? warfarin
C. antiplatelet therapy alone
A. cholesterol D. transposition of the left vertebral artery to the left common
B. creatinine carotid artery plus antiplatelet therapy
C. glucose E. warfarin therapy alone
D. international normalized ratio (INR)
E. platelet count

Continuum (Minneap Minn) 2014;20(2):275–279 www.ContinuumJournal.com 275


Copyright © American Academy of Neurology. Unauthorized reproduction of this article is prohibited.
Self-Assessment

9. A 68-year-old woman is being evaluated in the emergency 14. Which of the following symptoms and signs in late infancy or
department for left hemiparesis. She says that she awoke 3 childhood is most suggestive of a previously undiagnosed
hours ago, and she fell when she tried to get out of bed. In perinatal stroke?
trying to stand, she realized that she couldn’t move her left A. dysphagia
leg. Her examination is notable for severe weakness and B. early hand dominance
diminished sensation of the left lower extremity, with mild C. failure to thrive
weakness and diminished sensation of the left upper D. headache
extremity. She says that her left arm weakness has definitely E. visual impairment
become more pronounced over the past hour. Her health
was last known to be normal 10 hours ago. Which of the 15. A 72-year-old woman had an episode of numbness and weakness
following treatments should be administered? of the left arm and leg, which resolved spontaneously within
A. aspirin 3 minutes. Her neurologic examination is normal in the
B. intra-arterial rtPA emergency department 2 hours later. She has a history of
C. IV heparin hypertension, well controlled on hydrochlorothiazide and
D. IV recombinant tissue-type plasminogen activator (rtPA) lisinopril, and her blood pressure in the emergency department
E. warfarin is 106/68 mm Hg. She has the following lipid levels: total
cholesterol, 210 mg/dL; low-density lipoprotein (LDL), 140 mg/dL;
10. In the Carotid Revascularization Endarterectomy versus high-density lipoprotein (HDL), 50 mg/dL; triglycerides,
Stenting Trial (CREST), the results were better (ie, a significantly 422 mg/dL; total cholesterol/HDL ratio, 4.2. To reduce the risk
lower rate of the primary outcome) for patients in the stenting of subsequent stroke, the evidence is most compelling for
group than they were for patients in the endarterectomy which of the following treatment targets?
group when the analysis was restricted to patients belonging A. HDL cholesterol level greater than 60 mg/dL
to which of the following subgroups? B. LDL cholesterol level less than 70 mg/dL
A. female patients C. total cholesterol level less than 160 mg/dL
B. male patients D. total cholesterol level/HDL ratio less than 3.0
C. patients older than 70 years E. triglyceride level less than 120 mg/dL
D. patients younger than 70 years
E. patients with asymptomatic carotid stenosis 16. Which of the following statements best summarizes the available
evidence regarding the relative efficacy of endovascular and
11. Fabry disease is most commonly associated with strokes in surgical management of unruptured intracranial aneurysms?
which of the following arterial distributions? A. endovascular management is equivalent to surgical manage-
A. anterior cerebral artery ment, based on large randomized controlled trials
B. carotid artery B. endovascular management is inferior to surgical management,
C. lenticulostriate arteries based on large randomized controlled trials
D. middle cerebral artery C. endovascular management is superior to surgical manage-
E. posterior circulation ment, based on large randomized controlled trials
D. large randomized controlled trials have yielded contradictory
12. Which of the following factors has the best established results
correlation with the likelihood of benefit from IV recombinant E. no large randomized controlled trials have been conducted
tissue-type plasminogen activator (rtPA) in patients with
acute ischemic stroke? 17. In patients known to have unruptured intracranial aneurysms,
A. age of patient which of the following features is associated with a reduced
B. degree of diffusion/perfusion mismatch on imaging studies risk of rupture?
C. severity of neurologic deficits at the time treatment is initated A. alcohol abuse
D. time since patient was last known well B. aneurysm diameter of less than 3 mm
E. weight of patient C. familial aneurysm syndrome
D. hypertension
13. Magnetic resonance angiography (MRA) and CT angiography E. tobacco use
(CTA) have greater than 90% sensitivity for detecting unruptured
intracranial aneurysms as long as the aneurysm diameter
exceeds what cutoff?
A. 3 mm for CTA and 3 mm for MRA
B. 3 mm for CTA and 10 mm for MRA
C. 10 mm for CTA and 3 mm for MRA
D. 10 mm for CTA and 10 mm for MRA

276 www.ContinuumJournal.com April 2014

Copyright © American Academy of Neurology. Unauthorized reproduction of this article is prohibited.


Self-Assessment

18. A 76-year-old woman presents to the emergency department 23. Which of the following clinical factors is weighted most
because of three 5-minute episodes of weakness and heavily when calculating the ABCD2 scale?
numbness of the right face and arm, associated with a right A. age greater than or equal to 60 years
visual field deficit and difficulty with language. Vascular B. blood pressure greater than or equal to 140/90 mm Hg
imaging with CT angiography reveals 40% stenosis of the left C. diabetes mellitus
internal carotid artery just distal to its origin and 90% stenosis D. speech/language impairment without weakness
of the left middle cerebral artery. In addition to aggressive E. unilateral weakness
management of risk factors, which of the following is the
most appropriate management of this patient? 24. In light of recent clinical trials, which of the following is the
A. angioplasty and stenting of the left middle cerebral artery, standard treatment for a patient with an acute ischemic
plus antiplatelet therapy stroke who comes to medical attention 2 hours after the
B. angioplasty and stenting of the left middle cerebral artery, onset of deficits?
plus warfarin A. intra-arterial recombinant tissue-type plasminogen activator
C. antiplatelet therapy alone (rtPA)
D. angioplasty of the left middle cerebral artery, plus B. IV rtPA
antiplatelet therapy, but no stenting C. IV rtPA followed by intra-arterial rtPA
E. warfarin alone D. IV rtPA followed by mechanical endovascular clot retrieval
E. mechanical endovascular clot retrieval
19. A 33-year-old man who has had an ischemic stroke in the
territory of the right middle cerebral artery is most likely to 25. For which of the following conditions is the evidence strongest
have which of the following risk factors? that the risk of stroke can be reduced by aggressively treating
A. dyslipidemia the condition?
B. fibromuscular dysplasia A. hypernatremia
C. Kawasaki disease B. hypertension
D. prothrombin gene (G20210A) mutation C. insulin resistance
E. pulmonary arteriovenous malformation D. obesity
E. physical inactivity
20. The US Food and Drug Administration (FDA) has approved
the use of three novel oral anticoagulants for nonvalvular 26. Which of the following skeletal abnormalities is most commonly
atrial fibrillation: dabigatran, rivaroxaban, and apixaban. associated with cerebral autosomal recessive arteriopathy with
Which of the following is a disadvantage of these therapies subcortical infarcts and leukoencephalopathy (CARASIL)?
compared to warfarin?
A. ankylosing spondylitis
A. greater need for monitoring B. atlantoaxial subluxation
B. increased incidence of intracranial hemorrhage C. osteogenesis imperfecta
C. more complicated dosing regimens D. spina bifida
D. more frequent drug-drug interactions E. spondylosis
E. no validated reversal strategy
27. Which of the following laboratory tests may be a useful
21. For patients with diabetes mellitus, the evidence is strongest adjunctive test in making a diagnosis of acute cerebral
that aggressive control of which of the following serum levels venous thrombosis?
is effective for secondary stroke prevention?
A. D-dimer
A. fasting glucose B. erythrocyte sedimentation rate
B. hemoglobin A1c C. international normalized ratio
C. low-density lipoprotein D. prothrombin time
D. random glucose E. partial thromboplastin time
E. triglyceride
28. A 72-year-old man presents to the emergency department
22. An increased risk of atherosclerotic disease is associated with after awakening with severe right leg weakness. CT of the
elevated levels of which of the following? brain obtained on admission shows a small acute ischemic
A. cyanocobalamin stroke in the left anterior cerebral artery territory. By the next
B. homocysteine day, he has improved to the point where he has only mild
C. niacin weakness of right ankle dorsiflexion, with no sensory deficits.
D. riboflavin Carotid ultrasound and CT angiography both show 70% to
E. vitamin A 80% stenosis of the left internal carotid artery just distal to its
origin. When should endarterectomy be performed in this
patient?
A. at least 12 weeks after the stroke
B. 6 to 12 weeks after the stroke
C. 2 to 4 weeks after the stroke
D. within 2 weeks of the stroke
E. neverVendarterectomy is not indicated

Continuum (Minneap Minn) 2014;20(2):275–279 www.ContinuumJournal.com 277


Copyright © American Academy of Neurology. Unauthorized reproduction of this article is prohibited.
Self-Assessment

29. An 83-year-old woman in an acute rehabilitation facility 4 weeks 34. In a 27-year-old woman with a right middle cerebral artery
after a left hip replacement surgery abruptly develops aphasia stroke, which of the following features would provide the most
and right hemiparesis during a physical therapy session. She has compelling reason to test for the factor V Leiden mutation?
a history of chronic obstructive pulmonary disease (and had an A. fibromuscular dysplasia
arterial blood gas performed 1 day ago), and had a right B. history of five prior pregnancies, all carried to term
occipital stroke 2 months ago. A head CT scan shows the prior C. history of recent chiropractic manipulation
stroke, but no acute changes. Which of the following would be D. nonwhite ethnic background
the most compelling contraindication to the use of IV recombi- E. right-to-left cardiac shunt
nant tissue-type plasminogen activator (rtPA) in this patient?
A. her age 35. Which of the following statements best characterizes the
B. her hip surgery 4 weeks ago results of each of the three randomized controlled trials that
C. her stroke 2 months ago have been published to date comparing medical therapy to
D. history of upper gastrointestinal bleeding 6 months ago closure in patients with stroke and patent foramen ovale?
E. radial artery puncture 24 hours ago A. there were fewer recurrent strokes with closure, and the
results were statistically significant
30. Clinical monitoring alone, without cerebrovascular imaging to B. there were fewer recurrent strokes with closure, but the
evaluate for aneurysm, would be most appropriate for a results were not statistically significant
patient with which of the following? C. there were fewer recurrent strokes with medical therapy,
A. autosomal dominant polycystic kidney disease and the results were statistically significant
B. headache associated with transient loss of consciousness D. there were fewer recurrent strokes with medical therapy,
C. intermittent unilateral headache, alternating sides but the results were not statistically significant
D. two first-degree relatives with unruptured intracranial aneurysms E. there were no consistent trends in favor of either approach
E. unilateral oculomotor nerve palsy
36. In patients who stop smoking after having smoked more than
31. Which of the following is the most appropriate timing for 40 cigarettes a day for 10 years, at what point does the risk of
cerebrovascular imaging to screen for unruptured intracranial stroke return to nonsmoker levels?
aneurysms in asymptomatic patients belonging to high-risk A. 6 months after stopping smoking
subpopulations? B. 5 years after stopping smoking
A. at 5 years of age; if negative, every 5 to 10 years C. 15 years after stopping smoking
B. at 5 years of age; if negative, no need to repeat D. 30 years after stopping smoking
C. at 20 years of age; if negative, every 5 to 10 years E. never
D. at 20 years of age; if negative, no need to repeat
E. never, as long as the patient remains asymptomatic 37. A 66-year-old woman with a history of headaches since
adolescence volunteered as a healthy control for a cerebro-
32. A 35-year-old woman presents to the office because of severe, vascular imaging study, and was found to have an aneurysm of
recurrent migraines that are often associated with visual aura. the anterior communicating artery. Which of the following
Family history reveals an aunt with strokes in her 50s and a historical features would be most likely to affect management?
sister with recently diagnosed multiple sclerosis. MRI of the A. heterozygosity for factor V Leiden
patient’s brain shows multiple areas of white matter abnor- B. history of nausea and vomiting accompanying her headaches
mality. White matter lesions in which of the following locations C. history of visual aura
would be most supportive of the diagnosis of cerebral D. recent cataract surgery
autosomal dominant arteriopathy with subcortical infarcts E. recent headache associated with severe neck stiffness
and leukoencephalopathy (CADASIL) in this patient?
A. anterior temporal pole 38. In patients being evaluated for cervicocephalic arterial
B. centrum semiovale dissection, magnetic resonance angiography combined with
C. corpus callosum axial T1-weighted cervical MRI with fat suppression is superior
D. periventricular to CT angiography for identifying which of the following
E. thalamus abnormalities?
A. high-grade stenoses
33. A 2-year-old girl with known sickle cell disease undergoes B. intimal flaps
screening for her stroke risk with transcranial Doppler. Her mean C. pseudoaneurysms
flow velocities remain greater than 200 cm/sec on repeat D. small intramural hematomas
screening done a month after an initial abnormal scan. She has E. vertebral artery dissections
never had a stroke. Which of the following therapies is most
appropriate for stroke prevention in this patient?
A. bone marrow transplant
B. exchange transfusion
C. phlebotomy
D. phlebotomy and hydroxyurea
E. warfarin

278 www.ContinuumJournal.com April 2014

Copyright © American Academy of Neurology. Unauthorized reproduction of this article is prohibited.


Self-Assessment

39. In the Carotid Revascularization Endarterectomy versus Stenting 40. A 4-year-old boy with normal development is seen for three
Trial (CREST), patients in the endarterectomy group had which of paroxysmal episodes of speech arrest and alternating left- and
the following patterns of perioperative complications (relative to right-sided weakness. He is investigated for epilepsy, with
patients in the stenting group)? negative results. His parents say that all three spells occurred
A. fewer perioperative myocardial infarctions, fewer perioperative in the setting of crying excessively. Which of the following
strokes vascular abnormalities is most consistent with this presentation?
B. fewer perioperative myocardial infarctions, more perioperative A. carotid dissection
strokes B. cerebral vasculitis
C. more perioperative myocardial infarctions, fewer perioperative C. cerebral venous thrombosis
strokes D. moyamoya disease
D. more perioperative myocardial infarctions, more perioperative E. multiple cavernous malformations
strokes
E. the same rate of both perioperative myocardial infarctions
and perioperative strokes

Continuum (Minneap Minn) 2014;20(2):275–279 www.ContinuumJournal.com 279


Copyright © American Academy of Neurology. Unauthorized reproduction of this article is prohibited.
Copyright © American Academy of Neurology. Unauthorized reproduction of this article is prohibited.
Editor’s Preface

* 2014, American Academy

Avoiding Hemorrhagic of Neurology.

Stroke and Confronting


All the Rest

When Guest Editor Adam Kelly, MD, need to be very knowledge-


and I began planning this issue, we able about as we provide our
envisioned covering the prevention, secondary preventive stroke
evaluation, and management of as wide risk reduction recommenda-
a variety of cerebrovascular disorders as tions to patients who have
possible in a single issue. To achieve had a stroke or TIA. Next,
this aim for our readers while at the Dr Anthony Kim discusses
same time avoiding unnecessary redun-
the evaluation and prevention
dancy with other issues, we decided to
of cardioembolic stroke. In his
defer coverage of the management of
article, Dr Kim provides a
hemorrhagic stroke to the upcoming
discussion and analysis of the
(2015) neurocritical care issue of
Continuum, where entities such as diagnostic evaluation to assess
intraparenchymal and subarachnoid for atrial fibrillation (AF)Va
hemorrhage will be covered extensively. common and preventable
I hope you will be as pleased with cause of recurrent strokeVas
the end result as I am. Within this issue Dr Kelly and his team a patient’s stroke mechanism,
Dr Kelly and his team of vascular of vascular neurology as well as risk stratification, the
neurology experts have assembled a
practical and contemporary review of
experts have assembled options (new and old) avail-
able for oral anticoagulation
the diagnosis, management, and sec- a practical and for stroke prevention in AF,
ondary prevention of the diverse etiol- contemporary review and the most recent data
ogies of ischemic strokeVboth arterial regarding other potential
and venousVthat can occur through- of the diagnosis,
mechanisms of cardioembolic
out the lifespan. management, and stroke. Drs Seemant Chaturvedi
The issue begins with a bang with
secondary prevention and Pratik Bhattacharya next
a discussion of the evaluation and
of the diverse review the management of
management of acute ischemic stroke
large artery disease. These
by Dr Pooja Khatri. Dr Khatri provides etiologies of ischemic authors provide a state-of-
a state-of-the-art and remarkably prac-
tical review of the emergency evalua-
strokeVboth arterial the-art review of the manage-
tion and time-dependent diagnostic and venousVthat ment of symptomatic and
and acute reperfusion (eg, IV throm- can occur throughout asymptomatic carotid artery
bolysis) therapeutic decisions required stenosis, including an analy-
for the contemporary management of the lifespan. sis of the data comparing
acute ischemic stroke, including a carotid endarterectomy and
thoughtful summary and analysis of very recent trials stenting and the current managementVand im-
assessing the role of endovascular reperfusion in- portant role of medical therapyVin patients
terventions. Next, the issue turns to stroke preven- with vertebral artery disease and intracranial
tion with an article by Drs Shyam Prabhakaran and atherosclerosis.
Ji Chong on risk factor management. In their ex- Drs Cheryl Bushnell and Gustavo Saposnik next
tremely thorough review, these authors summarize the review the evaluation and management of cerebral
current evidence for each of the medical and behavioral venous thrombosis. These authors provide a detailed
stroke risk factors and their managementVwith a and very clear discussion of the epidemiology,
focus on those that are modifiableVthat neurologists diagnosis, and management of this uncommon but

Continuum (Minneap Minn) 2014;20(2):281–282 www.ContinuumJournal.com 281


Copyright © American Academy of Neurology. Unauthorized reproduction of this article is prohibited.
Editor’s Preface

potentially devastating, though treatable, cause of acute stroke, an issue of particular concern in
stroke, which tends to affect young adults and situationsVas in the scenarios described by the
especially young women. Continuing with the theme authorVwhere such consultation is requested when
of stroke in young adults, Dr Jason Mackey discusses a preexisting telestroke contractual relationship is
specific considerations to keep in mind (in addition not in place. These theoretical scenarios will sound
to standard stroke risk factors) in the diagnosis, very familiar to many of us who have been on the re-
evaluation, and management of stroke in patients ceiving end of such calls or involved in similar, even
younger than age 50, including assessing for cervical if less urgent, ‘‘curbside’’ consultations. In the
artery dissections, patent foramen ovale, and poten- Practice section, Dr James Burke discusses cost
tial hypercoagulable syndromes, with an emphasis of and utility in the diagnostic evaluation of stroke.
the lack of current clarity on the management of the Using a very recognizable and common clinical
latter two scenarios. Moving on to an even younger scenario, the author provides a careful analysis of
age group, Drs Mahendra Moharir and Gabrielle de cost-benefit implications of different diagnostic
Veber discuss pediatric arterial ischemic stroke, and strategies in the evaluation of stroke etiology.
we benefit from their extensive combined ex- Finally, Drs Timothy Ingall and Bart Demaerschalk
perience in the diagnosis and management of discuss coding for telestroke, including both a brief
this increasingly recognized, but still probably overview of the practice and a review of the current
underdiagnosed, important cause of long-term coding, billing, and reimbursement considerations
morbidity in children. that relate to this activity.
In a Continuum issue that avoids hemorrhagic As with every Continuum issue, a number of
stroke, no article may be more apropos than opportunities exist for CME. If you need to earn
Dr Kelly’s article on unruptured intracranial credits specifically approved by the American Board
aneurysms, an entity in which the treatment, if of Psychiatry and Neurology (ABPN) for self-
indicated, is specifically intended to avoid sub- assessment, submit your answers to the multiple-
arachnoid hemorrhage (we chose to include this choice questions in the Self-Assessment Pretest that
critically important vascular neurology topic in were crafted by Drs Ronnie Bergen and Douglas
this issue since it is unlikely to be covered Gelb before you read the issue; review your results
elsewhere in the Continuum core curriculum). to better tailor your learning needs; and then
In this article, Dr Kelly provides a careful review complete the Postreading CME Test after reading
of the considerations surrounding and the most the issue. By doing so you may earn up to 12 AMA
recent studies addressing this complicated issue PRA Category 1 CME Creditsi toward self-
to help inform our screening, observation, or assessment. Alternatively, you may wish to receive
management recommendations in patients with credits toward CME only, in which case, reading
asymptomatic intracranial aneurysms. Next, the issue and submitting the Postreading CME
Drs Kevin Barrett and James Meschia review Test will allow you to earn up to 10 AMA PRA
genetic stroke syndromes. In their article, which Category 1 CME Credits. The Patient Management
is remarkably devoid of the complicated molecular Problem, written by Dr Kelly, involves the manage-
genetic information sometimes found in articles ment of a 64-year-old man presenting to the
with similar titles, these authors provide a very emergency department with an acute ischemic
clinically relevant discussion of the diagnostic stroke. By following his case and answering
evaluation and management of the most common multiple-choice questions corresponding to im-
genetic disorders associated with stroke. The final portant diagnostic and therapeuticVincluding
review article of the issue, by Dr David Rempe, acute stroke management and secondary
addresses the current status of outcome prediction preventionVdecision points along the course of
after transient ischemic attack and stroke. The his stroke and poststroke care (reinforcing a large
author carefully discusses and analyzes the various number of the same issues discussed in the
recently developed scores that attempt to predict previous articles) you will have the opportunity to
mortality and functional outcome after stroke and earn up to 2 AMA PRA Category 1 CME Credits.
the risk of symptomatic hemorrhage after throm- My sincere thanks to Dr Kelly and his team of
bolysis (as well as stroke following TIA). The article expert stroke neurologists (and wonderfully clear
includes a well-thought-out analysis as to why such writers and educators) for putting this volume
outcome prediction scores are not currently ready together, which covers so much material that will
for real-world stroke prognostication or for clinical help us provide the most up-to-date and informed
decision making in regard to thrombolysis. management and recommendations to our patients
In this issue’s Ethical Perspectives section, with cerebrovascular disease.
Dr Justin Sattin carefully dissects the ethical consid-
erations involved in telephone consultations for VSteven L. Lewis, MD, FAAN
tissue plasminogen activator (tPA) administration in Editor-in-Chief

282 www.ContinuumJournal.com April 2014

Copyright © American Academy of Neurology. Unauthorized reproduction of this article is prohibited.


Review Article

Evaluation and
Address correspondence
to Dr Pooja Khatri, Department
of Neurology, University of
Cincinnati, 260 Stetson Street,

Management of Acute ML 0525, Cincinnati, OH


45267-0525,
pooja.khatri@uc.edu.

Ischemic Stroke Relationship Disclosure:


Dr Khatri has received
research grants from the
NIH and research support
Pooja Khatri, MD, MSc from Penumbra Inc and
Genentech, Inc.
Unlabeled Use of
ABSTRACT Products/Investigational
Use Disclosure:
Purpose of Review: This review provides an overview of emergent evaluation of the Dr Khatri discusses the use of
stroke patient with an emphasis on practical issues regarding ischemic stroke treatment. IV tissue plasminogen activator
for minor stroke and of
Recent Findings: The IV recombinant tissue-type plasminogen activator (rtPA) endovascular therapy for stroke
treatment window has been expanded from 3 to 4.5 hours from symptom onset. treatment, neither of which are
The evidence for better outcomes with more rapid initiation of reperfusion therapies is approved by the US Food and
Drug Administration.
very strong. Adjunctive endovascular therapy has not been shown to benefit all
* 2014, American Academy
patients with moderate or severe strokes, and investigations are underway to identify of Neurology.
subgroups that may benefit from this approach. Endovascular therapy should be
considered for patients who are ineligible for IV rtPA and can begin treatment within
6 hours of stroke onset.
Summary: Effective emergent evaluation of a stroke patient requires well-organized
systems that maximize speed of assessment and administration of appropriate
therapies, including IV rtPA and endovascular therapies.

Continuum (Minneap Minn) 2014;20(2):283–295.

INTRODUCTION EMERGENT EVALUATION OF


This article provides a practical over- THE POTENTIAL ACUTE
view regarding the emergent evalua- REPERFUSION CANDIDATE
tion of a patient with acute ischemic Rapid evaluation and treatment is
stroke, including the decision to critical for the best outcomes. This
administer acute reperfusion therapy has been shown both in the setting of
and other acute supportive care. IV thrombolysis (IV recombinant
It should be noted that ‘‘acute re- tissue-type plasminogen activator
perfusion therapy’’ refers to treat- [rtPA]) and endovascular therapy.
ment aimed at emergently restoring With IV rtPA, the number needed to
blood flow in the acutely occluded treat to prevent one death or signifi-
cerebral artery, and may consist of IV cant disability is 8 when treating
thrombolysis and/or specific endovas- within 3 hours of symptom onset,
cular interventions such as intra- and 14 when treating from 3 to 4.5
arterial thrombolysis or mechanical hours.2,3 Figure 1-1 demonstrates the
embolectomy. odds of good outcome with IV rtPA
This discussion is framed around treatment among subjects pooled
the most recent American Heart from major trials to date.4 It has also
Association/American Stroke Associ- been demonstrated that every 30-
ation guidelines, which serve as use- minute delay in acute reperfusion
ful and comprehensive references to leads to a 10% relative reduction in
the reader.1 the likelihood of a good outcome.5

Continuum (Minneap Minn) 2014;20(2):283–295 www.ContinuumJournal.com 283


Copyright © American Academy of Neurology. Unauthorized reproduction of this article is prohibited.
Acute Stroke

KEY POINTS
h Rapid evaluation and
treatment are critical for
the best outcomes.
h A patient should receive
IV recombinant
tissue-type plasminogen
activator within 1 hour
of arrival to an
emergency department
(and sooner is better).

FIGURE 1-1 Odds of good outcome based on stroke onset to treatment time. Pooled
analysis of European Cooperative Acute Stroke Study (ECASS), Alteplase
Thrombolysis for Acute Noninterventional Therapy in Ischaemic Stroke
(ATLANTIS), National Institute of Neurological Disorders and Stroke (NINDS), and
Echoplanar Imaging Thrombolytic Evaluation Trial (EPITHET) subjects demonstrates that
later onset to treatment times lead to lower odds of a treatment effect of IV rtPA
compared with placebo. Treatment beyond 270 minutes appears to have no benefit.
4
Reprinted with permission from Lees KR, et al, Lancet. B 2010 Elsevier. www.sciencedirect.com/science/
article/pii/S0140673610604916.

A patient should receive IV rtPA DECISION TO ADMINISTER


within 1 hour of arrival to an emergency ACUTE REPERFUSION THERAPY
department (and sooner is better) with The following information is needed
the following goals.6 from the emergency department upon
& Emergency department physician initial consultation to guide clinical
sees patient within 10 minutes decision making.
of arrival & Time that patient was last known
& Stroke consultant is notified to be well
within 15 minutes of arrival & Any significant medical history
& CT scan is completed within & A brief neurologic examination
25 minutes of arrival (ideally the NIH Stroke Scale [NIHSS])
& CT is interpreted within 45 & Glucose level (finger stick) and any
minutes of arrival other significant laboratory results
Strategies likely to increase the & Current blood pressure
speed of treatment include early & Results of acute brain imaging,
stroke-team notification (preferably be- typically CT scan, when available
fore or concurrent with CT scan per- Each of these points and their role
formance); storage of rtPA in the in decision making are described in
emergency department; recognition subsequent sections.
that glucose level is the only necessary
laboratory result if no clinical suspicion
for bleeding diathesis is present; and Time
mixing of IV rtPA (1:1 ratio with sterile The patient’s ‘‘last known well’’ time
water or normal saline) early. determines potential treatment options.

284 www.ContinuumJournal.com April 2014

Copyright © American Academy of Neurology. Unauthorized reproduction of this article is prohibited.


KEY POINT

Case 1-1 h IV thrombolysis,


specifically using
A 78-year-old woman with a history of diabetes mellitus presented to the
recombinant tissue-type
emergency department with acute left-sided weakness at 4:00 PM. Her
plasminogen activator,
husband had called 9-1-1 immediately upon identifying her symptoms at
should be considered
3:30 PM. Emergency medical services (EMS) had promptly evaluated the
for patients for whom it
patient and brought her to the stroke-ready emergency department.
can be administered
En route, EMS had prenotified the hospital, where a CT scan was
within 4.5 hours of
performed upon the patient’s arrival. The CT scan showed minimal early
onset of stroke deficits.
ischemic changes and no intracranial hemorrhage. The patient’s
finger-stick glucose level was 95 mg/dL, and her examination revealed left
hemiplegia and anosognosia. Her husband reported that her only
medication was metformin and she had no recent surgeries or prior history
of intracranial hemorrhage. However, upon further discussion regarding
the time of onset with the stroke clinician, it became clear that, while the
patient’s symptoms were discovered at 3:30 PM, her husband had last seen
her well before he left for work at 9:00 AM. The patient reported feeling
fine until her husband came home. Her cell phone was checked, and she
had not spoken to anyone that day. Her neighbor was called by the husband,
and the patient had not been witnessed to be well by anyone else since
9:00 AM. Thus, it was 7 hours from last known well at this point, and IV
recombinant tissue-type plasminogen activator (rtPA) was not administered.
Comment. In this case, if the patient had not had anosognosia, it may
have been appropriate to identify the last known well time based on the
patient’s own history. However, given her lack of appreciation for her
deficit, her report could not be trusted. A normal CT scan has not been
demonstrated to identify patients who might benefit from IV rtPA
regardless of time from last known well. However, an active and promising
area of investigation is to develop imaging markers that can identify
patients who will benefit from IV rtPA treatment based on physiologic
time. Had it not been for the unknown and potentially greater than
4.5-hour time of onset, the patient in this case would have been eligible
for IV rtPA.

It is important to distinguish the time (European Cooperative Acute Stroke


last known well from the time that the Study III).3 Specifically, within 3 hours
stroke deficits were first discovered, as of onset, despite a 6% increased risk of
demonstrated by Case 1-1. symptomatic intracranial hemorrhage,
IV thrombolysis, specifically using the overall absolute benefit of treatment
rtPA, should be considered for patients is a 13% lower rate of significant dis-
for whom it can be administered within ability (defined as a modified Rankin
4.5 hours of onset. IV rtPA (at the dose Scale of 0 or 1). This benefit is estimated
of 0.9 mg/kg, maximum dose 90 mg, to be as high as 33% for any improve-
with 10% given as an initial bolus) has ment in the spectrum of independence
been shown to improve clinical out- to severe disability or death; Figure 1-27
comes for patients within 3 hours of illustrates potential treatment effects.8
onset by two randomized trials (jointly Several community cohorts,9 pooled
referred to as the National Institutes of analyses of other IV rtPA trials,4 and the
Neurological Disorders and Stroke 3-hour subgroup of the Third Interna-
[NINDS] tPA Stroke Study),2 and within tional Stroke Trial (IST-3) provide
3 to 4.5 hours by one randomized trial further supportive evidence.10 Of note,
Continuum (Minneap Minn) 2014;20(2):283–295 www.ContinuumJournal.com 285
Copyright © American Academy of Neurology. Unauthorized reproduction of this article is prohibited.
Acute Stroke

of, for example, time of onset,


coagulopathy, or recent surgery) who
have significant stroke deficits (typically,
an NIHSS score of 8 or higher),
endovascular therapy should be consid-
ered if treatment can be initiated within
6 hours (Case 1-2). Indirect evidence,
including the Prolyse in Acute Cerebral
Thromboembolism (PROACT) II trial
of recombinant prourokinase and the
Middle Cerebral Artery Embolism Local
Fibrinolytic Intervention Trial (MELT)
of urokinase,11Y13 suggests that intra-
arterial rtPA initiated within 6 hours of
onset will lead to better clinical out-
comes than supportive care alone (ie,
no reperfusion therapy). More recent
evidence suggests that endovascular
devices open major arterial occlusions
more effectively, leading to their fre-
quent clinical use in this setting; spe-
cifically, the most recent generation of
mechanical embolectomy devices, in-
cluding the Penumbra Aspiration Sys-
tem, the Solitaire Stent Retriever, and
the TREVO2 Stent Retriever, report
revascularization rates exceeding
80%.14Y16 Comparable safety and effi-
cacy between IV rtPA within 3 hours
FIGURE 1-2 Changes in final outcome as a result of intravenous versus endovascular therapy within 6
recombinant tissue-type plasminogen activator
(IV rtPA) treatment within 3 hours of onset. hours in the SYNTHESIS trial also sup-
7
Reprinted from Saver JL, Medscape. emedicine.
ports the approach for IV rtPAYineligible
medscape.com/article/1160840-overview. patients who present early.17 Case series
data also suggest reasonable safety for
the use of intra-arterial thrombolysis in
KEY POINT the US Food and Drug Administration the nonneurosurgical postoperative set-
h For patients ineligible has only approved the 3-hour time ting.18 Devices available in the United
for IV recombinant window for IV rtPA, whereas clinical States are shown in Figure 1-4.
tissue-type plasminogen guidelines recommend treatment up The role of imaging selection to
activator who have to 4.5 hours based on the current identify patients beyond 6 hours from
significant stroke
evidence.1 It should also be noted that onset who will benefit from acute
deficits (typically, an NIH
no other lytic agent, such as reteplase reperfusion therapies remains to be
Stroke Scale score of 8
or greater), endovascular
or tenecteplase, has been demonstrated determined. A phase 2b study, the
therapy should be as an effective treatment for acute Mechanical Retrieval and Recanalization
considered if treatment ischemic stroke. of Stroke Clots Using Embolectomy
can be initiated within Endovascular therapy for IV re- (MR RESCUE) trial, tested the penum-
6 hours. combinant tissue-type plasminogen bral hypothesis.19 A favorable CT-based
activatorYineligible patients. For pa- or magnetic resonanceYbased penum-
tients ineligible for IV rtPA (because bral selection pattern was defined as a
286 www.ContinuumJournal.com April 2014

Copyright © American Academy of Neurology. Unauthorized reproduction of this article is prohibited.


Case 1-2
A 65-year-old woman who was postoperative from coronary artery bypass graft surgery was found to have
right-sided weakness and confusion in the postoperative anesthesia care unit (PACU) upon regaining consciousness
from general anesthesia at 3:00 PM. She was last known to be neurologically normal at 12:00 PM. The on-call
neurologist, who was immediately activated by the PACU nurse, identified a left middle cerebral artery
syndrome with global aphasia and right hemiparesis (NIH Stroke Scale score of 20). Given the recent surgery,
she deemed the patient to be ineligible for IV recombinant tissue-type plasminogen activator (rtPA), and
alerted the neurointerventional team regarding a potential endovascular case while transporting the patient to
the CT scanner. The CT scan revealed no intracranial hemorrhage, no early ischemic changes, and a hyperdense
left middle cerebral artery; the patient was briskly taken directly from the CT scanner to the neuroangiography
suite. Groin stick was performed at 4 hours after her last time seen normal, and a left proximal middle cerebral
artery thrombus was evacuated by mechanical embolectomy at 4.5 hours with complete angiographic reperfusion
observed on final angiogram, as demonstrated in Figure 1-3. Upon reexamination after the procedure, the patient
had regained strength on her right side and her aphasia (now primarily expressive) was improved. Follow-up CT
scan showed a small branch middle cerebral artery infarct.

FIGURE 1-3 Example of middle cerebral artery (MCA) infarction with hyperdense left middle cerebral artery on baseline CT
scan (A). Proximal MCA (M1) occlusion on pretreatment digital subtraction angiogram in coronal (B) and
sagittal (C) planes, mechanical extraction of intact thrombus (D), and complete angiographic reperfusion in
coronal (E), and sagittal (F) planes.
Courtesy of Aaron Grossman, MD, PhD, and Todd Abruzzo, MD, University of Cincinnati Medical Center.

Comment. Given the recent surgery, IV rtPA could not be safely administered in this patient. In this case,
emergent endovascular treatment was a consideration. Rapid time to angiographic reperfusion likely
contributed to the improved clinical outcome seen after the procedure. Of note, despite the observation of
complete angiographic reperfusion, some infarct burden remained. This may have been due to a distal
embolus or brain tissue that was irreversibly infarcted by the time angiographic reperfusion was achieved.

Continuum (Minneap Minn) 2014;20(2):283–295 www.ContinuumJournal.com 287


Copyright © American Academy of Neurology. Unauthorized reproduction of this article is prohibited.
Acute Stroke

FIGURE 1-4 Devices cleared by the US Food and Drug Administration for acute stroke clot removal: A, Merci Retriever; B, Solitaire
stent retriever; C, Penumbra aspiration system; and D, TREVO2 stent retriever.

Panel A courtesy of Concentric Medical, Inc; Panel B courtesy of Covidien; Panel C courtesy of Penumbra, Inc; Panel D courtesy of Stryker.

KEY POINTS predicted infarct core comprising 70% Medical History


h Rapidly improving or more of ‘‘at-risk’’ tissue as identified In addition to eligibility based on time
deficits should not be by a complex voxel-by-voxel algorithm, from onset, acute reperfusion treat-
considered a
and incorporating baseline NIHSS score ment options are further determined
contraindication unless
for the CT-based cases as well. Patients by eliciting key medical history.
the remaining deficit
is minor.
for whom endovascular treatment could IV rtPA exclusion criteria and relative
be initiated within 8 hours of symptom contraindications are listed in Box 1-1.1
h Additional exclusion onset were randomized to mechanical
criteria for IV
A consensus definition of deficits that
embolectomy versus supportive care, should typically be considered disabling
recombinant tissue-type
plasminogen activator
and stratified by the presence of penum- (regardless of total NIHSS score) is
within the 3- to bra. The presence of penumbra did not shown in Table 1-1.
4.5-hour time window discriminate between subjects who would Based on limited evidence, poten-
include a history of differentially benefit from mechanical em- tial eligibility for endovascular therapy
stroke and diabetes bolectomy, but did predict better clinical in IV rtPAYineligible patients might
mellitus, NIH Stroke outcomes among those with penumbra include age younger than 85 years,
Scale score greater than regardless of treatment arm. Evidence for NIHSS greater than or equal to 8, and
25, age greater than newer definitions of penumbra, and its treatment within 6 hours of onset.1,10,11
80 years, and warfarin use beyond 8 hours from onset, are
use (regardless of limited to nonrandomized cohort studies Brief Examination and NIHSS
international normalized
of treated patients only.20,21 These single- The examination helps the clinician
ratio value).
arm trials are unable to discriminate determine that the patient is indeed
h All clinicians who care between a predictor of treatment effect having a stroke and gauge its severity.
for stroke patients in the and a marker of better clinical outcome The NIHSS score, in particular, is a useful
acute setting should be
regardless of treatment. This is an active way to describe and follow the patient’s
NIH Stroke Scale
area of investigation, and several planned examination (Appendix A). All clinicians
certified.
and ongoing randomized trials will likely who care for stroke patients in the
inform this discussion in the future.22 acute setting should be NIHSS certified
288 www.ContinuumJournal.com April 2014

Copyright © American Academy of Neurology. Unauthorized reproduction of this article is prohibited.


BOX 1-1 IV rtPA Exclusion and Relative Contraindication Criteria
Key IV rtPA Exclusion Criteria
& Stroke or significant head trauma within 3 months
& Major surgery or serious trauma within 14 days
& Gastrointestinal or urinary hemorrhage within 21 days
& Arterial puncture at a noncompressible site within 7 days
& History of intracranial hemorrhage
& Intracranial neoplasm, arteriovenous malformation, or aneurysm
) Some experts consider treating patients with remotely secured or unruptured aneurysms
& Symptoms of subarachnoid hemorrhage
& Active internal bleeding
& Pretreatment blood pressure with systolic 9185 mm Hg or diastolic 9110 mm Hg
& Clear and large hypodensity on CT scan
& Current bleeding diathesis including
) International normalized ratio (INR) 91.7
) Heparin within 48 hours resulting in abnormal partial thromboplastin time (PTT)
) Platelets G100,000/mm3
) Direct thrombin inhibitor (eg, dabigatran) or factor Xa inhibitor (eg, rivaroxaban, apixaban)
use within 48 hours
h Optimal laboratory testing thresholds for safe IV recombinant tissue-type plasminogen
activator (rtPA) use in this setting remain to be determined and are an area of active
investigation.
& Serum glucose G50
) If persistent symptoms after correction, or infarct is verified/supported by imaging, most experts
would consider IV rtPA treatment.
Relative Contraindications for IV rtPA1
& Minor deficit
) Rapidly improving deficits should not be considered a contraindication unless the remaining
deficit is minor.23
) A common definition of minor deficits is an NIH Stroke Scale (NIHSS) score e5 and not clearly
disabling.
) A consensus definition of deficits that should typically be considered disabling (regardless of total
NIHSS score) is shown in Table 1-1.
& Myocardial infarction in the past 3 months
) Some experienced centers treat this as a contraindication only if the myocardial infarction is
subacute and transmural, or other signs suggest a high risk of hemopericardium, such as clinical
or ECG evidence of pericarditis.
) Concurrent acute myocardial infarction may benefit from IV rtPA as well and should be
considered in consultation with a cardiologist; however, only lower stroke dosing of 0.9 mg/kg
(not higher cardiac dosing of approximately 1.1 mg/kg) should be used in this setting.
& Seizure at presentation
) If stroke is verified by imaging, IV rtPA treatment should be considered. However, the severity of
concurrent stroke must be judged in the context of ictal/postictal presentation.
& Pregnancy
) Must weigh risks and benefits in the individual circumstances.

Continued on next page

Continuum (Minneap Minn) 2014;20(2):283–295 www.ContinuumJournal.com 289


Copyright © American Academy of Neurology. Unauthorized reproduction of this article is prohibited.
Acute Stroke

BOX 1-1 IV rtPA Exclusion and Relative Contraindication Criteria (continued)

Additional Exclusion Criteria for IV rtPA Within the 3- to 4.5-Hour Time Window1
& History of stroke AND diabetes mellitus
& NIHSS score 925
& Age 980 years old
& On warfarin (regardless of INR value)

KEY POINTS (http://nihss-english.trainingcampus. The role of adjunctive endovascular


h The risk of intracranial net/uas/modules/trees/windex.aspx). treatment for severe IV rtPAYtreated
hemorrhage when However, emergency physicians are ischemic strokes remains to be deter-
treating a stroke mimic not always NIHSS trained, and a de- mined. The only randomized trial of
with IV recombinant scriptive examination can be used the combined IV rtPA/endovascular
tissue-type plasminogen to estimate the NIHSS score and approach to date, the Interventional
activator inadvertently Management of Stroke (IMS) III trial,
thereby the severity of the stroke in that
is extremely low (less
circumstance. did not demonstrate its superiority
than 1%).
Treating a stroke mimic, such as over IV rtPA alone among subjects of
h The combined IV/ a complicated migraine or seizure, 18 to 83 years of age with severe
endovascular approach
with IV rtPA is inevitable sometimes. strokes (NIHSS score of 8 or higher),
in all patients with an
Fortunately, the risk of intracranial although safety parameters were
NIH Stroke Scale score
hemorrhage when treating a stroke comparable.25 The trial results also
of 8 or greater is not
superior to IV mimic with IV rtPA inadvertently is suggested that the clinical benefit of
recombinant tissue-type extremely low (less than 1%) based on successful flow restoration may be lost
plasminogen activator case series of tPA-treated stroke beyond 7 hours from stroke onset in
alone despite mimics and the cardiology literature.24 the average patient. It has been hy-
comparable safety. Therefore, time to treatment should pothesized that subgroups of patients
not be lost with ancillary testing such may benefit from endovascular therapy,
as MRI or CT angiography if stroke such as those with the most severe
seems likely but not definitive. deficits (eg, an NIHSS score greater than

a,b
TABLE 1-1 Proposed Operational Definition of Disabling Deficits

The following typically should be considered disabling deficits:


Complete hemianopsia (Q2 on NIH Stroke Scale [NIHSS] question 3), or
Severe aphasia (Q2 on NIHSS question 9), or
Visual or sensory extinction (Q1 on NIHSS question 11), or
Any weakness limiting sustained effort against gravity (Q2 on NIHSS
question 6 or 7),
Any deficits that lead to a total NIHSS 95, or
Any remaining deficit considered potentially disabling in the view of the
patient and the treating practitioner. Clinical judgment is required.
a
Reprinted with permission from Re-examining Acute Eligibility for Thrombolysis (TREAT) Task
Force, Stroke.23 stroke.ahajournals.org/content/44/9/2500.abstract.
b
All neurologic deficits present at the time of the treatment decision should be considered in
the context of individual risk and benefit, as well as the patient’s baseline functional status.

290 www.ContinuumJournal.com April 2014

Copyright © American Academy of Neurology. Unauthorized reproduction of this article is prohibited.


KEY POINT
20), patients with demonstrated large directly affect cerebral perfusion pres- h One should not delay IV
proximal arterial occlusions (such as sure. In this setting, increased systemic recombinant tissue-type
internal carotid artery terminus occlusions blood pressure will improve blood flow plasminogen activator
or occlusions that are greater than 8 mm to the cerebral infarct, often via collateral administration for any
in length), and those whose occlusions blood vessels, and thereby may reduce laboratory result other
are recanalized more rapidly or effectively. the extent of irreversible ischemia. than finger-stick glucose
Several randomized trials of more selec- The upper limits of blood pressure level unless a clinical
tive patient subgroups are now underway. control are dictated by the decision of suspicion for an
whether to administer acute reperfu- abnormality exists.
Glucose and Other Laboratory sion therapy. The blood pressure
Results must be less than 185 mm Hg systolic
Testing for serum glucose level is and 110 mm Hg diastolic for IV rtPA
necessary before the IV rtPA treatment eligibility, and must be maintained
decision.1 This should take minimal below 180/105 mm Hg during and
time when performed by finger stick after IV rtPA administration. Gentle
and is often done by EMS en route. blood pressure reduction can usually
This test allows the clinician to exclude be achieved with labetalol 10 mg IV. If
stroke mimics of hypoglycemia or this dose is not enough, then it might
hyperglycemia. If the hypoglycemic be doubled. If no response occurs, a
patient’s symptoms resolve after glu- nicardipine infusion (5 mg per hour
cose administration, then IV rtPA may IV, titrate up by 2.5 mg per hour every
not be indicated. Hyperglycemia can 5 to 15 minutes, maximum 15 mg per
present with choreiform movements hour) may be needed and typically
that can be mistaken for stroke symp- achieves the blood pressure goals. If
toms. Experts generally believe that nicardipine is not available, or contra-
neither hypoglycemia nor hyperglyce- indications exist for this or labetalol,
mia should preclude IV rtPA treatment another consideration is IV enalaprilat
if the deficits are believed to be caused 3.25 mg to 6.5 mg.
by concurrent ischemia.
Other laboratory results should be so- Acute Brain Imaging
licited if available, but they are not ne- Imaging of the brain parenchyma serves
cessary. In particular, INR, PTT, and the primary role of ruling out intracranial
platelet count will help determine IV rtPA hemorrhage, including intracerebral,
eligibility. However, IV rtPA administration subarachnoid, and epidural/subdural lo-
should not be delayed for any laboratory cations. At most centers, a noncontrast
result other than finger-stick glucose CT scan is the most rapid modality
level unless a clinical suspicion for an available, although some centers may
abnormality exists. Fewer than three in use limited MRI (specifically consisting
1000 patients will have unsuspected of diffusion-weighted imaging [DWI],
thrombocytopenia,26 and fewer than four either susceptibility-weighted [SW] im-
in 1000 patients will have an unsus- aging or gradient echo [GRE] se-
pected INR greater than 1.7.27 quences, and fluid-attenuated inversion
recovery [FLAIR] sequences) if rapidly
Blood Pressure available. Rarely, an acute CT scan will
Blood pressure is generally maintained reveal a large and clear hypodensity that
relatively high after acute ischemic will lead to questioning the time of
stroke. In the setting of cerebral ischemia, onset of the stroke and the possibility
a loss of cerebral autoregulation occurs of an undiagnosed subacute stroke,
such that systemic blood pressures both of which would contraindicate IV
Continuum (Minneap Minn) 2014;20(2):283–295 www.ContinuumJournal.com 291
Copyright © American Academy of Neurology. Unauthorized reproduction of this article is prohibited.
Acute Stroke

KEY POINTS
h Subtle ischemic changes rtPA. Subtle changes may also be seen 4.5 hours, and the role of penumbral
on CT scan are not on CT scan, such as loss of gray-white imaging for selecting patients for acute
contraindications to differentiation or sulcal effacement, to reperfusion treatment beyond 4.5 hours
acute reperfusion support the diagnosis of ischemic remains to be determined (as discussed
therapy. stroke. While subtle ischemic changes earlier in this review).
h Standard postYtissue on CT scan are not contraindications to
plasminogen activator acute reperfusion therapy, they do
portend worse outcomes compared POSTREPERFUSION THERAPY
management for the
with those with lesser acute ischemic CARE
first 24 hours includes
aggressive blood pressure changes on CT scan.28 Standard post-tPA management for the
treatment if systolic blood Advanced imaging, specifically head first 24 hours includes the following:
pressure is greater than and neck vascular imaging with CT & admission to a step-down or
180 mm Hg or diastolic angiography or magnetic resonance intensive care unit
blood pressure is greater
than 105 mm Hg.
angiography, can be valuable in plan- & maintenance of nothing-by-mouth
ning the approach to endovascular (NPO) status until dysphagia
therapy if indicated, determining the screening is performed to avoid
etiology of stroke for secondary pre- aspiration pneumonia
vention (eg, carotid endarterectomy), & administration of isotonic IV fluids
or obtaining supportive evidence for (not dextrose containing because
an ischemic stroke diagnosis. Using of risk of hyperglycemia)
this as a first-line imaging approach for & blood pressure and neurologic
all ‘‘rule-out stroke patients’’ in an monitoring every 15 minutes for
emergency department is likely to be 2 hours, then every 30 minutes
inappropriate, given the additional for 6 hours, then every hour for
radiation exposure to a diverse group 16 hours after treatment
of patients. In settings where an expert & aggressive blood pressure treatment
physician triages stroke patients, CT if systolic blood pressure is greater
angiography or magnetic resonance than 180 mm Hg or diastolic blood
angiography may be useful in selected pressure is greater than 105 mm Hg
cases (ie, IV rtPAYineligible patients who & emergent CT scan of the brain if
arrive in the emergency department neurologic decline, acute increase
early) to identify symptomatic occlu- in blood pressure, nausea, vomiting,
sions amenable to acute endovascular or new headache is present to rule
therapy while the neurointerventional out hemorrhagic transformation
team is being mobilized. In all cases, & repeat brain imaging at 24 hours
advanced neuroimaging should not de- to assess for asymptomatic
lay the administration of IV rtPA. hemorrhage and to allow initiation
To date, there is no known role for of antiplatelet therapy
other forms of advanced imaging,
such as perfusion studies with CT
perfusion or magnetic resonance OTHER ACUTE ISCHEMIC
perfusion-weighted imaging, to select STROKE MANAGEMENT
patients for acute reperfusion thera- CONSIDERATIONS
pies. In particular, no data have Blood Pressure in Patients With
supported the utility of identifying No Reperfusion Therapies
penumbra (ie, brain at risk for infarc- Data are very limited to dictate the
tion without reperfusion) for IV rtPA optimal blood pressure in this set-
decisions, as the majority of these ting. Experts generally recommend
patients will have penumbra within permissive hypertension if tolerated
292 www.ContinuumJournal.com April 2014

Copyright © American Academy of Neurology. Unauthorized reproduction of this article is prohibited.


KEY POINTS
(up to 220/120 mm Hg) in the non- combined aspirin and clopidogrel admin- h Experts generally
reperfusion therapy setting and con- istration may better prevent early stroke recommend permissive
sideration of lowering blood pressure recurrence in minor strokes or TIAs, and hypertension if tolerated
by 15% over the first 24 hours. 1 further randomized study is currently (up to 220/120 mm Hg)
Less commonly, blood pressure underway in the United States.31 in the nonreperfusion
may be low in the acute setting, and It is well established that acute therapy setting.
stroke deficits may improve with raising anticoagulation (including unfractionated h Current recommendations
the pressure. Most experts will attempt and low-molecular-weight heparin) are to treat hyperglycemia
IV fluid boluses to raise blood pressure does not improve clinical outcomes to a level lower than
in the setting of relative hypotension after acute ischemic stroke compared 180 mg/dL.
and see if stroke symptoms improve with antiplatelet therapy in unselected h Early aspirin treatment
with this intervention. If the stroke patients. There may be a role for acute leads to a 1% absolute
deficits appear to be pressure depen- anticoagulation in specific circum- reduction of stroke over
dent, more aggressive blood pressure stances in which early stroke recurrence the next 2 weeks.
support with pressors may be consid- risk is high, but clinical data are lacking. h Short-term combined
ered. Evidence for the role of pressors aspirin and clopidogrel
in the acute stroke setting is limited, administration may
however. SUMMARY better prevent early
stroke recurrence in
In summary, effective emergent evalua-
Glucose in the Acute Setting minor strokes or TIAs.
tion of a stroke patient requires well-
Hyperglycemia (greater than 140 mg/dL) organized systems that maximize speed
during the first 24 hours after stroke is a of assessment and administration of
poor prognostic indicator. Whether acute appropriate therapies. This article pro-
correction will lead to better clinical out- vides a practical overview of this pro-
comes after stroke is unknown. Current cess, and the reader is referred to the
recommendations are to treat hyper- American Heart Association/American
glycemia to achieve a level lower than Stroke Association clinical guidelines
180 mg/dL, and stricter glucose control for more detailed discussions. When
is under study in a major randomized these systems cannot appropriately be
clinical trial.1 implemented at a given hospital, it is
imperative that triage and bypass plans
Antithrombotic Therapy in the
be implemented to maximize clinical
Acute Setting
outcomes after stroke.
Aspirin should be initiated within 48
hours in all patients and is typically
initiated in the emergency department
REFERENCES
if no acute reperfusion therapy is
1. Jauch EC, Saver JL, Adams HP, et al.
administered. If IV rtPA or acute Guidelines for the early management of
endovascular therapy is administered, patients with acute ischemic stroke: a guideline
aspirin is initiated at approximately 24 for healthcare professionals from the
American Heart Association/American Stroke
hours and only after confirmation of no Association. Stroke 2013;44(3):870Y947.
hemorrhagic transformation on the
2. Tissue plasminogen activator for acute
24-hour CT scan. Early aspirin treat- ischemic stroke. The National Institute of
ment leads to a 1% absolute reduction Neurological Disorders and Stroke rt-PA
of stroke over the next 2 weeks.29,30 Stroke Study Group. N Engl J Med 1995;333(24):
1581Y1587.
Recent evidence from a randomized
3. Hacke W, Donnan G, Fieschi C, et al;
trial in China of TIA and minor stroke
ATLANTIS Trials Investigators; ECASS Trials
patients who were not treated with IV Investigators; NINDS rt-PA Study Group
tPA has suggested that short-term Investigators. Association of outcome with

Continuum (Minneap Minn) 2014;20(2):283–295 www.ContinuumJournal.com 293


Copyright © American Academy of Neurology. Unauthorized reproduction of this article is prohibited.
Acute Stroke

early stroke treatment: pooled analysis of revascularisation of large vessel occlusions in


ATLANTIS, ECASS, and NINDS rt-PA stroke acute ischaemic stroke (TREVO 2): a randomised
trials. Lancet 2004;363(9411):768Y774. trial. Lancet 2012;380(9849):1231Y1240.
4. Lees KR, Bluhmki E, von Kummer R, et al. 15. Saver JL, Jahan R, Levy EI, et al. Solitaire flow
Time to treatment with intravenous restoration device versus the Merci Retriever
alteplase and outcome in stroke: an updated in patients with acute ischaemic stroke
pooled analysis of ECASS, ATLANTIS, NINDS, (SWIFT): a randomised, parallel-group,
and EPITHET trials. Lancet non-inferiority trial. Lancet 2012;380(9849):
2010;375(9727):1695Y1703. 1241Y1249.
5. Khatri P, Abruzzo T, Yeatts SD, et al. Good 16. Bose A, Henkes H, Alfke K, et al. The
clinical outcome after ischemic stroke with Penumbra System: a mechanical device for
successful revascularization is time-dependent. the treatment of acute stroke due to
Neurology 2009;73(13):1066Y1072. thromboembolism. AJNR Am J Neuroradiol
6. Marler JR, Jones PW, Emr W. Proceedings of 2008;29(7):1409Y1413.
a national symposium on rapid identification 17. Ciccone A, Valvassori L, Nichelatti M, et al.
and treatment of acute stroke. Bethesda, Endovascular treatment for acute ischemic
MD: The National Institute of Neurologic stroke. N Engl J Med 2013;368(10):904Y913.
Disorders and Stroke (NINDS), National
Institutes of Health, 1997;97:4239. 18. Chalela JA, Katzan I, Liebeskind DS, et al. Safety
of intra-arterial thrombolysis in the postoperative
7. Saver JL. Thrombolytic therapy in stroke. period. Stroke 2001;32(6), 1365Y1369.
Medscape. emedicine.medscape.com/article/
1160840-overview. Updated September 18, 19. Kidwell CS, Jahan R, Gornbein J, et al. A trial
2012. Accessed February 6, 2014. of imaging selection and endovascular
treatment for ischemic stroke. N Engl J Med
8. Gadhia J, Starkman S, Ovbiagele B, et al. 2013;368(10):914Y923.
Assessment and improvement of figures to
visually convey benefit and risk of stroke 20. Lansberg MG, Straka M, Kemp S, et al. MRI
thrombolysis. Stroke 2010;41(2):300Y306. profile and response to endovascular
reperfusion after stroke (DEFUSE 2): a
9. Graham GD. Tissue plasminogen activator
prospective cohort study. Lancet Neurol
for acute ischemic stroke in clinical practice:
2012;11(10):860Y867.
a meta-analysis of safety data. Stroke
2003;34(12):2847Y2850. 21. Jovin TG, Liebeskind DS, Gupta R, et al.
Imaging-based endovascular therapy for
10. The IST3 Collaborative Group. The benefits
and harms of intravenous thrombolysis with acute ischemic stroke due to proximal
recombinant tissue plasminogen activator intracranial anterior circulation occlusion
treated beyond 8 hours from time last seen
within 6 h of acute ischaemic stroke (the
third international stroke trial [IST-3]): a well: retrospective multicenter analysis of
randomised controlled trial. Lancet 237 consecutive patients. Stroke 2011;42(8):
2012;379(9834):2352Y2363. 2206Y2211.

11. Furlan A, Higashida R, Wechsler L, et al. 22. Parsons MW, Albers GW. MR RESCUE: is the
Intra-arterial prourokinase for acute glass half-full or half-empty? Stroke
ischemic stroke. The PROACT II study: a 2013;44(7):2055Y2057.
randomized controlled trial. Prolyse in 23. The Re-examining Acute Eligibility for
acute cerebral thromboembolism. JAMA Thrombolysis (TREAT) Task Force; Levine SR,
1999;282(21):2003Y2011. Khatri P, Broderick JP, et al. Review, historical
12. Ogawa A, Mori E, Minematsu K, et al. context, and clarifications of the NINDS
Randomized trial of intraarterial infusion of rt-PA stroke trials exclusion criteria part 1:
urokinase within 6 hours of middle cerebral rapidly improving stroke symptoms. Stroke
artery stroke: the middle cerebral artery 2013;44(7):2055Y2057.
embolism local fibrinolytic intervention 24. Tsivgoulis G, Alexandrov AV, Chang J, et al.
trial (MELT) Japan. Stroke 2007;38(10):
Safety and outcomes of intravenous
2633Y2639. thrombolysis in stroke mimics: a 6-year,
13. Fields JD, Khatri P, Nesbit GM, et al. single-care center study and a pooled
Meta-analysis of randomized intra-arterial analysis of reported series. Stroke
thrombolytic trials for the treatment of 2011;42(6):1771Y1774.
acute stroke due to middle cerebral artery
25. Broderick JP, Palesch YY, Demchuk AM,
occlusion. J Neurointerv Surg 2011;3(2):151Y155.
et al. Endovascular therapy after intravenous
14. Nogueira RG, Lutsep HL, Gupta R, et al. t-PA versus t-PA alone for stroke. N Engl J
Trevo versus Merci retrievers for thrombectomy Med 2013;368(10):893Y903.

294 www.ContinuumJournal.com April 2014

Copyright © American Academy of Neurology. Unauthorized reproduction of this article is prohibited.


26. Cucchiara BL, Jackson B, Weiner M, Messe 29. CAST: randomized placebo-controlled trial
SR. Usefulness of checking platelet count of early aspirin use in 20,000 patients with
before thrombolysis in acute ischemic acute ischaemic stroke. CAST (Chinese Acute
stroke. Stroke 2007;38(5):1639Y1640. Stroke Trial) Collaborative Group. Lancet
1997;349(9066):1641Y1649.
27. Rost NS, Masrur S, Pervez MA, et al.
Unsuspected coagulopathy rarely prevents 30. The International Stroke Trial (IST): a randomised
IV thrombolysis in acute ischemic stroke. trial of aspirin, subcutaneous heparin, both, or
Neurology 2009;73(23):1957Y1962. neither among 19435 patients with acute ischaemic
stroke. Lancet 1997;349(9065):1569Y1581.
28. Demchuk AM, Hill MD, Barber PA, et al.
Importance of early ischemic computed 31. Wang Y, Wang Y, Zhao X, et al. Clopidogrel
tomography changes using ASPECTS in with aspirin in acute minor stroke or
NINDS rtPA Stroke Study. Stroke 2005;36(10): transient ischemic attack. N Engl J Med
2110Y2115. 2013;369(1):11Y19.

Continuum (Minneap Minn) 2014;20(2):283–295 www.ContinuumJournal.com 295


Copyright © American Academy of Neurology. Unauthorized reproduction of this article is prohibited.
Review Article

Risk Factor Management


Address correspondence to
Dr Shyam Prabhakaran, 710
North Lake Shore Drive, Suite
1417, Chicago, IL 60611,
sprabhak@nmff.org.
Relationship Disclosure:
Dr Prabhakaran has served as
for Stroke Prevention
an author of UpToDate, and
has received research support Shyam Prabhakaran, MD, MS; Ji Y. Chong, MD
as a principle investigator
from the National Institute of
Neurological Disorders and
Stroke and the American ABSTRACT
Stroke Association. Dr Chong
reports no disclosure.
Purpose of Review: This review focuses on the recommendations for manage-
Unlabeled Use of ment of hypertension, dyslipidemia, diabetes mellitus, diet, physical activity, and
Products/Investigational lifestyle choices commonly encountered in neurologic practice. Specific studies,
Use Disclosure: including those relevant to lipid targets, blood pressure targets, and adherence to
Drs Prabhakaran and Chong
report no disclosures. medications after stroke, are reviewed.
* 2014, American Academy Recent Findings: In addition to traditional risk factors such as hypertension,
of Neurology. dyslipidemia, and diabetes mellitus, this review discusses sleep apnea, diet, physical
activity, and other novel risk factors that are potentially modifiable. Recent studies
confirm that pharmacologic strategies to achieve aggressive targets for lipid and
blood pressure lowering have significant impact on recurrent stroke risk.
Summary: Optimal secondary prevention strategies can prevent as much as 80%
of all recurrent strokes.

Continuum (Minneap Minn) 2014;20(2):296–308.

Risk factors for stroke include those that One-third of adults in the United States
are nonmodifiable and those that are have hypertension, defined as blood
modifiable. Studies suggest that 90% of pressure (BP) 140/90 mm Hg or greater,
strokes can be explained by modifiable and another 30% may have prehyper-
risk factors, and 80% of recurrent strokes tension, defined as BP between 120
can be prevented with optimal risk factor and 139 mm Hg (systolic) and 80 and
control and/or elimination.1,2 Further- 89 mm Hg (diastolic).4 Studies have con-
more, on a population level, reductions sistently shown a linear relationship
in the burden of these conditions and between BP and stroke risk such that
adherence to medical treatment regi- no threshold exists where risk could
mens have been associated with declines not be further reduced. In prospective
in stroke incidence.3 studies, every 10Ymm Hg reduction in
This review focuses on modifiable BP is associated with a 33% lowering of
medical and behavioral risk factors stroke risk in primary prevention.5
(Table 2-1) commonly encountered The benefits of antihypertensive
by the neurologist after ischemic therapy on secondary stroke preven-
stroke or TIA. Evidence-based tools tion are clear, with a 25% relative-risk
and strategies for risk factor manage- reduction in patients with and without
ment will be addressed. preexisting hypertension.6,7 There-
fore, if BP lowering is reasonable and
MEDICAL RISK FACTORS safe, it should be considered in every
Hypertension stroke patient. Even among patients
Hypertension is the single most im- with intracranial stenosis (Case 2-1), a
portant modifiable risk factor for both condition that may be associated with
hemorrhagic and ischemic stroke. cerebral hypoperfusion, elevated BP

296 www.ContinuumJournal.com April 2014

Copyright © American Academy of Neurology. Unauthorized reproduction of this article is prohibited.


KEY POINT

TABLE 2-1 Modifiable Risk Factors in Secondary Stroke Prevention h Studies suggest that
90% of strokes can be
Risk Factor Treatment Goals explained by modifiable
risk factors, and 80% of
Hypertension Patients with hypertension: G140/90 mm Hg;
recurrent strokes can be
10Ymm Hg reduction in systolic BP and 5Ymm Hg
reduction in diastolic BP from baseline prevented with optimal
risk factor control
Patients with diabetes mellitus or renal disease: and/or elimination.
G130/80 mm Hg; 10Ymm Hg reduction in systolic BP
and 5Ymm Hg reduction in diastolic BP from baseline
Patients without hypertension: G120/80 mm Hg;
10Ymm Hg reduction in systolic BP and 5Ymm Hg
reduction in diastolic BP from baseline
Dyslipidemia Atherosclerotic stroke or TIA: low-density
lipoprotein (LDL) cholesterol level G70 mg/dL or
50% reduction in LDL-cholesterol level from baseline
Nonatherosclerotic stroke or TIA: per National
Cholesterol Education Program (NCEP) Adult
Treatment Panel III (ATP-III) goals
Diabetes mellitus HgA1c level G7%
Cigarette smoking Complete cessation
Alcohol consumption Men: two or fewer drinks per day
Nonpregnant women: one or fewer drinks per day
Physical activity At least 30 minutes of moderate intensity
physical exercise 1Y3 times per week
Diet Low-fat, low-sodium, and Mediterranean or
Dietary Approaches to Stop Hypertension diets
(diabetic diet when applicable)
Obesity Goal body mass index of 18.5Y25 kg/m2

in the postacute phase was associated defined as 120 to 139 mm Hg.10 These
with recurrent stroke.8 In the recent data suggest that a goal BP of approx-
Secondary Prevention of Small Sub- imately 120 to 140 mm Hg (systolic) and
cortical Strokes (SPS3) trial, a lower 80 to 90 mm Hg (diastolic) may be ideal.
systolic BP target (less than 130 mm Whether specific classes of antihyper-
Hg) was associated with a decrease in tensive therapy are superior to others is
recurrent stroke (hazard ratio [HR] debated, although specific subpopula-
0.81, P=.08) with a significant reduc- tions may preferentially benefit from
tion in hemorrhagic stroke events in certain drug classes (Table 2-2). Current
patients with lacunar stroke (HR 0.37, guidelines support any antihypertensive
P=.03).9 However, a recent post hoc agent in patients with stroke, although
analysis of the Prevention Regimen for diuretics with or without angiotensin-
Effectively Avoiding Second Strokes converting enzyme inhibitors (ACE-I)
(PRoFESS) trial found that very low may be reasonable evidence-based,
systolic BP (less than 120 mm Hg) first-line approaches (Figure 2-1). In
after recent stroke was associated with most patients, goal BP lowering should
increased risk of recurrent stroke strive for a 10Ymm Hg and 5Ymm Hg
compared with those with normal BP reduction in systolic BP and diastolic

Continuum (Minneap Minn) 2014;20(2):296–308 www.ContinuumJournal.com 297


Copyright © American Academy of Neurology. Unauthorized reproduction of this article is prohibited.
Risk Factor Management for Stroke

KEY POINT
h Goal blood pressure (BP)
lowering should strive
Case 2-1
A 73-year-old woman with hypertension presented with transient arm weakness
for a 10Ymm Hg and
and expressive aphasia. She reported taking metoprolol for hypertension
5Ymm Hg reduction in
at home, but initial blood pressure (BP) in the emergency department was
systolic BP and diastolic
176/99 mm Hg. Diagnostic imaging revealed left subcortical acute infarcts.
BP, respectively, from
Magnetic resonance angiography demonstrated a left middle cerebral artery
baseline even in
flow gap consistent with high-grade stenosis. Aspirin 325 mg/d was started,
nonhypertensive
and metoprolol was halved for 24 hours, after which it was resumed at the
patients, and less than
prior dose and then increased slowly to achieve BP of 135/80 mm Hg. At a
140/90 mm Hg in
follow-up 1 month later, she was doing well without recurrent stroke or TIA
hypertensive patients
but had not resumed BP medications. BP was 210/105 mm Hg in the clinic.
(less than 130/80 mm
Comment. Given the negative effects of hypertension on stroke (and
Hg in patients with
other vascular) risk, it is recommended that gradual BP lowering be
diabetes mellitus and
initiated as soon as neurologic stability is confirmed (ie, no changes in
chronic kidney disease).
examination in the first 24 hours). While acute management of BP in acute
stroke remains controversial, it is generally safe to begin lowering BP
gradually (15% daily) after the first 24 hours and monitoring for symptom
recurrence even while in the hospital. The long-term goal would be less
than 140/90 mm Hg with careful titration of combination drug therapy.
Frequent BP measurements and follow-up are advised until goals are achieved.

BP, respectively, from baseline even in with diabetes mellitus and chronic kidney
nonhypertensive patients, and less than disease).11 Although very recently pub-
140/90 mm Hg in hypertensive patients lished guidelines12 liberalized these
(less than 130/80 mm Hg in patients goals and recommended initiation of

TABLE 2-2 Antihypertensive Therapy Considerations in Specific


Subpopulations With Stroke or Transient Ischemic
Attack Based on Joint National Committee-7 Guidelinesa

Subpopulation Preferred Drugs


People with congestive Diuretics, angiotensin-converting
heart failure enzyme inhibitor (ACE-I), angiotensin
receptor blocker (ARB), beta-blocker,
and/or aldosterone antagonist
People with postYmyocardial Beta-blockers, ACE-I, aldosterone
infarction antagonist
People with coronary Diuretics, beta-blocker, calcium
artery disease channel blocker (CCB), ACE-I
People with diabetes mellitus Diuretics, beta-blocker, CCB, ACE-I,
and/or ARB
People with chronic ACE-I or ARB
kidney disease
African Americans Diuretics or CCB
People with left ventricular Any class but avoid direct vasodilators
hypertrophy (eg, hydralazine/minoxidil)
a
Data from Chobanian AV, et al. JAMA.11 jama.jamanetwork.com/article.aspx?articleid=196589.

298 www.ContinuumJournal.com April 2014

Copyright © American Academy of Neurology. Unauthorized reproduction of this article is prohibited.


FIGURE 2-1 Algorithm for management of hypertension in patients with ischemic stroke and TIA.
TIA = transient ischemic attack; SBP = systolic blood pressure; DBP = diastolic
blood pressure; ACE-I = angiotensin-converting enzyme inhibitor;
ARB = angiotensin receptor blocker.
a 11
Data from Chobanian AV, et al. JAMA. jama.jamanetwork.com/article.aspx?articleid=196589.

antihypertensive therapy at a higher lipids and stroke risk are less clear
threshold (9 150/90 mm Hg in patients compared with coronary or peripheral
older than 60 years without kidney disease vascular disease, this is mostly attributed
or diabetes mellitus, and 9 140/90 mm Hg to the heterogeneity of stroke (eg,
in those with either condition), these re- hemorrhagic stroke, nonatherosclerotic
commendations were made for primary subtypes of ischemic stroke). A large
prevention and not specifically for pa- meta-analysis found that low-density
tients with stroke (ie, secondary preven- lipoprotein (LDL) cholesterol level,
tion). In addition to lifestyle modification nonYhigh-density lipoprotein (HDL)
and nonpharmacologic approaches, pa- cholesterol level, and very low-density
tient educationVincluding explaining the lipoprotein level (but not HDL choles-
specific medications, doses, risks, and terol and triglyceride levels) were asso-
importance of adherenceVis paramount. ciated with ischemic stroke.
LDL-cholesterol level lowering is the
Dyslipidemia primary goal of dyslipidemia manage-
Approximately one-quarter of adults in ment following stroke, primarily
the United States have elevated choles- through use of statins. A meta-analysis
terol levels.4 While associations between of statins in primary or secondary

Continuum (Minneap Minn) 2014;20(2):296–308 www.ContinuumJournal.com 299


Copyright © American Academy of Neurology. Unauthorized reproduction of this article is prohibited.
Risk Factor Management for Stroke

KEY POINT
h The most recent prevention of stroke found that for each equivalents (Class I, Level of Evidence
American Heart 1 mmol/L (or approximately 39 mg/dL) B). Although data on monitoring
Association guidelines reduction in LDL cholesterol level, a patients with an LDL-cholesterol level
recommend strict goals 21% relative reduction in stroke risk was on statin therapy are limited, the ATP-III
of low-density achieved.13 The Stroke Prevention by recommends checking LDL-cholesterol
lipoprotein (LDL) Aggressive Reduction of Cholesterol levels approximately 6 weeks after
cholesterol level Levels (SPARCL) trial randomized pa- initiation and every 6 weeks until
lowering by 50% or tients with stroke or TIA within 6 targets are achieved; thereafter, mea-
more or an absolute LDL months who had LDL-cholesterol levels surements every 6 to 12 months is
goal level of less than between 100 mg/dL and 190 mg/dL, and reasonable. Monitoring for side ef-
70 mg/dL.
without evident coronary artery disease to fects such as myalgia and liver dys-
atorvastatin 80 mg daily or placebo. function should occur when symptoms
The study observed a modest but sig- or signs are reported or discovered. The
nificant reduction in stroke risk at AHA recently updated their recommen-
5 years in favor of atorvastatin (2.2% dations for cholesterol management.18
absolute and 16% relative risk reduc- While no longer targeting specific LDL
tions).14 Those patients who achieved a goals, guidelines continue to emphasize
50% reduction in LDL-cholesterol level high-dose statin therapy for those with
had the most benefit (38% relative reduc- clinical cardiovascular disease like
tion in stroke risk) followed by those who atherothrombotic stroke and reserve
achieved an LDL-cholesterol level of less moderate intensity statin therapy in
than 70 mg/dL (28% relative risk reduc- patients over age 75 or intolerant of
tion).15 Although potency varies by statin high-dose statin therapy.
drug, with rosuvastatin and atorvastatin For low HDL-cholesterol levels
considered the most potent (capable of (less than 40 mg/dL), niacin may be a
achieving greater than 50% LDL- reasonable treatment option in stroke
cholesterol level lowering at medium to prevention despite the lack of clear
high doses), choice of statin may be benefit in cardiovascular risk reduc-
determined by other factors, including tion. While triglyceride levels may be a
costs and patient tolerance. risk factor for ischemic stroke, partic-
The Adult Treatment Panel (ATP) III ularly large artery atherosclerosis sub-
National Cholesterol Education Pro- types, studies of fibrate medications in
gram (NCEP) has recommended that stroke prevention have not yielded
patients with atherosclerosis such as positive results. Other lipoproteins
carotid artery disease be placed into may be important treatment targets,
the high-risk category, where the goal although evidence for clinical utility of
LDL is less than 100 mg/dL.16 The most this treatment is less compelling. Tri-
recent American Heart Association glycerides, lipoprotein A, lipoprotein
(AHA) guidelines on secondary stroke phospholipase A2 (LpPLA2), and apo-
prevention recommend even stricter lipoproteins are considered risk factors
goals: LDL lowering by 50% or more for cardiovascular disease. Elevated
or an absolute LDL-cholesterol level levels may be markers of stroke risk,
goal of less than 70 mg/dL.17 These but it is unclear whether treatment to
should apply to stroke or TIA patients attain normal levels (eg, dietary modi-
with evidence of atherosclerosis (ie, fication, niacin, fibrates, and statins)
large or small artery disease), those reduces risk.
with an LDL-cholesterol level greater Although a meta-analysis of statin trials
than 100 mg/dL, or those who have concluded that there was no elevated risk
had coronary heart disease or its of intracerebral hemorrhage (ICH)
300 www.ContinuumJournal.com April 2014

Copyright © American Academy of Neurology. Unauthorized reproduction of this article is prohibited.


KEY POINTS
among patients taking statins,19 the stroke) have failed to show benefit, h While the risk of
SPARCL trial found an increased risk especially when targets for HgA1c are intracerebral
(absolute difference 0.9%) of ICH less than 6.5%.23 Given this result, hemorrhage may not be
among stroke patients treated with glycemic control should aim for HgA1c increased in patients
atorvastatin 80 mg daily compared with of less than 7% using diet, exercise, and without prior stroke,
placebo (Case 2-2). Those with prior medications such as oral hypoglycemic patients with prior lobar
lobar hemorrhage, who have higher drugs and insulin.17 Aggressive targets hemorrhage, who
rates of recurrent spontaneous ICH for BP (less than 130/80 mm Hg) and have higher rates of
than those with deep hemorrhage, lipid-lowering (LDL-cholesterol level of recurrent spontaneous
may be particularly at risk on statin less than 70 mg/dL) in patients with intracerebral
hemorrhage than those
therapy.20 diabetes mellitus are associated with
with deep hemorrhage,
decreased stroke risk.24,25
may be particularly at
Diabetes Mellitus
risk on statin therapy.
New guidelines define diabetes Obstructive Sleep Apnea
mellitus as a hemoglobin A1c (HgA1c) The relationship between obstructive h Glycemic control should
aim for hemoglobin A1c
greater than 6.5%.21 It is estimated sleep apnea (OSA) and stroke is com-
of less than 7% using
that 19 million adults in the United plex. OSA has been associated with diet, exercise, and
States have diabetes mellitus, but several physiologic changes, including medications such as oral
nearly 90 million may have prediabe- BP, cardiac structure, and atrial fibrilla- hypoglycemic drugs and
tes and insulin resistance, a number tion, all of which may contribute to an insulin.
that is expected to climb.4 Diabetes increased risk of stroke. Several pro-
mellitus is a risk factor for recurrent, spective studies have found sleep-
particularly lacunar, stroke.22 Despite disordered breathing to be an inde-
the benefits of glycemic control on pendent risk factor for stroke. The
microvascular complications of diabe- odds ratio for incident stroke was 2.24
tes mellitus, trials of aggressive glyce- in a meta-analysis,26 and risk seems to
mic lowering for secondary prevention correlate with severity of OSA.27 OSA
of macrovascular events (including may also develop as a result of stroke,

Case 2-2
A 67-year-old man with diabetes mellitus, coronary artery disease, and
hyperlipidemia presented with left-sided neglect and hemianopsia.
Brain imaging showed a right parietal ischemic stroke with petechial
hemorrhagic transformation. Evaluation revealed right internal carotid
artery stenosis of 80%. Fasting LDL-cholesterol level on admission was
137 mg/dL. Right carotid endarterectomy was performed with good
technical results on hospital day 7. He was discharged on aspirin 325 mg
daily and lisinopril 20 mg daily. Despite concerns about hemorrhagic stroke
risk, the patient was started on atorvastatin 80 mg daily to achieve goal
LDL-cholesterol level of less than 70 mg/dL. He presented to the clinic
at 3 months after stroke for follow-up, at which time laboratory
assessment showed that his LDL level was 65 mg/dL.
Comment. While the Stroke Prevention by Aggressive Reduction of
Cholesterol Levels (SPARCL) trial showed a small risk of hemorrhagic stroke
among stroke patients started on atorvastatin, this patient’s atherosclerotic
disease (both carotid and coronary artery disease) and diabetes mellitus
place him at the highest risk for ischemic vascular events. The absolute risk
of hemorrhage in patients taking statins is small and would be outweighed
by the potential benefits in this case.

Continuum (Minneap Minn) 2014;20(2):296–308 www.ContinuumJournal.com 301


Copyright © American Academy of Neurology. Unauthorized reproduction of this article is prohibited.
Risk Factor Management for Stroke

KEY POINTS
and poststroke OSA may increase MEDICATION COMPLIANCE
h Despite the lack of
benefit in clinical trials, it morbidity and mortality after a Despite strong evidence to support
is reasonable to treat stroke.26,28,29 Complicating the rela- these medical strategies for recurrent
hyperhomocysteinemia tionship between OSA and stroke stroke risk reduction, medication ad-
(greater than 15 2mol/L), further are the shared risk factors herence is a significant problem and
if present, with vitamin between these conditions, eg, obesity has implications for long-term out-
B6, vitamin B12, and folic and hypertension. Further studies are comes. Adherence to antihypertensive
acid replacement in needed to further unravel these asso- medications after stroke is close to
patients with ciations. Randomized clinical trials 90% at 1 year 31 but declines to
atherosclerotic ischemic using continuous positive pressure approximately 75% by 2 years.32 In
stroke.
ventilation among stroke survivors studies of statin use after stroke,
h Effective means of are especially needed to evaluate this 76.3% of patients remained on statin
maintaining medication treatment option to lower recurrent therapy at 1 year, and the number
adherence can be stroke rates and reduce mortality.30 decreased further to 56.1% at 2
elusive, and few
For now, screening for OSA among years.31,32 Continuation of all dis-
randomized trials of
stroke patients and its treatment once charge secondary stroke prevention
interventions to improve
compliance are
diagnosed seem appropriate. medications was only 66% at 1 year.31
available. Patients should Adherence has been associated with
receive education on
Insulin Resistance, Metabolic lower risk of vascular events.33 Effec-
their medications and Syndrome, and Homocysteine tive means of maintaining medication
have adequate follow-up. In addition to hypertension, diabetes adherence can be elusive, and few
h Cigarette smoking is a mellitus, and dyslipidemia, other randomized trials of interventions to
well-established risk atherosclerotic risk factors have been improve compliance are available. Pa-
factor for stroke and has evaluated as potential targets for in- tients should receive education on
a strong association tervention. Insulin resistance may be a their medications and have adequate
with atherothrombotic risk factor for stroke. Metabolic syn- follow-up.
stroke. drome, a condition related to insulin
resistance, is defined as three or more BEHAVIORAL RISK FACTORS
of the following factors: elevated tri- Several behavioral factors, including
glyceride level, low HDL-cholesterol smoking, physical inactivity, diet, obe-
(HDL-C) level, elevated fasting glucose sity, and alcohol use, affect stroke risk.
level, high BP, and high waist circum- In addition, some interventional stud-
ference or abdominal obesity. Meta- ies support lifestyle modification strat-
bolic syndrome appears to be a risk egies in primary prevention, although
factor for stroke separate from its com- limited studies in secondary stroke
ponent parts. Hyperhomocysteinemia prevention have been done.
has also been consistently associated
with atherosclerotic disease. Despite Smoking
epidemiologic associations between Cigarette smoking is a well-established
these conditions and incident stroke, risk factor for stroke and has a strong
currently no compelling clinical trial association with atherothrombotic
data show that treatment of these stroke. In the Framingham study, in-
conditions lowers recurrent stroke risk. vestigators found the relative risk of
Despite the lack of benefit in clinical stroke in smokers, after adjusting for
trials, it is reasonable to treat hyper- age and hypertension, was 2.3 in men
homocysteinemia with vitamin B6, vita- and 3.1 in women. A significant dose-
min B12, and folic acid replacement in response relationship was found with
patients with atherosclerotic ischemic double the risk of stroke in heavy
stroke. smokers compared with light smokers.
302 www.ContinuumJournal.com April 2014

Copyright © American Academy of Neurology. Unauthorized reproduction of this article is prohibited.


KEY POINTS
After 5 years of smoking cessation, Another dietary component that h Meta-analyses of
stroke risk returned to nonsmoker has been attributed to increased sodium reduction trials
levels.34 stroke risk is sodium, because of its have demonstrated that
Because of the dose-response rela- association with hypertension. Meta- decreased sodium
tionship between smoking and stroke analyses of sodium reduction trials intake to 1800 mg per
risk, it is plausible that secondhand have demonstrated that decreased day was associated with
smoke, especially at high levels of sodium intake to 1800 mg per day a 2Ymm Hg reduction in
exposure, also increases stroke risk. was associated with a 2Ymm Hg reduc- systolic BP and 1Ymm
One study of Chinese women who tion in systolic BP and 1Ymm Hg reduc- Hg reduction in diastolic
never smoked found that those ex- tion in diastolic BP in nonhypertensive BP in nonhypertensive
patients and a 5Ymm
posed to more than 20 cigarettes a patients and a 5Ymm Hg (systolic BP)
Hg (systolic BP) and
day of passive smoke had nearly a and 2.7Ymm Hg (diastolic BP) drop in
2.7Ymm Hg (diastolic
twofold increased risk of ischemic hypertensive patients. These small BP) drop in hypertensive
stroke compared with women not changes in BP can still translate into a patients.
exposed to passive smoke.35 Smoking significant public health impact. There-
h Several studies have
cessation should be recommended for fore, the AHA recommends less than
suggested the
secondary stroke prevention, and en- 1500 mg per day of dietary sodium.38 Mediterranean diet as
vironmental smoke should also be Other dietary studies have examined beneficial in
avoided. Physician counseling alone several components of diet together. cardiovascular health.
has a modest but definite benefit; it The Dietary Approaches to Stop Hyper- The Mediterranean diet
has been associated with higher quit tension (DASH) diet encourages high encourages fresh fruits
rates (up to 7.5%) compared with fruit and vegetable, low-fat dairy, low and vegetables, fish,
spontaneous quit rates (3% to 5%). animal protein, and high plant protein legumes, white meat
Group counseling and support groups intake.39 In one study, higher DASH instead of red meat,
are associated with higher quit rates of adherence was associated with lower and wine.
up to 20%.36 Beyond counseling, first- risk of stroke; after adjusting for con-
line medications for smoking cessa- founders, the relative risk (RR) for
tion include nicotine replacement, stroke was 0.82 comparing the top
such as gum or patch. Other treatments versus bottom quintiles of the DASH
include bupropion and varenicline; score.
both appear efficacious. Combination Several studies have suggested the
therapy with nicotine replacement and Mediterranean diet as beneficial in
bupropion is US Food and Drug cardiovascular health. The Mediterra-
AdministrationYapproved and may be nean diet encourages fresh fruits and
superior to either alone.36 vegetables, fish, legumes, white meat
instead of red meat, and wine. A study
Diet of more than 7000 subjects compared
Many studies have evaluated the ef- the Mediterranean diet with a low-fat
fects of different dietary components diet in cardiovascular outcomes.40
on cardiovascular outcomes. In a This study randomized patients with
Cochrane meta-analysis of randomized diabetes mellitus or multiple other
trials of reduced dietary fat, a 14% risk factors to the Mediterranean diet
reduction in cardiovascular events was supplemented with either nuts or
noted in patients who adhered to a olive oil, or a low-fat diet. This trial
reduced-fat or modified-fat diet. Sub- was stopped after an interim analysis
group analysis suggested that modify- because a significant difference occurred
ing fat (reducing saturated fat) rather in the composite outcome of stroke,
than reducing total fat contributed to myocardial infarction, and cardiovascular
this risk reduction.37 death with the Mediterranean diet
Continuum (Minneap Minn) 2014;20(2):296–308 www.ContinuumJournal.com 303
Copyright © American Academy of Neurology. Unauthorized reproduction of this article is prohibited.
Risk Factor Management for Stroke

(HR 0.70) and for the secondary end group drinking less than one drink
point of stroke alone (HR 0.61) com- (RR 0.8) or one to two drinks per day
pared with controls. Overlap occurred (RR 0.7) compared with abstainers.42
between the DASH and Mediterranean Because of concerns about abuse and
diets (Table 2-3). Despite no studies of alcoholism, patients who do not drink
these diets in secondary stroke preven- alcohol should not be encouraged to start
tion, they may be considered in stroke and those who drink excessively should
patients. be advised to moderate their intake.

Alcohol Consumption Obesity


Several studies have been done on A meta-analysis including more than 2
alcohol consumption and stroke risk. million subjects assessed the relation-
One case-control study of an older, ship between normal weight (body mass
multiethnic population suggested that index [BMI] G25 kg/m2), overweight
moderate alcohol consumption (up to status (BMI 25 kg/m2 to 29.9 kg/m2),
two drinks per day) was associated with and obesity (BMI 930 kg/m2) and the
lower stroke risk. An increased risk was risk of stroke. It concluded that the RR
found in the group consuming more than for ischemic stroke was 1.64 in obese
seven drinks per day.41 A meta-analysis of versus healthy subjects and 1.22 in
observational studies of alcohol and overweight versus healthy subjects.43
stroke risk also found a J-shaped curve While BMI is the most commonly used
for ischemic stroke risk with the lowest assessment of obesity, other measures
relative risk for ischemic stroke in the of obesity may be relevant in evaluating

TABLE 2-3 Comparison of the Dietary Approaches to Stop Hypertension and


Mediterranean Diets

Dietary Approaches Recommended Recommended


to Stop Hypertension Servings Mediterranean Servings
Grains 6Y8 Servings per day Grains No recommendation
Vegetables 4Y5 Servings per day Vegetables Q2 Servings per day
Fruits 4Y5 Servings per day Fruits Q3 Servings per day
Fat-free or low-fat 2Y3 Servings per day Milk products No recommendation
milk/milk products
Lean meats, G6 Oz per day Fish and seafood Q3 Servings per week
poultry, fish
White meat instead of red
Red and processed meats G1 Serving per day
Nuts, seeds, legumes 4Y5 Servings per week Tree nuts and peanuts Q3 Servings per week
Legumes Q3 Servings per week
Sofrito Q2 Servings per week
Fats and oils 2Y3 Servings per week Olive oil Q4 Tbsp per day
Wine with meals, only Q7 Glasses per week
for habitual drinkers
Sweets and sugars 5 or Fewer servings per week Soda e1 Drink per day
Baked goods/pastries e3 Servings per week

304 www.ContinuumJournal.com April 2014

Copyright © American Academy of Neurology. Unauthorized reproduction of this article is prohibited.


KEY POINTS
stroke risk. Studies of waist-hip ratio, women.46 The AHA recommends 30 h No completed
waist-height ratio, and waist circumfer- minutes of moderate activity 5 days a randomized trials have
ence suggest abdominal obesity is a risk week or 25 minutes of vigorous activity evaluated weight loss in
factor for stroke, and these measure- 3 days a week. preventing vascular
ments may be better than BMI for stroke Taken together, adherence to a events or in secondary
risk assessment.44 healthy lifestyle could have a signifi- stroke prevention,
Weight loss can lead to improvements cant effect on global stroke incidence. although maintenance
in BP and metabolic profile. However, no Data from the Health Professionals of body mass index
completed randomized trials have evalu- Follow-up Study and the Nurses’ within 18.5 kg/m2 to
ated weight loss in preventing vascular Health Study assessed a low-risk life- 25 kg/m2 is reasonable.
events or in secondary stroke preven- style defined as not smoking, BMI less h Adherence to a healthy
tion, although maintenance of BMI with- than 25 kg/m2, 30 minutes or more lifestyle could have a
in 18.5 kg/m2 to 25 kg/m2 is reasonable. per day of moderate activity, modest significant effect on
alcohol consumption (men 5 g/d to global stroke incidence.
Physical Activity 30 g/d, women 5 g/d to 15 g/d), and
In the Northern Manhattan Study, mod- healthy diet. A significant lowering
erate to heavy-intensity physical activity of ischemic stroke risk was observed
versus inactivity was protective for ische- with a healthier lifestyle score; half
mic stroke in men (adjusted HR 0.65 of the strokes in this population
[0.44 to 0.98]). A protective effect of light can be attributed to unhealthy life-
activity was not observed in women.45 styles.47 Patient education on stroke
Other studies have found that light prevention should include a discus-
activity (eg, walking) reduced cardio- sion of healthy lifestyle behaviors
vascular events, including stroke, in (Case 2-3).

Case 2-3
A 54-year-old man who had been diagnosed with a right occipital stroke after
having visual symptoms presented for outpatient follow-up. He had no prior
history of hypertension, diabetes mellitus, or high cholesterol. He admitted
to smoking several cigarettes a day since age 9 and consumed three to four
alcoholic drinks every day. His blood pressure (BP) was 140/80 mm Hg, weight
77 kg (170 lbs), and height 1.75 m (5 ft, 9 in). He had a left upper-quadrant visual
field cut; otherwise, neurologic examination was normal. His HDL-cholesterol
level was 80 mg/dL and LDL-cholesterol level was 115 mg/dL. His glucose and
hemoglobin A1c levels were normal. His homocysteine level was 24.8 Hmol/L.
He had been placed on aspirin but refused to take a statin.
Comment. This patient has behavioral risk factors of cigarette smoking
and heavy alcohol use. He also has a body mass index of 25.1, placing
him in the overweight category. His BP is elevated, as is his homocysteine
level. An optimal secondary stroke prevention strategy for him includes
several medication and behavioral modifications. Compliance with his
aspirin therapy was reinforced. His BP was elevated and a low-dose
diuretic was started. He was advised to quit smoking and was referred to a
smoking cessation clinic. He was counseled to decrease his alcohol intake
to 2 or fewer drinks per day. Given his chronic alcohol use and elevated
homocysteine level, he was advised to take a daily multivitamin that
included folate and B vitamins. He was willing to change his diet to low fat
and low sodium. He was provided with educational materials that
described a diet of fresh fruits and vegetables, lean meats, low-fat dairy
products, fish, whole grains, and legumes.

Continuum (Minneap Minn) 2014;20(2):296–308 www.ContinuumJournal.com 305


Copyright © American Academy of Neurology. Unauthorized reproduction of this article is prohibited.
Risk Factor Management for Stroke

CONCLUSION 9. SPS3 Study Group, Benavente OR, Coffey CS,


Conwit R, et al. Blood-pressure targets in
Most risk factors for stroke are modi- patients with recent lacunar stroke: the SPS3
fiable. Secondary stroke prevention randomised trial. Lancet 2013;382(9891):
strategies should include treatment of 507Y515.
hypertension, diabetes mellitus, and 10. Ovbiagele B, Diener HC, Yusuf S, et al. Level
high cholesterol with medications and of systolic blood pressure within the normal
range and risk of recurrent stroke. JAMA
lifestyle modification. Lifestyle modifi- 2011;306(19):2137Y2144.
cation after stroke includes adherence
11. Chobanian AV, Bakris GL, Black HR, et al.
to medications, smoking cessation, The seventh report of the Joint National
limiting alcohol intake, maintaining a Committee on Prevention, Detection,
normal weight, adhering to a healthy Evaluation, and Treatment of High Blood
Pressure: the JNC 7 report. JAMA
diet, and engaging in regular physical 2003;289(19):2560Y2572.
activity. These interventions taken
12. James PA, Oparil S, Carter BL, et al. 2014
together can have a significant effect evidence-based guideline for the management
on reducing recurrent stroke risk. of high blood pressure in adults: report from
the panel members appointed to the Eighth
REFERENCES Joint National Committee (JNC 8). JAMA
2014;311(5):507Y520.
1. Gorelick PB. Stroke prevention. Arch Neurol
1995;52(4):347Y355. 13. Amarenco P, Labreuche J. Lipid management
in the prevention of stroke: review and
2. O’Donnell MJ, Xavier D, Liu L, et al. Risk
updated meta-analysis of statins for stroke
factors for ischaemic and intracerebral prevention. Lancet Neurol 2009;8(5):
haemorrhagic stroke in 22 countries (the 453Y463.
INTERSTROKE study): a case-control study.
Lancet 2010;376(9735):112Y123. 14. Amarenco P, Bogousslavsky J, Callahan A
3rd, et al. High-dose atorvastatin after
3. Rothwell PM, Coull AJ, Giles MF, et al. stroke or transient ischemic attack. N Engl J
Change in stroke incidence, mortality, Med 2006;355(6):549Y559.
case-fatality, severity, and risk factors in
Oxfordshire, UK from 1981 to 2004 (Oxford 15. Amarenco P, Goldstein LB, Szarek M, et al.
Vascular Study). Lancet 2004;363 Effects of intense low-density lipoprotein
(9425):1925Y1933. cholesterol reduction in patients with stroke
or transient ischemic attack: the Stroke
4. Go AS, Mozaffarian D, Roger VL, et al. Heart Prevention by Aggressive Reduction in
disease and stroke statisticsV2013 update: a Cholesterol Levels (SPARCL) trial. Stroke
report from the American Heart Association. 2007;38(12):3198Y3204.
Circulation 2013;127(1):e6Ye245.
16. National Cholesterol Education Program
5. Lawes CM, Bennett DA, Feigin VL, Rodgers Expert Panel (NCEP) on Detection Evaluation,
A. Blood pressure and stroke: an overview of Treatment of High Blood Cholesterol in
published reviews. Stroke 2004;35
Adults (Adult Treatment Panel III). Third
(3):776Y785. report of the National Cholesterol Education
6. Rashid P, Leonardi-Bee J, Bath P. Blood Program (NCEP) Expert Panel on Detection,
pressure reduction and secondary prevention Evaluation, and Treatment of High Blood
of stroke and other vascular events: a Cholesterol in Adults (Adult Treatment Panel
systematic review. Stroke 2003;34(11): III) final report. Circulation 2002;106(25):
2741Y2748. 3143Y3421.
7. Thompson AM, Hu T, Eshelbrenner CL, et al. 17. Furie KL, Kasner SE, Adams RJ, et al.
Antihypertensive treatment and secondary Guidelines for the prevention of stroke in
prevention of cardiovascular disease events patients with stroke or transient ischemic
among persons without hypertension: a attack: a guideline for healthcare
meta-analysis. JAMA 2011;305(9):913Y922. professionals from the American Heart
association/American Stroke Association.
8. Turan TN, Cotsonis G, Lynn MJ, et al;
Stroke 2011;42(1):227Y276.
Warfarin-Aspirin Symptomatic Intracranial
Disease (WASID)Trial Investigators. 18. Stone NJ, Robinson J, Lichtenstein AH, et al.
Relationship between blood pressure and 2013 ACC/AHA Guideline on the Treatment
stroke recurrence in patients with intracranial of Blood Cholesterol to Reduce Atherosclerotic
arterial stenosis. Circulation 2007;115(23): Cardiovascular Risk in Adults: A Report of the
2969Y2975. American College of Cardiology/American

306 www.ContinuumJournal.com April 2014

Copyright © American Academy of Neurology. Unauthorized reproduction of this article is prohibited.


Heart Association Task Force on Practice cohort study. J Stroke Cerebrovasc Dis
Guidelines. Circulation 2013 Nov 12. [Epub 2011;20(5):401Y405.
ahead of print].
30. Tomfohr LM, Hemmen T, Natarajan L, et al.
19. McKinney JS, Kostis WJ. Statin therapy and Continuous positive airway pressure for
the risk of intracerebral hemorrhage: a treatment of obstructive sleep apnea in
meta-analysis of 31 randomized controlled stroke survivors: what do we really know?
trials. Stroke 2012;43(8):2149Y2156. Stroke 2012;43(11):3118Y3123.

20. Westover MB, Bianchi MT, Eckman MH, 31. Bushnell CD, Olson DM, Zhao X, et al.
Greenberg SM. Statin use following Secondary preventive medication persistence
intracerebral hemorrhage: a decision and adherence 1 year after stroke. Neurology
analysis. Arch Neurol 2011;68(5):573Y579. 2011;77(12):1182Y1190.

21. International Expert Committee. International 32. Glader EL, Sjolander M, Eriksson M,
Expert Committee report on the role of Lundberg M. Persistent use of secondary
the A1C assay in the diagnosis of diabetes. preventive drugs declines rapidly during the
Diabetes Care 2009;32(7):1327Y1334. first 2 years after stroke. Stroke
2010;41(2):397Y401.
22. Mast H, Thompson JL, Lee SH, et al.
Hypertension and diabetes mellitus as 33. Perreault S, Yu AY, Cote R, et al. Adherence
determinants of multiple lacunar infarcts. to antihypertensive agents after ischemic
Stroke 1995;26(1):30Y33. stroke and risk of cardiovascular outcomes.
Neurology 2012;79(20):2037Y2043.
23. Skyler JS, Bergenstal R, Bonow RO, et al.
Intensive glycemic control and the prevention 34. Wolf PA, D’Agostino RB, Kannel WB, et al.
of cardiovascular events: implications of the Cigarette smoking as a risk factor for stroke.
ACCORD, ADVANCE, and VA diabetes trials: a The Framingham Study. JAMA 1988;259
position statement of the American Diabetes (7):1025Y1029.
Association and a scientific statement of the 35. He Y, Lam TH, Jiang B, et al. Passive smoking
American College of Cardiology Foundation and risk of peripheral arterial disease and
and the American Heart Association. Circulation ischemic stroke in Chinese women who
2009;119(2):351Y357. never smoked. Circulation 2008;118(15):
24. UK Prospective Diabetes Study Group. 1535Y1540.
Tight blood pressure control and risk of 36. Chandler MA, Rennard SI. Smoking cessation.
macrovascular and microvascular complications Chest 2010;137(2):428Y435.
in type 2 diabetes: UKPDS 38. BMJ 1998;
317(7160):703Y713. 37. Hooper L, Summerbell CD, Thompson R,
et al. Reduced or modified dietary fat for
25. Colhoun HM, Betteridge DJ, Durrington PN, preventing cardiovascular disease. Cochrane
et al. Primary prevention of cardiovascular
Database Syst Rev 2012;5:22592684.
disease with atorvastatin in type 2 diabetes
in the Collaborative Atorvastatin Diabetes 38. Appel LJ, Frohlich ED, Hall JE, et al. The
Study (CARDS): multicentre randomised importance of population-wide sodium
placebo-controlled trial. Lancet reduction as a means to prevent cardiovascular
2004;364(9435):685Y696. disease and stroke: a call to action from the
American Heart Association. Circulation
26. Loke YK, Brown JW, Kwok CS, et al. Association
2011;123(10):1138Y1143.
of obstructive sleep apnea with risk of serious
cardiovascular events: a systematic review 39. Fung TT, Chiuve SE, McCullough ML, et al.
and meta-analysis. Circ Cardiovasc Qual Adherence to a DASH-style diet and risk of
Outcomes 2012;5(5):720Y728. coronary heart disease and stroke in women.
Arch Intern Med 2008;168(7):713Y720.
27. Redline S, Yenokyan G, Gottlieb DJ, et al.
Obstructive sleep apnea-hypopnea and 40. Estruch R, Ros E, Salas-Salvado J, et al.
incident stroke: the sleep heart health study. Primary prevention of cardiovascular disease
Am J Respir Crit Care Med 2010;182(2): with a Mediterranean diet. N Engl J Med
269Y277. 2013;368(14):1279Y1290.
28. Yaggi HK, Concato J, Kernan WN, et al. 41. Sacco RL, Elkind M, Boden-Albala B, et al.
Obstructive sleep apnea as a risk factor for The protective effect of moderate alcohol
stroke and death. N Engl J Med 2005;353 consumption on ischemic stroke. JAMA
(19):2034Y2041. 1999;281(1):53Y60.
29. Mansukhani MP, Bellolio MF, Kolla BP, et al. 42. Reynolds K, Lewis B, Nolen JDL, et al.
Worse outcome after stroke in patients with Alcohol consumption and risk of stroke: a
obstructive sleep apnea: an observational meta-analysis. JAMA 2003;289(5):579Y588.

Continuum (Minneap Minn) 2014;20(2):296–308 www.ContinuumJournal.com 307


Copyright © American Academy of Neurology. Unauthorized reproduction of this article is prohibited.
Risk Factor Management for Stroke

43. Strazzullo P, D’Elia L, Cairella G, et al. Excess Northern Manhattan Study. Neurology
body weight and incidence of stroke: 2009;73(21):1774Y1779.
meta-analysis of prospective studies with
46. Manson JE, Greenland P, LaCroix AZ, et al.
2 million participants. Stroke 2010;41(5):
Walking compared with vigorous exercise
e418Ye426.
for the prevention of cardiovascular events
44. Yatsuya H, Yamagishi K, North KE, et al. in women. N Engl J Med 2002;347(10):
Associations of obesity measures with 716Y725.
subtypes of ischemic stroke in the ARIC
47. Chiuve SE, Rexrode KM, Spiegelman D,
Study. J Epidemiol 2010;20(5):347Y354.
et al. Primary prevention of stroke by
45. Willey JZ, Moon YP, Paik MC, et al. Physical healthy lifestyle. Circulation 2008;118(9):
activity and risk of ischemic stroke in the 947Y954.

308 www.ContinuumJournal.com April 2014

Copyright © American Academy of Neurology. Unauthorized reproduction of this article is prohibited.


Review Article

Evaluation and Prevention


Address correspondence to
Dr Anthony S. Kim, UCSF
Department of Neurology,
Sandler Neurosciences Center,

of Cardioembolic Stroke 675 Nelson Rising Lane, Room


411B, San Francisco, CA 94158,
akim@ucsf.edu.
Anthony S. Kim, MD, MAS Relationship Disclosure:
Dr Kim has received research
grants from the American
Heart Association, the NIH,
ABSTRACT and SanBio, Inc; has served
as an expert witness providing
Purpose of Review: The potential for cardioembolic stroke has important record review in a court
implications for clinical management. This review describes the diagnostic workup case; and has received an
and management options for this key stroke subtype. honorarium and travel
expenses for speaking at
Recent Findings: The suspicion for a cardioembolic source for stroke is raised with a an American Neurological
large vessel occlusion or when strokes occur in multiple vascular territories. Diagnostic Association training course.
workup includes ECG, echocardiography, and cardiac monitoring. Atrial fibrillation is Unlabeled Use of
Products/Investigational
the most common cause of cardioembolic stroke and typically justifies anticoagulation Use Disclosure:
therapy. New data on other mechanisms of cardioembolic strokeVsuch as congestive Dr Kim reports no disclosure.
heart failure, prosthetic valves, and aortic arch diseaseVas well as the availability of * 2014, American Academy
novel oral anticoagulants have implications for optimizing stroke prevention. of Neurology.
Summary: Cardiogenic embolization is an important cause of stroke with important
implications for diagnosis, treatment, and prevention.

Continuum (Minneap Minn) 2014;20(2):309–322.

INTRODUCTION of cortical involvement, also suggests a


Cardioembolic stroke accounts for a cardioembolic source. Evidence of multi-
substantial proportion of the overall ple foci of concurrent or sequential
burden of disease from stroke. This ischemia, particularly in multiple cerebro-
disproportionate burden reflects the vascular or systemic vascular beds, is also
overall prevalence of cardioembolic strongly suggestive of a cardioembolic
stroke combined with the greater sever- source, although cardioembolism can
ity, less favorable prognosis, and high initially present with a single ischemic
recurrence risk of this stroke subtype.1 lesion. Certain examination and labo-
The potential for cardioembolism di- ratory findings can be informative in the
rectly affects primary prevention, acute diagnosis of cardioembolic stroke, as
management, and secondary prevention outlined in Table 3-1.
of stroke. Recent advances in diagnosis,
risk stratification, and management devel- Neuroimaging Studies
oped from clinical research, as well as the Neuroimaging studies can help to
availability of new therapeutic options, confirm or establish the vascular dis-
have had an impact on current clinical tribution of both clinically evident and
decision making. subclinical ischemic lesions. Occlusion
of a large- to medium-sized artery with
Initial Clinical Evaluation an otherwise normal appearance of
Cardioembolic stroke typically occurs the parent vessel may suggest cardio-
suddenly, with neurologic symptoms that embolic stroke rather than stroke
are maximal at onset. A stroke syndrome due to intrinsic atherosclerotic disease.
that localizes to a large artery territory, Carotid studies may help to define an
particularly when there is clinical evidence alternative artery-to-artery mechanism.
Continuum (Minneap Minn) 2014;20(2):309–322 www.ContinuumJournal.com 309
Copyright © American Academy of Neurology. Unauthorized reproduction of this article is prohibited.
Cardioembolic Stroke

KEY POINTS
h Cardioembolic strokes which involves the injection of agitated
TABLE 3-1 Clinical Evaluation
are often more severe for Cardioembolic saline as a contrast agent, can identify an
than other stroke Stroke intracardiac (eg, septal defect or patent
subtypes and have a foramen ovale) or extracardiac shunt.
high recurrence risk. b Physical Examination
h The suspicion for Temperature
ATRIAL FIBRILLATION
cardioembolic stroke is Cardiac auscultation An estimated 2.7 million Americans, in-
raised with large artery
Assessment of jugular venous
cluding approximately 8% of the popu-
occlusions or strokes in pressure or peripheral edema lation more than 80 years of age, have
multiple vascular
Lung auscultation atrial fibrillation (AF).3 Given the in-
territories. creased risk of AF with age and the
Retinal examination
h Transesophageal aging of the US population, the overall
Skin examination
echocardiography is burden of AF and consequently stroke
helpful for evaluating b Imaging Studies from AF is expected to surge in the
the aortic arch, aortic CT/MRI of brain coming decades.3,4 Because AF is res-
valve, atrial septum, and Vascular imaging ponsible for nearly half of all cardio-
left atrial appendage. Chest x-ray embolic strokes, and because strokes
h The incidence of stroke b Electrocardiography from AF are more severe and result in
from atrial fibrillation is greater disability and mortality than
Serial ECG
expected to surge in the other stroke subtypes, preventing
coming decades as the Cardiac telemetry
Extended cardiac monitoring
stroke in patients with AF has a consid-
US population ages.
erable impact on both individual patient
h Relatively short and b Echocardiography
outcomes and population health.
asymptomatic periods Transthoracic Most AF-related strokes are caused by
of atrial fibrillation may Transesophageal embolization from the left atrium and
be sufficient to form
b Laboratory Studies atrial appendage, which have direct
thrombus and cause
stroke. White blood cell count access to the cerebral arteries. AF is also
Erythrocyte sedimentation rate associated with enlargement of the left
C-reactive protein atrium, which contributes to relative
Blood cultures stasis, clot formation, and subsequent
Thyroid function tests
embolization. Relatively short periods of
AF may be sufficient to form thrombus
Troponin
because paroxysmal AF appears to pres-
Antiphospholipid antibodies
ent a similar risk for stroke as permanent
CT = computed tomography; MRI = magnetic or persistent AF, but whether there is a
resonance imaging; ECG = electrocardiogram.
threshold burden of paroxysmal AF that
confers this risk is uncertain.

Echocardiography Risk Stratification in Atrial


Transthoracic echocardiography (TTE) Fibrillation
provides a noninvasive assessment of AF confers a 5% annual risk of stroke
the structure and function of the heart overall. However, the estimated risk of
that is essential to the cardioembolic stroke for a particular patient may vary
stroke evaluation. Transesophageal echo- substantially depending on other risk
cardiography (TEE) has higher procedural factors. Risk stratification scores have
risks but is more sensitive than TTE for been developed to estimate this risk and
evaluating the aortic arch, the aortic valve, to inform the choice of antithrombotic
the atrial septum, and left atrial append- therapy. As illustrated in Case 3-1,
age.2 Bubble-contrast echocardiography, CHADS2 (congestive heart failure,
310 www.ContinuumJournal.com April 2014

Copyright © American Academy of Neurology. Unauthorized reproduction of this article is prohibited.


KEY POINTS

Case 3-1 h Patients with atrial


fibrillation and a history
An 80-year-old man with a history of hypertension suddenly developed
of stroke or TIA are in a
left face and arm weakness and neglect. Head CT showed no hemorrhage,
high-risk category that
a neck CT angiogram showed no carotid stenosis, and an MRI demonstrated a
typically justifies
small infarct in the distal right middle cerebral artery territory. His initial ECG
anticoagulation.
showed sinus rhythm, but he had a 2-minute episode of atrial fibrillation (AF)
captured on telemetry on the first hospital day. A transthoracic echocardiogram h The absence of
showed moderate left atrial enlargement. symptoms of atrial
His primary care provider expressed some concern about initiating fibrillation such as
anticoagulation given this patient’s age and risk of falls. The patient’s wife was reduced exercise
concerned about his taking ‘‘rat poison’’ because a family friend had sustained a tolerance, shortness of
warfarin-associated intracranial hemorrhage. After an extensive discussion, breath, or palpitations,
warfarin was initiated during the initial hospitalization given that a large infarct or an initial ECG
was not present, with a plan for close outpatient follow-up. Bridging aspirin showing sinus rhythm is
therapy was stopped once an international normalized ratio (INR) of 2 was not sufficient to exclude
achieved several days later. the possibility of
Comment. This case illustrates the clear indication for anticoagulation paroxysmal atrial
conferred by atrial fibrillation and stroke. Simple risk scores can provide fibrillation as a cause
recommendations based on estimated stroke risk, but a history of stroke of stroke.
and AF is typically sufficient to justify anticoagulation on its own. This
patient’s CHADS2 (congestive heart failure, hypertension, age Q 75 years,
diabetes mellitus, stroke/TIA symptoms previously) score was 4 (8.5%
annual stroke risk), and his CHA2D2-Vasc (congestive heart failure,
hypertension, age Q 75 years, diabetes mellitus, stroke, vascular disease,
age 65Y74 years, sex category) score was 5 (9.3% risk). Concerns about
falls, advanced age, and bleeding risks, while relevant, are likely to be
outweighed by the benefits of anticoagulation. (This patient’s HAS-BLED
[hypertension, abnormal liver or renal function, stroke, bleeding, labile INR,
elderly age, drug or alcohol use] score was 3 [5.8% annual risk]Va threshold
at which caution is warranted.)

hypertension, age Q 75 years, diabetes Electrocardiogram and Cardiac


mellitus, stroke/TIA symptoms previously) Monitoring
and CHA2DS2VASc (congestive heart Detecting an irregularly irregular heart-
failure, hypertension, age Q 75 years, beat on examination or by ECG may
diabetes mellitus, stroke, vascular dis- establish the diagnosis of AF. However,
ease, age 65Y74 years, sex category)Va since paroxysmal AF is often asymp-
refinement of CHADS2Vcombine read- tomatic, at least 24 hours of cardiac
ily available clinical factors into simple monitoring after an ischemic stroke is
scores that provide estimates of annual recommended. An initial ECG demon-
stroke risk (Table 3-2, Table 3-3).5,6 strating sinus rhythm does not exclude
For each of these scores, a history the possibility of a transient episode of
of stroke or TIA is typically sufficient new-onset paroxysmal AF that may
to justify anticoagulation therapy in have preceded the onset of stroke.7
isolation. Nonetheless, neurologists AF is likely to be systematically
may find utility by applying these scores underdiagnosed in patients with crypto-
in more complex cases, particularly genic stroke. Extended cardiac monitor-
when the estimated stroke risk is ing with ambulatory electrocardiograms
weighed against other risks such as the (Holter monitors), real-time continuous
risk of bleeding. heart monitors, external loop recorders,
Continuum (Minneap Minn) 2014;20(2):309–322 www.ContinuumJournal.com 311
Copyright © American Academy of Neurology. Unauthorized reproduction of this article is prohibited.
Cardioembolic Stroke

TABLE 3-2 The CHADS2 Risk Score a,bto Estimate Risk of Stroke in Patients
With Atrial Fibrillation

Annual Stroke 95% Confidence


CHADS2 Score Risk % Interval Recommendation
0 1.9 1.2Y3.0 No antithrombotic
1 2.8 2.0Y3.8 Antiplatelet or
anticoagulation
2 4.0 3.1Y5.1 Anticoagulation
3 5.9 4.6Y7.3 Anticoagulation
4 8.5 6.3Y11.1 Anticoagulation
5 12.5 8.2Y17.5 Anticoagulation
6 18.2 10.5Y27.4 Anticoagulation
a
Data from Gage BF, et al, JAMA.5 jama.jamanetwork.com/article.aspx?articleid=193912.
b
CHADS2
Congestive heart failure 1 point
Hypertension 1 point
Age Q75 years 1 point
Diabetes mellitus 1 point
Stroke/TIA symptoms previously 2 points

TABLE 3-3 The CHA2DS2-VASc Risk Score toa,b Estimate Risk of Stroke in
Patients With Atrial Fibrillation

CHA2DS2-VASc Annual 95% Confidence


Score Stroke Risk % Interval Recommendation
0 0.8 0.6Y1.0 No antithrombotic
1 2.0 1.7Y2.4 Antiplatelet or
anticoagulation
2 3.7 3.4Y4.1 Anticoagulation
3 5.9 5.5Y6.3 Anticoagulation
4 9.3 8.7Y9.9 Anticoagulation
5 15.3 14.3Y16.2 Anticoagulation
6 19.7 18.2Y21.4 Anticoagulation
7 21.5 18.8Y24.6 Anticoagulation
8 22.4 16.3Y30.8 Anticoagulation
9 23.6 10.6Y52.6 Anticoagulation
a 6
Data from Olesen JB, et al, BMJ. www.bmj.com/content/342/bmj.d124.
b
CHA2DS2-VASc
Congestive heart failure 1 point
Hypertension 1 point
Age Q75 years 2 points
Diabetes mellitus 1 point
Stroke/TIA/thromboembolism 2 points
Vascular disease history (previous myocardial infarction, peripheral arterial disease, or aortic plaque) 1 point
Age 65Y74 years 1 point
Sex category (female) 1 point

312 www.ContinuumJournal.com April 2014

Copyright © American Academy of Neurology. Unauthorized reproduction of this article is prohibited.


or implantable loop recorders all allow
for a longer sampling period to detect
transient episodes of AF after stroke.
The 30-Day Cardiac Event Monitor Belt
for Recording Atrial Fibrillation After a
Cerebral Ischemic Event (EMBRACE)
study evaluated 572 patients with cryp-
togenic ischemic stroke or TIA who were
randomized to either 30 days of home-
based cardiac monitoring or repeat
Holter monitoring.8 Significantly more
patients were diagnosed with new
AF in the 30-day monitoring group
compared with the repeat Holter mon- FIGURE 3-1 Adjusted odds ratios for ischemic stroke and
intracranial bleeding in relation to intensity of
itoring group (16% versus 3%, PG.001). anticoagulation. The dashed vertical lines delimit
However, the appropriate candidates the typical therapeutic range for warfarin therapy.
for monitoring, the optimal duration Reprinted with permission from European Heart Rhythm
and mode of monitoring, and the Association, et al, J Am Coll Cardiol.9 B 2006 American
College of Cardiology Foundation. content.onlinejacc.org/
clinical significance of fleeting episodes article.aspx?articleID=1137853.
of AF that can now be detected with
prolonged monitoring have not been
established. provides additional efficacy for stroke KEY POINTS
prevention in AF (2.4% annual risk) h Extended cardiac
Antithrombotic Therapy over aspirin alone (3.3% annual risk; monitoring increases
The mainstay of stroke prevention in AF relative risk of stroke for aspirin com- the sampling period for
detecting paroxysmal
has been oral anticoagulation with war- pared with aspirin and clopidogrel is
atrial fibrillation, but the
farin to an international normalized ratio 1.6; 95% CI 1.3 to 1.9), but this benefit
appropriate candidates
(INR) goal of 2.0 to 3.0 (Figure 3-19). appears to be offset by a higher risk of for monitoring, the
Warfarin can effectively reduce the risk major bleeding (2.0% versus 1.3% an- optimal duration and
of stroke by up to 68% (95% CI 50% to nual risk) that is similar to warfarin.12 mode of monitoring,
79%), which corresponds to an absolute Therefore, national guidelines recom- and the clinical
reduction in the annual risk of stroke mend aspirin alone rather than the significance of fleeting
from 4.5% to 1.4%.10 In contrast, aspirin combination of aspirin and clopidogrel episodes of atrial
is substantially less effective (21% rela- for patients with AF and clear contra- fibrillation have not
tive risk reduction; 95% CI 0% to 38%) indications to oral anticoagulation ther- been established.
and is reserved for patients who are apy, particularly in patients with h Anticoagulation is
unable to take oral anticoagulants. hemorrhagic risks.11 Apixaban 5 mg preferred over
Adding aspirin to warfarin increases twice daily may be a potential alterative antiplatelet agents for
bleeding risk and does not provide for high-risk patients (see the section secondary stroke
clear additional benefits, and therefore on novel oral anticoagulants).13 prevention in most
is generally discouraged.11 The use of For patients with AF and ischemic patients with atrial
fibrillation.
medications for rate control or rhythm stroke despite therapeutic anticoagulation,
control for AF does not obviate the the optimal management strategy is not h Medications for rate or
need for appropriate antithrombotic defined. Most patients with ischemic rhythm control for atrial
therapy to prevent stroke. stroke while on warfarin therapy are fibrillation do not
obviate the need for
For patients deemed unsuitable for found to have a subtherapeutic INR. The
antithrombotic therapy
oral anticoagulation because of a spe- risk of ischemic stroke rapidly rises as the
in patients with atrial
cific bleeding risk or patient prefer- INR drops below 2 (Figure 3-19),14 and fibrillation.
ence, adding clopidogrel to aspirin the percentage of time that patients

Continuum (Minneap Minn) 2014;20(2):309–322 www.ContinuumJournal.com 313


Copyright © American Academy of Neurology. Unauthorized reproduction of this article is prohibited.
Cardioembolic Stroke

KEY POINT
h Potential benefits of spend in the therapeutic range may be Food and Drug Administration (FDA)
novel oral anticoagulants 60% to 70% at most. For selected has approved the direct thrombin
compared to warfarin patients, an increased INR goal could inhibitor dabigatran15,16 and two fac-
include noninferior be considered, but bleeding risks are tor Xa inhibitors (rivaroxaban17,18 and
(rivaroxaban) or superior increased and direct evidence of ben- apixaban19,20) for prevention of stroke
efficacy (dabigatran and efit is lacking. in nonvalvular AF (Table 3-4).
apixaban), lower These agents have fixed dosing, do
intracranial bleeding, Timing of Initiating not require frequent laboratory moni-
fixed dosing, fewer Anticoagulation After toring, have a rapid onset of anticoag-
drug-drug interactions, Acute Stroke ulant effect, and have fewer drug
rapid onset of
During the first few weeks after a new interactions than warfarin. All were
anticoagulant effect,
ischemic stroke, the risks of hemor- found to be noninferior to warfarin
and a mortality benefit
(apixaban).
rhagic transformation, particularly for for stroke and systemic embolism
patients with large-territory infarcts or (Rivaroxaban Versus Warfarin in Non-
with evidence of associated hemor- valvular Atrial Fibrillation [ROCKET-AF],18
rhage, likely outweigh the potential Randomized Evaluation of Long-
benefits of anticoagulation to prevent Term Anticoagulant Therapy [RE-LY],15
early recurrent cardioembolic events and Apixaban for the Prevention of
for most patients. For high-risk patients Stroke in Subjects With Atrial Fibril-
with smaller strokes, well-controlled lation [ARISTOTLE] 20). Dabigatran
blood pressure, and no evidence of (150 mg dose) and apixaban also met
bleeding, earlier initiation of anticoagu- superiority end points. Apixaban showed
lation can be considered, particularly a mortality benefit over warfarin. Intra-
with warfarin since the achieving a full cranial hemorrhage rates were lower for
therapeutic effect typically takes several all three agents compared with warfarin,
days anyway. but major gastrointestinal bleeding was
higher with dabigatran and rivaroxaban
Novel Oral Anticoagulants compared with warfarin.
The intensive laboratory monitoring, As illustrated in Case 3-2, these
narrow therapeutic range, and numer- agents can be considered as an alter-
ous drug interactions associated with native to warfarin in most patients
warfarin have prompted the develop- with AF and stroke, particularly given
ment of alternative oral anticoagu- the lower intracranial hemorrhage
lants. As of December 2013, the US rates seen in these initial studies. But

TABLE 3-4 Properties of Oral Anticoagulant Therapies

Dabigatran
Warfarin Rivaroxaban Etexilate Apixaban
Mechanism Vitamin K antagonist Factor Xa inhibitor Direct thrombin Factor Xa inhibitor
inhibitor
Typical Dose for Variable 20 mg daily 150 mg twice daily 5 mg twice daily
Atrial Fibrillation
Renal Dose No Yes Yes Yes
Adjustment
Half-Life 20Y60 hours 5Y9 hours 12Y17 hours about 12 hours
Onset of Action 24Y72 hours 3Y4 hours 0.5Y2 hours 3Y4 hours

314 www.ContinuumJournal.com April 2014

Copyright © American Academy of Neurology. Unauthorized reproduction of this article is prohibited.


KEY POINTS

Case 3-2 h Potential drawbacks


of novel oral
A 65-year-old man with a history of paroxysmal atrial fibrillation and a
anticoagulants
right middle cerebral artery stroke 1 year ago had been maintained on
compared to warfarin
warfarin therapy. He reported good medication adherence, and his
include the lack of a
international normalized ratio was usually between 2 and 3 at his
validated reversal
follow-up visits to the anticoagulation clinic. Renal function was normal.
strategy or laboratory
He would like to discuss whether he is a candidate for a novel oral
test of anticoagulant
anticoagulant.
effect, uncertainty
Comment. For patients who are already well managed on warfarin,
about candidacy for
continuing warfarin therapy may be reasonable. However, the lower risks
acute thrombolytic
of intracranial hemorrhage that have been reported with the novel oral
therapy, and cost.
anticoagulants could justify making the transition from warfarin to a novel
oral anticoagulant in carefully selected patients. Renal disease, liver h A significant proportion
disease, medication interactions, medication adherence, availability of of patients with atrial
reversal agents, and cost are all considerations when choosing an oral fibrillation who would
anticoagulant. benefit from
anticoagulation are not
on anticoagulation
patients with renal impairment, liver benefits of anticoagulation. Patients therapy.
disease, mechanical valves or other at risk for falls with a CHADS2 score
h Overestimating the
valvular disease, or an anticipated of 2 or higher generally benefit
bleeding risk associated
need for reversal of anticoagulant from anticoagulation therapy, even with falls and advanced
effect for gastrointestinal bleeding when there is an increased risk of age and
(for dabigatran and rivaroxaban) may hemorrhage from falls.22 Although underappreciating
be well served to remain on warfarin. elderly patients have higher risks the benefits of
A validated reversal strategy and a for both ischemic and hemorrhagic anticoagulation with
laboratory test of anticoagulant effect stroke, a meta-analysis of 12 clinical age contribute to the
are not yet available for these novel trials found that the benefits of underuse of
anticoagulants. Poor adherence to a anticoagulation appear to extend to anticoagulation for
twice-daily dosing schedule could re- elderly patients. This benefit was atrial fibrillation.
sult in a subtherapeutic anticoagulant driven by hemorrhage rates that were
effect more quickly compared with similar with aspirin and with warfarin
warfarin. Higher drug costs and con- therapy.23 Ultimately though, the deci-
troversy regarding whether patients sion to initiate anticoagulation is often
on these agents could be candidates complex and should incorporate patient
for acute thrombolytic therapy are preferences.
also relevant considerations. The HAS-BLED (Hypertension, Ab-
normal liver or renal function, Stroke,
Underutilization of Bleeding, Labile INR, Elderly age,
Anticoagulation Drug or alcohol use) scale was devel-
Up to two-thirds of patients with AF, oped to estimate the risk of hemor-
particularly those at highest risk of stroke, rhage with anticoagulation and to
are not on appropriate anticoagulation highlight modifiable factors that are
therapy.21 The underuse of anticoagula- associated with increased risk such as
tion for AF is based, in part, on the concomitant antiplatelet therapy or
challenges of warfarin therapy; however, alcohol intake (Table 3-5 24,25 ). 24
other significant barriers remain. HAS-BLED can provide additional in-
The bleeding risks associated with formation for clinical decision making,
frequent falls or advanced age are often but should not be used as the sole
outweighed by the stroke-prevention reason to withhold anticoagulation

Continuum (Minneap Minn) 2014;20(2):309–322 www.ContinuumJournal.com 315


Copyright © American Academy of Neurology. Unauthorized reproduction of this article is prohibited.
Cardioembolic Stroke

TABLE 3-5 HAS-BLEDa,bScore to Estimate the Risk of Hemorrhage With


Warfarin

HAS-BLED Bleeding 95% Confidence


Score Risk % Interval Recommendation
0 0.9 0.4Y1.9 None
1 3.4 2.5Y4.6 None
2 4.1 2.9Y5.6 None
3 5.8 3.9Y8.3 Caution warranted
4 8.9 5.2Y14.0 Caution warranted
5 9.1 1.1Y29.2 Caution warranted
Q6 Insufficient data Insufficient data Caution warranted
a
Data from Pisters R, et al, Chest,24 journal.publications.chestnet.org/article.aspx?articleid=1045174.
and Lip GY, et al, J Am Coll Cardiol.25 www.sciencedirect.com/science/article/pii/S0735109710043378.
b
HAS-BLED
Hypertension 1 point
Systolic blood pressure 9160 mm Hg
Abnormal renal and/or liver function 1 or 2 points
Renal: dialysis, kidney transplant, or creatinine Q2.6 mg/dL (1 point)
Liver: cirrhosis or bilirubin 92 normal; or AST, ALT, or AlkPhos 93 normal (1 point)
Stroke 1 point
Bleeding predisposition or history 1 point
Labile international normalized ratio (INR) 1 point
Time spent in therapeutic range G60%
Elderly 1 point
Age 965 years
Drugs and/or excess alcohol use 1 or 2 points
Antiplatelet agents or nonsteroidal anti-inflammatory drugs
Maximum HAS-BLED Score is 9 points
AST = aspartate aminotransferase; ALT = alanine aminotransferase; AlkPhos = alkaline phosphatase.

therapy for patients who have other prevent stroke with AF has not been
indications for anticoagulation for established.
stroke secondary prevention.
VALVULAR DISEASE
Surgical Procedures for Atrial Infective Endocarditis
Fibrillation Stroke is often the primary presenting
Left atrial appendage closure devices and symptom of infective endocarditis.
left atrial appendage excision or exclu- Although direct infection of normal
sion procedures as well as Cox maze valves can occur, the risk of endocar-
procedures (a series of surgical incisions ditis is much higher when the native
to the atria to interrupt atrial reentry) and valve is compromised. Although rheu-
radioablation have been developed to matic heart disease is uncommon in
treat AF. For most patients, these pro- developed counties, it is still the most
cedures do not obviate the need for common cause of valvular disease
long-term anticoagulation therapy be- globally. Bicuspid aortic valve and
cause a continued risk for recurrent AF prosthetic valve implantation confer
remains even after treatment, and the higher risks for infection as well. IV
risk of recurrent stroke for patients drug use is more commonly associated
with AF who have already had a stroke with right-sided valvular disease (eg,
or TIA is high. Therefore, the precise tricuspid valve) and typically results in
role for these procedural approaches to septic embolization to the lungs rather
316 www.ContinuumJournal.com April 2014

Copyright © American Academy of Neurology. Unauthorized reproduction of this article is prohibited.


KEY POINTS
than ischemic stroke, but IV drug use edge of heart valves and is primarily h A transthoracic
still confers a higher risk for left-sided associated with systemic malignancy. echocardiogram is not
endocarditis. Staphylococcus aureus, Heparin or low-molecular-weight hepa- adequate to exclude the
Streptococcus viridans, and Enterococ- rin rather than warfarin is generally possibility of valvular
cus are the most common organisms.26 recommended.28 Libman-Sacks endo- vegetations when
Endocarditis is suggested by fever, carditis, which is associated with sys- endocarditis is
weight loss, a history of IV drug use or temic lupus erythematosus and the suspected. Rather, a
other source of bacteremia, a new antiphospholipid antibody syndrome, transesophageal
murmur on physical examination, or involves a similar deposition of immune echocardiogram is
evidence of systemic embolization to complexes that result in small inflamma- critical to the diagnostic
workup of possible
other vascular territories. Multiple blood tory vegetations. Direct evidence is
endocarditis.
cultures, serial ECG (to evaluate for lacking, but antiplatelet therapy for
myocardial infarction, heart block, or primary prevention or anticoagulation h Anticoagulation with
conduction delay), and TEE are critical for secondary prevention is reasonable. warfarin is strongly
indicated for all patients
elements of the diagnostic workup. TTE
Rheumatic Heart Disease with mechanical heart
is not adequate to exclude the possibility
valves.
of valvular vegetations when endocardi- Rheumatic heart disease, a complication of
tis is suspected.27 An elevated erythro- Group A streptococcal infection, most
cyte sedimentation rate or C-reactive commonly affects the mitral valve. Con-
protein, and a leukocytosis, which is current AF is common.11 The risk of
not always present, support this diag- stroke remains even after successful
nosis but are relatively nonspecific. valvuloplasty, so continued anti-
Brain MRI is better suited than coagulation to a goal INR of 2.0 to 3.0 is
head CT for demonstrating the multi- recommended.
ple foci of ischemia and cerebral
microbleeds that are typical for endo- Prosthetic Mechanical Valves
carditis. The lesions in endocarditis and Bioprosthetic Valves
are classically of differing ages and in Prosthetic mechanical valves present a
multiple vascular distributions or in a high risk for thromboembolization, es-
watershed distribution; subclinical pecially for devices with caged-ball or
brain lesions are common. tilting-disc mechanisms and for devices
Antibiotic therapy may reduce the risk in the mitral position. Anticoagulation
of continued embolization. Anti- is indicated for all patients with me-
thrombotic therapy in the acute setting chanical valves; the intensity of anti-
is associated with an increased risk of coagulation therapy and the specific
hemorrhage. For patients with AF or indications for concomitant antiplatelet
mechanical heart valve prostheses who therapy are based on the type and
present with endocarditis and stroke, position of the prosthetic valve.
short-term interruption of anticoagula- For patients with prosthetic valves
tion therapy may be necessary in some that present with embolic events such
cases, despite the high risks of throm- as stroke despite adequate anti-
bosis and recurrent stroke during the thrombotic therapy, national guidelines
period that anticoagulation is withheld. provide comprehensive suggestions on
intensifying antithrombotic therapy for
Nonbacterial Thrombotic various clinical scenarios, although de-
Endocarditis finitive data to support these recom-
Nonbacterial thrombotic endocarditis mendations are admittedly sparse
(marantic endocarditis) involves the ag- (Table 3-629). Bioprosthetic valves do
gregation of sterile vegetations along the not present the same degree of embolic
Continuum (Minneap Minn) 2014;20(2):309–322 www.ContinuumJournal.com 317
Copyright © American Academy of Neurology. Unauthorized reproduction of this article is prohibited.
Cardioembolic Stroke

TABLE 3-6 Suggested Intensification of Antithrombotic Therapy


for Stroke With Prosthetic Valve Replacementa

Therapy on Presentation
With New Stroke Dose Adjustment Option
Warfarin, international Increase INR goal to 2.5Y3.5
normalized ratio (INR) 2.0Y3.0
Warfarin, INR 2.5Y3.5 Increase INR goal to 3.5Y4.5
No aspirin Initiate aspirin 75Y100 mg/d
Warfarin + aspirin 75Y100 mg/d Increase aspirin dose to 325 mg/d
Aspirin 75Y100 mg/d Increase aspirin dose to 325 mg/d,
and/or add clopidogrel 75 mg/d,
and/or add warfarin
a
Data from Bonow RO, et al, J Am Coll Cardiol.29 www.sciencedirect.com/science/article/pii/S0735109708018330.

risk as mechanical valves and can tients with associated AF or left ven-
generally be managed with anti- tricular thrombus, anticoagulation is
platelet therapy. However, warfarin is generally preferred over antiplatelet
recommended for the first 3 months therapy.
after implantation of a mitral bio- As illustrated in Case 3-3, for patients
prosthetic valve. with heart failure and sinus rhythm, the
Dabigatran is not recommended for optimal choice of antiplatelet or
use in patients with mechanical heart anticoagulation therapy is uncertain. In
valves, and rivaroxaban and apixaban the Warfarin and Antiplatelet Therapy in
have not been evaluated for this indi- Chronic Heart Failure (WATCH) study,
cation. Therefore, warfarin remains the patients with symptomatic heart failure,
preferred treatment for this indication. sinus rhythm, and a left ventricular
ejection fraction of 35% or less were
Mitral Valve Prolapse randomized to aspirin (162 mg/d),
Mitral valve prolapse is a largely echo- clopidogrel (75 mg/d), or warfarin (INR
cardiographic diagnosis that is charac- 2.5 to 3.0).32 There were no significant
terized by abnormal movement of one differences between the three treat-
or more valve leaflets into the left atrium ment arms for the risk of the composite
during systole. Data on the relationship primary end point of death, myocardial
between mitral valve prolapse and infarction, or stroke. Similarly, in the
stroke are conflicting. Most patients with Warfarin and Aspirin in Patients With
stroke and mitral valve prolapse can be Heart Failure and Sinus Rhythm
managed with antiplatelet therapy.30 (WARCEF) study, patients with an ejection
fraction below 35% with sinus rhythm
CARDIOMYOPATHY AND were randomized to aspirin 325 mg/d
STRUCTURAL HEART LESIONS or adjusted-dose warfarin (goal INR of
Heart Failure 2.75; range 2.0 to 3.5).31 Anticoagulation
Heart failure is associated with an did not provide an overall benefit: a re-
increased risk of stroke from altered duction in ischemic stroke was offset by
flow within dilated heart chambers, an increase in major hemorrhage.
relative stasis of blood, increased risk For secondary prevention, warfarin,
of concomitant AF, and an associated aspirin, clopidogrel, or aspirin plus
relative prothrombotic state. For pa- extended dipyridamole are all reasonable

318 www.ContinuumJournal.com April 2014

Copyright © American Academy of Neurology. Unauthorized reproduction of this article is prohibited.


KEY POINTS

Case 3-3 h For patients with


congestive heart failure
A 64-year-old woman with a history of congestive heart failure presented
with sinus rhythm, the
with an acute right middle cerebral stroke. Her echocardiogram showed
choice of
biatrial enlargement and a left ventricular ejection fraction of 30%; no
anticoagulation or
thrombus or valvular disease was seen. ECG showed sinus rhythm. A brain
antiplatelet agent for
MRI showed multiple foci of restricted diffusion in the right middle
secondary stroke
cerebral artery territory, as well as three small foci of restricted diffusion in
prevention is uncertain.
the left hemisphere. The patient was discharged home on antiplatelet
therapy, but, given the suspicion for a cardioembolic source, she was h Patients with left
referred for extended cardiac monitoring. One week later, she had an ventricular thrombus
asymptomatic 5-minute episode of atrial fibrillation noted on cardiac should generally be
monitoring and returned to clinic to transition to anticoagulation therapy. treated with
Comment. This case illustrates the initial management of a stroke anticoagulation for at
patient with a history of congestive heart failure with a reduced ejection least 3 months.
fraction and sinus rhythm with antiplatelet therapy based on the Warfarin
and Aspirin in Patients With Heart Failure and Sinus Rhythm (WARCEF)
study.31 However, the distribution of the infarcts suggested a cardioembolic
source, and the documented paroxysmal atrial fibrillation demonstrated in
follow-up provided a clear justification for anticoagulation therapy.

options, but comparative data are lacking. Other Cardiac Lesions


Data on the use of novel oral anticoagu- Atrial myxoma, a tumor of mesenchy-
lants in this setting are not available. mal cells of the heart, is a rare cause of
cardioembolic stroke that is seen most
Myocardial Infarction and Left often in younger patients. An atrial mass
Ventricular Thrombus is seen on echocardiography and can be
Left ventricular thrombus, a complica- present even without a mitral stenosis
tion of myocardial infarction, presents murmur or a tumor plop on ausculta-
a high risk for embolization and tion. Papillary fibroelastoma is a rare
subsequent stroke. Left ventricular cause of stroke that is also apparent on
thrombus is more common with ante- echocardiography. For both lesions, the
rior ST-segment elevation myocardial ongoing risk of embolization typically
infarction because of the relative stasis warrants surgical resection.
of blood within the ventricle. There-
fore, for patients at high risk of AORTIC ARCH ATHEROSCLEROSIS
developing left ventricular thrombus, Aortic arch atherosclerosis confers a
a screening TTE is recommended. risk for both systemic and cerebrovas-
For patients with documented left cular embolization. This commonly oc-
ventricular thrombus and myocardial curs via thromboembolism of larger
infarction, at least 3 months of clots that lodge and fragment in medi-
anticoagulation is recommended.11 um and large vessels. However, choles-
Follow-up TTE to evaluate left ventric- terol embolization from the plaque
ular ejection fraction and residual itself may also occur and typically
thrombus may help with individual- showers much smaller emboli into the
ized decisions on the optimal dura- arterioles in multiple vascular terri-
tion of anticoagulation therapy. Again, tories. Cholesterol embolization from
novel oral anticoagulants have not the plaque itself can shower much
been evaluated in this setting and are smaller emboli into arterioles in multi-
not recommended. ple vascular territories.
Continuum (Minneap Minn) 2014;20(2):309–322 www.ContinuumJournal.com 319
Copyright © American Academy of Neurology. Unauthorized reproduction of this article is prohibited.
Cardioembolic Stroke

TEE is generally preferred over TTE ischemia of uncertain etiology. J Am Coll


Cardiol 1991;17(1):66Y72.
for assessing the aortic arch. Plaque
thickness greater than 4 mm, ulcera- 3. Go AS, Mozaffarian D, Roger VL, et al. Heart
disease and stroke statisticsV2013 update:
tion, and actively mobile elements a report from the American Heart
have been implicated as potential risk Association. Circulation 2013;127(1):e6Ye245.
factors for embolization.33Y35 4. Go AS, Hylek EM, Phillips KA, et al.
Increased plaque thickness suggests Prevalence of diagnosed atrial fibrillation
a complex plaque with overlying throm- in adults: national implications for rhythm
management and stroke prevention: the
bus. Ulceration is thought to increase AnTicoagulation and Risk Factors in Atrial
the risk of thrombus formation. Mobile Fibrillation (ATRIA) Study. JAMA 2001;285(18):
elements suggest that a thrombus is 2370Y2375.
superimposed on an unstable plaque. 5. Gage BF, Waterman AD, Shannon W, et al.
Aortic arch atherosclerosis shares Validation of clinical classification schemes
for predicting stroke: results from the
risk factors in common with stroke National Registry of Atrial Fibrillation.
more generally, so these patients often JAMA 2001;285(22):2864Y2870.
have clear indications for aggressive risk 6. Olesen JB, Lip GY, Hansen ML, et al. Validation
factor modification and antiplatelet of risk stratification schemes for predicting
therapy for secondary prevention stroke and thromboembolism in patients
with atrial fibrillation: nationwide cohort
irrespective of aortic arch disease. study. BMJ 2011;342:d124.
Statins may have the potential to stabi-
7. Jauch EC, Saver JL, Adams HP Jr, et al.
lize or even regress plaques. The Aortic Guidelines for the early management of
Arch Related Cerebral Hazard Trial patients with acute ischemic stroke: a
(ARCH) trial compared the combina- guideline for healthcare professionals from
tion of aspirin and clopidogrel with the American Heart Association/American
Stroke Association. Stroke 2013;44(3):870Y947.
adjusted-dose warfarin to prevent re-
current vascular events in patients with 8. Gladstone D, Spring M, Dorian P, et al.
Prolonged ambulatory cardiac monitoring
significant aortic arch disease and a improves the detection and treatment of
recent TIA or minor stroke.36 The final atrial fibrillation in patients with cryptogenic
results of this study are not yet avail- stroke: primary results from the EMBRACE
multicenter randomized trial. Honolulu, HI:
able; however, the trial was recently International Stroke Conference, 2013.
and prematurely terminated for futility.
9. European Heart Rhythm Association;
Heart Rhythm Society; Fuster V, Rydén LE,
SUMMARY Cannom DS, et al. ACC/AHA/ESC 2006
guidelines for the management of patients
Cardioembolism is an important cause with atrial fibrillationVexecutive summary:
of stroke that requires an initial clini- a report of the American College of
cal suspicion, a tailored diagnostic Cardiology/American Heart Association Task
Force on Practice Guidelines and the
workup, particularly for atrial fibrilla- European Society of Cardiology Committee
tion, and a prevention strategy that for Practice Guidelines (Writing Committee
draws upon current clinical evidence to Revise the 2001 Guidelines for the
and therapeutic options. Management of Patients With Atrial
Fibrillation). J Am Coll Cardiol 2006;48(4):
854Y906.
REFERENCES 10. Risk factors for stroke and efficacy of
1. Lin HJ, Wolf PA, Kelly-Hayes M, et al. Stroke antithrombotic therapy in atrial fibrillation.
severity in atrial fibrillation. The Framingham Analysis of pooled data from five randomized
Study. Stroke 1996;27(10):1760Y1764. controlled trials. Arch Intern Med 1994;154(13):
1449Y1457.
2. Pearson AC, Labovitz AJ, Tatineni S,
Gomez CR. Superiority of transesophageal 11. Furie KL, Kasner SE, Adams RJ, et al.
echocardiography in detecting cardiac Guidelines for the prevention of stroke in
source of embolism in patients with cerebral patients with stroke or transient ischemic

320 www.ContinuumJournal.com April 2014

Copyright © American Academy of Neurology. Unauthorized reproduction of this article is prohibited.


attack: a guideline for healthcare professionals 22. Gage BF, Birman-Deych E, Kerzner R, et al.
from the American Heart Association/American Incidence of intracranial hemorrhage in
Stroke Association. Stroke 2011;42(1): patients with atrial fibrillation who are prone
227Y276. to fall. Am J Med 2005;118(6):612Y617.
12. Connolly SJ, Pogue J, Hart RG, et al. Effect 23. van Walraven C, Hart RG, Connolly S, et al.
of clopidogrel added to aspirin in patients Effect of age on stroke prevention therapy
with atrial fibrillation. N Engl J Med in patients with atrial fibrillation: the atrial
2009;360(20):2066Y2078. fibrillation investigators. Stroke
2009;40(4):1410Y1416.
13. Furie KL, Goldstein LB, Albers GW, et al. Oral
antithrombotic agents for the prevention of 24. Pisters R, Lane DA, Nieuwlaat R, et al. A
stroke in nonvalvular atrial fibrillation: a novel user-friendly score (HAS-BLED) to
science advisory for healthcare professionals assess 1-year risk of major bleeding in
from the American Heart Association/ patients with atrial fibrillation: the Euro
American Stroke Association. Stroke Heart Survey. Chest 2010;138(5):1093Y1100.
2012;43(12):3442Y3453. 25. Lip GY, Frison L, Halperin JL, Lane DA.
14. American College of Cardiology Foundation; Comparative validation of a novel risk score
American Heart Association; European for predicting bleeding risk in anticoagulated
Society of Cardiology; Heart Rhythm Society, patients with atrial fibrillation: the HAS-BLED
Wann LS, Curtis AB, Ellenbogen KA, et al. (Hypertension, Abnormal Renal/Liver Function,
Management of patients with atrial fibrillation: Stroke, Bleeding History or Predisposition,
a report of the American College of Cardiology/ Labile INR, Elderly, Drugs/Alcohol Concomitantly)
American Heart Association Task Force on score. J Am Coll Cardiol 2011;57(2):173Y180.
Practice Guidelines. Circulation 2013;127(18): 26. Selton-Suty C, Celard M, Le Moing V,
1916Y1926. et al. Preeminence of Staphylococcus
15. Connolly SJ, Ezekowitz MD, Yusuf S, et al. aureus in infective endocarditis: a 1-year
Dabigatran versus warfarin in patients with population-based survey. Clin Infect Dis
atrial fibrillation. N Engl J Med 2009;361(12): 2012;54(9):1230Y1239.
1139Y1151. 27. Nishimura RA, Carabello BA, Faxon DP, et al.
16. Diener HC, Connolly SJ, Ezekowitz MD, et al. ACC/AHA 2008 guideline update on valvular
Dabigatran compared with warfarin in heart disease: focused update on infective
patients with atrial fibrillation and previous endocarditis: a report of the American
transient ischaemic attack or stroke: a College of Cardiology/American Heart
subgroup analysis of the RE-LY trial. Association Task Force on Practice Guidelines
Lancet Neurol 2010;9(12):1157Y1163. endorsed by the Society of Cardiovascular
Anesthesiologists, Society for Cardiovascular
17. Hankey GJ, Patel MR, Stevens SR, et al. Angiography and Interventions, and Society
Rivaroxaban compared with warfarin in of Thoracic Surgeons. Circulation 2008;118(8):
patients with atrial fibrillation and previous 887Y896.
stroke or transient ischaemic attack: a
subgroup analysis of ROCKET AF. Lancet 28. Whitlock RP, Sun JC, Fremes SE, et al.
Neurol 2012;11(4):315Y322. Antithrombotic and thrombolytic therapy
for valvular disease: Antithrombotic Therapy
18. Patel MR, Mahaffey KW, Garg J, et al. and Prevention of Thrombosis, 9th ed:
Rivaroxaban versus warfarin in nonvalvular American College of Chest Physicians
atrial fibrillation. N Engl J Med Evidence-Based Clinical Practice Guidelines.
2011;365(10):883Y891. Chest 2012;141(2 suppl):e576SYe600S.
19. Easton JD, Lopes RD, Bahit MC, et al. 29. Bonow RO, Carabello BA, Chatterjee K, et al.
Apixaban compared with warfarin in 2008 focused update incorporated into
patients with atrial fibrillation and previous the ACC/AHA 2006 guidelines for the
stroke or transient ischaemic attack: a management of patients with valvular heart
subgroup analysis of the ARISTOTLE trial. disease: a report of the American College of
Lancet Neurol 2012;11(6):503Y511. Cardiology/American Heart Association Task
20. Granger CB, Alexander JH, McMurray JJ, Force on Practice Guidelines (Writing
et al. Apixaban versus warfarin in patients Committee to revise the 1998 guidelines for
the management of patients with valvular
with atrial fibrillation. N Engl J Med
2011;365(11):981Y992. heart disease). Endorsed by the Society of
Cardiovascular Anesthesiologists, Society
21. Birman-Deych E, Radford MJ, Nilasena DS, for Cardiovascular Angiography and
Gage BF. Use and effectiveness of warfarin Interventions, and Society of Thoracic
in Medicare beneficiaries with atrial Surgeons. J Am Coll Cardiol 2008;52(13):
fibrillation. Stroke 2006;37(4):1070Y1074. e1Ye142.

Continuum (Minneap Minn) 2014;20(2):309–322 www.ContinuumJournal.com 321


Copyright © American Academy of Neurology. Unauthorized reproduction of this article is prohibited.
Cardioembolic Stroke

30. Lansberg MG, O’Donnell MJ, Khatri P, et al. 33. The French Study of Aortic Plaques Group.
Antithrombotic and thrombolytic therapy Atherosclerotic disease of the aortic arch as
for ischemic stroke: Antithrombotic Therapy a risk factor for recurrent ischemic stroke.
and Prevention of Thrombosis, 9th ed: N Eng J Med 1996;334(19):1216Y1221.
American College of Chest Physicians
34. The Stroke Prevention in Atrial Fibrillation
Evidence-Based Clinical Practice Guidelines. Investigators Committee on Echocardiography.
Chest 2012;141(2 suppl):e601SYe636S. Transesophageal echocardiographic correlates
31. Homma S, Thompson JL, Pullicino PM, et al. of thromboembolism in high-risk patients
Warfarin and aspirin in patients with heart with nonvalvular atrial fibrillation. Ann Intern
failure and sinus rhythm. N Engl J Med Med 1998;128(8):639Y647.
2012;366(20):1859Y1869. 35. Amarenco P, Cohen A, Tzourio C, et al.
32. Massie BM, Collins JF, Ammon SE, et al. Atherosclerotic disease of the aortic artch
Randomized trial of warfarin, aspirin, and and the risk of ischemic stroke. N Engl J Med
1994;331(22):1474Y1479.
clopidogrel in patients with chronic heart
failure: the Warfarin and Antiplatelet 36. Aortic Arch Related Cerebral Hazard Trial
Therapy in Chronic Heart Failure (WATCH) (ARCH). www.clinicaltrials.gov/ct/show/
trial. Circulation 2009;119(12):1616Y1624. NCT00235248. Accessed August 26, 2013.

322 www.ContinuumJournal.com April 2014

Copyright © American Academy of Neurology. Unauthorized reproduction of this article is prohibited.


Review Article

Large Artery
Address correspondence to
Dr Seemant Chaturvedi,
Department of Neurology,
Wayne State University,

Atherosclerosis: Carotid 8C-UHC, 4201 St. Antoine,


Detroit, MI 48201,
schaturv@med.wayne.edu.

Stenosis, Vertebral Relationship Disclosure:


Dr Chaturvedi has received
compensation for expert

Artery Disease, witness testimony and research


support from AstraZeneca,
Daiichi Sankyo, and Johnson
& Johnson Services, Inc.

and Intracranial Dr Chaturvedi serves as a


consultant for Abbott Vascular
and W. L. Gore & Associates,

Atherosclerosis Inc; on the executive committee


of the Asymptomatic Carotid
Trial (ACT)Y1 and Carotid
Revascularization Endarterectomy
Seemant Chaturvedi, MD, FAAN; Pratik Bhattacharya, MD, MPH versus Stenting Trial (CREST)Y2
studies; and as a contributing
editor to NEJM Journal Watch
Neurology. Dr Bhattacharya
ABSTRACT has received a research grant
Purpose of Review: Large artery atherosclerosis is an important cause of ischemic from the Ethel & James Flinn
stroke. Recent randomized clinical trials have helped clarify the treatment options Foundation to study poststroke
depression in stroke patients.
for conditions such as carotid stenosis and intracranial atherosclerosis. This review Unlabeled Use of
outlines the primary findings of these trials and provides current recommendations Products/Investigational
for treatment. Use Disclosure:
Drs Chaturvedi and Bhattacharya
Recent Findings: Carotid revascularization is preferred in patients with severe report no disclosures.
symptomatic carotid stenosis. Carotid endarterectomy achieves lower rates of * 2014, American Academy
stroke or death than carotid artery stenting. The risk of stroke or death with of Neurology.
stenting is higher among older patients and women. Intensive medical therapy
achieves low stroke and death rates in asymptomatic stenosis. Medical therapy and
treatment of atherosclerotic risk factors are the mainstay of therapy for intracranial
atherosclerosis, and medical therapy is recommended for patients with vertebral
artery origin atherosclerosis.
Summary: Contemporary medical therapy is paramount in large artery atheroscle-
rosis. Patient demographics, comorbidities, and the periprocedural risks of stroke
and death should be carefully weighed while choosing a revascularization
procedure for carotid stenosis.

Continuum (Minneap Minn) 2014;20(2):323–334.

Atherosclerosis of the large arteries is cular therapies for some large artery
responsible for about 15% of all ische- atherosclerotic lesions, specifically carotid
mic strokes. Within the last decade, disease, the importance of intensive
there has been significant progress in medical management cannot be over-
the medical management of athero- emphasized. In this article, we will
sclerosis. Blood pressure lowering and discuss contemporary management prin-
control of dyslipidemia have im- ciples of three conditions: cervical ca-
proved, resulting in enhanced sec- rotid atherosclerosis, intracranial
ondary stroke prevention. Even with the atherosclerosis, and vertebral artery ori-
availability of surgical and endovas- gin atherosclerosis.

Continuum (Minneap Minn) 2014;20(2):323–334 www.ContinuumJournal.com 323


Copyright © American Academy of Neurology. Unauthorized reproduction of this article is prohibited.
Large Artery Atherosclerosis

KEY POINTS
CERVICAL CAROTID [P=.009]).2 Perioperative MI was signif-
h Symptomatic carotid
ATHEROSCLEROSIS icantly higher among the CEA patients
stenosis has a markedly
increased risk of Carotid atherosclerosis accounts for (1.1% CAS versus 2.3% CEA [P=.03]).
stroke compared with about 7% of ischemic strokes. In the The significant increase in strokes
asymptomatic stenosis. Framingham Heart study, the degree of among the CAS patients was noted up
h In the Carotid stenosis was predicted by common to 4 years (6.2% CAS versus 4.7% CEA
Revascularization baseline atherosclerotic risk factors such [P=.049]).2
Endarterectomy versus as older age, cigarette smoking, systolic The traditionally accepted outcome
Stenting Trial (CREST) blood pressure, and total cholesterol.1 of stroke and death in the perioperative
study, periprocedural In studies from the prestatin era, pa- period, and stroke up to 4 years of
stroke was higher with tients with an asymptomatic carotid follow-up (excluding MI) was significantly
carotid artery stenting, stenosis less than 75% had an annual higher in the CAS arm (6.4% with CAS
whereas periprocedural stroke risk of 1.3%; with a stenosis and 4.7% with CEA [P=.03]).2 A note-
myocardial infarction greater than 75% the annual risk of worthy feature of the assessment of
was higher with carotid
stroke was 2.0% to 2.5%. On the other outcomes in CREST was the regular
endarterectomy.
hand, symptomatic carotid stenosis screening for MIs with ECGs and cardiac
h In the CREST study, greater than 70% carries an annual enzymes before and after the proce-
periprocedural stroke stroke risk of 10% to 15%. Intensive dure.2 Some critics question the inclu-
had a greater impact on
medical therapy and carotid revascular- sion of MI as a primary outcome while
patients’ quality of life
ization procedures reduce these risks. evaluating procedures intended for
than periprocedural
myocardial infarction.
Clinical trials in carotid stenosis are stroke prevention.3 As outcomes go,
geared toward answering two questions: does a stroke or an MI have greater
Which patients should opt for revascu- impact? The physical component of the
larization procedures (versus intensive Short Form-36 questionnaire for health-
medical therapy alone), and which is related quality of life was significantly
the appropriate revascularization proce- worse at 1 year among stroke patients,
dure (carotid endarterectomy [CEA] but showed an uncertain effect among
versus carotid artery stenting [CAS])? MI patients. The mental component in
Our understanding of the contem- CREST was also significantly worse
porary management of carotid stenosis among stroke patients at 1 year.2 On
is primarily shaped by the findings of the other hand, long-term mortality
the Carotid Revascularization Endarter- rates were higher among patients who
ectomy versus Stenting Trial (CREST). had an MI in the perioperative period,
After 4 years of follow-up, the primary even after adjustment of baseline co-
outcome (stroke, myocardial infarction morbid factors.4 Whether the perioper-
[MI], or death in the periprocedural ative MI event is causally linked with
period, or any ipsilateral stroke within later mortality or is a marker of patients
4 years) occurred in 7.2% of the 1262 with a greater atherosclerotic disease
patients in the CAS group and 6.8% of burden is unclear.
the 1240 CEA patients (P=.51).2 How-
ever, upon review of the individual
components of the outcome, certain Symptomatic Carotid Stenosis
significant differences were observed. The North American Symptomatic Ca-
In comparison with the CEA group, rotid Endarterectomy Trial (NASCET)
patients in the CAS group had signifi- has set the paradigm for revasculariza-
cantly higher perioperative strokes tion of patients with moderate (50% to
(4.1% CAS versus 2.3% CEA [P=.01]) 69%) to severe (70% to 99%) symptom-
and perioperative minor ipsilateral atic carotid stenosis. In this trial, pa-
strokes (2.9% CAS versus 1.4% CEA tients with symptomatic carotid stenosis
324 www.ContinuumJournal.com April 2014

Copyright © American Academy of Neurology. Unauthorized reproduction of this article is prohibited.


KEY POINT
greater than or equal to 30% were brain, resulting in intracerebral hemor- h Carotid revascularization
randomized to medical therapy versus rhage.7 To determine the ideal time for within 2 weeks of a
CEA.5 revascularization surgery, data from stroke or TIA is
After a mean follow-up of 2 years, 5893 patients with 33,000 patient-years preferred over delaying
patients with severe stenosis showed a of follow-up from NASCET and the revascularization for
dramatic risk reduction of any ipsilateral European Carotid Surgery Study were 6 weeks or more.
stroke from 26% in the medical arm to analyzed. Patients with moderate and
9% in the CEA arm. The absolute risk severe stenosis showed significant ben-
reduction was 17% (PG.001), which efits from CEA if the procedure was
translated to a number needed to treat performed within 2 weeks of the index
of six.6 Among patients with moderate event (Case 4-1). The benefit declined
stenosis, CEA showed a lower, but thereafter and was no longer statistically
significant, risk reduction from 22.2% significant beyond 2 weeks in the 50%
in the medical arm to 15.7% in the CEA to 69% group.8 Based on these data, the
arm. The absolute risk reduction was American Heart Association recom-
6.5% (P=.045), resulting in a number mends that it is reasonable to perform
needed to treat of 15.5 Patients with CEA when indicated for patients with
mild stenosis of 30% to 49% did not stroke or TIA within 2 weeks, rather
achieve a significant risk reduction of than delaying surgery.9 Surgery within 2
any ipsilateral stroke following CEA.5 weeks may be contraindicated for pa-
The highest risk for a recurrent tients who have very large infarctions at
stroke is during the first month; the high risk of hemorrhagic conversion, or
risk decreases with time because of already have hemorrhagic conversion of
plaque stabilization and the develop- their infarctions.
ment of collaterals.7 Therefore, the In CREST, about 53% of the study
benefit obtained from carotid revascu- population had symptomatic carotid
larization declines with time from the stenosis. The periprocedural stroke,
initial event. However, very early revas- death, and MI rates in the symptomatic
cularization procedures could potentially group were not significantly different
lead to hemodynamic changes within between the CAS and CEA groups
an acutely necrotic area of infarcted (6.7% with CAS versus 5.4% with CEA

Case 4-1
An 81-year-old woman presented to the emergency department because
of a 1-hour episode of transient expressive speech difficulty and right hand
weakness. The patient was on aspirin 81 mg/d but not on a statin. MRI
of the brain showed a 1 cm infarct in the left precentral gyrus on
diffusion-weighted imaging. Magnetic resonance angiography (MRA)
showed severe left internal carotid stenosis. Carotid duplex
ultrasonography confirmed severe left internal carotid artery stenosis
(80% to 99%) and less than 30% stenosis on the right side. An uneventful
carotid endarterectomy (CEA) was performed 4 days after admission.
Comment. Given the patient’s age and symptomatic status, it was felt
that she should undergo carotid revascularization, with CEA preferred.
CEA is preferred over carotid artery stenting (CAS) given the higher
complication rate with CAS in patients aged 70 years or older. One
might suggest that a trial of aggressive medical therapy is warranted
because she was not on a statin at the time of presentation.

Continuum (Minneap Minn) 2014;20(2):323–334 www.ContinuumJournal.com 325


Copyright © American Academy of Neurology. Unauthorized reproduction of this article is prohibited.
Large Artery Atherosclerosis

KEY POINT
h Symptomatic patients [P=.30]).10 The periprocedural rate of indications where CAS may be a pre-
appear to have a lower stroke and death was higher in CAS ferred option over CEA.
complication rate with versus CEA (6.0% with CAS versus 3.2% Data regarding intensive medical
carotid endarterectomy with CEA [P=.02]).10 The rates of MI therapy for symptomatic carotid steno-
than with carotid artery and cranial nerve palsies on the other sis are lacking. Only about 15% of
stenting. hand were higher with CEA compared patients in NASCET were on lipid-
with CAS. lowering medication. New clinical trials
Similar trends were noted in a should be undertaken for symptomatic
preplanned meta-analysis of patient patients with extracranial carotid steno-
data from three randomized controlled sis utilizing the aggressive medical ther-
European trials: Endarterectomy versus apy regimen from the Stenting and
Angioplasty in Patients with Symptom- Aggressive Medical Management for
atic Severe Carotid Stenosis (EVA-3S) Preventing Recurrent Stroke in Intracra-
trial, the Stent-Protected Angioplasty nial Stenosis (SAMMPRIS) trial (de-
versus Carotid Endarterectomy (SPACE) scribed in the section on intracranial
trial, and the International Carotid atherosclerosis).
Stenting Study (ICSS).11 The data in-
cluded 3433 patients with symptomatic Asymptomatic Carotid Stenosis
carotid stenosis, randomly assigned to The risks of stroke in patients with
CAS versus CEA. In the first 120 days asymptomatic stenosis are lower than
after randomization, significantly more in those with symptomatic stenosis.
strokes and deaths occurred among the Increasing evidence shows that inten-
CAS group (8.9%) compared with the sive medical therapy only, without
CEA group (5.8%; P=.0006).11 In aggre- revascularization, can reduce the is-
gate, these data show that, for patients chemic stroke risk dramatically. Over
with symptomatic carotid stenosis, the the past decade, the intensity of
short-term outcomes for stroke and medical therapy has improved signifi-
death are better with CEA compared cantly. In NASCET, only 16% of pa-
with CAS. Table 4-1 provides a list of tients assigned to the medical arm and

TABLE 4-1 High Surgical Risk Factors for Carotid Endarterectomy Where
Carotid Artery Stenting May Be Considered as an Option

b Anatomic Factors (Surgically Inaccessible Carotid Stenosis)


Obesity
High carotid bifurcation
Severe cervical spine arthritis
b Clinical Factors
Clinically significant cardiac disease (congestive heart failure, abnormal stress
test, need for open-heart surgery)
Severe lung disease
Contralateral carotid occlusion
Contralateral laryngeal nerve palsy
Previous radical neck surgery
Previous radiation to the neck
Recurrent stenosis after carotid endarterectomy

326 www.ContinuumJournal.com April 2014

Copyright © American Academy of Neurology. Unauthorized reproduction of this article is prohibited.


KEY POINTS
13% of patients assigned to the CEA asymptomatic carotid stenosis. Asymp- h A new clinical trial,
arm were on lipid-lowering medica- tomatic patients with ongoing micro- CREST-2, will evaluate
tions.5 In the Asymptomatic Carotid emboli14 and men younger than 75 years whether optimal medical
Surgery Trial-1 (ACST-1), use of lipid- merit consideration for CEA. Ongoing therapy alone is the
lowering medications improved from microemboli can be detected using trans- preferred treatment for
7% to 11% in the early 1990s at the cranial Doppler studies, but these are asymptomatic carotid
start of the trial, to 80% to 82% in the not in widespread clinical use. Men stenosis.
late 2000s toward the end of long- below 75 years of age are considered h Intensified medical
term follow-up.12 The use of antihy- candidates more frequently than women therapy has reduced the
pertensive drugs similarly improved, as a result of uncertainty regarding stroke rate for patients
corresponding to a drop in mean whether asymptomatic women benefit. with asymptomatic
diastolic blood pressures over time.12 In the CREST study, the rate of the carotid stenosis.
Abbott analyzed 11 asymptomatic primary outcome among asymptomatic
carotid intervention studies between patients was similar: 5.6% with CAS and
1985 and 2007.13 Raw data from these 4.9% with CEA.10 However, the stroke
trials were used to calculate rates of and death rate by 4 years was 4.5% in
ipsilateral stroke, ipsilateral stroke/ the CAS arm and 2.7% in the CEA arm
TIA, any territory stroke, and any (P=.07).10 The study methods did not
territory stroke/TIA in the medical provide enough power to identify sig-
therapy arm; the rate of each outcome nificant differences in the asymptomatic
decreased during this time period.13 subgroup. A similar trial designed with
The outcome rates from medical ther- sufficient power in the asymptomatic
apy in the more contemporary studies subgroup could plausibly have found
were quite similar, if not better, than the above difference in stroke and
in the CEA arm in the ACAS trial.13 In death rate to be significant.3
conclusion, as seen in Case 4-2, ex-
tremely low stroke rates (1% per year or Special Considerations
less) can be achieved by intensive Carotid revascularization in women.
medical therapy among patients with Across most carotid revascularization

Case 4-2
An 80-year-old man was referred for left carotid stenosis detected by his
internist after a carotid bruit was heard. He had a history of hypertension
and dyslipidemia. He was on aspirin 325 mg/d, atorvastatin 80 mg/d,
ramipril 10 mg/d, and a diuretic. Blood pressure was 136/78 mm Hg and
heart rate was 64 beats/min and regular. Neurologic examination was
normal, except for the presence of a left carotid bruit. Carotid duplex
ultrasonography revealed 80% to 99% stenosis on the left and 30% to
49% on the right. CT angiography (CTA) was interpreted as showing
70% to 80% stenosis on the left and 40% on the right. Low-density
lipoprotein was 51 mg/dL. The patient was counseled regarding the
uncertain benefit of revascularization in his age group.
Comment. This patient was continued on aggressive medical therapy
and has been symptom-free for 3 years. This type of patient could be
considered for enrollment in a clinical trial such as CREST-2, which is comparing
outcomes with aggressive medical therapy alone versus aggressive medical
therapy plus carotid endarterectomy or carotid artery stenting.

Continuum (Minneap Minn) 2014;20(2):323–334 www.ContinuumJournal.com 327


Copyright © American Academy of Neurology. Unauthorized reproduction of this article is prohibited.
Large Artery Atherosclerosis

KEY POINTS
h In CREST, women had trials, women have carried more ICSS.11 The event rate for any stroke or
a higher complication perioperative risk than men. One death in the CEA arm was 5.7% in
rate with carotid artery potential mechanism of this phenom- subjects younger than 70 years and
stenting, compared with enon is the smaller diameter of the 5.9% in subjects 70 years and older.
carotid endarterectomy. carotid artery in women.15 Sex differ- On the other hand, any stroke or death
h Patients older than ences in perioperative risk from ca- occurred in 5.8% of subjects younger
70 years have a higher rotid revascularization in CREST have than 70 years in the CAS arm and 12%
complication rate with been published. The rate of 30-day of subjects 70 years and older.11
carotid artery stenting stroke, death, or MI in the CAS arm While the above lines of evidence
than with carotid was 4.3% among men and 6.8% suggest that CAS in older adults has high
endarterectomy. among women. 16 Among women, rates of 30-day stroke and death, very
h Ethnic differences exist the 30-day stroke and death rate in few clinical trial data of complication
with regard to the the CAS arm was 5.5%, and 2.2% in rates from CEA or medical therapy in
frequency of intracranial the CEA arm (P=.013). 16 Among this subgroup are available. The ACST-1
atherosclerosis. symptomatic women, CAS resulted study had 30 of 1662 patients in the CEA
in a higher 30-day stroke and death arm who were over the age of 75 years,
rate compared with CEA (7.5% versus and 38 of 1701 subjects in the medical
2.7%, P=.03). In summary, women arm over the age of 75 years.12 Within
appear to have higher periprocedural this subgroup of elderly patients in
stroke risk compared with men, with ACST 1, medical therapy achieved equiv-
risks being potentially greater with alent results to revascularization by CEA
CAS compared with CEA. (odds ratio: 0.81, 95% CI 0.43 to 1.51).12
Carotid revascularization in older In view of the uncertain benefit of CEA
adults. An important finding in the in asymptomatic, elderly patients, the
CREST study was an effect modification American Academy of Neurology guide-
of the primary outcome by age. Patients lines recommend consideration of CEA
older than 70 years appeared to do only in asymptomatic patients younger
better with CEA, whereas patients youn- than 75 years.18
ger than 70 years fared better with CAS
(P=.02 for interaction).2 The rate of INTRACRANIAL ATHEROSCLEROSIS
primary outcome with CEA was 6.1% for Intracranial atherosclerotic disease is
subjects younger than 65 years, 6.8% for an important cause of ischemic stroke.
subjects between 65 and 74 years, and An impressive racial difference exists,
7.4% for subjects 75 years and older.17 such that intracranial atherosclerotic
In contrast, the rate of primary outcome diseaseYrelated strokes comprise 9%,
rose much more steeply with age in the 17%, and 15% of all ischemic strokes
CAS arm: 3.9% in patients younger than among white, African American, and
65 years, 6.3% in the 65 to 74 years age Hispanic patients, respectively.19 The
group, and 12.7% in patients 75 years proportion is even higher in the Asian
and older.17 This age-treatment interac- population. Patients with 50% to 99%
tion was found in symptomatic and stenosis of intracranial vessels who
asymptomatic subjects.17 A similar inter- develop symptoms are at a 12% to 14%
action was also found when the stroke risk for a recurrent stroke during a 2-year
end point (periprocedural stroke and follow-up, in spite of antiplatelet therapy
postprocedure ipsilateral stroke) was or anticoagulation therapy. The annual
examined in CREST. risk may exceed 20% in high-risk groups.20
The interaction with age was also In the Warfarin-Aspirin Symptomatic
seen in a meta-analysis of three carotid Intracranial Disease Trial (WASID), sub-
intervention trials: EVA-3S, SPACE, and jects with stroke or TIA attributed to
328 www.ContinuumJournal.com April 2014

Copyright © American Academy of Neurology. Unauthorized reproduction of this article is prohibited.


KEY POINTS
stenosis of an intracranial artery (intra- no improvement in blood pressure h Antiplatelet therapy,
cranial carotid, middle cerebral, intra- control. They did show improvements rather than
cranial vertebral, or basilar) were in the proportion of patients with total anticoagulation, is
randomized to receive warfarin (titrated cholesterol less than 200 mg/dL (54.6% preferred for intracranial
to an international normalized ratio [INR] to 79.2%, PG.001) and low-density lipo- atherosclerosis.
of 2 to 3) versus aspirin 1300 mg/d.21 protein (LDL) cholesterol less than h Control of hyperlipidemia
The arterial stenosis was verified at 50% 100 mg/dL (28.7% to 55.9%, PG.001). and blood pressure is
to 99% by cerebral angiogram. After a In a multivariate analysis, systolic blood critical in patients with
mean follow-up of 1.8 years, the primary pressure greater than 140 mm Hg, no intracranial atherosclerosis.
end point (ischemic stroke, brain hem- alcohol consumption, and total choles-
orrhage, or vascular death other than terol greater than 200 mg/dL increased
from stroke) occurred in 22.1% in the the risk of recurrent stroke, MI, or vas-
aspirin group and 21.8% in the warfarin cular death.22 This study laid the foun-
group (P=.83). While not providing any dation of intensive medical management
efficacy benefit, the warfarin group had a in contemporary studies of intracranial
significantly higher rate of death, major atherosclerosis.
hemorrhage, and MIs,21 and the study The SAMMPRIS trial investigated the
was stopped early for safety concerns. safety and efficacy of percutaneous
Thus, in patients with symptomatic transluminal angioplasty and stenting
intracranial atherosclerotic disease, (PTAS, self-expanding Wingspan stent)
antiplatelet drugs are the mainstay of with aggressive medical management
secondary prevention.9 versus aggressive medical management
The WASID investigators described only, in patients with recent TIA or
the importance of risk factor control in stroke within 30 days, attributed to 70%
prevention of recurrent ischemic stroke to 99% stenosis of a major intracranial
in patients with intracranial atheroscle- artery.23 The various components of ag-
rotic disease.22 Over the 2-year follow-up gressive medical management are out-
period, the patients in WASID showed lined in Table 4-2. The primary end

TABLE 4-2 Intensive Medical Therapy for Patients With Intracranial


Atherosclerosis, Used in the Stenting and Aggressive
Medical Management for Preventing Recurrent Stroke in
Intracranial Stenosis (SAMMPRIS) Trial

b Aspirin 325 mg daily


b Clopidogrel 75 mg daily for the first 90 days after enrollment
b Management of primary risk factors
Systolic blood pressure: target G140 mm Hg (G130 mm Hg if diabetic)
Low-density lipoprotein cholesterol: target G70 mg/dL.
b Management of secondary risk factors
Diabetes mellitus: target hemoglobin A1c G7%
NonYhigh-density lipoprotein cholesterol: target G100 mg/dL
Smoking cessation
Excess weight: body mass index (BMI) G25 kg/m2 if the enrollment BMI is
25 kg/m2Y27 kg/m2 or 10% weight loss if the enrollment BMI is 927 kg/m2
Insufficient exercise: moderate intensity exercise at least 3 times/wk for
30 min/session

Continuum (Minneap Minn) 2014;20(2):323–334 www.ContinuumJournal.com 329


Copyright © American Academy of Neurology. Unauthorized reproduction of this article is prohibited.
Large Artery Atherosclerosis

KEY POINTS
h In the Stenting and point was stroke or death within 30 days try (4.5%).25 The medical arm, on the
Aggressive Medical or following a revascularization proce- other hand, fared much better than
Management for dure of the target lesion during follow-up; historically expected from event rates
Preventing Recurrent or ischemic stroke during the long-term in WASID at 1 month (5.8% observed
Stroke in Intracranial follow-up. The trial was stopped early for rate in SAMMPRIS versus 10.7%
Stenosis (SAMMPRIS) reasons of safety and futility.23 expected based on WASID) and at 1 year
trial, medical therapy SAMMPRIS stopped enrolling after (12.2% observed versus 25% expected).23
was superior to 451 patients were randomized, when Differences in the intensity of medical
intracranial stenting for the primary end point at 30 days management, including dual antiplatelet
stroke prevention. (stroke or death) occurred in 14.7% in therapy for the first 3 months and
h Procedural strokes the PTAS group and 5.8% in the aggressive blood pressure and LDL cho-
following intracranial medical management group (P=.002) lesterol reduction were probably respon-
stenting can be due to (Case 4-3).23 A third of the strokes in sible for the improvement in event rates.
diverse mechanisms. the PTAS arm were brain hemorrhages. A subsequent detailed analysis of the
All strokes in the PTAS arm occurred 30-day events in the PTAS arm revealed
within the first week after the proce- that a large number of the ischemic
dure; most occurred within the first 24 strokes occurred from occlusion of
hours. Periprocedural ischemic strokes perforators (more commonly the basilar
were associated with older age, diabetes perforators to the pons, or the
mellitus, basilar stenosis, and non- lenticulostriate perforators from the
smoking.24 Five (2.2%) stroke-related middle cerebral artery). Stenting can
deaths occurred in the PTAS arm and result in a ‘‘snowplowing’’ effect in
one (0.4%) nonYstroke-related death which atheromatous debris is pushed
occurred with medical management. into the perforators and occludes
The 1-year rate of the primary end them.26 A few wire perforations resulted
point was significantly higher in the in periprocedural subarachnoid hemor-
PTAS arm (20.0%) versus the medical rhages. A significant number of
arm (12.2%; P=.009).23 The differences intraparenchymal hemorrhages were
in 1-year event rates were primarily also noted. They were associated with
driven by the increased 30-day events a higher degree of stenosis, a high
in the PTAS arm. In SAMMPRIS, the activated clotting time (a laboratory
event rates in the PTAS arm (14.7%) test that measures intraprocedural
were significantly higher than antici- anticoagulation) maintained during the
pated from the Wingspan stent regis- procedure, along with a loading dose of

Case 4-3
A 64-year-old woman presented with multiple episodes of dizziness,
diplopia, and transient visual loss affecting both eyes over a 1-week
period. Each episode lasted 5 minutes. She had a history of uncontrolled
hypertension, dyslipidemia, diabetes mellitus, and smoking. CTA showed
severe stenosis in the proximal third of the basilar artery. She was placed
on aspirin, clopidogrel, and 40 mg rosuvastatin, and her blood pressure
medication was intensified. After 3 months of dual antiplatelet therapy,
clopidogrel was discontinued, and she continued on aspirin.
Comment. This patient should be placed on a SAMMPRIS-style regimen
(Table 4-2). At present, there is no proven role for endovascular
intervention in this type of patient. The importance of medication
compliance and lifestyle modifications should be reinforced.

330 www.ContinuumJournal.com April 2014

Copyright © American Academy of Neurology. Unauthorized reproduction of this article is prohibited.


clopidogrel 600 mg.24 Increased perfu- perfusion to metabolism mismatch in
sion in the territory supplied by a the ischemic areas of the brain.28 The
previously stenosed vessel (hyper- ischemic preconditioning concept orig-
perfusion syndrome) is also a proposed inated from the study of cardiac dis-
mechanism.24 ease, in which brief episodes of
Balloon angioplasty alone, without myocardial ischemia were associated
stenting, has been proposed as a with reduced MI size, suggesting that
method to reduce perforator strokes. brief, reversible ischemia could trigger
No randomized trials have compared vasculoprotective factors such as im-
angioplasty against intensive medical proved collateral flow. Further studies
management. In a retrospective case are needed to assess the preconditioning
series of 74 patients, technical success paradigm for the brain.
(less than 50% residual stenosis) was
achieved in 92%, and periprocedural EXTRACRANIAL VERTEBRAL
success (no event within 72 hours) was ARTERY DISEASE
achieved in 88%. The 30-day stroke and Vertebral artery atherosclerosis, com-
death rate was 5%.27 While this compli- monly affecting the V1 segment (origin
cation rate is lower than seen with of the artery from the subclavian artery),
SAMMPRIS, there is a concern about is found in about 20% of ischemic
the long-term durability of angioplasty strokes in the vertebrobasilar territory,
and whether long-term restenosis rates but is often associated with other causes
will be favorable. of ischemic stroke.29 Many of these
The high rate of recurrent events in patients often have stenosis of bilat-
patients with intracranial atherosclerotic eral vertebral artery origins and, less
disease has led to a search for novel commonly, coexist with subclavian
approaches to reduce stroke risk. In a artery stenosis, making it difficult to
proof-of-concept study, 68 patients with identify which lesion is truly symp-
symptomatic intracranial atherosclerotic tomatic.29 Patients with symptomatic
disease were randomized to either vertebrobasilar atherosclerosis of 50%
standard medical management or med- or more have a higher rate of recurrent
ical management with brief, repetitive, events compared with patients with
bilateral arm ischemic preconditioning symptomatic carotid stenosis.30
within 30 days of symptoms.28 Ischemic Various operative approaches are
preconditioning was applied using a described for vertebral origin stenosis,
blood pressure tourniquet. The pres- including transposition of the vertebral
sure was raised to 200 mm Hg and artery to the common carotid artery.
maintained for 5 minutes followed by However, these methods are not sys-
5 minutes of reperfusion. Five such tematically studied and are not com-
cycles were performed in each arm, monly performed. In a large series of
and this treatment was continued 369 extracranial vertebral artery recon-
daily for 300 days. A marked reduction structions from the 1990s, perioperative
was seen in the incidence of recurrent morbidity and mortality were low. Long-
ischemic stroke in the ischemic term patency of the vessel was 80%,
preconditioning group compared with and stroke-free survival was 97%.31 In a
standard medical treatment at 90 days literature review of 300 endovascular
(5% versus 23.3%, PG.01) and 300 days interventions in symptomatic vertebral
(7.9% versus 26.7%, PG.01).28 Upon artery origin stenosis, periprocedural
single-photon emission CT imaging, neurologic complications occurred in
ischemic preconditioning reduced the 5.5% and the restenosis rate was 26%.32
Continuum (Minneap Minn) 2014;20(2):323–334 www.ContinuumJournal.com 331
Copyright © American Academy of Neurology. Unauthorized reproduction of this article is prohibited.
Large Artery Atherosclerosis

KEY POINTS
h No high-quality studies Nevertheless, at long-term follow-up stenosis. It is generally recommended
have been done to (mean 14.2 months), the risk of death that medical therapy should follow the
evaluate treatment was 0.3%, and the risk for posterior principles and targets of the medical
methods for vertebral stroke was 0.7%. The risks of adverse treatment of carotid atherosclerosis.34
artery origin stenosis. events are generally higher with distal Based on expert consensus, patients
h Aggressive medical vertebral or basilar interventions and with acute vertebrobasilar ischemic syn-
therapy is preferred as when interventions are performed for dromes who have angiographic evidence
initial treatment for urgent revascularization.32 of a thrombus in the extracranial verte-
vertebral artery origin In the Carotid and Vertebral Artery bral artery may benefit from 3 months of
stenosis. Transluminal Angioplasty Study anticoagulation.34 Endovascular interven-
(CAVATAS), 16 patients with symptom- tions may be considered on an individual
atic vertebral artery stenosis were ran- basis, such as in the patient with
domized to receive best medical recurrent vertebrobasilar symptoms in
therapy versus angioplasty or stenting.33 spite of maximal medical therapy.
Over a mean follow-up period of 4.7
years, no vertebrobasilar territory CONCLUSION
strokes occurred in either arm of the Large artery atherosclerosis is an im-
study. In both arms of the study, three portant, medically treatable, cause of
patients died of either MIs or carotid ischemic stroke. Strict control of ath-
territory strokes. Thus, in this small erosclerotic risk factors is essential.
group of patients, optimal medical Surgical and endovascular options ben-
treatment had equivalent outcomes to efit patients with symptomatic, moder-
endovascular stenting.33 ate to severe carotid stenosis, although
In the Oxford Vascular Study the benefit is not as robust in women.
(OXVASC), a population-based study of Endovascular or surgical interventions
141 patients with vertebrobasilar strokes for asymptomatic cervical carotid
or TIAs, 37 (26.2%) had 50% or greater stenosis must be held to stringent
stenosis in the extracranial vertebral safety standards, given the low stroke
(n = 23) or intracranial vertebral/basilar rates achieved by contemporary in-
arteries (n = 14).30 Medical therapy was tensive medical therapy. Aggressive
determined by the patient’s general medical therapy is also beneficial for
practitioners. The 90-day risk of recur- symptomatic intracranial atherosclerosis
rent vertebrobasilar events (strokes or and will serve as a benchmark for any
TIAs) was high (46%) in the patients future comparisons of endovascular
with 50% or greater stenosis compared treatment.
with patients who did not have 50% or
greater stenosis (21%). The incidence
of recurrent vertebrobasilar strokes in REFERENCES
1. Fine-Edelstein JS, Wolf PA, O’Leary DH, et al.
the OXVASC population (excluding Precursors of extracranial carotid atherosclerosis
TIAs) was 22% within 90 days.30 This is in the Framingham Study. Neurology 1994;
higher than the recurrence rates of 44(6):1046Y1050.
events in patients with carotid stenosis, 2. Brott TG, Hobson RW 2nd, Howard G, et al.
although the medical therapy was not Stenting versus endarterectomy for treatment
of carotid-artery stenosis. N Engl J Med 2010;
standardized in this study. 363(1):11Y23.
In the absence of clinical trial evi-
3. Chaturvedi S, Wechsler LR. Carotid
dence of the superiority of revasculariza- revascularization strategies: the need for
tion methods, optimal medical therapy more data. Stroke 2012;43(4):929Y930.
should be the default treatment modal- 4. Blackshear JL, Cutlip DE, Roubin GS, et al.
ity for symptomatic vertebral artery Myocardial infarction after carotid stenting

332 www.ContinuumJournal.com April 2014

Copyright © American Academy of Neurology. Unauthorized reproduction of this article is prohibited.


and endarterectomy: results from the carotid death from carotid endarterectomy:
revascularization endarterectomy versus systematic review. BMJ 1997;315(7122):
stenting trial. Circulation 2011;123(22): 1571Y1577.
2571Y2578.
16. Howard VJ, Lutsep HL, Mackey A, et al.
5. Barnett HJ, Taylor DW, Eliasziw M, et al. Influence of sex on outcomes of stenting
Benefit of carotid endarterectomy in patients versus endarterectomy: a subgroup analysis
with symptomatic moderate or severe stenosis. of the Carotid Revascularization
North American Symptomatic Carotid Endarterectomy versus Stenting Trial (CREST).
Endarterectomy Trial Collaborators. N Engl J Lancet Neurol 2011;10(6):530Y537.
Med 1998;339(20):1415Y1425.
17. Voeks JH, Howard G, Roubin GS, et al. Age
6. North American Symptomatic Carotid and outcomes after carotid stenting and
Endarterectomy Trial Collaborators. Beneficial endarterectomy: the carotid revascularization
effect of carotid endarterectomy in endarterectomy versus stenting trial. Stroke
symptomatic patients with high-grade carotid 2011;42(12):3484Y3490.
stenosis. N Engl J Med 1991;325(7):445Y453.
18. Chaturvedi S, Bruno A, Feasby T, et al.
7. Rajamani K, Chaturvedi S. Surgery insight: Carotid endarterectomyVan evidence-based
carotid endarterectomyVwhich patients to review: report of the Therapeutics and
treat and when? Nat Clin Pract Cardiovasc Technology Assessment Subcommittee of
Med 2007;4(11):621Y629. the American Academy of Neurology.
8. Rothwell PM, Eliasziw M, Gutnikov SA, et al. Neurology 2005;65(6):794Y801.
Endarterectomy for symptomatic carotid
19. White H, Boden-Albala B, Wang C, et al.
stenosis in relation to clinical subgroups Ischemic stroke subtype incidence among
and timing of surgery. Lancet 2004;363 whites, blacks, and Hispanics: the Northern
(9413):915Y924. Manhattan Study. Circulation 2005;
9. Furie KL, Kasner SE, Adams RJ, et al. Guidelines 111(10):1327Y1331.
for the prevention of stroke in patients with
20. Qureshi AI, Feldmann E, Gomez CR, et al.
stroke or transient ischemic attack: a guideline
Intracranial atherosclerotic disease: an
for healthcare professionals from the
update. Ann Neurol 2009;66(6):730Y738.
American Heart Association/American Stroke
Association. Stroke 2011;42(1):227Y276. 21. Chimowitz MI, Lynn MJ, Howlett-Smith H,
et al. Comparison of warfarin and aspirin
10. Silver FL, Mackey A, Clark WM, et al. Safety of
for symptomatic intracranial arterial stenosis.
stenting and endarterectomy by symptomatic
N Engl J Med 2005;352(13):1305Y1316.
status in the Carotid Revascularization
Endarterectomy Versus Stenting Trial (CREST). 22. Chaturvedi S, Turan TN, Lynn MJ, et al. Risk
Stroke 2011;42(3):675Y680. factor status and vascular events in patients
with symptomatic intracranial stenosis.
11. Carotid Stenting Trialists’ Collaboration,
Neurology 2007;69(22):2063Y2068.
Bonati LH, Dobson J, Algra A, et al.
Short-term outcome after stenting versus 23. Chimowitz MI, Lynn MJ, Derdeyn CP, et al.
endarterectomy for symptomatic carotid Stenting versus aggressive medical therapy
stenosis: a preplanned meta-analysis of for intracranial arterial stenosis. N Engl J
individual patient data. Lancet 2010;376 Med 2011;365(11):993Y1003.
(9746):1062Y1073.
24. Fiorella D, Derdeyn CP, Lynn MJ, et al.
12. Halliday A, Harrison M, Hayter E, et al. Detailed analysis of periprocedural strokes
10-year stroke prevention after successful in patients undergoing intracranial stenting
carotid endarterectomy for asymptomatic in Stenting and Aggressive Medical
stenosis (ACST-1): a multicentre randomised Management for Preventing Recurrent
trial. Lancet 2010;376(9746):1074Y1084. Stroke in Intracranial Stenosis (SAMMPRIS).
Stroke 2012;43(10):2682Y2688.
13. Abbott AL. Medical (nonsurgical) intervention
alone is now best for prevention of stroke 25. Bose A, Hartmann M, Henkes H, et al. A novel,
associated with asymptomatic severe carotid self-expanding, nitinol stent in medically
stenosis: results of a systematic review and refractory intracranial atherosclerotic stenoses: the
analysis. Stroke 2009;40(10):e573Ye583. Wingspan study. Stroke 2007;38(5):1531Y1537.
14. Spence JD, Coates V, Li H, et al. Effects of 26. Derdeyn CP, Fiorella D, Lynn MJ, et al.
intensive medical therapy on microemboli and Mechanisms of stroke after intracranial
cardiovascular risk in asymptomatic carotid angioplasty and stenting in the SAMMPRIS
stenosis. Arch Neurol 2010;67(2):180Y186. trial. Neurosurgery 2013;72(5):777Y795.
15. Rothwell PM, Slattery J, Warlow CP. Clinical 27. Nguyen TN, Zaidat OO, Gupta R, et al. Balloon
and angiographic predictors of stroke and angioplasty for intracranial atherosclerotic

Continuum (Minneap Minn) 2014;20(2):323–334 www.ContinuumJournal.com 333


Copyright © American Academy of Neurology. Unauthorized reproduction of this article is prohibited.
Large Artery Atherosclerosis

disease: periprocedural risks and short-term symptomatic vertebral artery stenosis


outcomes in a multicenter study. Stroke compared with medical treatment in the
2011;42(1):107Y111. Carotid And Vertebral Artery Transluminal
Angioplasty Study (CAVATAS): a randomized
28. Meng R, Asmaro K, Meng L, et al. Upper limb trial. Stroke 2007;38(5):1526Y1530.
ischemic preconditioning prevents recurrent
stroke in intracranial arterial stenosis. Neurology 34. Brott TG, Halperin JL, Abbara SG, et al.
2012;79(18):1853Y1861. 2011 ASA/ACCF/AHA/AANN/AANS/ACR/
ASNR/CNS/SAIP/SCAI/SIR/SNIS/SVM/SVS
29. Caplan LR, Wityk RJ, Glass TA, et al. New guideline on the management of patients
England Medical Center Posterior Circulation
with extracranial carotid and vertebral
registry. Ann Neurol 2004;56(3):389Y398. artery disease: executive summary: a report
30. Marquardt L, Kuker W, Chandratheva A, et al. of the American College of Cardiology
Incidence and prognosis of 9 or = 50% Foundation/American Heart Association Task
symptomatic vertebral or basilar artery stenosis: Force on Practice Guidelines, and the American
prospective population-based study. Brain Stroke Association, American Association of
2009;132(pt 4):982Y988. Neuroscience Nurses, American Association
of Neurological Surgeons, American College
31. Berguer R, Flynn LM, Kline RA, Caplan L. of Radiology, American Society of
Surgical reconstruction of the extracranial Neuroradiology, Congress of Neurological
vertebral artery: management and outcome. Surgeons, Society of Atherosclerosis Imaging
J Vasc Surg 2000;31(1 pt 1):9Y18. and Prevention, Society for Cardiovascular
32. Eberhardt O, Naegele T, Raygrotzki S, et al. Angiography and Interventions, Society
of Interventional Radiology, Society of
Stenting of vertebrobasilar arteries in
symptomatic atherosclerotic disease and NeuroInterventional Surgery, Society for
acute occlusion: case series and review of Vascular Medicine, and Society for Vascular
Surgery. Developed in collaboration with
the literature. J Vasc Surg 2006;43(6):
1145Y1154. the American Academy of Neurology and
Society of Cardiovascular Computed
33. Coward LJ, McCabe DJ, Ederle J, et al. Long-term Tomography. Catheter Cardiovasc Interv
outcome after angioplasty and stenting for 2013;81(1):E76YE123.

334 www.ContinuumJournal.com April 2014

Copyright © American Academy of Neurology. Unauthorized reproduction of this article is prohibited.


Review Article

Evaluation and
Address correspondence to
Dr Cheryl Bushnell, Wake
Forest School of Medicine,
Department of Neurology,

Management of Cerebral Medical Center Boulevard,


Winston Salem, NC 27157,
cbushnel@wakehealth.edu.

Venous Thrombosis Relationship Disclosure:


Dr Bushnell has received
grants from the Hazel K.
Goddess Fund for Stroke
Cheryl Bushnell, MD, MHS; Gustavo Saposnik, MD, MSc, FAHA, FRCP(C) Research in Women, the
North Carolina Stroke Care
Collaborative, the World
Federation of Neurology, and
ABSTRACT the World Stroke Organization.
Purpose of Review: Thrombosis of the dural sinus and/or cerebral veins (CVT) is a Dr Saposnik is supported by the
rare but potentially devastating type of stroke that tends to occur in young adults, Distinguished Clinician Scientist
Award from the Heart and
especially women. In this article, the abbreviation CVT refers to either cerebral Stroke Foundation of Canada.
venous thrombosis or dural sinus thrombosis. The purpose of this review is to Unlabeled Use of
review the most up-to-date literature on the epidemiology, diagnosis, manage- Products/Investigational
Use Disclosure:
ment, and prognosis of CVT. In addition, illustrative cases that represent the Drs Bushnell and Saposnik
spectrum of CVT are provided. report no disclosures.
Recent Findings: CVT represents about 0.5% of all strokes and can be challenging * 2014, American Academy
to diagnose because headache, rather than focal neurologic symptoms, is the of Neurology.
prominent feature. The diagnosis is confirmed with MRI and magnetic resonance
venogram (MRV). The mainstay of acute management is anticoagulation, although,
in the cases of severe hemorrhagic conversion of a venous infarction, endovascular
mechanical thrombectomy may be potentially lifesaving. The evaluation of
underlying causes from transient triggers, eg, pregnancy, oral contraceptives, or
infection, versus chronic triggers, eg, cancer and thrombophilia, will often influence
the duration of anticoagulation. The outcomes after CVT are generally favorable,
and the risk of recurrence is low.
Summary: CVT is an important diagnosis to keep in mind when evaluating patients
with headache in the emergency department, and it is important that it not be
overlooked because it is treatable.

Continuum (Minneap Minn) 2014;20(2):335–351.

EPIDEMIOLOGY AND of those with CVT, likely a result of the


PATHOGENESIS sex-specific risk factors that occur in
Thrombosis of the dural sinus and/or women, including oral contraceptives,
cerebral veins (CVT) is relatively rare, pregnancy, and hormone therapy use.
accounting for only about 0.5% of CVT causes dysfunction by two
patients with stroke and with a preva- mechanisms. (1) Thrombosis of cere-
lence of only 5 per 1 million.1 CVT is bral veins causes localized edema of
also primarily a stroke type that affects the brain and venous infarction. (2)
individuals younger than the age of 50. Thrombosis of the major sinuses leads
In the 624 cases included in the to intracranial hypertension as a result
International Study of Cerebral Venous of increased venous pressure and
Thrombosis (ISCVT), the median age impaired absorption of CSF.4 Generally,
was 37 years.2 The incidence of CVT in some underlying trigger or pre-
children is 0.67 per 100,000 per year, disposing risk factor that increases the
with 43% occurring in neonates.3 In risk of thrombosis is the reason for
addition, women make up about 75% the timing and occurrence of CVT. In the
Continuum (Minneap Minn) 2014;20(2):335–351 www.ContinuumJournal.com 335
Copyright © American Academy of Neurology. Unauthorized reproduction of this article is prohibited.
Cerebral Venous Thrombosis

KEY POINTS
h Cerebral venous ISCVT, 85% of patients with CVT had at commonly present with deep cerebral
thrombosis is a least one identifiable risk factor for venous occlusion of the internal cere-
cerebrovascular disease thrombosis and 44% had multiple pre- bral vein or vein of Galen (11%), or
of the young, primarily disposing factors.2 These predisposing straight sinus (18%), but they are at a
of women. factors are discussed in detail below. nearly threefold higher risk of death
h Headache is the most and dependency during follow-up.2
common symptom of
CLINICAL PRESENTATION These patients may present with bilat-
cerebral venous The presentation of patients with CVT eral thalamic or basal ganglia infarction,
thrombosis. may be variable. It can be categorized and will often have decreased level of
h The most common into four clinical syndromes: (1) iso- consciousness and rapid neurologic
clinical syndromes in lated intracranial hypertension (eg, deterioration.2 Some symptoms may
cerebral venous patients may present with headache, guide the localization or extension of
thrombosis include (1) diplopia, visual impairment [severe the CVT. For example, patients with
intracranial hypertension, papilledema], decreased level of con- deep cerebral venous occlusion or
(2) focal neurologic sciousness, or sixth nerve palsy), (2) extensive CVT (involving the superior
deficits (eg, motor focal neurologic deficit (eg, motor sagittal plus other sinuses) may present
weakness, sensory deficit, weakness, sensory deficit, aphasia), with decreased level of consciousness
aphasia), (3) encephalopathy, and (4) seizures with bilateral papilledema.7,8 The de-
(3) encephalopathy, and
(accompanied or not by a focal neu- velopment of headaches and fever in
(4) seizure disorder.
rologic deficit).1,4 the context of an underlying otitis or
Headache is the most common symp- sinusitis may raise suspicion about CVT
tom, affecting approximately 90% of pa- involving the transverse or sigmoid
tients with CVT. Isolated headaches are sinuses.7,9 Delays in diagnosis are com-
less common, but may constitute a clinical mon, likely related to an insidious
challenge (Table 5-1).2,5 The presenta- course in up to two-thirds of patients.2
tion of headache associated with focal
symptoms or seizures may suggest the DIAGNOSIS
presence of a venous infarction.6 Hem- The diagnosis of CVT requires a high
orrhagic conversion was reported in 35% level of clinical suspicion and confir-
to 39% of patients.2 Demographic char- mation by neuroimaging. The differ-
acteristics associated with hemorrhagic ential diagnosis of CVT is summarized
conversion include older age, female sex, in Table 5-2. D-Dimer, a product of
and an acute onset (48 hours).6 fibrin degradation, has been studied in
The most commonly affected venous the context of CVT. Most patients with
sinus is the superior sagittal (62%) an acute and extensive CVT have a
D-dimer level greater than 500 6g/L. A
7
followed by the transverse sinus (41%
to 45%; Figure 5-17).2 Patients less recent meta-analysis of 14 studies

TABLE 5-1 Clinical Manifestations of Cerebral Venous Thrombosis

Clinical Presentation Frequency2


Headaches 90%
Seizures 40%
Focal deficit (eg, hemiparesis, aphasia) 20%
Decreased level of consciousness 14%
Isolated headaches5 15%
Visual loss 13%

336 www.ContinuumJournal.com April 2014

Copyright © American Academy of Neurology. Unauthorized reproduction of this article is prohibited.


FIGURE 5-1 Cerebral venous thrombosis: most commonly affected sinuses. The sagittal
sinus is the most commonly affected, followed by the transverse sinuses.
Modified from Saposnik G, et al, Stroke.7 B 2011 American Heart Association, Inc.
stroke.ahajournals.org/content/42/4/1158.long.

reported similar findings, with a strong clinical suspicion of CVT should


weighted mean sensitivity of 93.9% not prohibit further evaluation if the
and specificity of 89.7%.10 However, a D-dimer level is normal.
low D-dimer level does not rule out Recent advances in brain imaging have
CVT in patients with high suspicion or played an important role in improving
in the subacute/chronic phase of the the diagnosis of CVT. CT brain, CT
disease, since the D-dimer levels de- venogram, MRI brain, magnetic reso-
cline with time from onset of symp- nance venogram (MRV), and conven-
toms.7,10,11 In addition, those with a tional angiogram are the common
lesser clot burden may have false investigations used in the diagnosis of
negative D-dimer levels. Therefore, a CVT.12 The American Heart Association

TABLE 5-2 Differential Diagnosis of Cerebral Venous Thrombosis

b Idiopathic intracranial hypertension (pseudotumor cerebri)


b Arterial ischemic stroke
b Primary intracerebral hemorrhage
b Hemorrhagic stroke due to a vascular malformation
b Meningitis/encephalitis
b Brain abscess
b Systemic lupus erythematosus (by far the most common differential
diagnosis, affecting mostly women as in cerebral venous thrombosis [CVT])
b Sarcoidosis (may have a presentation similar to CVT)
b Antiphospholipid syndrome (may have a presentation similar to CVT)
b Gliomatosis cerebri

Continuum (Minneap Minn) 2014;20(2):335–351 www.ContinuumJournal.com 337


Copyright © American Academy of Neurology. Unauthorized reproduction of this article is prohibited.
Cerebral Venous Thrombosis

(AHA) and the European Federation of available.7,13 One of the advantages of


Neurological Societies (EFNS) guidelines MR is the use of novel sequences, eg,
recommend MRI/MRV as the preferred T2* susceptibility-weighted imaging, that
brain image, whereas CT/CT venogram improve the detection of isolated corti-
is an acceptable option when MRI is not cal venous thrombosis identified as a
hypointense area.14,15

CT
Spontaneous density (dense cord sign)
represents the thrombosed cortical vein
or sinus that can be seen in noncontrast
CT (Figure 5-2A).14,16 The presence of
bilateral intracerebral hemorrhages, hem-
orrhagic infarction, or an infarction out-
side the boundaries for an arterial
territory are useful signs to suspect
cerebral venous sinus thrombosis.17 With
contrasted CT, the delta sign represents
the thrombosis of the posterior portion
of the superior sagittal sinus or transverse
sinus (Figure 5-2B).14 CT venogram may
reveal the thrombosis of major sinuses
up to the jugular vein (Figure 5-2B).
Three-dimensional reconstitution of the
CT venogram may facilitate visualization
of the thrombosed sinus (Figure 5-2C).
The advantage of CT/CT venogram
includes its wide availability and
prompt assessment of major sinuses.
Radiation, exposure to the risk of IV
contrast, and poor visualization of
deep and cortical venous thrombosis
are known limitations.7

MRI
The most common findings using MRI
include visualization of the thrombus in
T1-weighted images or loss of signal in
the venous system on MRV (Case 5-1,
Figure 5-3A). Changes of blood prod-
ucts using MRI constitute a limitation for
FIGURE 5-2 A young woman presenting with headache in visualization of CVT in the acute phase.
the emergency department. A, Plain CT
shows increased attenuation involving the For example, in the first 5 days after
superior sagittal sinus, straight sinus, the confluence, and CVT, the thrombus is isointense on T1-
right transverse sinus (yellow arrows). B, CT venogram
reveals absence of contrast in the superior sagittal sinus, weighted images and hypointense on
straight sinus, right transverse sinus (green arrow), right T2-weighted images because of in-
sigmoid sinus, and right internal jugular vein (yellow arrow). creased deoxyhemoglobin.7,14 In the
C, Three-dimensional reconstruction of the CT venogram shows
thrombosis of the superior sagittal sinus (yellow arrows). subacute stage (5 to 15 days), the
thrombus becomes hyperintense on
338 www.ContinuumJournal.com April 2014

Copyright © American Academy of Neurology. Unauthorized reproduction of this article is prohibited.


KEY POINTS
T1-weighted and T2-weighted images. Transient risks and triggers are fre- h MRI/magnetic
After 15 days, the thrombus will become quently considered to be related to a resonance venography
homogeneous and hypointense in all temporary condition, such as pregnancy/ is the recommended
image sequences.7,14 Contrast-enhanced puerperium, CNS or ear/sinus/mouth/ imaging for the
MRV is more sensitive than time-of-flight face infections, exposure to drugs (eg, diagnosis of cerebral
MRV in demonstrating the thrombus oral contraceptives, steroids, oncology venous thrombosis.
within small veins.7,18 treatments), head trauma, or procedures Specific sequences (eg, T2*
Conventional angiography is less (eg, lumbar puncture, jugular catheter susceptibility-weighted
commonly used to assess for CVT placement; Table 5-3). Chronic triggers imaging) are useful to
and is typically reserved for selected include hereditary or acquired thrombo- assist in the diagnosis of
isolated cortical venous
situations when the diagnosis is un- philias (disorders that increase the likeli-
thrombosis.
certain or other brain imaging tech- hood of blood clotting) that are
niques are unavailable.7,13,14 established causes of venous thromboem- h Transient risk factors for
bolism. Specific types include deficiencies cerebral venous
RISK FACTORS AND TRANSIENT thrombosis include
of protein S, protein C, or antithrombin
VERSUS CHRONIC TRIGGERS pregnancy/puerperium,
III, as well as antiphospholipid antibody
CNS or ear/sinus/mouth/
Once a CVT has been diagnosed, it is syndrome, dysfibrinogenemia, increased face infections, exposure
important to search for the potential factor VIII levels, factor V Leiden mutation to drugs (eg, oral
causes and predisposing factors. Female (causing resistance to protein C), and contraceptives, steroids,
sex is one of the major risk factors for prothrombin 20210A mutation. Each cancer treatments), head
CVT, as cases in women outnumber cases thrombophilia has been associated with trauma, or procedures (eg,
in men 3 to 1. In sex-specific analysis of CVT, with varying annual risk of recur- lumbar puncture, jugular
those enrolled in the ISCVT, women were rence of venous thromboembolism catheter placement).
younger, had a shorter time from onset of (Table 5-420).7 In one study of 145 h Chronic triggers of
symptoms to admission, and more often patients with CVT, the prothrombin cerebral venous
had headache at presentation.19 Sixty-five mutation was found in 19%,21 although thrombosis include
percent of identifiable causes of CVT in a separate study reported antiphos- hereditary or acquired
women were related to a risk factor pholipid antibodies as the most preva- thrombophilias that are
unique to women, including oral contra- lent thrombophilia in that group of established causes
ceptives, pregnancy (Case 5-2), the patients (5.9%).2 It is important to note of venous
thromboembolism.
puerperium, and hormone therapy.19 that multiple predisposing factors are

Case 5-1
A 26-year-old right-handed woman with a history of ulcerative colitis and anemia presented to the
emergency department with a new onset of headaches, somnolence, and numbness in the fingertips of
both hands. She described progressive worsening of sharp headaches within 72 hours accompanied by
nausea and vomiting. Current medications included oral contraceptives that were started 2 years ago. On
presentation, her blood pressure was 154/84 mm Hg, and her heart rate was 62 beats per minute. She was
afebrile. On neurologic examination, she was somnolent, opening her eyes to verbal stimuli and oriented to
place and time. Funduscopic examination revealed bilateral papilledema. Cranial nerve examination was
otherwise normal, and the remainder of her neurologic examination was unremarkable. Her D-dimer level
was greater than 3000 6g/L. A plain CT head scan demonstrated increased attenuation of the superior
sagittal and transverse sinuses (Figure 5-2A). CT venography revealed occlusion of the superior sagittal sinus,
straight sinus, the confluence of sinuses, and right transverse sinus extending to the right jugular vein
(Figure 5-2B and C). Brain MRI and brain magnetic resonance venogram confirmed the observed CT
findings with no evidence of venous infarction (Figure 5-3A and B).

Continued on page 340

Continuum (Minneap Minn) 2014;20(2):335–351 www.ContinuumJournal.com 339


Copyright © American Academy of Neurology. Unauthorized reproduction of this article is prohibited.
Cerebral Venous Thrombosis

Continued from page 339

FIGURE 5-3 A young woman with headache. A, Magnetic resonance venogram (MRV)
showing an extensive thrombosis throughout the posterior two-thirds of the
superior sagittal sinus, involving the torcular herophili, straight sinus, the right
transverse and sigmoid sinuses, and extending inferiorly into the right internal jugular vein
(arrows). The left transverse and sigmoid sinuses and left internal jugular veins are spared. B, MRV
(sagittal view) showing the thrombosis of two-thirds of the superior sagittal sinus, involving the
torcular herophili and the straight sinus (arrow). C, MRV at 1 year showing complete recanalization
of the previously thrombosed sinuses.

Comment. This case illustrates the common clinical presentation with signs and symptoms suggestive
of intracranial hypertension, elevated D-dimer level, and imaging finding consistent with CVT
involving multiple sinuses extending to the right jugular vein.

340 www.ContinuumJournal.com April 2014

Copyright © American Academy of Neurology. Unauthorized reproduction of this article is prohibited.


Case 5-2
A 27-year-old woman, 19 weeks pregnant with a history of migraine headaches, developed severe
occipital headache that increased in severity over the course of 5 days, unlike her usual migraine. Her
neurologic examination was normal. Magnetic resonance venogram (MRV) showed acute thrombosis
of the transverse and sigmoid sinuses on the left, and MRI showed adjacent left mastoiditis (Figure 5-4A and B).
She was treated with antibiotics and unfractionated heparin subcutaneously until delivery, and then
switched to adjusted-dose warfarin daily after delivery with a targeted international normalized ratio of 2
to 3. The repeat MRI/MRV on follow-up 1 year later revealed complete recanalization of the left transverse
and sigmoid sinuses, and the mastoiditis had resolved (Figure 5-4C). Warfarin was discontinued.

FIGURE 5-4 A young pregnant woman with mastoiditis. A, Magnetic resonance venogram showing acute thrombosis of
the left transverse and sigmoid sinuses. B, T1-weighted MRI of the brain showing left mastoid effusion and
adjacent thrombosis that include the cortical veins of the tentorium. C, Resolution of the left transverse and
sigmoid thrombosis 20 months later.

Comment. This case illustrates the transient trigger of both pregnancy and mastoiditis in a young
woman who later had complete resolution of the thromboses as well as the mastoiditis.

often found in these patients. For for patients with CVT. Anticoagulation is KEY POINT
example, in the study of 145 patients used to prevent thrombus growth, to h Anticoagulation is the
with CVT, the combination of oral facilitate recanalization, and to prevent main treatment for the
contraceptives and thrombophilia was other thrombotic events (eg, deep acute management of
present in 37% of women.21 venous thrombosis and pulmonary em- cerebral venous
bolism). A recent meta-analysis of two thrombosis.
ACUTE MANAGEMENT trials22,23 including 79 patients revealed
The acute management of CVT involves that anticoagulation therapy was asso-
strategies aimed at the recanalization of the ciated with a lower relative risk of
thrombosed sinus or sinuses (Figure 5-5) death (0.33, 95% confidence interval
and the prevention of medical complica- [CI], 0.08 to 1.21) and of death or
tions. The treatment of the underlying dependency (0.46, 95% CI, 0.16 to
causes is discussed in another section. 1.31).24 A sensitivity analysis, in which
two of the excluded (one reported as an
Anticoagulation abstract, and another one not having
Despite the limited evidence from large vascular imaging) randomized studies
randomized clinical trials, initial anti- (n = 176) were added, revealed a
coagulation is the standard treatment significant reduction in the odds of death

Continuum (Minneap Minn) 2014;20(2):335–351 www.ContinuumJournal.com 341


Copyright © American Academy of Neurology. Unauthorized reproduction of this article is prohibited.
Cerebral Venous Thrombosis

TABLE 5-3 Common Underlying Causes of Cerebral Venous Thrombosis or Dural Sinus
Thrombosis Designated by Transient Versus Chronic/High-Risk Triggers

Condition Prevalence (%)a Trigger Type


Prothrombotic conditions 34.1 Chronicb
Pregnancy and puerperium 21.0 Transient
Oral contraceptives/hormone replacement 54.3 Transient
Drugs (eg, danazol, lithium, vitamin A, 7.5 Transient
IV immunoglobulin, ecstasy)
Cancer related 7.4 Transient or chronic
Local compression
Hypercoagulability
Antineoplastic drugs (eg, tamoxifen,
L-asparaginase)

Infection 12.3 Transient


Parameningeal (eg, ear, sinus, mouth,
face, and neck)
Sepsis
Mechanical precipitants 4.5 Transient
Complication of epidural blood patch
Spontaneous intracranial hypotension
Lumbar puncture
Dehydration No prevalence data but Transient
is a recognized trigger
Other hematologic disorders 12.0 Chronic
Paroxysmal nocturnal hemoglobinuria
Iron deficiency anemia
Thrombocytopenia
Nephrotic syndrome
Polycythemia
Systemic diseases 7.2 Chronic
Systemic lupus erythematosus 1.0
Behc$et disease 1.0
Inflammatory bowel disease 1.6
Thyroid disease 1.7
Sarcoidosis 0.2
Other 1.7
None identified 12.5 Unknown
a
Prevalence rates among patients with CVT from the International Study of Cerebral Venous Thrombosis.2
b
Excludes the acquired and potentially transient deficiencies associated with the acute phase of systemic conditions (eg, nephrotic syndrome).

KEY POINT (odds ratio [OR] 0.33; 95% CI, 0.14 to Data from the International Study on
h All available guidelines 0.78; PG.006) with anticoagulation Cerebral Vein and Dural Sinus Throm-
recommend therapy.24 Since intracerebral hemor- bosis suggest low-molecular-weight hep-
anticoagulation therapy rhage (ICH) may be a consequence of arin (LMWH) might be safer and perhaps
for the acute
venous hypertension caused by the more efficacious than unfractionated
management of
thrombosed vein or sinus, guidelines heparin. For example, among CVT par-
cerebral venous
thrombosis. recommend anticoagulation therapy ticipants in the study, 119 (28%) patients
even in the presence of ICH or hemor- received LMWH and 302 (72%) received
rhagic transformation (Table 5-5).7,13,25 unfractionated heparin.26 CVT patients

342 www.ContinuumJournal.com April 2014

Copyright © American Academy of Neurology. Unauthorized reproduction of this article is prohibited.


TABLE 5-4 Thrombophilias: Annual Risk and Recurrence Rate
of Venous Thromboembolism Among Families With
Hereditary Disordersa

Defect Grade Annual Risk Risk of Recurrence


Antithrombin deficiency
Protein C deficiency High 1.52%Y1.90% 40% (at 5 years)
Protein S deficiency 55% (at 10 years)
Factor V Leiden
Prothrombin 20210A Moderate 0.34%Y0.49% 11% (at 5 years)
mutation 25% (at 10 years)
Elevated factor VIII
a
Data from Lijfering WM, et al, Blood.20 bloodjournal.hematologylibrary.org/content/113/21/5314.long.

FIGURE 5-5 Algorithm for the management of cerebral venous thrombosis.


CVT = cerebral venous thrombosis; MRI = magnetic resonance imaging;
MRV = magnetic resonance venography; CTV = computed tomography
venography; ICH = intracerebral hemorrhage.
a
Anticoagulation remains the principal therapy and is aimed at preventing
thrombus propagation and increasing recanalization. This algorithm is not
comprehensive, nor is it applicable to all clinical scenarios; patient management
must be individualized.
b
Limited evidence is available on the benefits of decompressive hemicraniectomy
and endovascular therapy for the management of cerebral venous thrombosis.

Modified from Saposnik G, et al, Stroke.7 B 2011 American Heart Association, Inc.
stroke.ahajournals.org/content/42/4/1158.long.

Continuum (Minneap Minn) 2014;20(2):335–351 www.ContinuumJournal.com 343


Copyright © American Academy of Neurology. Unauthorized reproduction of this article is prohibited.
Cerebral Venous Thrombosis

TABLE 5-5 Comparison Among Guidelines for the Acute Management Options of Cerebral
Venous Thrombosis

American Heart European Federation American College


Association of Neurological of Chest Physicians
Guidelines Societies Guidelines Guidelines
Anticoagulation Therapy Class IIa Level B Grade 2C
(Despite Intracerebral
Hemorrhage)
Intrasinus Thrombolysis Class IIba ‘‘Good practice point’’ NR
Mechanical Thrombectomy Class IIba NR NR
a
Decompressive Class IIb ‘‘Good practice point’’ NR
Hemicraniectomy
NR = not rated.
a
To be considered in case of progressive neurologic deterioration despite intensive medical treatment.

KEY POINT receiving LMWH were more likely to be underway (Thrombolysis or Anti-
h Endovascular functionally independent at 6 months coagulation for Cerebral Venous
interventions are compared with patients receiving Thrombosis [TO-ACT] trial).30
reserved for patients
unfractionated heparin (OR = 2.4; Mechanical thrombectomy is another
with progressive
95% CI, 1.0 to 5.7). Post hoc analysis alternative to improve recanalization in
neurologic deterioration
despite intensive
also revealed a nonsignificant lower patients with CVT (Case 5-3). A recent
medical treatment. chance of ICH (adjusted OR = 0.29; literature review from 1990 to 2012
95% CI, 0.07 to 1.3) for patients included 64 patients who received me-
receiving LMWH than for those receiving chanical thrombectomy.31 The most
unfractionated heparin.26 commonly used device was the AngioJet
(46.9%), followed by balloon venoplasty
Endovascular Therapy without stenting (18.7%) and the Penum-
Different endovascular approaches may bra system (4.7%). Mortality was reported
be considered for patients with CVT in nine (16.1%) patients; 40 (62.5%)
who develop progressive neurologic patients had no disability or minor
deterioration despite intensive medical disability.31 A large randomized multi-
treatment, including anticoagulation. center trial is needed before reaching
Unfortunately, the evidence is limited conclusions about the efficacy of these
by a nonrandomized design and small devices and which patients to target.
sample size for most studies.7
Thrombolysis is regarded as a poten- Decompressive
tial treatment to improve the probability Hemicraniectomy
of early recanalization for patients who Rarely, patients with CVT may develop
were unsuccessful with anticoagulation. a malignant cerebral infarction
A combination of four studies may (Case 5-4). Limited evidence is available
suggest a higher recanalization (87%) regarding the role of decompressive
compared with anticoagulation alone hemicraniectomy in CVT. In a small
(range 47% to 100%).27Y29 However, case series including 10 patients (median
invasive procedures carry some risk of age 41 years) who underwent decom-
complications, including intracranial pressive hemicraniectomy following CVT,
bleeding. A randomized trial comparing five patients recovered without disability
thrombolysis versus anticoagulation is at 12 months (modified Rankin Scale

344 www.ContinuumJournal.com April 2014

Copyright © American Academy of Neurology. Unauthorized reproduction of this article is prohibited.


Case 5-3
A 59-year-old woman with a history of psoriatic arthritis and migraine headaches had the onset of
severe occipital headaches over the course of several days and presented to an outside emergency
department multiple times during the week before admission. Brain CT scans from these emergency
department visits had been read as normal. On the day of admission, she presented with worsening
headache, visual disturbances, and left-sided weakness. Her neurologic examination showed a
normal mental status, but she had a gaze deviation to the right, left homonymous hemianopia, left
facial droop, and moderate left-sided weakness. Brain CT revealed a right temporal ICH, and MRI
brain showed a right sigmoid and transverse sinus thrombosis, extending into the right jugular vein
(Figure 5-6A and B). Because of the size of her hemorrhage, the extension of the venous sinus
thrombosis, and progressive neurologic deterioration, she underwent endovascular mechanical
thrombectomy and balloon venoplasty of the right mid-sigmoid sinus, which resulted in successful
recanalization and antegrade flow through the sigmoid and transverse sinuses and right internal
jugular vein (Figure 5-6C). This procedure was followed by an unfractionated heparin drip. She had
immediate improvement of her headache, but it returned the next day. Her left-sided visual field deficits
and weakness resolved during the next 3 weeks.

FIGURE 5-6 Hemorrhagic venous infarction and endovascular therapy. A, Head CT at presentation showing the large
right temporal venous infarct with hemorrhagic conversion and mass effect. B, Magnetic resonance
venogram (MRV) showing thrombosis of the right transverse and sigmoid sinus. C, Interventional
angiogram with catheter in the right transverse sinus, status post thrombolysis and balloon angioplasty, and
resolution of the thrombus.

Comment. This case illustrates the progression of CVT and the difficulty with recognition of this
condition by CT scan until venous infarction with hemorrhagic conversion leads to progression of
neurologic deficits. In addition, it illustrates the dilemma of treatment in the setting of severe
thrombosis and significant hemorrhage causing mass effect and edema. Aggressive anticoagulation
in the setting of this much bleeding was thought to be contraindicated acutely, leading to the
decision to perform endovascular mechanical thrombectomy.

score of 0 to 1) and two patients died of (modified Rankin Scale score of 3 or


progressive cerebral edema and expan- less).33 This procedure should be re-
sion of the hemorrhagic infarcts.32 In served for young CVT patients who
another review including 13 patients develop a malignant hemispheric
with severe CVT who underwent stroke with progressive neurologic de-
decompressive hemicraniectomy, 11 terioration despite intensive medical
(84.6%) achieved a favorable outcome treatment.

Continuum (Minneap Minn) 2014;20(2):335–351 www.ContinuumJournal.com 345


Copyright © American Academy of Neurology. Unauthorized reproduction of this article is prohibited.
Cerebral Venous Thrombosis

KEY POINT
PREVENTION OF MEDICAL In the ISCVT, 39% of patients
h Seizures are a common
COMPLICATIONS presented with seizures and 6.9%
manifestation, observed
in 30% to 50% of
Seizures experienced an early seizure within 2
patients with cerebral Seizures may occur in one-third of adults weeks after diagnosis. Supratentorial
venous thrombosis. and nearly one-half of children with CVT parenchymal lesions on CT/MRI at the
(Case 5-5).3 The most common factors time of the diagnosis (present in 58%)
associated with the development of were associated with occurrence of
seizures include the presence of a early seizures (OR = 3.1; 95% CI, 1.6
venous infarction, hemorrhagic conver- to 9.6).34 No benefit has been found
sion, and intracranial hypertension. for the routine use of antiepileptic

Case 5-4
A 37-year-old woman was found by her mother with incoherent speech and reports of headaches. When the
paramedics arrived, she developed a generalized tonic-clonic seizure. On arrival to the emergency department,
she was lethargic and had an expressive aphasia and right-sided weakness. Medical history was remarkable for
menorrhagia and anemia. A noncontrast CT on admission revealed an extensive left frontal hypodensity with
mass effect (Figure 5-7). A CT venogram confirmed thrombosis of several left frontal cortical veins with
extension into the
superior sagittal
sinus. MRI confirmed
the extensive cortical
vein thrombosis and
a venous infarction
with hemorrhagic
conversion
(Figure 5-8A–D). The
patient underwent
hemicraniectomy for
a malignant venous
infarction secondary
to CVT. Despite
anticoagulation and
intensive medical and
surgical treatment of
her cerebral edema
with osmotic agents,
the patient developed
neurologic
deterioration with an
Mass effect from the hemorrhagic conversion of a venous infarction (AYD, arrows)
increased intracranial FIGURE 5-7 within the left frontal lobe lesion. Signs of brain herniation are evident, as is
pressure (35 mm Hg) thrombosis of several left frontal cortical veins with very subtle extension in the
after hemicraniectomy. superior sagittal sinus. The lack of visualization of residual lumen represents the
occluded cortical veins and the associated severe mass effect from the adjacent brain
Brain CT revealed edema (C, D, arrows).
enlargement of the
venous infarction with
associated multifocal hemorrhagic components. Figure 5-9 shows herniation of the brain through the
craniectomy with signs of severe mass effect on the midline structures and basal cisterns. The patient died 24
hours after the increased intracranial pressure was diagnosed.
Comment. This case illustrates the challenges in the management and early complications of
malignant edema from CVT, despite aggressive surgical measures.
Continued on page 347

346 www.ContinuumJournal.com April 2014

Copyright © American Academy of Neurology. Unauthorized reproduction of this article is prohibited.


Continued from page 346

FIGURE 5-8 Large area of altered signal and swelling with


restricted diffusion (A, B) involving the cortex
and underlying white matter of most of the
left frontal lobe. Several areas of hypointense signal are
present, more obvious on the gradient-echo and
susceptibility-weighted imaging sequence in keeping with
multiple hemorrhagic foci (D). The magnetic resonance
venogram (C) shows few multifocal filling defects involving the
superior sagittal sinus. These defects are nonobstructive. A focal
severe attenuation of the central lumen of the superior sagittal
sinus (B, C, thick arrows) is present, as is a paucity of expected
opacified left convexity cortical veins compared with the right
frontal lobe (thin arrow).

FIGURE 5-9 Noncontrast CT post craniectomy showing


further enlargement of the large left
hemispheric lesion and associated multifocal
hemorrhagic components and brain herniating through the
craniectomy site with persistent signs of severe mass effect
on the midline structures and basal cisterns. A, Inferior
section, and B, superior section, both of which illustrate the
mass effect and the hemorrhagic conversion.

Continuum (Minneap Minn) 2014;20(2):335–351 www.ContinuumJournal.com 347


Copyright © American Academy of Neurology. Unauthorized reproduction of this article is prohibited.
Cerebral Venous Thrombosis

Case 5-5
A 32-year-old woman presented to the emergency department after having a generalized
tonic-clonic seizure witnessed by her husband. One week before admission, she delivered a baby via
an uneventful cesarean delivery. Her medical history was remarkable for a prior deep venous
thrombosis. She had no family history of thrombophilia. On neurologic examination, she was drowsy
and had left-sided weakness and an enlarged pupil on the right side. A noncontrast brain CT revealed
foci of intraparenchymal hemorrhage in the right parietal subcortical region; CT venography revealed
thrombus within a cortical vein. IV heparin was initiated. MRI revealed heterogeneous signal intensity
on fluid-attenuated inversion recovery (FLAIR)/T2 and T1 sequences with hyperintense components
and susceptibility artifact on the gradient sequence, indicating acute focal bleeding in the right
parietal lobe.
The right
posterior parietal
hyperintensity
in the FLAIR
sequence is
consistent with a
cortical venous
thrombosis
(Figure 5-10A).
Her level of
consciousness
decreased (ie, she
was able to open
her eyes only to FIGURE 5-10 Seizures and intracerebral hemorrhage due to cerebral venous thrombosis in
the puerperium. A, Fluid-attenuated inversion recovery (FLAIR) T2 MRI shows
painful stimuli) hyperintensity in the right posterior parietal region consistent with acute cortical
6 hours after vein thrombosis. B, A head CT post hemicraniectomy demonstrates a large right
admission, and frontoparietal intraparenchymal hemorrhage with associated mass effect and mild herniation
of the brain through the craniectomy defect.
she underwent
hematoma
evacuation and hemicraniectomy (Figure 5-10B). The patient was discharged on warfarin and phenytoin
10 days after admission. Neurologic examination 6 months later was unremarkable, and her functional
status with activities of daily living was intact (Barthel Index of 100). Blood testing at that time revealed a
significantly low protein S level (0.26 mg/L) and free protein S antigen level at 0.38 mg/L. Lifelong
anticoagulation was recommended by a hematologist because of a protein S deficiency identified as a
severe thrombophilia, with high risk of recurrence of venous thromboembolism (Table 5-4).7,20
Comment. This case illustrates the development of CVT in the puerperium presenting with seizures
and an intracerebral hemorrhage. Treatment with phenytoin was initiated after the initial seizures in
the presence of intracerebral hemorrhage.

KEY POINT drugs for patients with CVT. Guide- transverse sinuses) may affect CSF
h Antiepileptic drug lines for the management of CVT drainage, resulting in communicating
treatment for cerebral recommend early initiation of anti- hydrocephalus. In a large study of
venous thrombosis is only
epileptic drugs in patients with CVT 11,400 patients with CVT, hydrocepha-
recommended for patients
and a single or multiple seizures.7 lus was observed in 15.0% of patients.
who have had a seizure
with or without
Hydrocephalus was associated with 3
Hydrocephalus times higher risk of death (OR = 3.2;
parenchymal lesions (eg,
venous infarction, intracranial The main sites for CSF absorption are 95% CI, 5.5 to 15.1).35 The appropriate
hemorrhage). Prophylactic the arachnoid granulations, which pro- treatment includes measures to decrease
use of antiepileptic drugs trude across the sinuses. The thrombo- the production of CSF, recanalization of
is not recommended. sis of major sinuses (eg, superior sagittal, the thrombosed sinus, and management

348 www.ContinuumJournal.com April 2014

Copyright © American Academy of Neurology. Unauthorized reproduction of this article is prohibited.


KEY POINTS
of intracranial hypertension. Neurosurgical CVT have been reported to develop h Intracranial hypertension
consultation for evacuation of CSF either deep venous thrombosis, pulmonary may be seen in 15% to
with ventriculostomy or ventriculo- embolism, ischemic stroke, and acute 40% of patients with
peritoneal shunt may be necessary.7 limb ischemia in addition to recurrent cerebral venous
CVT, but these events were mostly in thrombosis.
Intracranial Hypertension those not currently anticoagulated.2 h The risk of recurrence
The development of intracranial hyper- The presence of thrombophilia, after an unprovoked
tension is usually caused by venous and whether this is considered mild cerebral venous
outflow obstruction and CSF malab- or severe, has been used as a decision thrombosis is low, but
sorption. Intracranial hypertension point for risk of recurrence and dura- severe thrombophilias
may be seen in 15% to 40% of patients tion of treatment. Mild thrombophilias are associated with a
with CVT.2,7 include heterozygous factor V Leiden high risk of recurrence
Symptomatic treatment includes and prothrombin G20210A mutations, of cerebral venous
the use of diuretics (ie, acetazolamide) and dysfibrinogenemia, whereas pro- thrombosis as well as
systemic venous
and serial lumbar punctures. Steroids tein C, protein S, and antithrombin III
thromboembolism.
are ineffective.7,36 In refractory cases, deficiencies; antiphospholipid anti-
lumboperitoneal shunt may be required.37 bodies; homozygous factor V Leiden
However, no randomized trials have mutation; and multiple abnormalities
tested any of these interventions. are categorized as severe. A follow-up
study of participants with CVT revealed
Visual Loss that patients with severe thrombophilia
The persistent and prolonged pres- were 4 times more likely to have a
sure on the optic nerves can result in deep venous thrombosis or pulmonary
loss of vision, reported in up to 13% embolism (adjusted hazard ratio
of patients with CVT.2 However, early 4.19; 95% CI, 1.25 to 14.0) than those
recognition and the use of advanced without thrombophilia. The risk of
imaging suggest this has become a less recurrence was similar in those with
common phenomenon. Ophthalmic con- mild thrombophilia and those without
sultation, monitoring for papilledema, thrombophilia.21 Among hereditary
and visual field testing are commonly thrombophilias, the highest risk of
recommended measures. For patients at recurrence of venous thromboembo-
high risk of blindness, optic nerve fenes- lism (not specifically CVT) is with
tration may be considered.7,13 homozygous factor V Leiden and anti-
thrombin III, protein C, and protein S
PROGNOSIS AND RISK OF deficiencies (40% at 5 years), whereas
RECURRENCE the risk with heterozygous factor V
Outcomes after CVT are generally favor- Leiden, prothrombin G20210A muta-
able. Women tend to have a better tions, and elevated factor VIII is much
prognosis than men; in the ISCVT, 81% less (11% at 5 years) (Table 5-4).20
of women had complete recovery versus
71% of men (P=.01).19 However, women CHRONIC ANTICOAGULATION:
without sex-specific risk factors tended to HOW LONG TO TREAT
have a worse outcome than women with Duration of anticoagulation following
these risk factors (OR for poor outcome CVT has not been studied in any
3.7; 95% CI, 1.9 to 7.4).19 randomized controlled trials, and
The recurrence of any thrombotic most recommendations are based on
event after a CVT is about 6.5% per observational data.7 Considerations
year, including systemic venous should be based on the risk for
thromboembolism.2,38 Patients with recurrence, patient preference, bleeding
Continuum (Minneap Minn) 2014;20(2):335–351 www.ContinuumJournal.com 349
Copyright © American Academy of Neurology. Unauthorized reproduction of this article is prohibited.
Cerebral Venous Thrombosis

KEY POINT
h For provoked cerebral risk, and the risk of thrombosis without REFERENCES
venous thrombosis, anticoagulation.7,20,21 Guideline recom- 1. Bousser MG, Ferro JM. Cerebral venous
mendations for the duration of thrombosis: an update. Lancet Neurol
anticoagulation 2007;6(2):162Y170.
consisting of anticoagulation in patients with CVT
2. Ferro J, Canhao P, Stam J, et al. Prognosis
adjusted-dose warfarin are dependent on the presence of of cerebral vein and dural sinus thrombosis:
is recommended for 3 to provoked (based on a transient risk results of the International Study on
6 months. An evaluation factor) versus unprovoked events, as Cerebral Vein and Dural Sinus Thrombosis
for thrombophilia will well as the presence of mild versus (ISCVT). Stroke 2004;35(3):664Y670.
potentially aid in the severe thrombophilia. For those with 3. deVeber G, Andrew M, Adams C, et al.
decision for the duration provoked CVT, warfarin (target interna- Cerebral sinovenous thrombosis in children.
of anticoagulation. N Engl J Med 2001;345(6):417Y423.
tional normalized ratio of 2.0 to 3.0) for
3 to 6 months is recommended.7 4. Stam J. Thrombosis of the cerebral veins
and sinuses. N Engl J Med 2005;352(17):
Patients with unprovoked CVT may be 1791Y1798.
treated with warfarin for 6 to 12 months. 5. Crassard I, Bousser MG. Headache in
In patients with recurrent CVT, venous patients with cerebral venous thrombosis
thromboembolism after CVT, or first [in French]. Rev Neurol (Paris) 2005;161(6Y7):
CVT (provoked or unprovoked) with 706Y708.

severe thrombophilia, indefinite 6. Girot M, Ferro JM, Canhão P, et al. Predictors


of outcome in patients with cerebral venous
anticoagulation may be considered. It is thrombosis and intracerebral hemorrhage.
important to note that these recommen- Stroke 2007;38(2):337Y342.
dations are based on consensus.7 It is 7. Saposnik G, Barinagarrementeria F, Brown R,
recommended that patients with CVT et al. Diagnosis and management of
undergo thrombophilia testing, includ- cerebral venous thrombosis: a statement for
healthcare professionals from the American
ing functional testing for protein C, Heart Association/American Stroke Association.
protein S, antithrombin III; quantitative Stroke 2011;42(4):1158Y1192.
measurement of antiphospholipid anti- 8. Biousse V, Ameri A, Bousser MG. Isolated
bodies (lupus anticoagulant and intracranial hypertension as the only sign
of cerebral venous thrombosis. Neurology
anticardiolipin antibodies); and genetic 1999;53(7):1537Y1542.
testing for factor V Leiden and pro-
9. Roach ES, Golomb MR, Adams R, et al.
thrombin G20210A mutations. Consul- Management of stroke in infants and
tation with a physician with expertise in children: a scientific statement from a
coagulation disorders, such as a hema- Special Writing Group of the American
Heart Association Stroke Council and the
tologist, may also be helpful.7 Council on Cardiovascular Disease in the
Young. Stroke 2008;39(9):2644Y2691.
10. Dentali F, Squizzato A, Marchesi C, et al.
CONCLUSION D-dimer testing in the diagnosis of cerebral

In conclusion, CVT is an uncommon vein thrombosis: a systematic review and a


meta-analysis of the literature. J Thromb
cause of stroke, but it requires accu- Haemost 2012;10(4):582Y589.
rate diagnosis since the pathophysiol-
11. Hiltunen S, Putaala J, Haapaniemi E, et al.
ogy and treatment differ from arterial D-dimer and clinicoradiologic features in
stroke. Understanding the transient cerebral venous thrombosis. J Neurol Sci
and chronic risk factors is the key to 2013;327(1Y2):12Y14.
prognosis for recurrent CVT and 12. Leach MJ, Gall SL, Dewey HM, et al. Factors
whether long-term treatment with associated with quality of life in 7-year
survivors of stroke. J Neurol Neurosurg
anticoagulation is needed. In addition,
Psychiatry 2011;82(12):1365Y1371.
although the outcomes after CVT are
13. Einhäupl K, Bousser MG, de Bruijn SF, et al.
generally favorable, they also depend
EFNS guideline on the treatment of cerebral
on patient factors, such as sex and the venous and sinus thrombosis. Eur J Neurol
female-specific risk factors. 2006;13(6):553Y559.

350 www.ContinuumJournal.com April 2014

Copyright © American Academy of Neurology. Unauthorized reproduction of this article is prohibited.


14. Leach JL, Fortuna RB, Jones BV, Gaskill- 27. Stolz E, Trittmacher S, Rahimi A, et al.
Shipley MF. Imaging of cerebral venous Influence of recanalization on outcome in
thrombosis: current techniques, spectrum dural sinus thrombosis: a prospective study.
of findings, and diagnostic pitfalls. Stroke 2004;35(2):544Y547.
Radiographics 2006;26(suppl 1):S19YS41.
28. Baumgartner RW, Studer A, Arnold M,
15. Selim M, Fink J, Linfante I, et al. Diagnosis of Georgiadis D. Recanalisation of cerebral
cerebral venous thrombosis with echo-planar venous thrombosis. J Neurol Neurosurg
T2*-weighted magnetic resonance imaging. Psychiatry 2003;74(4):459Y461.
Arch Neurol 2002;59(6):1021Y1026.
29. Strupp M, Covi M, Seelos K, et al. Cerebral
16. Rodallec MH, Krainik A, Feydy A, et al. venous thrombosis: correlation between
Cerebral venous thrombosis and recanalization and clinical outcomeVa
multidetector CT angiography: tips and long-term follow-up of 40 patients. J Neurol
tricks. Radiographics 2006;26(suppl 1):S5YS18. 2002;249(8):1123Y1124.
17. Wasay M, Azeemuddin M. Neuroimaging of
30. Coutinho JM, Ferro JM, Zuurbier SM, et al.
cerebral venous thrombosis. J Neuroimaging
Thrombolysis or anticoagulation for cerebral
2005;15(2):118Y128.
venous thrombosis: rationale and design of
18. Ganeshan D, Narlawar R, McCann C, et al. the TO-ACT trial. Int J Stroke 2013;8(2):135Y140.
Cerebral venous thrombosisVa pictorial
31. Borhani Haghighi A, Mahmoodi M, Edgell
review. Eur J Radiol 2010;74(1):110Y116.
RC, et al. Mechanical thrombectomy for
19. Coutinho JM, Ferro JM, Canhão P, et al. cerebral venous sinus thrombosis: a
Cerebral venous and sinus thrombosis in comprehensive literature review [printed
women. Stroke 2009;40(7):2356Y2361. online ahead of print January 7, 2013].
20. Lijfering WM, Brouwer JL, Veeger NJ, Clin Appl Thromb Hemost 2013. doi:10.1177/
et al. Selective testing for thrombophilia in 1076029612470968.
patients with first venous thrombosis: results 32. Zuurbier SM, Coutinho JM, Majoie CB, et al.
from a retrospective family cohort study Decompressive hemicraniectomy in severe
on absolute thrombotic risk for currently cerebral venous thrombosis: a prospective
known thrombophilic defects in 2479 case series. J Neurol 2012;259(6):1099Y1105.
relatives. Blood 2009;113(21):5314Y5322.
33. Coutinho JM, Majoie CB, Coert BA, Stam J.
21. Martinelli I, Bucciarelli P, Passamonti SM, Decompressive hemicraniectomy in cerebral
et al. Long-term evaluation of the risk of sinus thrombosis: consecutive case series and
recurrence after cerebral sinus-venous thrombosis. review of the literature. Stroke 2009;40(6):
Circulation 2010;121(25):2740Y2746. 2233Y2235.
22. Einhäupl KM, Villringer A, Meister W, 34. Ferro JM, Canhão P, Bousser MG, et al; ISCVT
et al. Heparin treatment in sinus venous Investigators. Early seizures in cerebral vein
thrombosis. Lancet 1991;338(8767):597Y600. and dural sinus thrombosis: risk factors and
role of antiepileptics. Stroke 2008;39(4):
23. de Bruijn S, Stam J. Randomized, 1152Y1158.
placebo-controlled trial of anticoagulant
treatment with low-molecular-weight 35. Nasr DM, Brinjikji W, Cloft HJ, et al.
heparin for cerebral sinus thrombosis. Stroke Mortality in cerebral venous thrombosis:
1999;30(3):484Y488. results from the national inpatient sample
database. Cerebrovasc Dis 2013;35(1):
24. Coutinho J, de Bruijn SF, Deveber G, Stam J. 40Y44.
Anticoagulation for cerebral venous sinus
thrombosis. Cochrane Database Syst Rev 36. Canhão P, Cortesão A, Cabral M, et al. Are
2011;(8):CD002005. steroids useful to treat cerebral venous
thrombosis? Stroke 2008;39(1):105Y110.
25. Lansberg MG, O’Donnell MJ, Khatri P, et al.
Antithrombotic and thrombolytic therapy 37. Hanley DF, Feldman E, Borel CO, et al.
for ischemic stroke: Antithrombotic Therapy Treatment of sagittal sinus thrombosis
and Prevention of Thrombosis, 9th ed: associated with cerebral hemorrhage and
American College of Chest Physicians intracranial hypertension. Stroke 1988;19(7):
Evidence-Based Clinical Practice Guidelines. 903Y909.
Chest 2012;141(2 suppl):e601SYe636S.
38. Ferro JM, Lopes MG, Rosas MJ, et al;
26. Coutinho JM, Ferro JM, Canhão P, et al. Cerebral Venous Thrombosis Portuguese
Unfractionated or low-molecular weight Collaborative Study Group. Long-term
heparin for the treatment of cerebral prognosis of cerebral vein and dural sinus
venous thrombosis. Stroke 2010;41(11): thrombosis. Results of the VENOPORT study.
2575Y2580. Cerebrovasc Dis 2002;13(4):272Y278.

Continuum (Minneap Minn) 2014;20(2):335–351 www.ContinuumJournal.com 351


Copyright © American Academy of Neurology. Unauthorized reproduction of this article is prohibited.
Review Article

Evaluation and
Address correspondence to
Dr Jason Mackey, 355 West
16th Street, Suite 3200,
Indianapolis, IN 46202,
jasonmackey78@gmail.com.
Relationship Disclosure:
Dr Mackey has served as an
Management of Stroke
expert medical record
reviewer in vascular neurology
for court cases and has received
in Young Adults
funding as the principal
investigator for outcomes and Jason Mackey, MD, MS
processes of care in intracerebral
hemorrhage for the Indiana
University Health Values Fund
and the Indiana Clinical and
ABSTRACT
Translational Sciences Institute Purpose of Review: This article provides an overview of the evaluation and
Project Development Team. management of ischemic stroke in young adults, with an emphasis on cervical
He is also a recipient of a NIH
Loan Repayment Program award. artery dissection, patent foramen ovale, and hypercoagulable states.
Unlabeled Use of Recent Findings: The incidence of ischemic stroke in young patients is increasing,
Products/Investigational although the reasons remain unclear. Patients with ischemic stroke at a young age
Use Disclosure:
Dr Mackey reports no
are more likely to die at an early age than their peers. Well-established vascular risk
disclosure. factors are prevalent in young stroke patients. Recent studies have informed the
* 2014, American Academy treatment of dissection and patent foramen ovale among others. The utility of
of Neurology. testing for hypercoagulable states in ischemic stroke is unclear.
Summary: Ischemic stroke in young adults is a major public health problem. A
wide range of etiologies of ischemic stroke is found in this age group. A careful
history, thorough examination, and methodical workup are essential. Specific
management is predicated on identification of the underlying etiology.

Continuum (Minneap Minn) 2014;20(2):352–369.

INTRODUCTION tients. In the largest study, Finnish in-


Stroke is often considered a disease of vestigators retrospectively evaluated 1008
older people, but an estimated 10% of consecutive ischemic stroke patients
patients with stroke are younger than aged 15 to 49 in a single university
50 years. Recent evidence suggests that center registry from 1994 to 2007.
the incidence of ischemic stroke in Conventional risk factors for stroke
young adults is rising,1 although the were highly prevalent in this young
reasons for that increase are unclear. stroke population: hypertension (39%),
These trends are especially concerning dyslipidemia (60%), and smoking
in light of recent evidence that young (44%).3 Investigators at a university cen-
stroke patients are at a greatly increased ter in the United States found similarly
risk of early death compared with the high rates of conventional risk factors.4
general population.2 The purpose of this Given the prevalence of conventional
article is to provide an overview of some risk factors in young stroke patients,
of the potential stroke etiologies to initial evaluation should approximate that
consider in patients under 50 years of age. of older stroke patients. Further evalua-
tion may be necessary pending initial
STANDARD RISK FACTORS results. A suggested initial evaluation is
Classic vascular risk factors, including shown in Table 6-1.
hypertension, dyslipidemia, diabetes As Table 6-25 demonstrates, there
mellitus, and tobacco abuse are prevalent are multiple potential etiologies of is-
in several studies of young stroke pa- chemic stroke in young adults. Of

352 www.ContinuumJournal.com April 2014

Copyright © American Academy of Neurology. Unauthorized reproduction of this article is prohibited.


KEY POINTS

TABLE 6-1 Initial Evaluation of a Young Adult With Ischemic Stroke h The incidence of stroke
in young adults appears
b Thorough history to be increasing.
b Physical examination h Stroke in young adults
b Brain MRI with diffusion-weighted images and gradient recall echo or is associated with
susceptibility-weighted imaging sequences increased risk of early
b Head and neck angiography (MR, CT, or conventional) death.

b Transesophageal echocardiography h Standard vascular risk


b Continuous cardiac telemetry factors are prevalent in
young stroke patients.
b ECG
b Standard laboratory evaluation
Complete blood count
Comprehensive metabolic panel
Coagulation studies
Erythrocyte sedimentation rate/C-reactive protein
Fasting lipid panel
Urine cocaine and methamphetamine
MRI = magnetic resonance imaging; MR = magnetic resonance; CT = computed tomography; ECG =
electrocardiogram.

particular interest for this review for about 2% of all ischemic strokes,6
are three categories especially germane they account for a higher proportion
to a discussion of stroke in young in young adults.3 A population-based
adults: (1) cervical artery dissections, study in Olmsted County, Minnesota
(2) patent foramen ovale (PFO), and estimated the incidence as 1.72 per
(3) hypercoagulable syndromes, includ- 100,000 per year for internal carotid
ing inherited thrombophilias and the artery dissections and 0.97 per
antiphospholipid antibody syndrome. 100,000 for vertebral artery dissec-
tions.7 The peak incidence occurs in
ARTERIAL DISSECTION the fifth decade of life. Intracranial
Dissection is the separation of the dissections appear to comprise ap-
structural components of the connec- proximately 10% of published dissec-
tive tissue network of the arterial wall tion cases. In 13% to 16% of cases of
and can occur in the carotid, vertebral, dissection, multiple arteries are in-
and intracranial arteries. The second volved.8 Recent analyses have noted
most common lesion of the cervical significant differences in the clinical
arteries after atherosclerosis, dissec- presentation, risk factors, and func-
tion has traditionally been character- tional outcomes of internal carotid
ized as either spontaneous or artery versus vertebral dissections.9,10
secondary to major trauma. Several The classic teaching is that cervical
connective tissue disorders have also artery dissections arise from an intimal
been associated with dissection, in- tear and that blood enters the wall and
cluding vascular Ehlers-Danlos syn- forms an intramural hematoma. But
drome (type IV), Marfan syndrome, the absence of a demonstrable intimal
and osteogenesis imperfecta, among tear on some histologic examinations
others. Although dissections account suggests that at least some dissections

Continuum (Minneap Minn) 2014;20(2):352–369 www.ContinuumJournal.com 353


Copyright © American Academy of Neurology. Unauthorized reproduction of this article is prohibited.
Stroke in Young Adults

a
TABLE 6-2 Uncommon Causes of Stroke

b Infectious and Inflammatory


Varicella-zoster virus
Neurosyphilis
Tuberculous meningitis
Fungal meningitis
Cysticercosis
Lyme disease
Chagas disease
HIV
Takayasu disease
Behçet syndrome
Neurosarcoidosis
Kawasaki disease
Buerger disease
Vasculitis (primary angiitis of the CNS, Sjögren syndrome, Wegener
granulomatosis, Churg-Strauss syndrome)
Temporal arteritis
b Genetic
Pulmonary arteriovenous malformations
Hereditary hemorrhagic telangiectasia
Cerebral autosomal dominant arteriopathy with subcortical infarcts and
leukoencephalopathy (CADASIL)
Fabry disease
Marfan syndrome
Pseudoxanthoma elasticum
Ehlers-Danlos syndrome
Progeria
Mitochondrial encephalomyopathy, lactic acidosis, and strokelike episodes
(MELAS) and other mitochondrial diseases
Sturge-Weber disease
Von Hippel-Lindau disease
Neurofibromatosis
Menkes disease
Wyburn-Mason syndrome
b Vascular Conditions of the Eyes, Ears, and Brain
Eales retinopathy
Acute posterior multifocal placoid pigment epitheliopathy
Susac syndrome
Hereditary endotheliopathy with retinopathy, nephropathy, and stroke
(HERNS)
Cogan syndrome

Continued on next page

354 www.ContinuumJournal.com April 2014

Copyright © American Academy of Neurology. Unauthorized reproduction of this article is prohibited.


a
TABLE 6-2 Uncommon Causes of Stroke (Continued)

b Systemic Disorders That Also Involve the Cerebrovascular System


Microscopic polyangiitis and polyarteritis nodosa
Systemic lupus erythematosus
Rheumatoid arthritis
Substance abuse
Kohlmeier-Degos syndrome
Inflammatory bowel disease
Sweet syndrome
Nephrotic syndrome
Epidermal nevus syndrome
Sneddon syndrome
Scleroderma
b Noninflammatory Disorders of the Arterial Wall
Cervicocephalic arterial dissections
Cerebral amyloid angiopathy
Moyamoya disease
Dilatative dolichoectasia
Fibromuscular dysplasia
b Thrombophilias (Table 6-3 and Table 6-4)
b Vasospasm Related
b Reversible Cerebral Vasoconstriction Syndrome
b Eclampsia
b Other
Radiation vasculopathy
Migrainous infarct
Lymphomatoid granulomatosis
Hypoperfusion
Intravascular lymphoma
Henoch-Schönlein purpura
a
Data from Caplan LR, ed, Cambridge University Press.5

are caused by a primary intramural factor appears to be trauma. Dissec-


hematoma.11 One study found outer tion occurs commonly with major
wall abnormalities in superficial tem- trauma but is also associated with
poral arteries of dissection patients minor trauma, such as chiropractic
and proposed that an underlying manipulation, coughing, and sneezing,
arteriopathy affecting the media and among others.13 Recent infection has
adventitia played a crucial role in also been associated with dissection.
dissection pathogenesis.12 Dissection Several lines of evidence suggest a
is likely a multifactorial process, and genetic contribution to dissection,
both environmental and genetic risk including reports of familial dissec-
factors have been implicated. The tion cases. Several candidate genes
most important environmental risk have been proposed, including the

Continuum (Minneap Minn) 2014;20(2):352–369 www.ContinuumJournal.com 355


Copyright © American Academy of Neurology. Unauthorized reproduction of this article is prohibited.
Stroke in Young Adults

KEY POINTS
h CT angiography and intercellular adhesion molecule 1 gene dissection include posterior circula-
magnetic resonance (ICAM1), the collagen, type III, alpha 1 tion strokes, subarachnoid hemor-
angiography (MRA) gene (COL3A1), and the methylene- rhage (in intracranial vertebral artery
have supplanted tetrahydrofolate reductase (NAD[P]H) dissection), and spinal cord ischemia.
conventional angiography gene (MTHFR). The Cervical Artery Internal carotid artery or vertebral
in the diagnosis of Dissections and Ischemic Stroke Pa- artery dissections resulting in arterial
dissection. tients (CADISP) consortium is actively occlusion or stenosis are more likely
h CTA appears to have investigating these and other genes as to result in ischemic manifestations,
some advantages over well as gene-environment interactions.8 while local manifestations are more
MRA in the diagnosis Clinical manifestations of cervical likely to occur in the absence of luminal
of dissection. artery dissections include both local narrowing.14 Case 6-1 illustrates a com-
signs and symptoms and ischemic mon presentation of a young adult with
events. Local manifestations of inter- carotid dissection.
nal carotid artery dissections include Conventional angiography, the di-
Horner syndrome (ipsilateral), neck agnostic mainstay of cervicocephalic
pain, headache, tinnitus, facial pain, dissection for many years, has now
and cranial nerve palsies (IX to XII been largely replaced by noninvasive
most commonly). Local manifestations means such as CT angiography (CTA)
in vertebral dissections include poste- and MR angiography (MRA). Conven-
rior headache or neck pain, cervical tional angiography is limited both by
root involvement (most commonly its invasiveness and its inability to
C5-C6 level), and lower brainstem visualize intramural hematoma. The
compression (in intracranial vertebral luminal contour can also at times
dissection). Ischemic manifestations appear normal on conventional angi-
of internal carotid artery dissection ography. MRA can be combined with
include stroke (most commonly in axial T1-weighted cervical MRI with fat
the middle cerebral artery distribu- suppression to better identify small
tion), amaurosis fugax, ischemic optic intramural hematomas and does not
neuropathy, and retinal infarction. require radiation. CTA, on the other
Ischemic manifestations in vertebral hand, might be superior to MRA with

Case 6-1
A 34-year-old man presented with a right posterior headache, neck pain,
and left-sided weakness that he noted when he awakened in the morning.
He had recently started a yoga routine. He had no prior significant medical
history, although he was a smoker. General examination was
unremarkable. Neurologic examination was remarkable for a mild left
hemiparesis. Brain MRI demonstrated restricted diffusion in the right
middle cerebral artery distribution. CT angiography of the head and
neck demonstrated dissection of the right internal carotid artery.
Echocardiography was unremarkable. The patient was started on a
heparin drip and warfarin 5 mg daily. A follow-up CT angiogram at
3 months showed resolution of the dissection, and anticoagulation was
transitioned to aspirin 325 mg daily. He has had no further complications
in 5 years of follow-up.
Comment. This patient sustained a right middle cerebral artery stroke
due to dissection apparently related to minor trauma. Both his arterial
healing time and the absence of recurrence are common in carotid artery
dissection.

356 www.ContinuumJournal.com April 2014

Copyright © American Academy of Neurology. Unauthorized reproduction of this article is prohibited.


KEY POINT
regard to diagnosis of intimal flaps, A recent meta-analysis found that h There is no
pseudoaneurysms, and high-grade ste- thrombolysis in patients with stroke contraindication to tissue
nosis15 in addition to superior identifi- related to dissection had similar safety plasminogen activator
cation of vertebral artery dissections. and outcome profiles compared with in strokes due to
Figure 6-1 demonstrates the superiority patients with strokes unrelated to dissection.
of CTA in the setting of high-grade dissection,17 although other studies
stenosis. High-resolution 3-tesla MRI is have questioned the efficacy of throm-
also emerging as a potential tool for bolysis for dissection.18 Patients are
detection of cervicocephalic dissection commonly prescribed anticoagulants
and delineation of intramural thrombus given concerns about recurrent
versus intraluminal thrombus.16 events. A common practice is to
The treatment of cervicocephalic initiate an anticoagulant in the acute
dissections is controversial owing to a setting and then to repeat angiogra-
lack of randomized controlled trials. In phy in 3 months. If the dissection has
the hyperacute setting, there is no resolved, the patient is typically
clear contraindication to thrombolysis. transitioned to an antiplatelet agent.

FIGURE 6-1 Example of a long-segment high-grade stenosis (string sign) from


dissection identified on CT angiography (CTA), which appeared occluded
on MRI/magnetic resonance angiography (MRA). A, B, Curved planar
reformatted images of the left internal carotid artery demonstrate flamelike tapering of
the proximal vessel with wall thickening and long-segment high-grade stenosis (arrow).
C, Axial plane from CTA shows the tiny residual lumen of the vessel (arrow). D,
Contrast-enhanced MRA acquired on the same day demonstrates apparent occlusion
just distal to the carotid bifurcation (arrow).
15
Modified with permission from Vertinsky AT, et al, AJNR Am J Neuroradiol. B 2008 American Society of
Neuroradiology. www.ajnr.org/content/29/9/1753.long.

Continuum (Minneap Minn) 2014;20(2):352–369 www.ContinuumJournal.com 357


Copyright © American Academy of Neurology. Unauthorized reproduction of this article is prohibited.
Stroke in Young Adults

If irregularities persist, anticoagulation tients compared with closure,24 but


is generally continued until the arterial randomized data were not available.
dissection resolves, completely oc- Three randomized controlled trials,
cludes, or stabilizes, at which time an each investigating the role of PFO for
antiplatelet agent is initiated. Recent secondary stroke prevention, have
work, however, has questioned whether now been published. The first of
this approach is warranted. A large these, the CLOSURE-I trial, investigated
nonrandomized study found that ische- the STARFlex device in patients 60
mic recurrence rates were very low both years old or younger with stroke or
in patients treated with antiplatelet TIA and a PFO.25 The primary end
agents and those treated with anticoag- point was a composite of stroke or TIA
ulants.19 An ongoing randomized clinical in 2 years of follow-up, 30-day all-cause
trial compares antiplatelet agents and mortality, and death from neurologic
anticoagulation in acute dissection20; the cause between 31 days and 2 years.
nonrandomized portion of this study More than 900 patients were enrolled.
found no evidence of superiority for The primary end point occurred in 5.5%
anticoagulation.21 of the closure group and 6.8% of the
Recurrence rates of both ischemic medical group (hazard ratio [HR] 0.78;
events and dissections are generally 95% confidence interval [CI], 0.45 to
reported as low. One-year ischemic re- 1.35; P=.37). Major periprocedural
currence rates range from 0% to 13%,8 complications occurred in 13 (3.2%)
and recurrences generally occur in the patients in the closure group. The
first few weeks following initial presen- authors found plausible alternative ex-
tation. Dissection recurrence rates are planations to a PFO-related cause for
low in population-based studies.7 Aca- stroke or TIA in a majority of patients
demic centers report higher rates but in both the closure and medical
may be subject to referral bias.8 groups.
The PC trial investigated a different
PATENT FORAMEN OVALE device, the Amplatzer PFO occluder, in
PFO, an embryonic defect in the atrial patients younger than 60 years with stroke,
septum, has been postulated as a TIA, or peripheral thromboembolic
source for paradoxical embolism for event who had a PFO.26 More than
decades. The proportion of the gen- 400 patients were enrolled. The prima-
eral population with PFO is approxi- ry end point (a composite of non-
mately 20%, but in young stroke fatal stroke, TIA, peripheral embolism,
patients this proportion is about 50%. and death) occurred in 3.4% of the
Co-occurrence with atrial septal aneu- closure group and 5.2% of the medical
rysm (greater than 10-mm septal ex- group (HR 0.63; 95% CI, 0.24 to 1.62;
cursion during a cardiac cycle) seems P=.34). The authors reported three
to confer additional risk.22,23 The (1.5%) minor procedural complications
presence of PFO in the setting of an in the closure group.
inherited or acquired thrombophilia is The Randomized Evaluation of Re-
especially important because it pro- current Stroke Comparing PFO Clo-
vides a potential mechanism for ve- sure to Established Current Standard
nous material to pass into the arterial of Care Treatment (RESPECT) trial was
circulation (ie, paradoxical emboliza- published concurrently with the PC
tion). Dozens of observational studies trial and also investigated the PFO
have suggested a higher rate of recur- Amplatzer occluder.27 The investiga-
rent stroke in medically treated pa- tors enrolled nearly 1000 patients with
358 www.ContinuumJournal.com April 2014

Copyright © American Academy of Neurology. Unauthorized reproduction of this article is prohibited.


KEY POINTS
cryptogenic strokes and PFO. The cascade is a complex system with multi- h Patent foramen ovale
primary end point was a composite ple sites in which alteration could result closure trials have yet
of nonfatal and fatal ischemic stroke in thrombophilia. to demonstrate
and early death. Nine patients with Several conditions are purported to superiority over medical
recurrent stroke were in the closure predispose patients to thromboembo- management.
group and 16 in the medical group lism and, by extension, acute ischemic h The best long-term
(HR 0.49; 95% CI, 0.22 to 1.11; P=.08). stroke. Most of these conditions are antithrombotic agent
Procedure-related adverse events occurred primarily associated with venous throm- for stroke patients with
in 21 (4.2%) patients in the closure group. bosis, although a few conditions such as patent foramen ovale
Taken individually, these trials did not homocystinuria and antiphospholipid remains unclear.
demonstrate the superiority of PFO antibody syndrome are associated with h The thrombophilias
closure over medical management; how- arterial thrombosis. The presumed with the best evidence for
ever, a recent meta-analysis of the trials mechanism for arterial stroke in a venous thromboembolism
suggests potential benefit.28 Given the patient with a venous thrombophilia is are factor V Leiden,
relatively large statistical effect sizes in paradoxical embolus, most commonly prothrombin gene
the trials, further studies are likely forth- through a PFO. A pressure gradient mutation, protein S
coming in an attempt to identify the promoting passage of venous material deficiency, protein C
patient population most likely to benefit. to the arterial circulation may be gener- deficiency, and
antithrombin III deficiency.
The optimal management of cryp- ated via the Valsalva maneuver. The
togenic stroke in a young person with inherited thrombophilias with the best-
PFO remains unclear at this point. established evidence base for venous
Given the negative trial data, for now thromboembolism are factor V Leiden
closure should be limited to the and activated protein C resistance,
setting of a clinical trial. The optimal prothrombin gene mutation, protein S
long-term antithrombotic agent for deficiency, protein C deficiency, and
the stroke patient with PFO is also antithrombin III deficiency. The most
unclear. No specific guidelines have common acquired thrombophilia is
been developed for the co-occurrence antiphospholipid antibody syndrome.
of PFO and atrial septal aneurysm. Table 6-329 and Table 6-429 list the most
Given the lack of definitive data, the common inherited and acquired
decision regarding antiplatelet versus thrombophilic states, respectively.
anticoagulant therapy should be in-
formed by patient risk stratification Factor V Leiden
and risk tolerance, although in the Approximately 90% to 95% of patients
absence of a deep vein thrombosis or with activated protein C resistance have
known hypercoagulable state antiplatelet the identical mutation, which is a single
therapy is typically recommended. Mod- point mutation in the factor V gene
ifiable risk factors should be aggressively (factor V R506Q). Factor V Leiden in-
addressed. creases the risk of thrombosis by in-
creasing thrombin production. Factor V
HYPERCOAGULABLE CAUSES Leiden is by far the most common
OF STROKE genetic risk factor for thrombophilia.
Up to 4% of ischemic strokes are The prevalence of factor V Leiden
thought to be due to coagulation disor- heterozygosity varies within American
ders, and young patients in particular are populations; the frequency is 5.3% in
often screened for hypercoagulable syn- whites, 2.2% in Hispanics, 1.3% in Native
dromes. Hypercoagulable syndromes Americans, 1.2% in African Americans,
may include inherited deficiencies and and 0.5% in Asian Americans.30 Factor V
acquired conditions. The coagulation Leiden is not fully penetrant, and only

Continuum (Minneap Minn) 2014;20(2):352–369 www.ContinuumJournal.com 359


Copyright © American Academy of Neurology. Unauthorized reproduction of this article is prohibited.
Stroke in Young Adults

a
TABLE 6-3 Primary/Inherited Hypercoagulable States

b Factor V Leiden mutation/activated protein C resistance


b Prothrombin G20210A mutation
b Protein S deficiency
b Protein C deficiency
b Antithrombin deficiency
b Heparin cofactor II deficiency
b Fibrinogen disorders
b Elevated thrombin-activatable fibrinolysis inhibitor
b Elevated factor VIII
b Elevated factor IX
b Elevated factor XI
b Fibrinolytic system disorders
b Methylenetetrahydrofolate reductase (NAD[P]H) (MTHFR) gene mutation
a
Modified with permission from Biller J, ed, Saunders.29 B 2009 Elsevier.

5% to 10% of factor V Leiden heterozy- risk of thrombosis. Prevalence of the


gotes will experience a symptomatic mutation varies in populations; up to
venous thromboembolic event. Homo- 7% of the white population carries the
zygotes are much rarer but have a mutation, while it is very rare in black
markedly elevated thrombotic risk (the or Asian populations.36 Homozygotes are
relative risk for first episode of ve- at increased risk for DVT and cerebral
nous thrombosis is 7 in heterozy- venous sinus thrombosis, especially
gotes and 80 in homozygotes).31 in patients taking oral contraceptives.
Activated protein C resistance is associ- Conclusive evidence of increased risk
ated with several other factors, includ- for arterial thrombosis in patients with
ing elevated factor VIII levels, oral prothrombin deficiency has not been
contraceptives, pregnancy, hormone re- established.
placement, cancer, antiphospholipid an-
tibody syndrome, and smoking. Patients Protein S Deficiency
with factor V Leiden frequently have Protein S is a vitamin K-dependent
other thrombophilias,32Y34 and these protein that enhances fibrinolysis, inhibits
patients appear to be at increased risk prothrombin activation, and reduces
for thrombosis. Factor V Leiden has not thrombin production. Protein S circu-
been convincingly linked to arterial lates in both free and protein-bound
thrombosis. Case 6-2 is a typical exam- forms. Deficiencies of protein S can be
ple of a patient with factor V Leiden. genetic or acquired. In genetic cases,
inheritance is predominantly autoso-
Prothrombin Gene Mutation mal dominant. Heterozygotes are at
Prothrombin deficiency is the second risk for recurrent venous thromboem-
most common inherited thrombophilia bolism; homozygous protein S defi-
after factor V Leiden, and the source of ciency is usually incompatible with
the thrombophilia is a single point mu- life. Acquired protein S deficiency is
tation (G20210A). Carriers have higher associated with pregnancy, oral con-
prothrombin levels and are at higher traceptive pill use, disseminated
360 www.ContinuumJournal.com April 2014

Copyright © American Academy of Neurology. Unauthorized reproduction of this article is prohibited.


thrombin generation. Protein C defi-
TABLE 6-4 Secondary/Acquired
Hypercoagulable ciency can be either genetic or ac-
Statesa quired. Heterozygosity occurs in 0.2%
to 0.5% of the general population.
b Antiphospholipid syndrome Acquired protein C deficiency is asso-
b Protein S deficiency ciated with severe infection (especially
b Protein C deficiency meningococcemia), liver disease, dis-
b Antithrombin deficiency
seminated intravascular coagulation,
acute respiratory distress syndrome,
b Myeloproliferative disorders
sepsis, and several medications, in-
b Paraproteinemia
cluding methotrexate, 5-fluorouracil,
b Cancer
and cyclophosphamide. A convincing
b Pregnancy link to arterial thrombosis has not
b Oral contraceptives been demonstrated.
b Polycythemia rubra vera
b Iron deficiency anemia Antithrombin Deficiency
b Hemoglobinopathies Antithrombin (formerly called anti-
b Sickle cell disease thrombin III) is an inhibitor of thrombin.
b Thalassemia Antithrombin deficiency can be both
b Platelet disorders genetic and acquired. The inheritance
b Essential thrombocythemia pattern is usually autosomal dominant.
b HeparinYinduced
The prevalence in the general popula-
thrombocytopenia tion may be 1 in 500. Acquired
b Thrombotic thrombocytopenic antithrombin deficiency is associated
purpura with disseminated intravascular coagula-
b Homocystinuria tion, sepsis, liver disease, nephrotic
b Hyperhomocysteinemia syndrome, oral contraceptives, estrogen,
b Paroxysmal nocturnal
and asparaginase. Patients with anti-
hemoglobinuria thrombin deficiency appear to be at no
b Diabetes mellitus increased risk for arterial thrombosis.
b Ovarian hyperstimulation
syndrome
Antiphospholipid Antibody
Syndrome
b Nephrotic syndrome
b Disseminated intravascular
The antiphospholipid antibody syndrome
coagulation (APS) is an autoimmune condition
b Plasma glutathione peroxidase associated with both venous and arterial
deficiency thrombosis. APS often but not always
a
Modified with permission from Biller J, ed, occurs with SLE. The most common
Saunders.29 B 2009 Elsevier. thrombotic event in patients with APS is
DVT; stroke is the most common
intravascular coagulation, HIV, nephrotic arterial event. The diagnostic criteria
syndrome, systemic lupus erythematosus for APS are found in Table 6-537. Any
(SLE), liver disease, and asparaginase patient with rheumatologic disease and
therapy. Protein S deficiency has not been stroke should be screened for APS; it
convincingly linked to arterial thrombosis. may also be reasonable to screen any
patient with stroke younger than
Protein C Deficiency age 45 for APS.38 Treatment generally
Activated protein C inactivates factors consists of long-term anticoagulation.
Va and VIIIa, which are important for Case 6-3 is an example of SLE-associated

Continuum (Minneap Minn) 2014;20(2):352–369 www.ContinuumJournal.com 361


Copyright © American Academy of Neurology. Unauthorized reproduction of this article is prohibited.
Stroke in Young Adults

Case 6-2
A 42-year-old woman without a significant prior medical history presented
with a 2-day history of difficulty seeing to the left. She did not take oral
contraceptives, smoke, or use illicit drugs. She had one previous
miscarriage but no history of deep venous thromboses (DVTs), pulmonary
emboli, or other bleeding or clotting issues. She did, however, have a
strong family history of factor V Leiden mutation in her mother and
brother, and her maternal grandmother died of blood clots. Neurologic
examination demonstrated a left homonymous hemianopia. Brain MRI
demonstrated restricted diffusion in the right occipital lobe. CT
angiography of the head and neck was unremarkable. Transesophageal
echocardiography demonstrated a small right-to-left shunt via foramen
ovale on the saline contrast study, and an atrial septal aneurysm. She was
heterozygous for the factor V Leiden mutation. The remainder of her
hypercoagulable workup was unremarkable. Lower extremity Doppler
ultrasound demonstrated no DVT in the legs. Magnetic resonance
venography of the pelvis demonstrated DVT in the left common iliac vein
due to May-Thurner syndrome. She was initially treated with aspirin 325 mg/d
and then transitioned
to anticoagulation with
warfarin.
Comment. This case
emphasizes the
importance of a
methodical approach in
cryptogenic stroke. A
strong family history
prompted an
investigation into
thrombophilia,
transesophageal
echocardiography
demonstrated a shunt,
and further evaluation
yielded a venous source.
May-Thurner syndrome
(iliocaval compression)
(Figure 6-235) is an
important consideration FIGURE 6-2 Anatomic diagram of May-Thurner syndrome
in the setting of with the right common iliac artery overlying
the left common iliac vein. Iliocaval compression
cryptogenic stroke, can lead to thrombus formation.
thrombophilia, and 35
Reprinted with permission from Kiernan TJ, et al, Stroke.
patent foramen ovale. B 2009 American Heart Association. stroke.ahajournals.
org/content/40/4/1502.long.

APS manifesting with multiple posterior antibody syndrome and is characterized


circulation strokes. by three or more new organ thromboses
Catastrophic antiphospholipid anti- within 1 week. Definitive diagnosis re-
body syndrome occurs in a small subset quires demonstration of microthrombus
of patients with antiphospholipid on biopsy.39 Treatment of catastrophic

362 www.ContinuumJournal.com April 2014

Copyright © American Academy of Neurology. Unauthorized reproduction of this article is prohibited.


TABLE 6-5 Revised Classification Criteria for the Antiphospholipid
Syndromea

Clinical Criteria
1. Vascular thrombosis
One or more clinical episodes of arterial, venous, or small vessel thrombosis in
any tissue or organ. Thrombosis must be confirmed via imaging, Doppler studies,
or histopathology, with the exception of superficial venous thrombosis.
2. Pregnancy morbidity
a. One or more unexplained deaths of a morphologically normal fetus at or
beyond the 10th week of gestation with normal fetal morphology
documented by ultrasound or examination, or
b. One or more premature births of a morphologically normal neonate
before the 34th week of gestation because of preeclampsia or severe
placental insufficiency, or
c. Three or more unexplained consecutive spontaneous abortions before the
10th week of gestation with maternal anatomic or hormonal abnormalities
and exclusion of paternal and maternal chromosomal causes.
Laboratory Criteria
1. Anticardiolipin antibody of IgG and/or IgM isotype and measured by a
standardized enzyme-linked immunoabsorbent assay or antiY"2-glycoprotein 1 of
IgG and/or IgM isotype in blood, present in medium or high titer, on two or more
occasions 12 weeks or more apart.
2. Lupus anticoagulant present in plasma on two or more occasions 12 weeks
or more apart and detected according to the guidelines of the International
Society on Thrombosis and Haemostasis in the following steps:
a. Demonstration of a prolonged phospholipid-dependent coagulation
screening test, eg, activated partial thromboplastin time, Kaolin clotting
time, dilute Russell viper venom time, dilute Prothrombin time, Textarin time.
b. Failure to correct the prolonged screening test by mixing with normal
platelet-poor plasma.
c. Shortening or correction of the prolonged screening test by the addition of excess
phospholipid.
d. Exclusion of other coagulopathies as appropriate, eg, factor VIII inhibitor, heparin.
IgG = immunoglobulin G; IgM = immunoglobulin M.
a
Modified from Miyakis S, et al, J Thromb Haemost.37 B 2006, John Wiley & Sons, Inc. onlinelibrary.wiley.com/
doi/10.1111/j.1538-7836.2006.01753.x/full.

antiphospholipid antibody syndrome reliability of nongenetic tests for protein


includes treatment of any underlying S, protein C, and antithrombin deficiency
predisposing condition, heparin, and depends on the clinical scenario. Defin-
steroids, with the addition of plasma itive diagnosis can be challenging given
exchange if warranted.40 testing limitations, timing considerations
in the acute phase, and potential
Diagnostic Approach confounding due to medications (espe-
Factor V Leiden diagnostic testing in- cially anticoagulants). For protein S defi-
cludes functional activated protein C ciency, both free and total protein S
assays or direct mutation testing via antigen and functional protein S assays
PCR. Prothrombin mutation is diag- are commonly used. The diagnosis of
nosed via G20210A PCR testing. The protein C deficiency is made somewhat

Continuum (Minneap Minn) 2014;20(2):352–369 www.ContinuumJournal.com 363


Copyright © American Academy of Neurology. Unauthorized reproduction of this article is prohibited.
Stroke in Young Adults

KEY POINT
h Studies have not
demonstrated a
Case 6-3
A 24-year-old woman with systemic lupus erythematosus (SLE) and a
convincing association
history of migraine headaches presented with headache and difficulty
between hypercoagulable
walking. Neurologic examination was remarkable for somnolence,
states and ischemic
dysarthria, and a central right facial droop. No focal weakness or dystaxia
stroke.
was noted. Brain MRI demonstrated acute infarcts in the bilateral cerebellar
hemispheres, the bilateral pons, and the left thalamus. CT angiogram of the
head and neck demonstrated diminished flow in the right vertebral artery
and occlusion or near occlusion of the distal basilar artery but no dissection.
Transesophageal echocardiography showed no left atrial appendage
abnormality, valvular abnormality, or right-to-left shunt. Anticardiolipin
and antiY"2-glycoprotein titers were markedly elevated. Antiphospholipid
antibody syndrome was suspected. She was started by a rheumatologist on
steroids, rituximab, and an antiplatelet agent (aspirin 325 mg/d) initially
and was subsequently transitioned to anticoagulation (warfarin).
Follow-up studies 4 months after the initial event confirmed the diagnosis
of antiphospholipid antibody syndrome.
Comment. The presumed mechanism of stroke in this young woman was
antiphospholipid antibody syndrome associated with SLE. Antiphospholipid
antibody syndrome can affect both the venous and arterial circulations,
and the most common arterial manifestation is ischemic stroke.

difficult by the wide range of normal thrombophilia is associated with an


values in the population. Normal pro- increased risk for ischemic stroke. An
tein C antigen ranges from 70% to extensive review43 identified six case-
140% of normal; levels less than 55% control studies of 50 or more ischemic
are likely due to a genetic abnormality. stroke cases with protein S deficiency,
Protein C functional levels are the protein C deficiency, and/or antithrom-
preferred screening test. Antithrombin bin deficiency. In four of the six studies,
deficiency is diagnosed using the the proportion of cases with the
antithrombin-heparin cofactor assay. deficiency was greater than that of
A reasonable approach for protein S, the controls, but this difference was
protein C, and antithrombin deficiency not statistically significant in any of
testing would be to repeat studies a few the studies (Table 6-6). One of these
months after the stroke (and with the studies also stratified by presumed
patient off anticoagulants for at least 2 mechanism and inherited thrombophilia
weeks) before committing the patient to and did not find significant differences.44
long-term anticoagulation. Interpretation This review also identified 16 case-
can be challenging. One study found control studies of 50 or more cases of
that approximately 30% of testing for factor V Leiden or prothrombin muta-
hypercoagulable states in routine aca- tion. Fourteen of these studies found
demic practice was performed in set- no statistical difference between
tings in which the results were not the cases and controls with regard
interpretable.41 Another study found to factor V Leiden or prothrombin
that nearly 75% of the diagnosed mutation (Table 6-7). The authors of
thrombophilias were subsequently not that review also reviewed nine case-
confirmed on follow-up testing.42 control studies (without limiting the
Even if a thrombophilia is reliably number of cases) that evaluated the
identified, it is unclear whether the relationship between PFO, ischemic
364 www.ContinuumJournal.com April 2014

Copyright © American Academy of Neurology. Unauthorized reproduction of this article is prohibited.


TABLE 6-6 Case-Control Studies of Protein C Deficiency, Protein S Deficiency, and Antithrombin
Deficiency in Patients With Ischemic Strokea

Number of
Cases and Thrombophilia % Identified % Identified Statistical
Reference Age Controls Tested in Cases in Controls Significance
Sastry et al, 16Y39 101 101 PC, PS, AT 7.9 8.9 NS
2006
Jerrard-Dunne e65 130 130 PC, PS, AT 8.5b 6.3b NS
et al, 2003
Hankey et al, Mean 219 205 PC, PS, AT 7.3 6.8 NS
2001 66
Maragaglione 3Y50 202 1036 PC, PS, AT 0 0 NS
et al, 1999
De Stefano 2Y50 72 198 PC, PS, AT 1.4 0 NS
et al, 1998
Mayer et al, 939 94 94 PS 21 20 NS
1993
PC = protein C deficiency; PS = protein S deficiency; AT = antithrombin deficiency; NS = not significant.
a
Reprinted with permission from Morris et al, Stroke.43 B 2010 American Heart Association. stroke.ahajournals.org/content/41/12/2985.long.
b
Using ethnicity-specific reference ranges.

stroke, and inherited thrombophilias. antiphospholipid antibody syndrome) KEY POINT


None of the studies that evaluated seems low yield. Young adults with h Young adults with
stroke and a right-to-left
protein C, protein S, or antithrombin stroke and a right-to-left shunt should
shunt should be
deficiency found an association with be evaluated for deep venous thrombo-
evaluated for deep
PFO and stroke. The studies were mixed sis with compression ultrasonography of venous thrombosis
with regard to factor V Leiden and the lower extremities and venography of with compression
prothrombin mutation, with a few dem- the pelvis. ultrasonography of the
onstrating a significant association.43 lower extremities and
Most experts recommend a tailored venography of the pelvis.
approach to testing for hypercoagulable Treatment
states, in which high-risk individuals The best treatment regimen for most
(ie, younger, undetermined cause of hypercoagulable states is unknown, and
stroke, history of venous thrombosis, few large studies to direct treatment
multiple miscarriages, frequent throm- decisions are available. One exception
botic recurrences, family history of is the Antiphospholipid Antibodies
venous thromboembolic disease, etc) in Stroke Study/Warfarin-Aspirin Recur-
are selected for testing.45 Testing for rent Stroke Study collaboration, which
factor V Leiden and prothrombin compared warfarin and aspirin for
G20210A in nonwhite populations is prevention of recurrent stroke in pa-
likely to be very low-yield, given the tients with antiphospholipid antibodies
very low prevalence of the mutations in and found no statistical difference in
these populations. A suggested ap- outcomes.46
proach to testing for thrombophilic Many clinicians use long-term anti-
states is shown in Table 6-8. In the coagulation for patients with inherited
absence of a right-to-left shunt, testing coagulation disorders. The optimal dura-
for thrombophilias (other than possibly tion of treatment is unclear. Whether the

Continuum (Minneap Minn) 2014;20(2):352–369 www.ContinuumJournal.com 365


Copyright © American Academy of Neurology. Unauthorized reproduction of this article is prohibited.
Stroke in Young Adults

TABLE 6-7 Case Control Studies of Factor V Leiden and Prothrombin Mutation in Young Patients
(Younger Than 60 Years) With Ischemic Strokea

Number of
Cases and Thrombophilia % Identified % Identified Statistical
Reference Age Controls Tested in Cases in Controls Significance
Sastry et al, 16Y39 101 FVL 4.0 7.9 NS
2006 101 PTM 2.0 0.0 NS
Slooter et al, 20Y49 193 FVL 7.8 5.5 OR 1.8
2005b 767 PTM 2.7 2.4 OR 1.0
Lalouschek et al, G60 468 FVL 6.2 6.4 NS
2005 468 PTM 5.0 2.0 P=.032
Madonna et al, 0.5Y50 132 FVL 5.3 6.5 NS
2002 162 PTM 7.6 6.1 NS
Austin et al, 18Y50 67 FVL 7.4 5.1 NS
2002 79 PTM 4.4 2.5 NS
Lopaciuk et al, e45 100 FVL 3.0 4.2 NS
2001 283 PTM 2.0 2.1 NS
Voetsch et al, 15Y45 153 FVL 3.3 3.6 NS
2000 225 PTM 4.6 2.2 NS
Maragaglione et al, 3Y50 202 FVL 19.6 4.2 PG.0001
1999 1036 PTM 5.0 4.2 NS
De Stefano et al, 2Y50 72 FVL 5.5 2.5 NS
1998 198 PTM 12.5 2.5 P=.0001
Nabavi et al, 15Y45 225 FVL 8.4 6.0 NS
1998 200
Longstreth et al, 18Y44 106 FVL 0.9 4.1 NS
1998b 391 PTM 1.9 1.6 NS
Bentolila et al, 18Y49 125 FVL 6.7 5.9 NS
1997 134 PTM 6.4 3.7 NS
Martinelli et al, 43 T 13 155 FVL 3.2 1.3 NS
1997 155 PTM 3.8 3.2 NS
Sanchez et al, 6Y52 66 FVL 4.5 4.5 NS
1997 66
Landi et al, 5Y44 95 FVL 4.2 1.6 NS
1996 190
Kontula et al, G60 236 FVL 4.5 2.9 NS
1995 137
FVL = Factor V Leiden; PTM = prothrombin mutation; NS = not significant; OR = odds ratio.
a
Reprinted with permission from Morris et al, Stroke.43 B 2010 American Heart Association. stroke.ahajournals.org/content/41/12/2985.long.
b
Only women were included in this study.

cost-benefit ratio is favorable over the thrombophilia. Smoking cessation is


long term for ischemic stroke patients essential.
identified with thrombophilias is also
unclear. For acquired thrombophilias, SUMMARY
therapy is targeted toward the specific The incidence of ischemic stroke in
predisposing condition. Many experts young adults is increasing. The list of
recommend discontinuing oral contra- potential stroke etiologies is extensive,
ceptives and hormones in patients with and a thorough history, examination,

366 www.ContinuumJournal.com April 2014

Copyright © American Academy of Neurology. Unauthorized reproduction of this article is prohibited.


TABLE 6-8 Thrombophilia Screening Panel

b Factor V Leiden mutation PCR or activated protein C resistance (low yield in


nonwhite populations)
b Prothrombin G20210A mutation (low yield in nonwhite populations)
b Protein S functional, protein S antigenVtotal and free
b Protein C functional
b Antithrombin: antithrombin-heparin cofactor assay
b Anticardiolipin antibodies (IgG and IgM)
b Lupus anticoagulant (dilute Russell viper venom time, kaolin, etc)
b "2-Glycoprotein 1 antibodies (IgG and IgM)
b Additional considerations
Methylenetetrahydrofolate reductase (NAD[P]H) (MTHFR) mutation
Homocysteine level
Factor VIII level
Fibrinogen testing

and tailored workup are essential. Cer- dissection: a population-based study.


Neurology 2006;67(10):1809Y1812.
vical artery dissection, PFO, and
thrombophilias are potential causes of 8. Debette S, Leys D. Cervical-artery dissections:
predisposing factors, diagnosis, and outcome.
stroke in young adults. Lancet Neurol 2009;8(7):668Y678.
9. von Babo M, De Marchis GM, Sarikaya H,
et al. Differences and similarities between
REFERENCES
spontaneous dissections of the internal
1. Kissela BM, Khoury JC, Alwell K, et al. Age
carotid artery and the vertebral artery.
at stroke: temporal trends in stroke
Stroke 2013;44(6):1537Y1542.
incidence in a large, biracial population.
Neurology 2012;79(17):1781Y1787. 10. Debette S, Grond-Ginsbach C, Bodenant M,
et al. Differential features of carotid and
2. Rutten-Jacobs LC, Arntz RM, Maaijwee NA,
vertebral artery dissections: the CADISP
et al. Long-term mortality after stroke
study. Neurology 2011;77(12):1174Y1181.
among adults aged 18 to 50 years. JAMA
2013;309(11):1136Y1144. 11. Schievink WI. Spontaneous dissection of
3. Putaala J, Metso AJ, Metso TM, et al. Analysis of the carotid and vertebral arteries. N Engl J
1008 consecutive patients aged 15 to 49 with Med 2001;344(12):898Y906.
first-ever ischemic stroke: the Helsinki young 12. Völker W, Dittrich R, Grewe S, et al. The
stroke registry. Stroke 2009;40(4):1195Y1203. outer arterial wall layers are primarily
4. Ji R, Schwamm LH, Pervez MA, Singhal AB. affected in spontaneous cervical artery
Ischemic stroke and transient ischemic attack dissection. Neurology 2011;76(17):
in young adults: risk factors, diagnostic 1463Y1471.
yield, neuroimaging, and thrombolysis. 13. Engelter ST, Grond-Ginsbach C, Metso TM,
JAMA Neurol 2013;70(1):51Y57. et al. Cervical artery dissection: trauma and
5. Caplan LR, ed. Uncommon causes of stroke. other potential mechanical trigger events.
2nd edition. Cambridge, UK: Cambridge Neurology 2013;80(21):1950Y1957.
University Press, 2008. 14. Baumgartner RW, Arnold M, Baumgartner I,
6. Giroud M, Fayolle H, André N, et al. et al. Carotid dissection with and without
Incidence of internal carotid artery dissection ischemic events: local symptoms and cerebral
in the community of Dijon. J Neurol Neurosurg artery findings. Neurology 2001;57(5):827Y832.
Psychiatry 1994;57(11):1443.
15. Vertinsky AT, Schwartz NE, Fischbein NJ,
7. Lee VH, Brown RD, Mandrekar JN, Mokri B. et al. Comparison of multidetector CT
Incidence and outcome of cervical artery angiography and MR imaging of cervical

Continuum (Minneap Minn) 2014;20(2):352–369 www.ContinuumJournal.com 367


Copyright © American Academy of Neurology. Unauthorized reproduction of this article is prohibited.
Stroke in Young Adults

artery dissection. AJNR Am J Neuroradiol 27. Carroll JD, Saver JL, Thaler DE, et al. Closure
2008;29(9):1753Y1760. of patent foramen ovale versus medical
therapy after cryptogenic stroke. N Engl J
16. Bachmann R, Nassenstein I, Kooijman H,
Med 2013;368(12):1092Y1100.
et al. High-resolution magnetic resonance
imaging (MRI) at 3.0 Tesla in the short-term 28. Kitsios GD, Thaler DE, Kent DM. Potentially
follow-up of patients with proven cervical large yet uncertain benefits: a meta-analysis
artery dissection. Invest Radiol 2007; of patent foramen ovale closure trials.
42(6):460Y466. Stroke 2013;44(9):2640Y2643.
17. Zinkstok SM, Vergouwen MD, Engelter ST, 29. Biller J, ed. Stroke in children and young
et al. Safety and functional outcome of adults. 2nd edition. Philadelphia, PA:
thrombolysis in dissection-related ischemic Saunders, 2009.
stroke: a meta-analysis of individual patient
data. Stroke 2011;42(9):2515Y2520. 30. Ridker PM, Miletich JP, Hennekens CH,
Buring JE. Ethnic distribution of factor V
18. Engelter ST, Dallongeville J, Kloss M, et al. Leiden in 4047 men and women. Implications
Thrombolysis in cervical artery for venous thromboembolism screening.
dissectionVdata from the Cervical Artery JAMA 1997;277(16):1305Y1307.
Dissection and Ischaemic Stroke Patients
(CADISP) database. Eur J Neurol 31. Rosendaal FR, Koster T, Vandenbroucke JP,
2012;19(9):1199Y1206. Reitsma PH. High risk of thrombosis in
patients homozygous for factor V Leiden
19. Georgiadis D, Arnold M, von Buedingen HC, (activated protein C resistance). Blood
et al. Aspirin vs anticoagulation in carotid 1995;85(6):1504Y1508.
artery dissection: a study of 298 patients.
Neurology 2009;72(21):1810Y1815. 32. Koeleman BP, Reitsma PH, Allaart CF,
Bertina RM. Activated protein C resistance
20. Cervical Artery Dissection in Stroke Study as an additional risk factor for thrombosis in
Trial Investigators. Antiplatelet therapy vs. protein C-deficient families. Blood 1994;84(4):
anticoagulation in cervical artery dissection: 1031Y1035.
rationale and design of the Cervical Artery
Dissection in Stroke Study (CADISS). Int J 33. Zöller B, Berntsdotter A, Garcı́a de Frutos P,
Stroke 2007;2(4):292Y296. Dahlbäck B. Resistance to activated protein
C as an additional genetic risk factor in
21. Kennedy F, Lanfranconi S, Hicks C, et al. hereditary deficiency of protein S. Blood
Antiplatelets vs anticoagulation for dissection: 1995;85(12):3518Y3523.
CADISS nonrandomized arm and
meta-analysis. Neurology 2012;79(7):686Y689. 34. van Boven HH, Reitsma PH, Rosendaal FR,
et al. Factor V Leiden (FV R506Q) in
22. Messé SR, Silverman IE, Kizer JR, et al.
families with inherited antithrombin
Practice parameter: recurrent stroke with deficiency. Thromb Haemost 1996;75(3):
patent foramen ovale and atrial septal 417Y421.
aneurysm: report of the Quality Standards
Subcommittee of the American Academy 35. Kiernan JT, Yan BP, Cubeddu RJ, et al.
of Neurology. Neurology 2004;62(7): May-Thurner syndrome in patients with
1042Y1050. cryptogenic stroke and patent foramen
ovale: an important clinical association.
23. Larrue V, Berhoune N, Massabuau P, et al.
Stroke 2009;40(4):1502Y1504.
Etiologic investigation of ischemic stroke in
young adults. Neurology 2011;76(23): 36. Rosendaal FR, Doggen CJ, Zivelin A, et al.
1983Y1988. Geographic distribution of the 20210 G to
A prothrombin variant. Thromb Haemost
24. Kitsios GD, Dahabreh IJ, Abu Dabrh AM,
1998;79(4):706Y708.
et al. Patent foramen ovale closure and
medical treatments for secondary stroke 37. Miyakis S, Lockshin MD, Atsumi T, et al.
prevention: a systematic review of International consensus statement on an
observational and randomized evidence. update of the classification criteria for
Stroke 2012;43(2):422Y431. definite antiphospholipid. J Thromb
Haemost 2006;4(2):295Y306.
25. Furlan AJ, Reisman M, Massaro J, et al.
Closure or medical therapy for cryptogenic 38. Brey R. Neurologic manifestations of
stroke with patent foramen ovale. N Engl J systemic lupus erythematosus and
Med 2012;366(11):991Y999. antiphospholipid antibody syndrome.
Continuum (Minneap Minn) 2008;14(1):
26. Meier B, Kalesan B, Mattle HP, et al.
94Y119.
Percutaneous closure of patent foramen
ovale in cryptogenic embolism. N Engl J Med 39. Erkan D, Espinosa G, Cervera R. Catastrophic
2013;368(12):1083Y1091. antiphospholipid syndrome: updated

368 www.ContinuumJournal.com April 2014

Copyright © American Academy of Neurology. Unauthorized reproduction of this article is prohibited.


diagnostic algorithms. Autoimmun Rev 43. Morris JG, Singh S, Fisher M. Testing for
2010;10(2):74Y79. inherited thrombophilias in arterial stroke:
can it cause more harm than good? Stroke
40. Lockshin MD, Erkan D. Treatment of the
2010;41(12):2985Y2990.
antiphospholipid syndrome. N Engl J Med
2003;349(12):1177Y1179. 44. Hankey GJ, Eikelboom JW, van Bockxmeer FM,
et al. Inherited thrombophilia in ischemic
41. Bushnell C, Siddiqi Z, Morgenlander JC, stroke and its pathogenic subtypes. Stroke
Goldstein LB. Use of specialized coagulation
2001;32(8):1793Y1799.
testing in the evaluation of patients with
acute ischemic stroke. Neurology 2001; 45. Seligsohn U, Lubetsky A. Genetic susceptibility
56(5):624Y627. to venous thrombosis. N Engl J Med 2001;
344(16):1222Y1231.
42. Bushnell CD, Siddiqi Z, Goldstein LB.
Improving patient selection for 46. Levine SR, Brey RL, Tilley BC, et al.
coagulopathy testing in the setting of Antiphospholipid antibodies and subsequent
acute ischemic stroke. Neurology 2001;57(7): thrombo-occlusive events in patients with
1333Y1335. ischemic stroke. JAMA 2004;291(5):576Y584.

Continuum (Minneap Minn) 2014;20(2):352–369 www.ContinuumJournal.com 369


Copyright © American Academy of Neurology. Unauthorized reproduction of this article is prohibited.
Review Article

Pediatric Arterial
Address correspondence to
Dr Mahendra Moharir, Division
of Neurology, The Hospital for
Sick Children, 555 University
Avenue, Toronto, Ontario M5G
1X8, Canada,
mahendranath.moharir@
Ischemic Stroke
sickkids.ca.
Mahendra Moharir, MD, MSc, FRACP; Gabrielle deVeber, MD
Relationship Disclosure:
Dr Moharir reports no disclosure.
Dr deVeber’s spouse holds an
investment of more than $10,000 ABSTRACT
in Thornhill Research Inc.
Unlabeled Use of
Purpose of Review: This article aims to provide a broad overview of pediatric
Products/Investigational arterial ischemic stroke, from recognition and diagnosis to the short-term and long-term
Use Disclosure: management based on current available literature.
Drs Moharir and deVeber
report no disclosures.
Recent Findings: Arterial ischemic stroke in children represents a significant disorder
* 2014, American Academy with a concerning high rate of adverse outcomes, including potentially preventable
of Neurology. recurrent stroke. Although awareness of pediatric stroke is increasing, diagnosis is still
commonly delayed or missed altogether, particularly in younger children. Current
vascular imaging techniques have limitations in accurate diagnosis of arteriopathies that
are now recognized as an important cause of childhood stroke. Significant variability
exists in treatment of pediatric stroke. Management is based on published consensus
guidelines; however, individual children require an individualized approach.
Summary: As pediatric stroke specialists become increasingly available, the collabo-
ration of such experts on individual management is crucial. Definitive evidence-based
treatment for pediatric stroke awaits the development of randomized controlled trials.

Continuum (Minneap Minn) 2014;20(2):370–386.

INTRODUCTION of the neurologic systems differ be-


Awareness about childhood stroke is tween children and adults. Third, risk
increasing. Stroke is among the top 10 factors for pediatric AIS are frequently
causes of death and a significant cause multiple, age-dependent across child-
of long-term morbidity in childhood.1 hood, and distinct from those in adults,
Stroke is traditionally classified into necessitating a complex set of etiologic
ischemic and hemorrhagic subtypes. investigations. Fourth, there is still a
This review focuses on pediatric arte- paucity of robust data on the rates of
rial ischemic stroke (AIS) only. outcomes in children, including stroke
AIS occurs with a frequency of 2 to volume, recurrent stroke, and chronic
3/100,000 children per year.2 Many cases disability. Consequently, evidence
occur during the perinatal or neonatal supporting specific treatments is lack-
period, during which the incidence is ing, leading to variability in manage-
approximately 1 in 4000 live births.3 ment. Consensus-based pediatric stroke
Boys are affected more frequently.4 treatment guidelines differ from each
Pediatric AIS differs from adult AIS in other in some aspectsVfor example,
several ways. First, AIS is rarer and has a initial aspirin versus anticoagulation for
subtler clinical presentation and a wider noncardiac stroke.6,7 Finally, the burden
differential diagnosis in children, which of pediatric AIS is longer lasting because
results in delayed and sometimes a child who survives AIS is expected to
missed diagnosis.5 Second, the coagula- live much longer than an adult stroke
tion, vascular, and adaptive components survivor.
370 www.ContinuumJournal.com April 2014

Copyright © American Academy of Neurology. Unauthorized reproduction of this article is prohibited.


KEY POINTS
RECOGNITION OF ARTERIAL mode of presentation seen in children h Arterial ischemic stroke
ISCHEMIC STROKE is a stuttering/fluctuating course of is rarer and has a subtler
Recognition of AIS in neonates and neurologic deficits.10 This should alert clinical presentation and
infants is challenging because acute the clinician to the possibility of an a wider differential
focal neurologic deficits, typically evi- underlying vasculopathy involving diagnosis in children
dent in older children and adults, are large vessels of the circle of Willis. than in adults, which
uncommon. Neonates usually present Recurrent TIAs also raise the possibil- results in delayed and
with seizures and other nonspecific ity of a vasculopathy or a thromboem- sometimes missed
symptoms.8 AIS should be suspected bolic source. A feature unique to diagnosis.
in any newborn with seizures, particu- children is a history of paroxysmal h Arterial ischemic stroke
larly when seizures begin more than episodes of focal neurologic dysfunc- should be suspected in
12 hours after birth.9 Neonates may tion precipitated by hyperventilation, any newborn with
also present with encephalopathy (ir- which should raise the suspicion of seizures, particularly
ritability, lethargy, and poor feeding) moyamoya. Exercise, strenuous bursts when seizures begin
more than 12 hours
or recurrent apnea. Focal deficits of physical activity, or excessive crying
after birth.
occur in less than 25% of all newborns are the usual causes for hyperventila-
with AIS. Sometimes acute neonatal tion. The mechanism underlying this h A feature of TIAs unique
AIS is not diagnosed until later infancy phenomenon is ischemia secondary to children is a history of
paroxysmal episodes of
or childhood when hemiparesis be- to cerebral vasoconstriction produced
neurologic dysfunction
comes gradually evident with matura- by carbon dioxide reduction during
precipitated by
tion. In such children, imaging shows hyperventilation. hyperventilation, which
remote AIS, which is presumed to have Based on the patient’s age, aware- should raise the
occurred in the prenatal or perinatal ness of stroke mimics such as focal suspicion of moyamoya.
period and is called ‘‘presumed peri- seizures, hemiplegic migraine, demy-
natal ischemic stroke.’’ The typical elination, tumor with hemorrhage,
history includes motor delay, early hypoglycemia, and conversion dis-
hand dominance (before the first birth- order is important in any child with
day), and focal seizures evolving into a acute focal deficits.11 Children with
classical hemiplegic cerebral palsy. progressive vasculopathies such as
With increasing age, acute AIS man- moyamoya or medium/small vessel
ifests with a clinical presentation sim- CNS vasculitis can present with pro-
ilar to that in adults, namely abrupt gressive neurocognitive decline due to
onset of focal neurologic deficits. subtle recurrent AIS events.
However, pediatric health care pro-
viders are still not always attuned to CLINICAL AND HISTORICAL
the possibility of stroke in childhood, SEARCH FOR ETIOLOGY AND
and delayed diagnosis is a conse- RISK FACTORS
quence. The most common mani- In addition to the history pertaining to
festation is hemiparesis; however, the acute presentation, a meticulous
depending on the vascular territory history of risk factors should also be
involved, aphasia, ataxia, dysarthria, obtained.12 Risk factor identification
visual deficits, and cranial nerve informs three key management ques-
palsies can occur. Seizures remain an tions regarding antithrombotic therapy
important manifestation even in older strategies: (1) Are any predisposing
children. Some children report head- risk factors present? (2) Does the
ache. A sudden-onset thunderclap patient have an acute trigger for
headache or neck, shoulder, or occip- thrombosis? (3) What is the stroke
ital pain should arouse suspicion of recurrence risk in the immediate and
cervical artery dissection. A typical distant future?
Continuum (Minneap Minn) 2014;20(2):370–386 www.ContinuumJournal.com 371
Copyright © American Academy of Neurology. Unauthorized reproduction of this article is prohibited.
Pediatric Arterial Ischemic Stroke

In neonatal AIS, details of maternal, older children, recurrence risk ranges


antenatal, perinatal, intrapartum, and from 10% to 60% and is determined by
immediate postnatal history are criti- the etiology and mechanism of stroke.15
cal. Risk factors associated with AIS Some children (10% to 15%) never
include maternal (gestational diabetes have a clear cause identified despite
mellitus, hypertension), perinatal (peri- exhaustive investigations.
natal depression, perinatal asphyxia,
complicated delivery, instrumentation), CONFIRMATION OF DIAGNOSIS:
and neonatal (congenital heart disease, NEUROIMAGING
dehydration, and infection). In the Initial imaging by CT frequently (in 40%
absence of any risk factors, a placental to 80% of cases) misses evidence that
source of thrombosis may be consid- would allow for AIS diagnosis.16 Hence,
ered. Placental vasculopathy may be an MRI of the brain and magnetic resonance
important but overlooked cause of angiography (MRA) of the intracranial
perinatal AIS.13 Placental examination (and ideally neck) arteries is indicated for
can be helpful and can demonstrate any child with suspected acute AIS.
placental infarction or infection, which Perfusion-weighted imaging, which may
predispose the patient to neonatal stroke. be useful in determining the ischemic
In older children with AIS, infection, penumbra in adults, has not been sys-
dehydration, trauma, head/neck ma- tematically studied in children. MRA has
nipulation, congenital heart disease, technical limitations in newborns that
systemic diseases (hepatic, renal, gas- may result in undercalling or overcalling
trointestinal, hematologic, and rheu- of vascular abnormalities. Sedation re-
matologic conditions), iron deficiency mains a major barrier to acute MRI in
anemia, and medication ingestion or young or noncooperative children,
substance abuse must be ruled out. delaying the diagnosis. Collaborative insti-
Dysmorphic features, neurocutaneous tutional protocols with neuroradiology
markers, and connective tissue disor- and anesthesia for emergent MRI under
ders may suggest syndromic genetic sedation can be helpful. CT remains an
vasculopathy. option in those who cannot get an MRI
Risk factors common to both neo- even though radiation exposure is an
nates and older children include con- issue. CT angiography (CTA) in the same
genital abnormalities of the coagulation/ setting can rule out major circle of Willis
fibrinolytic system (protein C, protein S, abnormalities. Although MRA and CTA
antithrombin, and genetic abnormali- have increased the diagnostic yield in
ties including factor V Leiden muta- pediatric AIS, invasive catheter cerebral
tion, prothrombin gene G20210A, angiography still plays an important role
and homozygous thermolabile either when MRA or CTA is unremark-
methylenetetrahydrofolate [MTHFR] able or when MRA or CTA has detected
mutations). A family history of a vascular abnormality of unclear etiol-
thrombophilia includes myocardial in- ogy. In the authors’ experience, catheter
farction or stroke, deep vein thrombosis, cerebral angiography has been of added
pulmonary embolism, recurrent sponta- value in cervical artery dissection17 and
neous miscarriages, and any family mem- CNS medium-vessel vasculitis (unpub-
ber on ‘‘blood thinners.’’ lished observations). It has a clear role
In general, perinatal/neonatal AIS in presurgical planning of revasculariza-
tends to be a one-time event unless the tion procedures for moyamoya by de-
patient has a risk factor such as congen- monstrating existing and potential
ital heart disease or thrombophilia.14 In external carotid artery collaterals. Arterial
372 www.ContinuumJournal.com April 2014

Copyright © American Academy of Neurology. Unauthorized reproduction of this article is prohibited.


KEY POINT
wall imaging, which is increasingly used cial effect on adult stroke outcomes. h When considering
in adults for detection of active athero- The main aim of neuroprotection is to antithrombotic therapy
sclerotic plaques in intracranial and prevent recruitment of the ischemic in any child with arterial
neck arteries and for diagnosis of CNS penumbra and avoidance of malignant ischemic stroke, it is
vasculitis and cervical artery dissection,18 cerebral edema. The American Heart critical to continually
appears to have a potential role in Association (AHA) guidelines have balance the risk
arteriopathy-related pediatric AIS, where reviewed the role of neuroprotection.6 (hemorrhagic
gadolinium enhancement of the arterial Simple measures such as avoidance of complication) of
wall at the site of stenosis may suggest hyperthermia, hypotension/hypertension, antithrombotic therapy
inflammatory etiologies.19 hypovolemia/hypervolemia, and with the risk of
nontreatment
The role of cranial ultrasound in AIS hypoglycemia/hyperglycemia are sug-
(extension of preexisting
diagnosis is limited to infants with an gested. Early detection and prompt treat-
infarction and
open anterior fontanelle. It can be used ment of seizures (particularly status recurrence of stroke).
only as a broad screening tool to detect epilepticus) are recommended to prevent
large areas of ischemia. It remains a infarct expansion. The authors generally
good bedside noninvasive modality but prefer to manage acute stroke patients in
is heavily operator dependent, thereby intensive care for about 48 to 72 hours,
precluding its use as a regular tool for particularly those with higher perceived
AIS diagnosis. The role of transcranial complication risk (large middle cerebral
Doppler in sickle cell disease is well artery [MCA] or posterior fossa strokes).
established; for more on sickle cell disease
and stroke, see ‘‘Genetic Stroke Syn- Antithrombotic Therapy
dromes’’ by Drs Kevin Barrett and James Antithrombotic therapy (anticoagulant
Meschia in this issue of . or antiplatelet therapy) is applicable to
vaso-occlusive territorial thromboem-
ACUTE MANAGEMENT AFTER bolic AIS only; therefore, a determina-
CONFIRMATION OF DIAGNOSIS tion of whether the AIS pattern is
After confirmation of AIS, neuroprotection, vaso-occlusive or watershed/borderzone
specific antithrombotic therapy, and on neuroimaging is critical. Antithrom-
concurrent search for etiology and risk botic therapy might not be an option
factors during the initial phase are the for watershed infarction because hypo-
cornerstones of acute management. perfusion (and not thromboembolism)
A multidisciplinary involvement of a is the underlying mechanism. However,
variety of pediatric subspecialty teams the two can coexist within the same
including neurology, general pediatrics, child in the setting of congenital heart
neuroradiology, intensive care, throm- disease. When considering anti-
bosis/hematology, and rehabilitation is thrombotic therapy in any child with
generally essential for most stroke AIS, it is critical to continually balance
cases. Additional involvement from dis- the risk (hemorrhagic complication) of
ciplines such as cardiology, interven- antithrombotic therapy with the risk of
tional neuroradiology, neurosurgery, nontreatment (extension of preexisting
infectious diseases, and rheumatology infarction and recurrence of stroke).
can be beneficial depending upon indi- Because of the lack of clinical trials,
vidual cases. antithrombotic therapy varies consider-
ably across centers based on physician
Neuroprotection preference. The AHA6 and American
Although clear evidence-based data are College of Chest Physicians7 guide-
lacking in children, neuroprotection is lines have extensively reviewed anti-
logical based on the presumed benefi- thrombotic therapy in pediatric AIS.
Continuum (Minneap Minn) 2014;20(2):370–386 www.ContinuumJournal.com 373
Copyright © American Academy of Neurology. Unauthorized reproduction of this article is prohibited.
Pediatric Arterial Ischemic Stroke

KEY POINTS
h The risk-benefit ratio of Thrombolytic therapy. IV tissue plas- for anticoagulant and antiplatelet
tissue plasminogen minogen activator (tPA) is the standard therapy 23Y25 ; the largest nonran-
activator in pediatric of care in adults with acute AIS and domized, multicenter, observational
arterial ischemic stroke improves outcomes. In children, dose, cohort study of more than 600 children
is unknown. safety, and efficacy of tPA (IV or local was conducted via the International
h After the diagnosis of intra-arterial) in AIS have not been Pediatric Stroke Study (IPSS) group.24
acute arterial ischemic established. The risk-benefit ratio of tPA The IPSS is a growing body of pediatric
stroke, antiplatelet and in pediatric AIS is unknown. Use of IV stroke investigators that collects com-
anticoagulant therapies tPA in children for systemic thrombolysis prehensive information on pediatric
in children are usually has been associated with a higher com- stroke cases at participating centers
selected based on the plication rate than in adults,20 and across the world through standardized
perceived mechanism children with AIS rarely present within data collection forms with a central
for arterial ischemic the 4.5-hour (for IV tPA) or 6-hour (intra- database housed in Toronto. In that
stroke. arterial tPA) window. Given the fact that IPSS cohort, initial acute therapy includ-
outcome from AIS in children is more ed anticoagulant therapy (27%), aspirin
favorable21 in general than in adults, the (28%), anticoagulant therapy and aspirin
drive to initiate IV or intra-arterial tPA combination (16%), and no treatment
even in a child presenting within the (30%). In that study, subtypes associat-
appropriate time window should be ed with any use of anticoagulant therapy
carefully scrutinized. Individual case re- were dissection and congenital heart
ports that have documented successful disease. Factors associated with nonuse
use of tPA in children are not generaliz- of anticoagulant therapy included sickle
able.22 The American College of Chest cell disease and location of the enroll-
Physicians guidelines7 currently recom- ment center in the United States. Anti-
mend against the use of tPA in childhood platelet therapy use was more frequent
AIS outside of clinical trials. The AHA in moyamoya, whereas nonuse was
guidelines6 take the same stance, although more frequent in dissection (anticoagu-
consensus is lacking regarding the use of lant therapy being the preferred medi-
tPA in older adolescents who otherwise cation), altered consciousness, and
meet the adult tPA eligibility criteria. The bilateral ischemia.
paucity of data with which to make any From a practical standpoint, consen-
recommendations regarding mechanical sus exists among pediatric stroke experts
thrombectomy is even greater. that some level of antithrombotic thera-
Anticoagulant therapy and py is beneficial in preventing acute
antiplatelet therapy. After the diagno- progression of thrombus and stroke
sis of acute AIS, antiplatelet and anti- recurrence. The authors of this article
coagulant therapies in children are generally tend to initiate anticoagulant
usually selected based on the per- therapy, regardless of perceived mecha-
ceived mechanism for AIS. The ratio- nism, unless contraindications are pres-
nale of initial therapy is to limit the ent. Either unfractionated heparin
extension of occlusive thrombosis and (UFH) or low-molecular-weight heparin
early recurrent thrombotic stroke, (LMWH) without a loading dose is
whereas subsequent maintenance ther- preferred. UFH is chosen when the
apy aims to prevent longer-term recur- perceived risk of hemorrhagic complica-
rence. To date, no randomized tions is higher and a need for rapid
controlled trials of anticoagulant or anti- reversal is anticipated or when invasive
platelet therapy have been conducted procedures (catheter cerebral angiogra-
in pediatric AIS. Some cohort studies phy, lumbar puncture, etc) are planned.
have assessed safety and failure rates We perform a screening head CT 3 days
374 www.ContinuumJournal.com April 2014

Copyright © American Academy of Neurology. Unauthorized reproduction of this article is prohibited.


KEY POINT
after starting heparin to rule out subclin- tional normalized ratio, partial thrombo- h From a practical
ical significant intracranial hemorrhage; plastin time, D-dimer, electrolyte and standpoint, consensus
if this is unremarkable, UFH is switched glucose levels, and renal or liver function exists among pediatric
to LMWH. In moyamoya and sickle cell tests. Investigations that may be helpful stroke experts that some
disease, we prefer to start aspirin over in individual cases include iron profile, level of antithrombotic
anticoagulant therapy because of per- inflammatory markers (erythrocyte sedi- therapy is beneficial in
ceived higher risk of hemorrhage with mentation rate, C-reactive protein, com- preventing acute
anticoagulant therapy. After risk factor plement, von Willebrand antigen, and progression of
screening is complete, antithrombotic CSF analysis for opening pressure, cell thrombus and stroke
therapy duration and type are reviewed. count, protein, and cultures), serology for recurrence.
When either aspirin or anticoagulant infectious agents, and genetic studies.
therapy fails (failure defined as radiologic The yield and influence of proth-
or clinical TIA/stroke recurrence), the rombotic workup on stroke manage-
authors of this article have given dual ment and recurrence have been recently
therapy with aspirin and anticoagulant reviewed.27 The presence of a these
therapy in selected cases. In children disorders appears to be an important
older than 12 years of age whose predisposing cause for thrombotic
treatment with aspirin fails, the authors stroke, frequently acting in concert with
have also switched over to clopidogrel. If other triggers for the stroke. A typical
hemorrhagic complications (4% symp- thrombophilia panel includes protein C,
tomatic risk on anticoagulant therapy26) protein S, antithrombin, lupus anticoag-
occur on antithrombotic therapy, a ulant, anticardiolipin antibody, factor
careful review of the need to prevent VIII, plasma homocysteine, factor V
stroke recurrence and avoid hemor- Leiden mutation and activated protein
rhagic complications is undertaken. C resistance, prothrombin gene G20210A
mutation, MTHFR mutation, and
Concurrent Search for Risk lipoprotein-a. Interpretation of abnormal-
Factors and Etiology of Arterial ities in a prothrombotic workup is
Ischemic Stroke challenging; any abnormality on the
A thorough search for stroke risk factors prothrombotic workup except genetic
should be concurrently undertaken tests should be repeated after at least
while neuroprotection and anti- 12 weeks of initial testing and off
thrombotic therapy is being adminis- anticoagulant therapy to determine
tered. Dedicated neck vessel imaging whether the abnormality was transient
(if not performed initially) by either MRA or persistent, as the latter may influence
or CTA can be helpful. Catheter cerebral duration of antithrombotic therapy.
angiography may need consideration if
cervical artery dissection17 or CNS vascu- MANAGEMENT BASED ON
litis are suspected. In moyamoya, early ETIOLOGIES OF PEDIATRIC
catheter cerebral angiography and ARTERIAL ISCHEMIC STROKE
cerebrovascular-reactivity-functional MRI Most pediatric AIS cases fall into one of
(fMRI) may be helpful if surgical revascu- the following broad categories: perinatal/
larization is indicated urgently. All cases neonatal, cardiogenic, arteriopathic
merit cardiac echo with bubbled saline (vasculopathic), prothrombotic, and
study to rule out a cardiac source of idiopathic/cryptogenic. This categoriza-
thromboembolism. Blood work that tion is helpful in tailoring management
merits immediate completion on confir- and estimating prognosis. Strategies for
mation of stroke includes complete antithrombotic therapy are summarized
blood cell count, platelet count, interna- in Table 7-1.
Continuum (Minneap Minn) 2014;20(2):370–386 www.ContinuumJournal.com 375
Copyright © American Academy of Neurology. Unauthorized reproduction of this article is prohibited.
Pediatric Arterial Ischemic Stroke

KEY POINTS
h Antithrombotic therapy TABLE 7-1 Long-Term Antithrombotic Therapy Based on Arterial
is rarely indicated in Ischemic Stroke Etiology
most cases of perinatal
or neonatal arterial Stroke Etiology Therapy
ischemic stroke because
Perinatal/neonatal arterial No antithrombotic therapy unless
of negligible recurrence ischemic stroke congenital heart disease or
risk except in the prothrombotic abnormalities are
presence of congenital identified
heart disease and Uncorrected congenital heart disease Low-molecular-weight heparin
abnormal prothrombotic (LMWH)/warfarin or aspirin
workup.
Fully corrected congenital heart Aspirin
h Because deficits from disease
perinatal or neonatal Nonmoyamoya arteriopathy Initial LMWH followed by long-term
arterial ischemic stroke LMWH or warfarin or aspirin
may only emerge with Inherited prothrombotic disease Initial LMWH followed by long-term
maturation, longitudinal warfarin
observation is necessary Sickle cell disease Chronic transfusion program
even in apparently with aspirin
healthy infants.
Idiopathic arterial ischemic stroke Initial LMWH/aspirin followed by
with normal (no residual stenosis) long-term aspirin (for at least
vessel imaging 2 to 5 years; duration controversial)
Moyamoya disease/moyamoya Aspirin, revascularization surgery
syndrome
Postvaricella angiopathy/transient Initial LMWH/aspirin, followed by
cerebral arteriopathy/large or long-term LMWH/aspirin depending
medium vessel CNS vasculitis on residual arterial stenosis with or
without immunosuppressive agents
CNS = central nervous system.

Perinatal/Neonatal Arterial emanating from the zone of primary


Ischemic Stroke infarction and best seen in the ipsilesional
Antithrombotic therapy is rarely indi- cerebral peduncle on the acute MRI. This
cated in most cases because of negli- signal represents acute Wallerian degen-
gible recurrence risk except in the eration29 and reliably predicts future
presence of congenital heart disease28 hemiplegia (Case 7-1).30 Recently,
and abnormal prothrombotic workup. constraint-induced therapy has been
The focus of acute management is demonstrated to be effective in infants
neuroprotection. Long-term manage- and children with hemiparesis due to
ment involves rehabilitation, including AIS. Neuropsychological testing can be
physiotherapy, occupational therapy, helpful during early school age.
and speech therapy. Because deficits
may only emerge with maturation, lon- Arterial Ischemic Stroke With
gitudinal observation is necessary even Congenital Heart Disease
in apparently healthy infants. It is possi- Multiple mechanisms interact to pro-
ble to select neonates at risk of duce AIS in congenital heart disease:
hemiparesis for early interventions based thromboembolism from valvular or
on the presence of acute restricted septal defects, cardiac catheterization,
diffusion signal in the corticospinal tract valvular/septal devices, intracardiac

376 www.ContinuumJournal.com April 2014

Copyright © American Academy of Neurology. Unauthorized reproduction of this article is prohibited.


Case 7-1
A 4-day-old newborn boy presented with apneic episodes, reduced
feeding, and recurrent twitching of the right upper and lower limb
beginning on the third day of life. Antenatal history and pregnancy
were unremarkable except the mother’s history of three spontaneous
miscarriages before this pregnancy, for which no cause was found. Birth
was unremarkable without any complications or need for assisted delivery.
Apgar scores were 7 and 9 at 1 minute and 5 minutes, respectively. The
baby was well on the first 2 days, without any neurologic or systemic
concerns, but he began having seizures as described above soon after he
was discharged home on day 3. Neonatal neurologic examination did not
reveal any focal findings. MRI of the brain revealed a large left middle
cerebral artery territory stroke; magnetic resonance angiogram appeared
normal. Diffusion-weighted imaging also showed the presence of restricted
diffusion along the corticospinal tracts (Figure 7-1). Seizures were well
controlled with phenobarbital. Despite the presence of stroke, the baby
was otherwise well and nonencephalopathic. Septic and prothrombotic
workup was normal.
Placenta was available for
histopathologic examination
and showed widespread
thrombosis and infarction,
suggesting placental
vasculopathy, which was
presumed to be the cause of
the baby’s stroke. Follow-up
at 2 years of age revealed
moderate-severe right-sided
hemiplegia and mild
language and cognitive delay.
Comment. This case
highlights the lack of
identifiable focal deficits on
neurologic examination in
neonatal stroke, although in
FIGURE 7-1 Coronal diffusion-weighted
this case focal seizures were image (DWI) of the patient in
a clue. The etiology of Case 7-1 showing a neonatal
acute left middle cerebral artery arterial ischemic stroke
‘‘idiopathic’’ neonatal stroke showing acute Wallerian degeneration along the
was likely placental descending corticospinal tract from the infarcted
vasculopathy. The case also area to the midbrain (cerebral peduncle) (arrows).
illustrates well the value of
restricted diffusion in the corticospinal tract as a marker of pre-Wallerian
degeneration in acute neonatal ischemic stroke and as a predictor of future
hemiplegia.

shunts, cardiopulmonary bypass, and cardiac arrest. Children with cardiac


direct surgical procedures; polycythe- disorders may be prothrombotic as a
mia in cyanotic congenital heart dis- result of chronic illness, infection, iron
ease; and hypotension. Additional deficiency anemia, persistent low-grade
factors include endocarditis, cardio- inflammation, and multiple surgical or
myopathy, cardiac arrhythmias, and circulatory support interventions.28

Continuum (Minneap Minn) 2014;20(2):370–386 www.ContinuumJournal.com 377


Copyright © American Academy of Neurology. Unauthorized reproduction of this article is prohibited.
Pediatric Arterial Ischemic Stroke

KEY POINTS
h Although congenital Antithrombotic therapy is strongly AIS cases.34 These can occur sponta-
heart disease correction recommended for all children with neously or in association with trivial
should theoretically congenital heart disease and AIS. mechanical injury or major head or
eliminate the Anticoagulant therapy is preferred neck trauma. The intradural-extradural
cardiogenic stroke risk, until risk factors causing stroke are junction of the ICA and C1-C2 portion
recent data fully corrected. Antithrombotic therapy of the VA (Case 7-2) are common sites
demonstrate that duration and type depends on the for dissection. Stroke occurs as a
recurrence risk remains child’s age, congenital heart disease, result of artery-artery embolism from
increased for extended and procedure type. Details of anti- intimal disruption, arterial occlusion
periods even after full thrombotic therapy recommendations causing hypoperfusion, or distal ex-
congenital heart disease
in congenital heart disease are avail- tension of dissection along the artery
correction.
able.6,7 Although congenital heart dis- wall, to disrupt the origin of branch
h Beyond the newborn ease correction should theoretically arteries. Anticoagulant therapy is usu-
period, arteriopathy eliminate the cardiogenic stroke risk, ally favored over aspirin as the first
(vasculopathy) accounts
recent data demonstrate that re- line of antithrombotic therapy in most
for approximately 60%
currence risk remains increased for centers, although recent adult data
of all childhood arterial
ischemic stroke and
extended periods even after full con- have suggested no difference between
predicts stroke genital heart disease correction.28 the two approaches.35 Published guide-
recurrence. lines recommend initial anticoagulant
Arterial Ischemic Stroke With therapy for pediatric AIS with cervical
h Transient cerebral Arteriopathy
arteriopathy of
artery dissection. Anticoagulant therapy
childhood is a Beyond the newborn period, arterio- duration ranges from 6 weeks to 6
well-described unilateral pathy (vasculopathy) accounts for ap- months after diagnosis. Anticoagulant
focal arteriopathy of proximately 60% of all childhood AIS therapy is followed by aspirin for several
presumed inflammatory and predicts stroke recurrence. 15 years. Stroke recurrence can occur even
origin. Transient Steno-occlusive arteriopathies cause several years after the initial event,
cerebral arteriopathy is stroke by local in situ thrombosis, longer than the commonly believed 6-
characterized by basal artery-to-artery embolism, or hypo- to 12-month period. The authors con-
ganglia infarction with perfusion. Classification has been sider degree of residual vessel stenosis,
ipsilateral irregular attempted based on vessel size, distri- stroke location (brainstem at risk), and
stenosis involving the
bution, the perceived pathology, tem- associated risk factors in deciding
carotid T-junction
poral evolution, and genetic or antithrombotic therapy duration. Anti-
represented by the
distal internal carotid
nongenetic basis.31,32 Recognition of coagulant therapy remains controver-
artery, proximal anterior a radiologic pattern of a vasculopathy sial for intracranial arterial dissection;
cerebral artery, and phenotype is beneficial in guiding management needs to be tailored to the
proximal middle management. This is feasible with individual situation.
cerebral artery. judicious use of vascular imaging tech- Transient cerebral arteriopathy.
niques such as MRA, CTA, catheter Transient cerebral arteriopathy of child-
cerebral angiography, and arterial wall hood is a well-described unilateral focal
imaging. This discussion focuses on arteriopathy of presumed inflammatory
the three commonly encountered origin. Transient cerebral arteriopathy is
arteriopathies associated with pediatric characterized by basal ganglia infarction
AIS: cervical artery dissection, transient with ipsilateral irregular stenosis involv-
cerebral arteriopathy, and moyamoya.33 ing the carotid T-junction represented
Cervical arterial dissection. Dissec- by the distal ICA, proximal anterior
tion involving the intracranial or extra- cerebral artery (ACA), and proximal
cranial portion of the internal carotid MCA (Case 7-3, Figure 7-5). Focal
artery (ICA) and vertebral artery (VA) cerebral arteriopathy is a recently coined
causes approximately 14% of pediatric term for this same pattern of radiologic

378 www.ContinuumJournal.com April 2014

Copyright © American Academy of Neurology. Unauthorized reproduction of this article is prohibited.


Case 7-2
A previously well
17-year-old girl presented
with progressive neck
pain, occipital headache,
nausea, and vomiting for
8 to 10 hours, which then
developed into left face
and tongue numbness,
dysarthria, and ataxia
over the next 2 to 3 hours.
She also noted episodic
diplopia with visual
blurring on lateral neck
movements. Over the past
month, she had
experienced episodic
vertigo, diplopia, visual
blurring, and bifrontal
headache 2 to 3 times
a week, as well as
photophobia and
phonophobia. Neurologic
examination revealed
horizontal and vertical
gaze-evoked bilateral
nystagmus, a left sixth
nerve palsy, left internuclear
ophthalmoplegia, mild
right arm and leg
hypotonia with pyramidal
type weakness, severe
bilateral appendicular
ataxia, and dysarthria.
Noncontrast head CT
revealed a hyperdense
basilar artery (Figure 7-2A).
Diffusion-weighted
imaging revealed
ischemia in the left
inferior cerebellar
peduncle and left pons FIGURE 7-2 Imaging studies at presentation of the patient in Case 7-2. A, Noncontrast
(Figure 7-2B, Figure 7-2C). head CT reveals hyperdense basilar artery (arrow). B and C, Diffusion-weighted
image (DWI) reveals ischemia (arrows) in the left inferior cerebellar peduncle
Magnetic resonance and left pons. D, Magnetic resonance angiography reveals greatly reduced flow in the entire
angiography (MRA) left vertebral artery throughout its course (arrows). E, Catheter cerebral angiography reveals an
showed greatly reduced intimal flap and double lumen (lower arrow) confirming left vertebral artery dissection at the
C1-C2 vertebral level and occlusion of the distal one-third of the basilar artery (upper arrow).
flow in the entire left
vertebral artery throughout
its course (Figure 7-2D). Unfractionated heparin was initiated as a left vertebral artery dissection was
suspected. Echocardiogram, lumbar puncture, inflammatory markers, and prothrombotic blood work
results were normal. Catheter cerebral angiography revealed an intimal flap and double lumen,
Continued on page 380
Continuum (Minneap Minn) 2014;20(2):370–386 www.ContinuumJournal.com 379
Copyright © American Academy of Neurology. Unauthorized reproduction of this article is prohibited.
Pediatric Arterial Ischemic Stroke

Continued from page 379


left vertebral artery dissection at the C1-C2 vertebral level, and occlusion of the distal third of the
basilar artery (Figure 7-2E). Unfractionated heparin was switched to low-molecular-weight heparin
(LMWH). One week later, the patient had new headache, nausea, and vomiting; repeat MRI revealed
a new ischemic stroke
in the right pons
(Figure 7-3A). Aspirin
was added; thereafter,
she remained stable
with no further TIAs or
stroke. A 3-month
follow-up catheter
cerebral angiography
revealed a
pseudoaneurysm at
the dissection site with
persistent basilar artery
occlusion (Figure 7-3B).
LMWH and aspirin
were continued for a
further 3 months.
Repeat catheter FIGURE 7-3 Imaging studies at follow-up of the patient in Case 7-2. A,
Diffusion-weighted image (DWI) shows a new ischemic stroke in the
cerebral angiography right pons (arrow) 1 week after initial presentation. B, Catheter cerebral
at 6 months was angiography 3 months after the initial presentation reveals pseudoaneurysm
unchanged; LMWH (lower arrow) at the site of dissection and persistent basilar artery occlusion
(upper arrow).
was stopped,
and aspirin was
continued. Follow-up MRA at 1 year and 2 years showed a stable pseudoaneurysm. The patient remained
on aspirin and made a full neurologic recovery except for a mild residual left gaze palsy.
Comment. This case illustrates the risk of recurrent stroke in the few days immediately after
diagnosis of pediatric arterial ischemic stroke, particularly in the presence of vasculopathies (in this
case arterial dissection), and highlights the need for close clinical and radiologic surveillance.

abnormalities.36 An inflammatory basis is methylprednisone) followed by tapering


presumed because postvaricella oral doses for 6 to 12 weeks (unpub-
angiopathy presents with an identical lished observations). Such treatment re-
clinicoradiologic syndrome following var- mains controversial pending published
icella infection within the preceding data supporting its safety and efficacy.
year. In postvaricella angiopathy, var- Acyclovir coverage can also be consid-
icella virus has been demonstrated in ered until varicella is ruled out if the
the arterial wall at the carotid T-junction patient has a history of chickenpox
on histopathology.37 Following that within a year preceding the stroke event.
description of postvaricella angiopathy, Follow-up imaging is helpful to decide
other infectious agents such as entero- on maintenance antithrombotic treat-
viruses, Borrelia, and Bartonella have ment, either anticoagulation or aspirin.
been associated with this syndrome. In In transient cerebral arteriopathy, the
addition to acute antithrombotic therapy, vasculopathy stabilizes by 6 months with
some children have been treated with a no further worsening of the arterial
short course of anti-inflammatory therapy, findings; progression makes transient
such as high-dose corticosteroids (IV cerebral arteriopathy unlikely, raising the

380 www.ContinuumJournal.com April 2014

Copyright © American Academy of Neurology. Unauthorized reproduction of this article is prohibited.


Case 7-3
An 11-year-old right-handed girl who had chickenpox 7 months before but was otherwise healthy
presented with stuttering right arm, leg, and facial weakness over 2 hours. Brain MRI demonstrated
diffusion restriction in the left internal capsule and globus pallidus (Figure 7-4A), with narrowing of
the distal left internal carotid artery on magnetic resonance angiography (MRA) (Figure 7-4B). MRA
pattern was consistent with transient cerebral arteriopathy. High-resolution coronal T1-weighted
two-dimensional fluid-attenuated inversion recovery (FLAIR) sequence demonstrated concentric wall
enhancement of the distal left internal carotid artery, proximal middle cerebral artery, and proximal
anterior cerebral artery, suggestive of inflammation (Figure 7-4C), compared with the opposite
side, which did not enhance. Serology was positive for varicella IgG; CSF white blood cell count was
2/mL, and CSF varicella PCR was negative. She was treated with IV methylprednisolone 30 mg/kg/d for
5 days, followed by a 3-month weaning course of oral prednisone starting at 2 mg/kg, in addition to
3 weeks of oral acyclovir. On follow-up imaging, the concentric wall enhancement was less intense
by 1 month and had completely resolved by 6 months, with normalization of vessel wall caliber.
Clinical recovery was complete. She remained on aspirin for secondary stroke prophylaxis.

FIGURE 7-4 Imaging studies at presentation of the patient in Case 7-3. Diffusion-weighted image (DWI)
demonstrating diffusion restriction in the left internal capsule and globus pallidus (A), with
narrowing of the distal left internal carotid artery (arrow) on magnetic resonance angiography
(B). Arterial wall imaging (C) shows concentric wall enhancement of the distal left internal carotid artery,
proximal middle cerebral artery, and proximal anterior cerebral artery (solid arrow) suggestive of inflammation
compared with the opposite side (dashed arrow) that did not enhance.

Comment. This case illustrates the ‘‘stuttering presentation’’ of pediatric arterial ischemic stroke in
the context of an arteriopathy such as postvaricella angiitis, as well as the potential utility of newer
vascular imaging techniques such as high-resolution arterial wall imaging. The typical unilateral
involvement (stenosis, occlusion, irregularity) of the region of bifurcation of the internal carotid
artery into anterior and middle cerebral artery should also raise the suspicion of transient cerebral
arteriopathy in a child with ischemic stroke.

suspicion of a progressive angiitis of such as intracranial arterial dissection,


the CNS. The radiologic outcome of which may have a very similar clinical
transient cerebral arteriopathy is either and radiologic presentation.38 Last, the
full recanalization of the affected arterial possibility of unilateral moyamoya may
segments or some degree of residual need consideration, although if initial
stenosis that eventually may dictate the angiography does not demonstrate the
need for long-term antithrombotic thera- classical hypertrophied collateral vessels,
py. Although an ‘‘inflammatory’’ basis has evolution to that entity appears rare.36
been presumed for this syndrome, it is Moyamoya vasculopathy.
important to consider other possibilities Moyamoya is a progressive occlusion of

Continuum (Minneap Minn) 2014;20(2):370–386 www.ContinuumJournal.com 381


Copyright © American Academy of Neurology. Unauthorized reproduction of this article is prohibited.
Pediatric Arterial Ischemic Stroke

KEY POINT
h Pediatric moyamoya
mainly presents with
ischemic stroke whereas
adult moyamoya often
presents with
hemorrhagic stroke.

FIGURE 7-5 A 4-year-old boy with transient cerebral arteriopathy. Diffusion-weighted image
(DWI) (A) shows acute left globus pallidus and putamen infarct. Magnetic
resonance angiography (B) shows irregular and reduced flow in the proximal
left middle cerebral artery (arrow). Catheter cerebral angiography (C) reveals significant
caliber reduction and irregularity (arrow) of the distal left internal carotid artery, proximal
middle cerebral artery, and absent flow in the anterior cerebral artery with cross filling from
the opposite side (D).

arteries of the circle of Willis, mainly the dition is typically bilateral, although it
distal ICA, proximal MCA, proximal ACA, can begin unilaterally and progress to
and sometimes posterior cerebral artery the other side. In some conditions,
(PCA). The presence of hypertrophied, moyamoya tends to stay unilateral, as
friable collateral vessels in the basal in NF-1. Pediatric moyamoya mainly
ganglia region (‘‘puff of smoke’’) presents with ischemic stroke, whereas
detected by prominent flow voids on T1 adult moyamoya often presents with
MRI and the ‘‘ivy sign’’ (leptomeningeal hemorrhagic stroke. The hallmarks of
high signal resembling creeping ivy) on moyamoya are recurrent TIAs, recurrent
fluid-attenuated inversion recovery clinical or radiologic infarcts (mainly in
(FLAIR) images representing prominent central white matter), and sometimes
leptomeningeal collaterals is typical territorial hypoperfusion-related or vaso-
(Case 7-4). The idiopathic variety is occlusive ischemic strokes. Presentation
called moyamoya disease, while that with recurrent headaches and progres-
occurring in the presence of conditions sive cognitive decline is also seen.
such as sickle cell disease, neurofibro- Aspirin is the favored antithrombotic
matosis (NF-1), and other systemic therapy for stroke prophylaxis, but
medical and genetic conditions is long-term data regarding the efficacy
termed moyamoya syndrome. The con- and safety of aspirin are lacking. Surgical
382 www.ContinuumJournal.com April 2014

Copyright © American Academy of Neurology. Unauthorized reproduction of this article is prohibited.


Case 7-4
A 6-year-old boy had a history of recurrent alternating left- and right-sided face, arm, and leg
weakness lasting several minutes for 3 to 4 weeks before presentation. Some of these episodes were
provoked by running on the playground. He had four to five episodes of migraine-type headaches for
a few months before presentation. He presented with the sudden onset of a left-sided hemiparesis
that lasted longer than the previous episodes. On examination, he had persistent mild left-sided
weakness. MRI showed acute patchy restricted diffusion in the right corona radiata and centrum
semiovale in the deep middle cerebral artery, middle cerebral artery/anterior cerebral artery, and
middle cerebral artery/posterior cerebral artery watershed zones (Figure 7-6A). T1-weighted MRI
sequence showed prominent flow voids in the basal ganglia region (Figure 7-6B). Fluid-attenuated
inversion recovery
(FLAIR) sequence
revealed bilateral
asymmetric (right
greater than left)
prominent
leptomeningeal
high signal,
consistent with
an ‘‘ivy sign’’
(Figure 7-6C).
Magnetic resonance
angiography
(Figure 7-6D)
revealed significant
stenosis of the
bilateral distal
internal carotid
arteries and
proximal middle
cerebral arteries, and
absent flow through
both anterior
cerebral
arteries with
bilateral moyamoya
collaterals. Aspirin FIGURE 7-6 Imaging studies at presentation of the patient in Case 7-4 A, Diffusion-weighted
image (DWI) shows acute patchy restricted diffusion in the right corona radiata
was started. He and centrum semiovale in the deep middle cerebral artery/anterior cerebral
artery, and middle cerebral artery/posterior cerebral artery watershed zones. B, Axial
underwent T1-weighted sequence shows prominent flow voids in the basal ganglia region (arrows).
catheter cerebral C, Axial fluid-attenuated inversion recovery (FLAIR) sequence reveals bilateral asymmetric (right 9
angiography, which left) prominent leptomeningeal high signal, consistent with an ‘‘ivy sign’’ (arrows). D, Magnetic
resonance angiography revealed significant stenosis (blue arrows) of bilateral distal internal
confirmed bilateral carotid arteries and proximal middle cerebral arteries, and absent flow through both anterior
moyamoya cerebral arteries with bilateral ‘‘moyamoya’’ collaterals (red arrows).
vasculopathy (right
greater than left).
Surgical revascularization (indirect bypass with bilateral pial synangiosis) was successfully performed. He
remained symptom-free at the time of his last follow-up after 4 years on aspirin.
Comment. This case illustrates the peculiar presenting symptom of hyperventilation-induced
(in this case, from running hard on the playground) paroxysmal episodes of focal neurologic
deficits unique to moyamoya. The case also demonstrates the success of revascularization surgery
in moyamoya.

Continuum (Minneap Minn) 2014;20(2):370–386 www.ContinuumJournal.com 383


Copyright © American Academy of Neurology. Unauthorized reproduction of this article is prohibited.
Pediatric Arterial Ischemic Stroke

KEY POINT
h Encephaloduroarteriosy- TABLE 7-2 Outcome From Perinatal/Neonatal Arterial Ischemic Stroke
nangiosis is preferred
over superficial b Deficits in neonates may become evident over time; long-term follow-up is
temporal arteryYmiddle essential
cerebral artery bypass in
b 30% to 70% of patients with perinatal/neonatal arterial ischemic stroke (AIS)
young children because
will have cerebral palsy
the small size of the
arteries makes surgery b As many as 90% of patients with perinatal/neonatal AIS will eventually walk
challenging. b Involvement of cerebral cortex and basal ganglia and internal capsule
predicts hemiparesis
b 20% to 50% of patients with perinatal/neonatal AIS may develop epilepsy
after 6-months post stroke
b Stroke recurrence risk is negligible

revascularization seems to have a bene- OUTCOMES FROM PEDIATRIC


ficial effect in reducing the stroke risk ARTERIAL ISCHEMIC STROKE
and is generally advocated in children Approximately 50% of neonates diag-
with clear vascular progression, recur- nosed with AIS will be left with a
rent stroke/TIA, and progressive cogni- cognitive, language, or motor deficit,
tive decline. Surgical outcome in typically hemiparesis40 as summarized
experienced centers is encouraging.39 in Table 7-2. Death is rare. Usually,
The main aim of surgery is to provide the cognitive deficits range from mild
an alternative blood supply to the brain to moderate, but boys seem to do
via either a direct (superficial temporal worse.41 However, in presumed perinatal
artery [STA]YMCA anastomosis) or indi- ischemic stroke, because such children
rect (encephaloduroarteriosynangiosis come to diagnosis only because of
[EDAS]) external carotidYinternal carotid clinical deficits (mainly hemiparesis), the
(EC-IC) bypass procedure. The latter outcome is of a more severe neurologic
involves laying of an STA branch over deficitVfor example, moderate to severe
the cerebral surface without direct anas- hemiparesis. Epilepsy occurs in 15% to
tomosis in the hope that new vessels will 40% of patients and can occasionally be
eventually grow from the overlaid refractory.41
branch. EDAS is preferred over STA- Outcome from childhood AIS is sum-
MCA bypass in young children because marized in Table 7-3. In older infants
the small size of the arteries makes and children, death due to stroke
surgery challenging. occurs in 5%, frequently from malignant

TABLE 7-3 Outcome From Childhood Arterial Ischemic Stroke

b Higher incidence of hemiparesis than neonatal stroke


b Dystonia may develop following basal ganglia involvement (typically after 6 months)
b Weaker cognitive performance compared to normative samples are seen in
combined cortical and subcortical stroke and with younger age at stroke
b Congenital heart disease and arteriopathy predict recurrence
b Stroke recurrence risk up to 19% and is highest in the first year after stroke
b Antithrombotic therapy reduces recurrence risk

384 www.ContinuumJournal.com April 2014

Copyright © American Academy of Neurology. Unauthorized reproduction of this article is prohibited.


MCA syndrome. The outcome in survi- Clinical Practice Guidelines. Chest 2012;141
(2 suppl):e737SYe801S.
vors ranges from 38% normal, 42% mild
disability, 8% moderate disability, to 8. Kirton A, Armstrong-Wells J, Chang T, et al.
Symptomatic neonatal arterial ischemic
12% severe handicap. Outcome at 1 stroke: the International Pediatric Stroke
year post stroke predicted final func- Study. International Pediatric Stroke Study
tional outcome, and arteriopathy Investigators. Pediatrics 2011;128(6):
e1402Ye1410.
predicted recurrence in one study.20
9. Rafay MF, Cortez MA, de Veber GA, et al.
Predictive value of clinical and EEG features
CONCLUSIONS in the diagnosis of stroke and hypoxic
ischemic encephalopathy in neonates with
Pediatric AIS, which is probably more seizures. Stroke 2009;40(7):2402Y2407.
common than previously thought, pre-
10. Braun KP, Rafay MF, Uiterwaal CS, et al.
sents unique diagnostic and manage- Mode of onset predicts etiological diagnosis
ment challenges that are distinct from of arterial ischemic stroke in children. Stroke
adult AIS. A multidisciplinary approach 2007;38(2):298Y302.
is essential to optimize short-term and 11. Shellhaas RA, Smith SE, O’Tool E, et al.
long-term management. Mimics of childhood stroke: characteristics
of a prospective cohort. Pediatrics
2006;118(2):704Y709.
REFERENCES 12. Mackay MT, Wiznitzer M, Benedict SL, et al.
1. National center for health statistics. Deaths, Arterial ischemic stroke risk factors: the
percentage of total deaths, and death rates International Pediatric Stroke Study. Ann
for the 10 leading causes of death in Neurol 2011;69(1):130Y140.
selected age groups, by race and sex: United 13. Elbers J, Viero S, MacGregor D, et al.
States, 2002. In: National Vital Statistics Reports Placental pathology in neonatal stroke.
Vol. 53, No. 17. CDC website [online]. Pediatrics 2011;127(3):e722Ye729.
Available at www.cdc.gov/nchs/data/nvsr/
nvsr53/nvsr53_17.pdf. Published March 7, 2005. 14. Rodan L, McCrindle BW, Manlhiot C, et al.
Accessed February 10, 2014. Stroke recurrence in children with
congenital heart disease. Ann Neurol 2012;
2. Fullerton HJ, Wu YW, Zhao S, Johnston SC. 72(1):103Y111.
Risk of stroke in children: ethnic and gender
disparities. Neurology 2003;61(2):189Y194. 15. Fullerton HJ, Wu YW, Sidney S, Johnston SC.
Risk of recurrent childhood arterial ischemic
3. Nelson KB, Lynch JK. Stroke in newborn stroke in a population-based cohort: the
infants. Lancet Neurol 2004;3(3):150Y158. importance of cerebrovascular imaging.
4. Golomb MR, Fullerton HJ, Nowak-Gottl U, Pediatrics 2007;119(3):495Y501.
Deveber G; International Pediatric Stroke 16. deVeber G. Delays in the timely diagnosis of
Study Group. Male predominance in stroke in children. Nat Rev Neurol 2010;6(2):
childhood ischemic stroke: findings from 64Y66.
the international pediatric stroke study.
Stroke 2009;40(1):52Y57. 17. Tan MA, DeVeber G, Kirton A, et al. Low
detection rate of craniocervical arterial
5. Rafay MF, Pontigon AM, Chiang J, et al. dissection in children using time-of-flight
Delay to diagnosis in acute pediatric arterial magnetic resonance angiography: causes
ischemic stroke. Stroke 2009;40(1):58Y64. and strategies to improve diagnosis. J Child
6. Roach ES, Golomb MR, Adams R, et al. Neurol 2009;24(10):1250Y1257.
Management of stroke in infants and 18. Swartz RH, Bhuta SS, Farb RI, et al.
children: a scientific statement from a
Intracranial arterial wall imaging using
Special Writing Group of the American high-resolution 3-tesla contrast-enhanced
Heart Association Stroke Council and the MRI. Neurology 2009;72(7):627Y634.
Council on Cardiovascular Disease in the
Young. Stroke 2008;39(9):2644Y2691. 19. Payne ET, Wei XC, Kirton A. Reversible
wall enhancement in pediatric cerebral
7. Monagle P, Chan AK, Goldenberg NA, et al; arteriopathy. Can J Neurol Sci 2011;38(1):
American College of Chest Physicians. 139Y140.
Antithrombotic therapy in neonates and
children: Antithrombotic Therapy and 20. Elbers J, Deveber G, Pontigon AM, Moharir M.
Prevention of Thrombosis, 9th ed: American Long-term outcomes of pediatric ischemic
College of Chest Physicians Evidence-Based stroke in adulthood [published online ahead

Continuum (Minneap Minn) 2014;20(2):370–386 www.ContinuumJournal.com 385


Copyright © American Academy of Neurology. Unauthorized reproduction of this article is prohibited.
Pediatric Arterial Ischemic Stroke

of print April 15 2013]. J Child Neurol 2013. stroke on acute MRI. Stroke 2009;40(3):
doi:10.1177/0883073813484358. 780Y787.
21. Gupta AA, Leaker M, Andrew M, et al. 31. Sébire G, Fullerton H, Riou E, deVeber G.
Safety and outcomes of thrombolysis with Toward the definition of cerebral
tissue plasminogen activator for treatment arteriopathies of childhood. Curr Opin
of intravascular thrombosis in children. Pediatr 2004;16(6):617Y622.
J Pediatr 2001;139(5):682Y688.
32. Bernard TJ, Manco-Johnson MJ, Lo W, et al.
22. Amlie-Lefond C, deVeber G, Chan AK, et al; Towards a consensus-based classification of
International Pediatric Stroke Study. Use of childhood arterial ischemic stroke. Stroke
alteplase in childhood arterial ischaemic 2012;43(2):371Y377.
stroke: a multicentre, observational, cohort
study. Lancet Neurol 2009;8(6):530Y536. 33. Amlie-Lefond C, Bernard TJ, Sebire G, et al.
Predictors of cerebral arteriopathy in chil-
23. Sträter R, Kurnik K, Heller C, et al. Aspirin dren with arterial ischemic stroke: results of
versus low-dose low-molecular-weight heparin:
the international pediatric stroke study.
antithrombotic therapy in pediatric ischemic Circulation 2009;119(10):1417Y1423.
stroke patients: a prospective follow-up study.
Stroke 2001;32(11):2554Y2558. 34. Rafay MF, Armstrong D, Deveber G, et al.
Craniocervical arterial dissection in children:
24. Goldenberg NA, Bernard TJ, Fullerton HJ,
clinical and radiographic presentation and
et al. Antithrombotic treatments, outcomes,
outcome. J Child Neurol 2006;21(1):8Y16.
and prognostic factors in acute
childhood-onset arterial ischaemic stroke: a 35. Arauz A, Ruiz A, Pacheco G, et al. Aspirin
multicentre, observational, cohort study. versus anticoagulation in intra- and extra
Lancet Neurol 2009;8(12):1120Y1127. cranial vertebral artery dissection. Eur J
Neurol 2013;20(1):167Y172.
25. Bernard TJ, Goldenberg NA, Tripputi M,
et al. Anticoagulation in childhood onset 36. Bulder MM, Braun KP, Leeuwis JW, et al. The
arterial ischemic stroke with non-moyamoya course of unilateral intracranial arteriopathy
arteriopathy: findings from the Colorado in young adults with arterial ischemic
and German (COAG) collaboration. Stroke stroke. Stroke 2012;43(7):1890Y1896.
2009;40(8):2869Y2871.
37. Gilden D, Cohrs RJ, Mahalingam R, Nagel
26. Schechter T, Kirton A, Laughlin S, et al. Safety
MA. Varicella zoster virus vasculopathies:
of anticoagulants in children with arterial diverse clinical manifestations, laboratory
ischemic stroke. Blood 2012;119(4):949Y956. features, pathogenesis, and treatment.
27. Kenet G, Lutkhoff LK, Albisetti M, et al. Lancet Neurol 2009;8(8):731Y740.
Impact of thrombophilia on risk of arterial
38. Dlamini N, Freeman JL, Mackay MT, et al.
ischemic stroke or cerebral sinovenous
Intracranial dissection mimicking transient
thrombosis in neonates and children: a
cerebral arteriopathy in childhood arterial
systematic review and meta-analysis of
ischemic stroke. J Child Neurol
observational studies. Circulation
2011;26(9):1203Y1206.
2010;121(16):1838Y1847.
28. Rodan L, McCrindle BW, Manlhiot C, et al. 39. Kim SK, Cho BK, Phi JH, et al. Pediatric
Stroke recurrence in children with congenital moyamoya disease: an analysis of 410
heart disease. Ann Neurol 2012;72(1): consecutive cases. Ann Neurol
103Y111. 2010;68(1):92Y101.

29. Jones KC, Hawkins C, Armstrong D, et al. 40. Golomb MR. Outcomes of perinatal arterial
Association between radiographic Wallerian ischemic stroke and cerebral sinovenous
degeneration and neuropathological thrombosis. Semin Fetal Neonatal Med
changespost childhood stroke. Dev Med 2009;14(5):318Y322.
Child Neurol 2013;55(2):173Y177.
41. Westmacott R, MacGregor D, Askalan R,
30. Domi T, deVeber G, Shroff M, et al. deVeber G. Late emergence of cognitive
Corticospinal tract pre-wallerian degeneration: deficits after unilateral neonatal stroke.
a novel outcome predictor for pediatric Stroke 2009;40(6):2012Y2019.

386 www.ContinuumJournal.com April 2014

Copyright © American Academy of Neurology. Unauthorized reproduction of this article is prohibited.


Review Article

Unruptured Intracranial
Address correspondence to
Dr Adam Kelly, University of
Rochester, 601 Elmwood
Avenue, PO Box 673,

Aneurysms: Screening Rochester, NY 14642-0001,


Adam_kelly@urmc.rochester.edu.
Relationship Disclosure:

and Management Dr Kelly reports no disclosure.


Unlabeled Use of
Products/Investigational
Use Disclosure:
Adam G. Kelly, MD Dr Kelly reports no disclosure.
* 2014, American Academy
of Neurology.
ABSTRACT
Purpose of Review: Unruptured intracranial aneurysms are found commonly in
the general public, and more frequently in certain populations. This article focuses
on the epidemiology, screening strategies, and management options for patients
with unruptured aneurysms.
Recent Findings: Recent epidemiologic studies show the overall prevalence of
intracranial aneurysms to be approximately 3%, with higher rates seen in familial
aneurysm syndromes and in certain medical conditions, such as autosomal
dominant polycystic kidney syndrome. Aneurysm treatment may include surgical
or endovascular techniques, with increasing utilization of endovascular strategies
over time. Increased aneurysm diameter, certain locations, and other anatomical
considerations may be associated with higher risks of aneurysm rupture.
Summary: Given the high morbidity and mortality associated with aneurysm
rupture, screening for unruptured aneurysms is generally recommended for high-
risk patients (patients who have at least two first-degree relatives with aneurysms,
and patients with autosomal dominant polycystic kidney disease). Screening may be
considered for other patients (eg, one first-degree relative with aneurysm) after
discussion of the risks and benefits of imaging. Following identification of an
aneurysm, decisions regarding observation or treatment should be based on patient
characteristics, features of the aneurysm, and provider expertise.

Continuum (Minneap Minn) 2014;20(2):387–398.

INTRODUCTION epidemiology of intracranial aneu-


Advances in the availability and reso- rysms, recommendations for screen-
lution of neuroimaging studies have ing for aneurysms in the general
led to more frequent identification of population and in select patient
intracranial aneurysms. Rupture of populations, and management options
intracranial aneurysms can lead to in patients found to have unruptured
devastating consequences, namely intracranial aneurysms.
subarachnoid hemorrhage, but
unruptured intracranial aneurysms ANEURYSM TYPES AND
can also be associated with significant DEFINITIONS
morbidity. Such morbidity can include Saccular (berry) aneurysms are the
local compressive symptoms such as most common type of intracranial
headache or cranial nerve palsies, aneurysm, consisting of a focal
strokes, and seizures, as well as pa- outpouching (ballooning) of the arte-
tient and family anxiety regarding the rial blood vessel wall. Although the
risk of rupture. This article reviews the exact pathophysiologic mechanism is

Continuum (Minneap Minn) 2014;20(2):387–398 www.ContinuumJournal.com 387


Copyright © American Academy of Neurology. Unauthorized reproduction of this article is prohibited.
Unruptured Intracranial Aneurysms

KEY POINT
h The prevalence of not clear, these are thought to be due (2) thrombus formation, resulting in
aneurysms in the to a congenital weakness in the vessel either occlusion of small perforating
general population is wall, which then slowly enlarges over vessels or downstream embolization.
estimated at time in the setting of arterial pressure. Mycotic aneurysms result from the
approximately 3%. Saccular aneurysms are most frequently deposition of septic emboli in small
seen at the branching points of blood pial arteries. The underlying infectious
vessels, often larger intracranial vessels agent (often bacteria, in the setting of
in or around the circle of Willis.1 A list of bacterial endocarditis and bacteremia)
common saccular aneurysm locations causes weakness and outpouching of
is listed in Table 8-1. Despite the the blood vessel wall, potentially
predilection for saccular aneurysms to resulting in subarachnoid or intra-
form on larger vessels near the skull parenchymal hemorrhage.4
base, aneurysms can be found in more
distal locations in smaller caliber arter- EPIDEMIOLOGY
ies. For example, distal anterior cere- Prevalence
bral artery aneurysms, also known as Although some past autopsy-based stud-
pericallosal artery aneurysms, are ies quote higher aneurysm identification
thought to represent 4% to 6% of rates, most population-based studies
intracranial aneurysms.2 quote unruptured intracranial aneurysm
Although this article focuses on saccu- prevalence rates of 2% to 3%.1,5,6 In a
lar aneurysms, other types of intracranial recent meta-analysis encompassing al-
aneurysms exist. Fusiform aneurysms are most 95,000 patients from 68 studies
seen in association with intracranial and 21 countries, the prevalence was
atherosclerotic disease or dissections of estimated at 2.8%.6 Subject age, sex, and
intracranial arteries and represent longi- comorbidity varied across studies; when
tudinal areas of vessel dilation, as op- data were normalized for a population
posed to the focal outpouching seen aged 50 years and with equal numbers
with saccular aneurysms. Fusiform aneu- of men and women, the prevalence was
rysmal segments are commonly seen in estimated to be 3.2%.6
conjunction with areas of atherosclerotic
narrowing, often seen in larger intracra- Risk Factors
nial arteries such as the basilar artery.3 Age, sex, and family history are all important
While there are occasional episodes of nonmodifiable risk factors for unruptured
rupture of dolichoectatic fusiform aneu- intracranial aneurysms (Table 8-2).7
rysms, they are more likely to become The prevalence of unruptured in-
symptomatic through (1) local mass tracranial aneurysms increases with
effect on adjacent brain structures or patient age, especially among women.

TABLE 8-1 Common Locations of Intracranial Saccular Aneurysms

b Origin of the anterior communicating artery (È30%)


b Origin of the posterior communicating artery (È25%)
b First bifurcation/branching of the middle cerebral artery (È20%)
b Terminal branching of the internal carotid artery (È7.5%)
b Terminal branching of the basilar artery (È7%)
b Origin of the posterior inferior cerebellar artery (È3%)

388 www.ContinuumJournal.com April 2014

Copyright © American Academy of Neurology. Unauthorized reproduction of this article is prohibited.


KEY POINTS

TABLE 8-2 Risk Factors for Intracranial Aneurysms h A family history of


aneurysms is a strong
b Age predictor of aneurysm
development, and
b Female sex
familial aneurysms may
b Family history of aneurysm or subarachnoid hemorrhage have a higher risk of
b Tobacco use rupture than aneurysms
of similar size and
b Excessive alcohol consumption
location in nonfamilial
b Autosomal dominant polycystic kidney disease syndromes.
b Connective tissue disorders (Marfan syndrome, Ehlers-Danlos syndrome type IV) h Autosomal dominant
polycystic kidney
disease is a strong
Specifically regarding family history, limited insight on other information predictor of the
presence of unruptured
the presence of an aneurysm in one of relevance for unruptured intracra-
intracranial aneurysms.
first-degree relative (parent, sibling, or nial aneurysm management (eg, size,
child) increases the risk of aneurysm location).10
from 2.3% to 4.0%, while presence in Certain connective tissue disorders
two or more first-degree relatives in- are associated with an increased risk of
creases the risk to 8.0% percent.5 Data aneurysms, such as Marfan syndrome
suggest that not only is the likelihood and Ehlers-Danlos syndrome type IV.1
of developing aneurysms heritable, A single study from the Mayo Clinic
but aneurysm location is concordant found higher than expected rates of
among individuals in families with unruptured intracranial aneurysms in
aneurysms.8 This may have implica- adults with coarctation of the aorta
tions regarding the developmental (10% versus 2%), although the num-
origin of aneurysms, their risk of ber of aneurysms was small (10 aneu-
rupture, and treatment decisions (see rysms among 100 studied patients).11
section on risk of rupture). The risk of Intracranial aneurysms can be seen in
aneurysm rupture may also be in- association with fibromuscular dyspla-
creased in familial aneurysm syn- sia,12,13 although it remains unclear
dromes compared with the overall whether the pathophysiology is related
population’s risk of rupture. Data to arterial dissection (commonly seen
from the Familial Intracranial Aneu- in fibromuscular dysplasia) or other
rysm study suggest that the risk of mechanisms causative of unruptured
aneurysm rupture in patients with a intracranial aneurysm development in
family history of aneurysm was 17 other populations.
times higher than the risk of rupture Although it is not a purely connec-
based on aneurysm size and location tive tissue disorder, autosomal domi-
that would be predicted from obser- nant polycystic kidney disease is
vational studies.9 The genetics of strongly associated with aneurysm
intracranial aneurysm formation is development. In a recent study from
complex and an area of ongoing China, patients with autosomal domi-
research. A large review of more than nant polycystic kidney disease were
116,000 patients included in genetic found to have a 12.4% prevalence of
and genome-wide association studies unruptured intracranial aneurysms,
identified several polymorphisms as- significantly elevated over the general
sociated with higher aneurysm inci- population rate.14 The risk increased
dence; however, this study offered with age (23.3% risk in patients aged

Continuum (Minneap Minn) 2014;20(2):387–398 www.ContinuumJournal.com 389


Copyright © American Academy of Neurology. Unauthorized reproduction of this article is prohibited.
Unruptured Intracranial Aneurysms

KEY POINT
h Most unruptured 60 to 69 years) and was twice as high ence of these symptoms may suggest
intracranial aneurysms in patients with a family history of that a presumed unruptured intracra-
are discovered when aneurysm.14 Risk of aneurysms is nial aneurysm may instead be an
asymptomatic, although possibly also increased among pa- aneurysm that has previously ruptured,
other presenting tients with the less common but and this may affect decision-making
symptoms include related condition of autosomal reces- regarding treatment.
compressive cranial sive polycystic kidney disease.15 Rarely, unruptured aneurysms can
neuropathies, seizures, Tobacco use, excessive alcohol con- be associated with focal neurologic
or ischemic stroke. sumption, and hypertension are im- symptoms, often from compression of
portant modifiable risk factors for adjacent structures such as cranial
aneurysm growth. While a congenital nerves. Classic examples include oculo-
weakness may underlie the develop- motor nerve palsies related to posterior
ment of an aneurysm, tobacco or communicating artery aneurysms or
alcohol abuse and hypertension can compression of the third, fourth, fifth,
lead to further vascular injury/weakness or sixth cranial nerves from aneurysms
and increased aneurysm size, which are of the carotid artery within the cavern-
factors associated with a higher risk of ous sinus. Giant aneurysms (measuring
aneurysm rupture.16 more than 25 mm in diameter) can
result in mass effect on nearby brain
SIGNS AND SYMPTOMS OF parenchyma and cause various symp-
ANEURYSMS toms depending on the specific location;
Subarachnoid hemorrhage (SAH) sec- such mass effect can also result in partial
ondary to aneurysm rupture is the most seizures. The turbulent, nonlaminar flow
feared complication of intracranial within aneurysms can result in thrombus
aneurysms. Common signs and symp- formation; downstream embolization
toms of SAH should be well known to and stroke may occur.
neurologists and can include sudden
severe headache, headache associated GUIDELINES FOR SCREENING
with concerning symptoms (loss of Which Patients to Assess for
consciousness, meningeal symptoms), an Unruptured Intracranial
or coma without other obvious cause.17 Aneurysm
In contrast to the presentation of All patients with symptoms of a possi-
SAH, most unruptured intracranial an- ble symptomatic unruptured aneurysm
eurysms are neurologically silent. Many described in the above section should
aneurysms will be identified when pa- be candidates for cerebrovascular im-
tients are imaged for an unrelated aging to evaluate for aneurysm.
neurologic indication, such as headache Regarding asymptomatic unruptured
(Case 8-1). Unless it has grown to a intracranial aneurysms, the American
significant size, an unruptured intracra- Heart Association and American Stroke
nial aneurysm would be unlikely to Association18 have established guide-
cause headache, and alternative expla- lines about specific patient populations
nations should be considered. How- in which screening of asymptomatic
ever, when an aneurysm is found, it is individuals should be considered. Pa-
important to inquire about possible tients with two first-degree relatives with
symptoms of prior aneurysmal rupture, known intracranial aneurysms (ruptured
such as an especially severe headache, or unruptured) are recommended to
headache associated with meningeal undergo screening imaging (Case 8-2).
symptoms, or headache associated Patients with a single first-degree
with loss of consciousness. The pres- relative affected by aneurysms may
390 www.ContinuumJournal.com April 2014

Copyright © American Academy of Neurology. Unauthorized reproduction of this article is prohibited.


Case 8-1
A 52-year-old woman was seen in the office 1 week after going to an
emergency department for a syncopal episode. At the time of her
presentation, she denied headache, nausea, neck stiffness, and any other
neurologic symptoms. Her overall medical examination and neurologic
examination in the emergency department had been unremarkable except
for mild orthostasis, and subsequent cardiologic evaluation was unrevealing.
Additional history in the office disclosed that her loss of consciousness had
been very brief and occurred in the setting of recent dehydration, and her
current neurologic examination was normal. Intravascular depletion at the
time of the syncopal episode was suspected as the primary contributor to her
event. Review of her evaluation in the emergency department, however,
included a head CT that showed a possible enlargement in the area of the
basilar bifurcation (Figure 8-1A). CT angiography followed by catheter
angiography confirmed an 8 mm aneurysm at the bifurcation of the basilar
artery (CT angiogram image shown in Figure 8-1B). Following discussion of
the risks and benefits of treatment, she underwent coil embolization of the
aneurysm. After 5 years of follow-up, there were no signs of aneurysm recurrence.

FIGURE 8-1 Imaging studies taken of the patient in Case 8-1 at presentation to the
emergency department. A, Noncontrast CT of the brain showing
enlargement of the basilar artery near the bifurcation (arrow). B, CT
angiogram (coronal view) showing basilar apex aneurysm (arrow).

Comment. As is often the case, this aneurysm was discovered incidentally,


following a CT scan of the brain that was ordered for other purposes (syncope,
in this case). Given the patient’s relatively young age, the aneurysm size being
greater than 7 mm, and the posterior circulation location of the aneurysm, the
decision to treat via endovascular coiling (as opposed to open surgery or
observation) seems appropriate. She received adequate follow-up, both
clinically and radiographically, to ensure there had been no recurrence of the
aneurysm at the treated site or new aneurysms forming elsewhere.

be candidates for screening imaging rysm detection (eg, anxiety, risks


based on a higher risk of unruptured of subsequent testing, difficulty
intracranial aneurysms in this setting; obtaining life insurance, occupational
however, the potential risks of aneu- concerns) need to be discussed

Continuum (Minneap Minn) 2014;20(2):387–398 www.ContinuumJournal.com 391


Copyright © American Academy of Neurology. Unauthorized reproduction of this article is prohibited.
Unruptured Intracranial Aneurysms

KEY POINTS
h Patients with two or
more first-degree
Case 8-2
A 33-year-old man was seen in clinic to discuss screening for intracranial
relatives with
aneurysms. He had no current neurologic symptoms and no active medical
unruptured intracranial
problems. He had never been prone to headache, and his history was
aneurysms or
completely unremarkable for any events suggestive of subarachnoid
subarachnoid
hemorrhage (eg, severe or sudden unexplained headache, headache with
hemorrhage, or patients
loss of consciousness, headache associated with meningeal symptoms). His
with autosomal
family history was notable for an unruptured intracranial aneurysm in his
dominant polycystic
sister, diagnosed at 40 years of age after she underwent an MRI of the
kidney disease, are
brain to evaluate her migraine headaches; this was subsequently treated
candidates for intracranial
with surgical clipping. In addition, his father died suddenly of unknown
imaging to screen for
causes at 51 years of age; an autopsy was not performed. The patient’s
unruptured intracranial
mother was alive and in good health, as were two other siblings (aged
aneurysms.
34 and 37 years) and his one daughter (aged 4 years). To his knowledge,
h Both CT angiography his other first-degree relatives had not undergone intracranial imaging for
and magnetic resonance any reason.
angiography are Comment. This patient may be a reasonable candidate for noninvasive
reasonable options for imaging to screen for unruptured intracranial aneurysms. He has at
noninvasive screening least one first-degree relative with an intracranial aneurysm (his sister). In
imaging for unruptured addition, his father’s sudden death may have been related to subarachnoid
intracranial aneurysms, hemorrhage secondary to aneurysm rupture. It would be important to
although catheter inquire about other possible causes of his father’s death (eg, cardiovascular
angiography remains disease, substance abuse); if none are present, noninvasive cerebrovascular
the gold standard for imaging with either CT angiography or magnetic resonance angiography
aneurysm identification. should be considered for aneurysm screening purposes.

with the patient along with the potential every 5 years, since studies show the
benefits (earlier detection and possi- risk of aneurysm identification following
ble treatment). Patients with auto- an initial negative study may be as high
somal dominant polycystic kidney as 7%.20
disease are also recommended to un-
dergo screening.5 Which Imaging Modality to Use
Catheter angiography remains the gold
When to Screen and Rescreen standard for aneurysm detection; al-
Aneurysms are uncommon in patients though its risks are low, they are not
younger than 20 years,19 so screening is inconsequential, and so typically nonin-
typically not recommended for children vasive imaging options are preferred for
and adolescents except in unusual initial screening. CT angiography and
circumstances. A screening imaging test magnetic resonance (MR) angiography
that fails to show an aneurysm does not are both associated with high specificity
rule out the possibility of subsequent in the detection of unruptured intracra-
aneurysm formation, especially since nial aneurysms. Sensitivity values range
the prevalence of aneurysms increases across studies, but CT angiography
with age. In high-risk subpopulations (sensitivity of 0.77 to 0.97) and MR
(eg, patients who have two or more angiography (0.69 to 0.99) are both felt
first-degree relatives with aneurysms, to have acceptable diagnostic test char-
patients with autosomal dominant poly- acteristics (see Table 8-3 for examples
cystic kidney disease), it may be rea- of sensitivities from one systematic
sonable to repeat noninvasive imaging review).1,21Y23 It is important to note
392 www.ContinuumJournal.com April 2014

Copyright © American Academy of Neurology. Unauthorized reproduction of this article is prohibited.


TABLE 8-3 Sensitivities of CT Angiography and MR Angiography for Aneurysm Detection
Based on Aneurysm Sizea,b

Detection of Detection of All Detection of Detection of


Aneurysms Aneurysms Aneurysms 93 mm Aneurysms
Imaging Study e3 mm 93 mm but G10 mmc Q10 mmc
CT angiography 61 (67/110) 96 (424/440) 93 (162/174) 100 (90/90)
51, 70 94,98 88, 96 96, 100
MR angiography 38 (20/52) 94 (239/254) 92 (178/193) 99 (81/82)
25, 53 90, 97 88, 96 93, 100
CT = computed tomography; MR = magnetic resonance.
a
Reprinted with permission from White PM, et al, Radiology.21 B 2000 Radiological Society of North America. pubs.rsna.org/doi/full/10.1148/
radiology.217.2.r00nv06361.
b
All data are sensitivity values, expressed as percentages. The numbers in parentheses are numbers of aneurysms, and the numbers on
the second line are 95% confidence intervals. The data on CT angiography were extracted from 12 of 16 studies with 556 subjects.
The data on MR angiography were extracted from 12 of 20 studies with 407 subjects.
c
From studies in which data on subdivisions of aneurysms Q3 mm could be extracted.

that the sensitivity of both modalities MANAGEMENT OF THE KEY POINT


drops considerably in the investigation UNRUPTURED ANEURYSM h The risk of aneurysm
of small aneurysms (less than 3 mm in Risk of Rupture rupture appears to
increase as aneurysm
diameter),23 although aneurysms of While other possible symptoms of size increases.
this size also have low rupture rates. unruptured intracranial aneurysms
The selection of the appropriate test were discussed in an earlier section,
should incorporate patient factors the major goal of unruptured aneu-
(eg, renal disease) and institutional rysm management is minimizing the
factors (eg, availability) as well; if re- risk of rupture and subsequent SAH.
peated screening is necessary, it may be There is a strong association between
reasonable to consider MR angiography aneurysm size and risk of rupture,
because of concerns about repeated shown in two large observational
radiation exposure associated with CT. studies. The International Study of
A recent cost-effectiveness analysis at- Unruptured Intracranial Aneurysms
tempted to address some of these po- (ISUIA)25 included a prospective ob-
tential questions for patients with at least servational cohort of 1692 patients in
two first-degree relatives with unruptured whom aneurysm treatment (surgery
intracranial aneurysms or SAH (ie, indi- or endovascular treatment) was not
viduals who appeared to be good candi- performed. Over 5 years, the risk of
dates for screening imaging). This analysis aneurysm rupture among aneurysms
used MR angiography as the initial nonin- in the anterior circulation (internal
vasive technique, followed by catheter carotid, middle cerebral, and anterior
angiography if needed to identify false cerebral arteries) was 0% in aneurysms
positives and to better characterize the less than 7 mm in diameter, 2.6% for
aneurysm. After varying the age to begin those 7 mm to 12 mm in diameter,
screening, the age to end screening, and 14.5% for those 13 mm to 24 mm in
the interval between imaging studies, the diameter, and 40% for diameters
authors determined that the optimal cost- greater than 25 mm. Across all diam-
effectiveness strategy was to begin screen- eter sizes, rupture risks were higher
ing at age 20 years and reimage every 7 for aneurysms in the posterior circu-
years until reaching 80 years of age.24 lation: 2.5% for less than 7 mm; 14.5%
Continuum (Minneap Minn) 2014;20(2):387–398 www.ContinuumJournal.com 393
Copyright © American Academy of Neurology. Unauthorized reproduction of this article is prohibited.
Unruptured Intracranial Aneurysms

for 7 mm to 12 mm; 18.4% for 13 mm between 5 mm and 6 mm in diameter


to 24 mm; and 50% for greater than (Figure 8-2).27 However, unlike ISUIA,
25 mm.25 Criticisms of the ISUIA data this study did not find an elevated risk of
include the fact that aneurysms of the rupture for aneurysms in the posterior
cavernous segment of the carotid circulation compared with those of
artery were included despite the fact similar size occurring in the anterior
that their intradural location prevents circulation. The study did show that,
the formation of SAH, as well as con- compared with unruptured intracranial
cerns regarding selection biases and aneurysms involving the middle cerebral
lower than expected rupture rates.26 artery, aneurysms involving the anterior
A large cohort of Japanese patients27 communicating or posterior communi-
with unruptured intracranial aneurysms cating arteries were more prone to
(6697 aneurysms in 5720 patients) rupture (hazard ratios 2.02 and 1.90,
showed some similarities to the ISUIA respectively). Finally, this cohort study
results but also some important differ- also showed an association between
ences. As in the ISUIA study, a clear rupture and the presence of a daughter
association was found between aneu- sac, an irregular protrusion from the
rysm size and risk of rupture. Specifically, aneurysmal wall, as opposed to a smooth
using aneurysms of 3 mm to 4 mm in wall (hazard ratio 1.63).27 Because this
diameter as a reference, the hazard ratio study involved only patients from Japan,
for aneurysm rupture was 3.35 for those the ability to generalize these results to
7 mm to 9 mm in diameter, 9.09 for other populations is uncertain.
those measuring 10 mm to 24 mm Despite these two large prospective
in diameter, and 76.26 for aneurysms studies, it still remains difficult to predict
25 mm or larger in diameter; no significant aneurysm rupture rates within individual
increase in risk was seen for aneurysms patients. In general, risks increase with

FIGURE 8-2 Probability of aneurysm rupture based on aneurysm size, as seen in a


large prospective cohort of Japanese patients.
27
Reprinted with permission from UCAS Japan Investigators, et al, N Engl J Med. B 2012
Massachusetts Medical Society. pubs.rsna.org/doi/full/10.1148/radiology.217.2.r00nv06361.

394 www.ContinuumJournal.com April 2014

Copyright © American Academy of Neurology. Unauthorized reproduction of this article is prohibited.


KEY POINTS
larger unruptured intracranial aneurysm enlarging, concerning features such as h Data on the risk of
size, although rupture rates were not daughter sacs are not developing, and aneurysm rupture based
zero for smaller aneurysms in the other aneurysms are not identified. on location are
Japanese cohort (0.36% annual risk for Again, highly regarded studies to guide inconsistent across
aneurysms measuring 3 mm to 4 mm in imaging in this setting are lacking, but a studies.
diameter).27 Aneurysms affecting the reasonable strategy may include noninva- h Certain anatomic
middle cerebral artery may have the sive imaging 6 to 12 months after initial features, such as the
lowest risk of rupture, and atypical wall discovery of an unruptured intracranial presence of a daughter
features are likely to increase risk. aneurysm, repeated annually until the sac, may increase the
In addition to the presence of a treatment team is confident that the risk of aneurysm
daughter sac, other anatomical fea- aneurysm has not changed significantly.31 rupture.
tures of aneurysms that possibly pre-
dict rupture are needed to guide Surgical/Endovascular
decisions regarding treatment. Recent Management
studies have looked at not only aneu- Full discussion of the surgical and
rysm diameter but also the ratio of the endovascular options for unruptured
aneurysm size to the size of the parent intracranial aneurysm management go
vessel.28,29 This size ratio was found to beyond the scope of this article, but
be especially predictive of aneurysm basic premises are reviewed here.
rupture in smaller aneurysms, which Historically, craniotomy followed by
are common and have a low but not surgical clipping was the preferred
0% rate of rupture.29 option, but endovascular aneurysm
treatment is now more common than
Medical Management surgeryVfrom 2001 through 2008, the
No high-quality randomized clinical trials proportion of unruptured intracranial
have been done to guide the medical aneurysms treated by endovascular
management of patients with un- techniques increased from 20% to
ruptured intracranial aneurysms. Given 63%.32 Figure 8-3 shows an example
the associations between (1) hyperten- of an aneurysm treated by endo-
sion and unruptured intracranial aneu- vascular coiling. Both options have
rysm growth and (2) larger unruptured their relative benefits and drawbacks,
intracranial aneurysm size and aneurysm and certain aneurysms may be best
rupture, treatment of hypertension is addressed by one strategy over the other.
generally recommended. National For example, aneurysms of the middle
guidelines from the Seventh Report of cerebral artery bifurcation are more read-
the Joint National Committee on the ily accessible via craniotomy and may be
Prevention, Detection, Evaluation, and more amenable to surgery compared
Treatment of High Blood Pressure with aneurysms in deeper locations (such
should be followed.30 Similarly, use of as the basilar artery apex). The diameter
tobacco or illicit drugs and excessive of the aneurysm neck may also play a role
alcohol consumption can lead to vascu- in treatment options, with wider-necked
lar injury and weaken blood vessel walls. aneurysms thought to be better treated
As part of good practice, patients with un- via surgical clipping; however, the devel-
ruptured intracranial aneurysms should opment of newer endovascular tech-
be counseled to avoid these habits. niques such as stent-assisted coiling now
If a conservative strategy is chosen, offer additional alternatives for wide-
follow-up noninvasive imaging should necked aneurysms.
be performed on a periodic basis to To further complicate decision mak-
ensure that the aneurysm is not rapidly ing, data regarding neurologic and other
Continuum (Minneap Minn) 2014;20(2):387–398 www.ContinuumJournal.com 395
Copyright © American Academy of Neurology. Unauthorized reproduction of this article is prohibited.
Unruptured Intracranial Aneurysms

KEY POINT
h If indicated,
interventional options
for unruptured
intracranial aneurysm
management include
endovascular and
surgical approaches;
large randomized trial
data to support one
strategy over the other
are lacking.

FIGURE 8-3 Catheter angiogram images showing a 6 mm aneurysm arising from the
origin of the left superior cerebellar artery, prior to treatment (A, arrow)
and following an endovascular coiling procedure (B).

functional outcomes after unruptured higher inpatient mortality among pa-


intracranial aneurysm treatment are of tients treated with surgery compared
limited quality because no randomized with those treated with endovascular
controlled trials have compared surgical techniques (1.2% versus 0.6%, respec-
and endovascular techniques for tively).32 The rate at which patients
unruptured intracranial aneurysms. Ret- were admitted to a long-term nursing
rospective or nonrandomized data are facility following hospitalization was
available from the ISUIA study, analyses also considerably higher for surgically
of the Nationwide Inpatient Sample treated patients than for those treated
(a large database of hospitalizations at by endovascular means (14.0% versus
institutions throughout the United 4.9%, respectively).32 Detailed infor-
States),33 and other sources. In the mation regarding patient-level and
ISUIA study, mortality rates at 1 year aneurysm-specific characteristics was
were 2.3% in the surgical group and not included in this study.
3.1% in the endovascular group; signif- Although not 100% effective, surgical
icant morbidity rates at 1 year were clipping may lead to a more definitive
9.8% among surgical patients and 6.4% eradication of the aneurysm, assuming
among patients treated by endovascular clip placement is appropriate, while
means.25 Treatment assignment was aneurysm coils may become compacted
not randomized, and it is difficult to over time and result in flow within the
make strong conclusions on these aneurysm neck again. Although the In-
values given this fact, as well as some ternational Subarachnoid Aneurysm Trial
differences in baseline characteristics (ISAT) was a study of outcomes following
between groups. Specifically, patients ruptured as opposed to unruptured
in the endovascular arm were older, aneurysms, late rebleeding events in this
had larger aneurysm sizes, and were study were seen more commonly in
more likely to have basilar artery aneu- patients treated by endovascular coiling
rysms; tobacco use and a family history than in those treated with clipping.34
of aneurysms were seen more com- The costs of either endovascular or
monly in surgically treated patients. surgical treatment of unruptured in-
A retrospective analysis of the Na- tracranial aneurysms are considerable:
tionwide Inpatient Sample found an analysis of Medicare patients from

396 www.ContinuumJournal.com April 2014

Copyright © American Academy of Neurology. Unauthorized reproduction of this article is prohibited.


2001 to 2008 showed that without 6. Vlak MH, Algra A, Brandenburg R, Rinkel GJ.
Prevalence of unruptured intracranial
complications, endovascular treat- aneurysms, with emphasis on sex, age,
ment was associated with approxi- comorbidity, country, and time period: a
mately $26,000 in costs, versus systematic review and meta-analysis. Lancet
Neurol 2011;10(7):626Y636.
approximately $23,500 for surgical
treatment (in 2008 US dollars).35 7. Rinkel GJE, Djibuti M, Algra A, van Gijn J.
Prevalence and risk of rupture of intracranial
However, given the significant mor- aneurysms: a systematic review. Stroke
bidity and mortality associated with 1998;29(1):251Y256.
aneurysmal rupture, an analysis found 8. Mackey J, Brown RD Jr, Moomaw CJ, et al.
treatment to be cost-effective for an- Familial intracranial aneurysms: is anatomic
eurysms with estimated annual rup- vulnerability heritable? Stroke 2013;44(1):
38Y42.
ture rates as low as 0.3%.36
9. Broderick JP, Brown RD, Sauerbeck L, et al.
Greater rupture risk for familial as compared
CONCLUSION to sporadic unruptured intracranial
aneurysms. Stroke 2009;40(6):1952Y1957.
Unruptured aneurysms are fairly prev-
10. Alg VS, Sofat R, Houlden H, Werring DJ.
alent and commonly identified Genetic risk factors for intracranial aneurysms:
through noninvasive brain imaging a meta-analysis in more than 116,000
studies. Certain populations, especially individuals. Neurology 2013;80(23):2154Y2165.
patients who have multiple family 11. Connolly HM, Huston J 3rd, Brown RD Jr,
members with aneurysms and patients et al. Intracranial aneurysms in patients with
coarctation of the aorta: a prospective
with polycystic kidney disease, are at magnetic resonance angiographic study of
especially high risk of unruptured 100 patients. Mayo Clin Proc 2003;78(12):
intracranial aneurysms and should be 1491Y1499.
screened with either CT angiography 12. Touze E, Oppenheim C, Trystram D, et al.
or MR angiography. Once an aneu- Fibromuscular dysplasia of cervical and
intracranial arteries. Int J Stroke 2010;5(4):
rysm is identified, management op- 296Y305.
tions include observation or surgical/
13. Slovut DP, Olin JW. Fibromuscular dysplasia.
endovascular treatment; if surgical or N Engl J Med 2004;350(18):1862Y1871.
endovascular options are being con-
14. Xu HW, Yu SQ, Mei CL, Li MH. Screening for
templated, referral to a comprehen- intracranial aneurysm in 355 patients with
sive cerebrovascular center with autosomal-dominant polycystic kidney
expertise in these treatment options disease. Stroke 2011;42(1):204Y206.
should be considered. 15. Chalhoub V, Abi-Rafeh L, Hachem K, et al.
Intracranial aneurysm and recessive polycystic
kidney disease: the third reported case.
REFERENCES JAMA Neurol 2013;70(1):114Y116.
1. Brisman JL, Song JK, Newell DW. Cerebral 16. van Gijn J, Kerr RS, Rinkel GJ. Subarachnoid
aneurysms. N Engl J Med 2006;355(9):928Y939. haemorrhage. Lancet 2007;369(9558):
2. Lehecka M, Dashti R, Lehto H, et al. Distal 306Y318.
anterior cerebral artery aneurysms. Acta 17. Suarez JI, Tarr RW, Selman WR. Aneurysmal
Neurochir Suppl 2010;107:15Y26. subarachnoid hemorrhage. N Engl J Med
3. Lou M, Caplan LR. Vertebrobasilar dilatative 2006;354(4):387Y396.
arteriopathy (dolichoectasia). Ann N Y Acad
18. Bederson JB, Awad IA, Wiebers DO, et al.
Sci 2010;1184(1):121Y133.
Recommendations for the management of
4. Kannoth S, Thomas S. Intracranial microbial patients with unruptured intracranial
aneurysm (infectious aneurysm): current aneurysms: a statement for healthcare
options for diagnosis and management. professionals from the Stroke Council of the
Neurocrit Care 2009;11(1):120Y129. American Heart Association. Circulation
2000;102(18):2300Y2308.
5. Rinkel GJE. Intracranial aneurysm screening:
indications and advice for practice. Lancet 19. Jordan LC, Johnston SC, Wu YW, et al. The
Neurol 2005;4(2):122Y128. importance of cerebral aneurysms in childhood

Continuum (Minneap Minn) 2014;20(2):387–398 www.ContinuumJournal.com 397


Copyright © American Academy of Neurology. Unauthorized reproduction of this article is prohibited.
Unruptured Intracranial Aneurysms

hemorrhagic stroke: a population-based 29. Kashiwazaki D, Kuroda S. Size ratio can


study. Stroke 2009;40(2):400Y405. highly predict rupture risk in intracranial
small (G5 mm) aneurysms. Stroke
20. Wermer MJ, Rinkel GJ, van Gijn J. Repeated
2013;44(8):2169Y2173.
screening for intracranial aneurysms in
familial subarachnoid hemorrhage. Stroke 30. Lenfant C, Chobanian AV, Jones DW,
2003;34(12):2788Y2791. Roccella EJ. Seventh report of the Joint
National Committee on the Prevention,
21. White PM, Wardlaw JM, Easton V. Can
Detection, Evaluation, and Treatment of
noninvasive imaging accurately depict
High Blood Pressure (JNC 7): resetting the
intracranial aneurysms? A systematic review.
hypertension sails. Hypertension
Radiology 2000;217(2):361Y370.
2003;41(6):1178Y1179.
22. Chappell ET, Moure FC, Good MC. Comparison
31. Williams LN, Brown RD. Management of
of computed tomographic angiography with
unruptured intracranial aneurysms. Neurol
digital subtraction angiography in the
Clin Pract 2013;3(2):99Y108.
diagnosis of cerebral aneurysms: a
meta-analysis. Neurosurgery 2003;52(3): 32. Brinjikji W, Rabinstein AA, Nasr DM, et al.
624Y631; discussion 630Y631. Better outcomes with treatment by coiling
relative to clipping of unruptured intracranial
23. White PM, Teasdale EM, Wardlaw JM,
aneurysms in the United States, 2001-2008.
Easton V. Intracranial aneurysms: CT
AJNR Am J Neuroradiol 2011;32(6):
angiography and MR angiography for
1071Y1075.
detection prospective blinded comparison in
a large patient cohort. Radiology 2001;219(3): 33. Healthcare Cost and Utilization Project.
739Y749. Overview of the Nationwide Inpatient
Sample (NIS). www.hcup-us.ahrq.gov/
24. Bor AS, Koffijberg H, Wermer MJ, Rinkel GJ.
nisoverview.jsp. Updated December 11,
Optimal screening strategy for familial
2013. Accessed December 26, 2013.
intracranial aneurysms: a cost-effectiveness
analysis. Neurology 2010;74(21):1671Y1679. 34. Molyneux AJ, Kerr RSC, Birks J, et al. Risk of
recurrent subarachnoid haemorrhage,
25. Wiebers DO, Whisnant JP, Huston J 3rd,
death, or dependence and standardised
et al. Unruptured intracranial aneurysms:
mortality ratios after clipping or coiling of
natural history, clinical outcome, and risks of
an intracranial aneurysm in the International
surgical and endovascular treatment. Lancet
Subarachnoid Aneurysm Trial (ISAT):
2003;362(9378):103Y110.
long-term follow-up. Lancet Neurol 2009;8(5):
26. White PM, Wardlaw J. Unruptured 427Y433.
intracranial aneurysms: prospective data
35. Brinjikji W, Kallmes DF, Lanzino G, Cloft HJ.
have arrived. Lancet 2003;362(9378):90Y91.
Hospitalization costs for endovascular and
27. UCAS Japan Investigators; Morita A, Kirino surgical treatment of unruptured cerebral
T, Hashi K, et al. The natural course of aneurysms in the United States are
unruptured cerebral aneurysms in a substantially higher than medicare payments.
Japanese cohort. N Engl J Med 2012;366(26): AJNR Am J Neuroradiol 2012;33(1):49Y51.
2474Y2482.
36. Greving JP, Rinkel GJ, Buskens E, Algra A.
28. Dhar S, Tremmel M, Mocco J, et al. Cost-effectiveness of preventive treatment
Morphology parameters for intracranial of intracranial aneurysms: new data and
aneurysm rupture risk assessment. Neurosurgery uncertainties. Neurology 2009;73(4):
2008;63(2):185Y196; discussion 185Y196. 258Y265.

398 www.ContinuumJournal.com April 2014

Copyright © American Academy of Neurology. Unauthorized reproduction of this article is prohibited.


Review Article

Genetic Stroke
Address correspondence to
Dr Kevin Barrett, Mayo Clinic,
4500 San Pablo Road South,
Cannaday 2EVNeurology,

Syndromes Jacksonville, Florida 32224-1865,


barrett.kevin@mayo.edu.
Relationship Disclosure:
Kevin M. Barrett, MD, MSc; James F. Meschia, MD Dr Barrett receives support
from the National Institute of
Neurological Disorders and
Stroke for his role in the
ABSTRACT Stroke Hyperglycemia Insulin
Network Effort (SHINE) trial
Purpose of Review: This review describes the clinical and radiographic features, and has received a speaking
genetic determinants, and treatment options for the most well-characterized honorarium for a Lee Memorial
monogenic disorders associated with stroke. Health System continuing
medical education course.
Recent Findings: Stroke is a phenotype of many clinically important inherited Dr Meschia receives support
disorders. Recognition of the clinical manifestations of genetic disorders associated from the National Institute of
with stroke is important for accurate diagnosis and prognosis. Genetic studies have Neurological Disorders and
Stroke for his roles in the
led to the discovery of specific mutations associated with the clinical phenotypes of Carotid Revascularization
many inherited stroke syndromes. Endarterectomy versus Stenting
Summary: Several inherited causes of stroke have established and effective Trial (CREST) and the Stroke
Genetics Network (SiGN).
therapies, further underscoring the importance of timely diagnosis. Unlabeled Use of
Products/Investigational
Continuum (Minneap Minn) 2014;20(2):399–411. Use Disclosure:
Drs Barrett and Meschia
report no disclosures.
* 2014, American Academy
of Neurology.
INTRODUCTION vertebrobasilar circulation, dolichoectasia
Ischemic stroke is a heterogenous of cerebral vessels (Figure 9-11), and
disorder with variable clinical presen- white matter abnormalities on brain
tations and a multitude of risk factors MRI.2 Other CNS manifestations include
and causative mechanisms. In many TIA, vertigo, hearing impairment, tinni-
cases, despite a comprehensive evalu- tus, and cognitive disturbances. Peripheral
ation, the stroke mechanism is not nervous system involvement manifests as
clearly elucidated. In cryptogenic small fiber peripheral neuropathy with
stroke and in patients with a family painful acroparesthesias, hypohidrosis,
history of stroke, it is particularly impaired temperature sensation, and in-
important to consider inherited syn- testinal dysmotility. Sensory and motor
dromes in the differential diagnosis. nerve conduction studies and EMG are
Recognition of clinical phenotypes typically normal.3 Major systemic mani-
associated with monogenic disorders festations include cardiac conduction
known to cause ischemic stroke can disturbances, cardiomyopathy, renal fail-
lead to specific testing for inherited ure, angiokeratomas, and corneal dystro-
syndromes. Establishing a definitive phy. Radiographically, hyperintensity in
diagnosis has both prognostic and the pulvinar region on T1-weighted
therapeutic implications for affected MRI sequences has been described in
patients and their families. addition to white matter abnormalities
(Figure 9-24).5 MRI or magnetic reso-
MONOGENIC DISORDERS nance (MR) angiography can show tortu-
ASSOCIATED WITH STROKE osity and dilatation of cerebral vessels.
Fabry Disease Fabry disease is caused by genetic
The cerebrovascular manifestations of mutations that result in deficient-
Fabry disease include ischemic stroke at activity of the lysosomal hydrolase
an early age, often involving the !-galactosidase A (GLA) (Case 9-1).
Continuum (Minneap Minn) 2014;20(2):399–411 www.ContinuumJournal.com 399
Copyright © American Academy of Neurology. Unauthorized reproduction of this article is prohibited.
Genetic Stroke Syndromes

KEY POINTS
h The cerebrovascular
manifestations of Fabry
disease include ischemic
stroke at an early age,
often involving the
vertebrobasilar
circulation, dolichoectasia
of cerebral vessels, and
white matter
abnormalities on brain
MRI.
h Fabry disease is caused
by genetic mutations
that result in deficient
activity of the lysosomal
hydrolase
!-galactosidase A. FIGURE 9-1 Vertebrobasilar dolichoectasia in Fabry disease. A, Magnetic resonance
angiography demonstrating tortuosity of the vertebrobasilar system. B,
T2-weighted MRI through the dolichoectatic vertebral-basilar junction (arrow).
Reprinted with permission from Flemming KD, et al, Cerebrovasc Dis.1 B 2005 S. Karger AG.
www.karger.com/Article/FullText/87710.

More than 300 distinct mutations have as globotriaosylceramide, in vascular en-


been identified. Inheritance is X-linked, dothelium, smooth-muscle cells, and au-
with men harboring the major disease tonomic and dorsal root ganglia. Lipid
manifestations and women being mildly deposits probably contribute to vessel oc-
affected carriers. Fabry disease is catego- clusion and tissue ischemia; however, the
rized as a lysosomal storage disorder with exact pathogenetic mechanism of cere-
GLA inactivity resulting in progressive bral infarction has yet to be elucidated.
accumulation of glycosphingolipids, such Dolichoectasia of the larger cerebral

FIGURE 9-2 Pulvinar sign in Fabry disease. Magnetic resonance findings in the posterior
thalamus (pulvinar). T1-weighted images through the thalamus in three patients
with mild (A), moderate (B), and marked (C) hyperintensity.
Reprinted with permission from Moore DF, et al, AJNR AM J Neuroradiol.4 B 2003 American
Society of Neuroradiology. www.ajnr.org/content/24/6/1096.long.

400 www.ContinuumJournal.com April 2014

Copyright © American Academy of Neurology. Unauthorized reproduction of this article is prohibited.


Case 9-1
A 21-year-old man presented to the emergency department reporting
bilateral blurry vision. The visual symptoms began abruptly 2 days earlier
and were preceded by a thunderclap headache the week before. The
patient admitted to using marijuana daily. The neurologic examination was
notable for uncorrected near visual acuity of 20/30 bilaterally. A right
homonymous hemianopia was found on confrontational visual field
testing. He was not able to read simple or complex sentences. Writing,
fluency, repetition, comprehension, and object naming were intact. Examination
showed normal gait and motor, sensory, and cerebellar function.
The abrupt onset of symptoms prompted consideration of a
vascular cause, such as ischemic or hemorrhagic stroke. The heralding
thunderclap headache raised the possibility of subarachnoid hemorrhage,
cervicocephalic dissection, cerebral
venous thrombosis, or reversible
cerebral vasoconstrictive syndrome.
Primary or secondary complications
related to marijuana or other
illicit substance abuse were also
considered. Laboratory testing was
remarkable for a serum creatinine
of 1.2 mg/dL, and ECG demonstrated
a right bundle branch block. Brain
MRI revealed multifocal areas of
acute infarction predominantly
involving the vertebrobasilar
circulation (Figure 9-3). Serum
!-galactosidase A (GLA) activity
was found to be zero, strongly
suggestive of Fabry disease.
Comment. This patient had
multisystem manifestations of
Fabry disease, including abnormal
cardiac conduction, cerebrovascular FIGURE 9-3 Fabry disease. Axial
diffusion-weighted MRI
involvement, and early renal sequence obtained at
dysfunction. At follow-up, the level of the thalami. Multifocal,
additional family history revealed bihemispheric regions of restricted
diffusion are seen with notable
that the patient’s sister had involvement of the calcarine cortex and
developed progressive visual splenium of the corpus callosum.
deterioration and painful Infarction of the splenium resulted in
the disconnection syndrome alexia
acroparesthesias during without agraphia, evident on
adolescence. A maternal half uncle neurologic examination.
developed renal failure requiring
hemodialysis at 32 years of age.
The patient was later able to recall a personal history of painful
acroparesthesia beginning in middle school. He was referred to a genetics
counselor; molecular testing was not believed to be necessary given
the family history and undetectable GLA activity. He was referred for
hematologic, ophthalmic, cardiologic, nephrologic, and dermatologic
evaluations. Enzyme replacement therapy was recommended.

Continuum (Minneap Minn) 2014;20(2):399–411 www.ContinuumJournal.com 401


Copyright © American Academy of Neurology. Unauthorized reproduction of this article is prohibited.
Genetic Stroke Syndromes

KEY POINTS
h Recombinant vessels may result from mechanical associated with costs per additional
!-galactosidase A is weakening of the vessel wall due to quality-adjusted life-year ranging from
available as enzyme glycosphingolipid deposition. €5.5 million to €7.5 million (US
replacement therapy for Measurement of leukocyte GLA $7,377,700 to $10,060,500). Identification
Fabry disease and activity can be used for diagnosis and of those patients most likely to benefit
should be considered is a standard enzymatic test in most from enzyme replacement therapy could
for affected individuals. laboratories. Sensitivity and specificity potentially enhance cost-effectiveness.
h Sickle cell disease is of the GLA activity assay approaches Clinicians must have a high index of
complicated by stroke in 100% in men, but only identifies about suspicion for Fabry disease because of its
25% of affected 50% of female carriers, where levels vague and variable presentation with
individuals before the range from normal to low. Molecular multiorgan system dysfunction requiring
age of 45 years. genetic testing can be used to diagnose the need for comprehensive multispe-
h Sickle cell disease is women or men who have marginal cialty care for optimal management.
caused by a point GLA activity. Skin biopsy or culture of
mutation that results in skin fibroblasts can help establish the Sickle Cell Disease
substitution of valine for diagnosis if other means are not avail- Sickle cell disease is complicated by
glutamic acid in position able. A diagnosis of Fabry disease stroke in 25% of affected individuals
number 6 of the should be considered when evaluating before the age of 45 years.8 Ischemic
"-polypeptide chain of stroke is most common when stroke
stroke in the young, particularly if the
hemoglobin.
patient has a family history of stroke onset occurs before the age of 20
and the posterior circulation is involved. years; hemorrhagic stroke is more
Once a diagnosis is established, addi- common when stroke occurs after
tional studies should include urinalysis the age of 20 years. The initial peak
for proteinuria, measurement of renal incidence of stroke occurs between 2
function, and an ECG to evaluate for years and 5 years of age. Recurrent
cardiac involvement. stroke risk is high and occurs in about
Recombinant GLA is available as two-thirds of patients within 2 years of
enzyme replacement therapy for Fabry the index event.9 A nonatherosclerotic
disease and should be considered for cerebral vasculopathy causing stenosis
affected individuals. Replacement ther- and occlusion of the proximal cerebral
apy has been shown to decrease left ven- vessels can be identified with nonin-
tricular mass, reduce pain, and increase vasive vascular imaging techniques,
quality of life.6 Reduction of stroke risk, including MR angiography, CT angiog-
progression of disease, or mortality has raphy, and transcranial Doppler (TCD)
not been established. Enzyme replace- ultrasonography. In some patients, a
ment therapy is typically started as soon network of lenticulostriate collaterals
as possible after diagnosis and adminis- develops, creating a moyamoya appear-
tered as an IV infusion (agalsidase alpha ance on conventional angiography. Cog-
0.2 mg/kg or agalsidase beta 1 mg/kg) nitive and behavioral changes may occur
every 2 weeks and continued indefinitely. as a result of silent cerebral infarction.
Treatment is expensive, with estimates of Systemic manifestations include vaso-
retail cost exceeding $200,000 per year. occlusive crises causing chest, back,
Cost-effectiveness of enzyme replace- and extremity pain, compensated hemo-
ment therapy compared with standard lytic anemia, and jaundice.
medical care has been evaluated recently Sickle cell disease is caused by a point
in a Dutch cohort of patients with Fabry mutation that results in substitution of
disease.7 In this cohort, treatment had valine for glutamic acid in position
limited effect on quality of life and pro- number 6 of the "-polypeptide chain of
gression to end-organ damage and was hemoglobin. The point mutation results

402 www.ContinuumJournal.com April 2014

Copyright © American Academy of Neurology. Unauthorized reproduction of this article is prohibited.


KEY POINT
in abnormal aggregation of hemoglobin, 200 cm/sec, rescreening in 2 to 4 weeks h Children with sickle cell
causing changes in red blood cell shape can identify children with persistently disease should be
and rheology. Polymerization of deoxy- elevated velocities that benefit from routinely screened with
genated hemoglobin, changes in red prophylactic transfusion therapy. transcranial Doppler,
blood cell membrane structure and A retrospective cohort study of chil- and those with mean
function, disordered cell volume, and dren with sickle cell disease and acute flow velocities greater
increased adherence to vascular endo- stroke (less than 24 hours from symp- than 200 cm/sec should
thelium play a role in the pathophysiol- tom onset) compared simple transfusion receive chronic
ogy of sickle cell disease.10 Inheritance is with exchange transfusion at the time of transfusion therapy to
autosomal recessive, and the disease is clinical stroke and found that children reduce the hemoglobin
S fraction to less than
most prevalent in individuals of African receiving simple transfusion had a five-
30%.
or African American descent. fold greater relative risk of recurrent
Measurement of mean blood flow stroke than those receiving exchange
velocity in the proximal middle cerebral transfusion therapy.13 Hydroxyurea,
artery or distal internal carotid artery by which increases fetal hemoglobin pro-
TCD can identify children at high risk for duction, has been utilized empirically as
stroke. Mean blood flow velocity greater an alternative to chronic transfusion
than 200 cm/sec is associated with in- therapy for secondary prevention. The
creased stroke risk. The Stroke Preven- Stroke with Transfusions Changing to
tion Trial in Sickle Cell Anemia study Hydroxyurea trial randomized children
randomized children without a history with sickle cell disease and stroke with
of stroke and with TCD velocities greater evidence of iron overload from chronic
than 200 cm/sec to observation or pro- transfusion to treatment with either
phylactic transfusion therapy targeted to continued transfusion and iron chelation
a hemoglobin S fraction of less than or hydroxyurea and phlebotomy.14 No
30%.11 The study was terminated after strokes occurred in the 66 subjects
enrollment of 130 patients demon- assigned to transfusion and chelation
strated a 92% reduction in stroke risk therapy compared with seven strokes
(PG.001) in the transfusion group. Trans- in the 67 subjects assigned to hydroxy-
fusion therapy also reduced the risk of urea and phlebotomy. Based on these
new silent cerebral infarction and stroke results, simple or exchange trans-
in a subgroup with evidence of prior fusion with chelation remains the op-
silent infarction on the initial brain MRI. A timal way to manage children with sickle
follow-up study found six additional cell disease, stroke, and iron overload.
stroke cases, with five occurring in the Discontinuation of transfusion therapy in
group that stopped transfusion therapy.12 children with sickle cell disease causes
The main risk associated with chronic high rates of reversion to abnormal intra-
transfusion therapy is iron overload that cranial blood flow velocities and
can be effectively managed with iron stroke.15 A challenge in managing these
chelation therapy. cases is that it is not known when, or if,
Children with sickle cell disease children grow out of the need for
should be routinely screened with TCD, transfusions.
and those with mean flow velocities
greater than 200 cm/sec should receive Cerebral Autosomal Dominant
exchange transfusion therapy to reduce Arteriopathy With Subcortical
the hemoglobin S fraction to less than Infarcts and
30%. In practice, screening is often per- Leukoencephalopathy
formed annually beginning at age 2 Recurrent subcortical brain infarction
years. If blood flow velocities exceed and TIA are common in cerebral
Continuum (Minneap Minn) 2014;20(2):399–411 www.ContinuumJournal.com 403
Copyright © American Academy of Neurology. Unauthorized reproduction of this article is prohibited.
Genetic Stroke Syndromes

KEY POINTS
h T2 hyperintensities autosomal dominant arteriopathy with changes in the medium and large
involving the white subcortical infarcts and leuko- cervicocephalic vessels are typically
matter of the anterior encephalopathy (CADASIL) and occur minimal or absent.
temporal poles in up to 85% of symptomatic individuals. CADASIL is associated with muta-
(O’Sullivan sign) are A retrospective cohort study found that tions in the NOTCH3 gene on chro-
seen in 90% of patients median age at stroke onset was 51 years mosome 19p13.2-p13.1, which codes
with cerebral autosomal in men and 53 years in women.16 for a large transmembrane protein
dominant arteriopathy Ischemic stroke may present with classic necessary for vascular smooth muscle
with subcortical small vessel lacunar syndromes, includ- differentiation and development. In-
infarcts and ing pure motor stroke, pure sensory heritance is autosomal dominant, with
leukoencephalopathy.
stroke, dysarthriaYclumsy hand, and affected family members in successive
h Cerebral autosomal ataxic-hemiparesis. Recurrent stroke can generations evident on pedigree anal-
dominant arteriopathy lead to cognitive decline, dementia, gait ysis. More than 80 different NOTCH3
with subcortical infarcts abnormalities, urinary incontinence, and mutations, the majority occurring in
and leukoencephalopathy
pseudobulbar affect. Migraine with aura the extracellular region of the recep-
is associated with
occurs in about 30% of cases and is an tor within an epidermal growth
mutations in the NOTCH3
gene on chromosome
early sign with onset often in the third factorYlike repeat domain, have been
19p13.2-p13.1, which decade of life (Case 9-2).17 Stroke identified in patients with CADASIL.21
codes for a large and vascular cognitive impairment are The underlying mechanism through
transmembrane protein the major causes of morbidity and which NOTCH3 mutations result in
necessary for vascular mortality. the clinical phenotype has not been
smooth muscle Although symptom onset typically fully elucidated. Transgenic mice ex-
differentiation and occurs in adulthood, brain MRI abnor- pressing a vascular NOTCH3 mutation
development. malities precede the onset of symp- or knockout mutation demonstrate
h False-negative results toms and are a useful screening tool enhanced cortical spreading depres-
can occur with genetic for diagnosis for both symptomatic sion, which may explain the co-
analysis and should and presymptomatic carriers. T2 prevalence of migraine with aura in
prompt skin biopsy to hyperintensities involving the white patients with CADASIL.22 Molecular
evaluate for granular matter of the anterior temporal poles genetic testing can establish the diag-
osmophilic material in (O’Sullivan sign) are seen in 90% of nosis in patients with a family history
the vascular basal patients with CADASIL. A similar pat- of stroke or dementia and clinical and
lamina, a finding highly
tern of regional involvement is un- radiographic features suggestive of
specific for cerebral
common in small vessel ischemia due CADASIL. False-negative results can
autosomal dominant
arteriopathy with
to hypertension or other traditional occur with genetic analysis and should
subcortical infarcts and cerebrovascular risk factors.18 Signal prompt skin biopsy to evaluate for
leukoencephalopathy. abnormalities in the extreme capsule granular osmophilic material in the
and corpus callosum are also distinc- vascular basal lamina, a finding highly
tive of CADASIL. Many patients with specific for CADASIL.
CADASIL have cerebral microbleeds.19 Specific treatment for CADASIL is
Gradient-recall echo sequences on not currently available. Aggressive
brain MRI can identify these areas of management of cerebrovascular risk
hemosiderin deposition with high factors such as hypertension, hyper-
sensitivity; however, microbleeds are lipidemia, and diabetes mellitus is
not unique to CADASIL as they occur likely valuable given that CADASIL is
in patients with cerebral amyloid angio- primarily a small vessel arteriopathy.
pathy and hypertensive vasculopathy. Antiplatelet therapy and migraine pro-
The MRI finding that correlates best phylaxis have been utilized in clinical
with clinical impairment in CADASIL practice, but neither has been clearly
is brain atrophy.20 Atherosclerotic established as beneficial in CADASIL.
404 www.ContinuumJournal.com April 2014

Copyright © American Academy of Neurology. Unauthorized reproduction of this article is prohibited.


Case 9-2
A 29-year-old woman presented for a second neurologic opinion for her history of recurrent neurologic
symptoms and stroke. Three years before presentation, she developed intermittent headaches
characterized by sharp, stabbing, and burning pain. Her headaches were occasionally accompanied by
nausea and impairment of peripheral vision bilaterally. She had an episode of right-sided weakness
that occurred 9 months ago. Six months ago, she suddenly began to vomit while washing dishes at
home. This was followed by headache and left-sided weakness, predominantly involving the arm and
hand. She also reported difficulty with concentration, memory, and multitasking at that time. She was
hospitalized and diagnosed with an ischemic stroke. Her mother and brother both had a history of
stroke. Neurologic examination demonstrated a left hemiparesis, reduced sensation on her left side,
and hyperreflexia in the left upper and lower extremities. Review of the brain MRI performed during
her hospitalization was remarkable for changes consistent with acute ischemic stroke. Additionally, T2
abnormalities involving the anterior temporal poles were seen, highly characteristic of cerebral
autosomal dominant arteriopathy with subcortical infarcts and leukoencephalopathy (CADASIL)
(Figure 9-4). Molecular genetic testing revealed a DNA sequence variation in the NOTCH3 gene.

FIGURE 9-4 Cerebral autosomal dominant arteriopathy with subcortical infarcts and
leukoencephalopathy (CADASIL). Axial fluid-attenuated inversion recovery (FLAIR)
MRI in a 29 year-old woman demonstrating T2 signal abnormalities in the temporal
polar white matter (A, B [arrows], C) and lateral ventricular margins (D, E, F) exceeding what
would be expected for a patient of this age.

Comment. The progression of symptoms from migraine with aura to TIA and stroke in a young person,
followed by symptoms of cognitive dysfunction, should lead to consideration of CADASIL, particularly
in patients with a family history of stroke or dementia. Missense mutations identified in the NOTCH3
gene resulting in gain or loss of a cysteine residue within one of the 23 exons encoding the 34 epidermal
growth factorYlike repeat domains of the protein have been associated with 90% of CADASIL cases.

Continuum (Minneap Minn) 2014;20(2):399–411 www.ContinuumJournal.com 405


Copyright © American Academy of Neurology. Unauthorized reproduction of this article is prohibited.
Genetic Stroke Syndromes

KEY POINTS
h Stroke with onset in the Cerebral Autosomal Recessive brain MRI demonstrates an enhancing
third decade, and Arteriopathy With Subcortical tumorlike lesion with cortical sparing
dementia with onset in Infarcts and that may resemble a primary CNS
the third through fifth Leukoencephalopathy malignancy. Small white matter lesions
decade, are characteristic Stroke with onset in the third decade, have also been described, which may
of cerebral autosomal and dementia with onset in the third resemble demyelinating lesions. Anec-
recessive arteriopathy through fifth decade, are characteristic dotally, spontaneous regression of the
with subcortical infarcts of cerebral autosomal recessive tumorlike lesions has been observed
and leukoencephalopathy. (Figure 9-5).25
arteriopathy with subcortical infarcts
Systemic manifestations Mutations in the TREX1 gene have
and leukoencephalopathy (CARASIL).
include premature
Systemic manifestations include prema- been associated with RVCL. TREX1
alopecia with onset in the
ture alopecia with onset in the teen years encodes a DNA exonuclease, and pa-
teen years and cervical or
lumbar spondylosis in the and cervical or lumbar spondylosis in the tients with RVCL have frame-shift muta-
second and third decades second and third decades of life. Hyper- tions affecting the C-terminal domain
of life. tension is not associated with develop- that result in a truncated protein
ment of subcortical infarction in product that prevents normal translo-
h Vision and memory loss,
seizures, hemiparesis, CARASIL. Linkage analysis has identified cation into the cell nucleus.26 The
apraxia, and dysarthria mutations in the high-temperature re- molecular pathophysiology has yet to
with onset in the fourth quirement A serine peptidase 1 (HTRA1) be elucidated; however, clinical mani-
decade of life followed gene on chromosome 10q as the cause festations are the result of a cerebral
by death 5 to 10 years of CARASIL.23 HTRA1 binds and cleaves and retinal microvasculopathy. Inheri-
after clinical the prodomain of proYtransforming tance is autosomal dominant, with
presentation is the growth factor "-1 (pro-TGF-"1), affected family members evident in
typical disease and the cleaved product is degraded successive generations. The prolifera-
progression in retinal tive retinopathy may respond to
by the endoplasmic reticulum. This
vasculopathy with intravitreal bevacizumab.27
mechanism regulates the amount of
cerebral
leukodystrophy.
mature TGF-"1.24 No disease-specific
therapy for CARASIL is available at
this time. Mitochondrial Encephalopathy,
Lactic Acidosis, and Strokelike
Episodes
Retinal Vasculopathy With Cerebrovascular manifestations of mi-
Cerebral Leukodystrophy tochondrial encephalopathy, lactic ac-
Retinal vasculopathy with cerebral leu- idosis, and strokelike episodes
kodystrophy (RVCL) has also been (MELAS) are characterized by stroke,
called cerebroretinal vasculopathy syn- often with onset before the age of
drome; hereditary vascular retinopa- 40 years, resulting in hemiparesis,
thy; or hereditary endotheliopathy, hemianopia, or cortical blindness.
retinopathy, nephropathy, and stroke Other neurologic signs and symptoms
(HERNS). Vision and memory loss, include focal and generalized seizures,
seizures, hemiparesis, apraxia, and dementia, recurrent migrainelike
dysarthria with onset in the fourth headaches, and muscle weakness. Sys-
decade of life followed by death 5 to temic manifestations include short
10 years after clinical presentation is stature, hearing loss, recurrent
the typical disease progression in vomiting, and diabetes mellitus.
RVCL. The retinopathy is character- MELAS often presents in childhood
ized by neovascularization of the optic after normal development, and the
disc, retinal hemorrhages, and macu- clinical course can be relapsing-
lar edema. In about 50% of patients, remitting with eventual development

406 www.ContinuumJournal.com April 2014

Copyright © American Academy of Neurology. Unauthorized reproduction of this article is prohibited.


KEY POINT
h The mitochondrial
encephalopathy, lactic
acidosis, and strokelike
episodes disorder often
presents in childhood
after normal
development, and the
clinical course can be
relapsing-remitting with
eventual development
of progressive
neurologic disability.

FIGURE 9-5 Retinal vasculopathy with cerebral leukodystrophy. Sequential axial MRI
showing an ovoid T2-hyperintense (A) and gadolinium-enhancing (B) lesion
adjacent to the frontal horn of the right lateral ventricle. At 6 months, a larger
lesion with surrounding edema occupied the right frontal lobe with a central zone of
presumed necrosis and gadolinium enhancement (C, D). At 12 months, the lesion had
reduced in size with persistent enhancement (E, F). At 18 months (not shown), the lesion
further decreased in size with near resolution of the surrounding edema. Fluorescein and
indocyanine green angiography with corresponding color photographs of the retina show
views of the macula of the right eye (GYI). Periarteriolar narrowing and sheathing, focal
leakage, telangiectasias, and cotton wool spots are present.

Reprinted with permission from Mateen FJ, et al, Neurology.25 B 2010, American Academy of Neurology.
www.neurology.org/content/75/13/1211.long.

of progressive neurologic disability.28 that a broader range of clinical pheno-


Early diagnostic criteria included types exists, including stroke onset
stroke before the age of 40 years, after the age of 40 years.30 MRI abnor-
encephalopathy characterized by sei- malities involve the cerebral cortex and
zures or dementia, and blood lactic may cross vascular territories with
acidosis or ragged red fibers on Gomori sparing of the deep white matter.31
trichrome staining of skeletal muscle.29 MELAS is a maternally inherited mi-
It has become increasingly apparent tochondrial disorder, with 80% of cases

Continuum (Minneap Minn) 2014;20(2):399–411 www.ContinuumJournal.com 407


Copyright © American Academy of Neurology. Unauthorized reproduction of this article is prohibited.
Genetic Stroke Syndromes

KEY POINTS
h The mitochondrial associated with the A3243G mutation in been associated with mutations in the
encephalopathy, lactic the gene encoding transfer RNALEU(UUR). RNF213 gene on chromosome
acidosis, and strokelike Additional mitochondrial mutations affect 17q25.3.38 In most families, inheritance
episodes syndrome is a respiratory chain enzymes, particularly is autosomal dominant with incomplete
maternally inherited complex I. Treatments for MELAS have penetrance, but other modes of transmis-
mitochondrial disorder, focused on reducing oxidative stress sion have been described.39 Moyamoya
with 80% of cases and include coenzyme Q10 (ubiquinone), disease is a nonatherosclerotic,
associated with the levocarnitine, L-arginine, and B vita- noninflammatory vasculopathy histopath-
A3243G mutation in the mins.32,33 Definitive disease-modifying ologically characterized by intimal hyper-
gene encoding transfer therapy has yet to be established. plasia, smooth muscle cell proliferation,
RNALEU(UUR).
Valproate should be avoided as anti- and disruption of the internal elastic
h The classic angiographic epileptic therapy for recurrent seizures lamina, which results in progressive
appearance of because severe paradoxical convulsions stenosis and occlusion of affected large
moyamoya is stenosis have been reported.34 Statins should also vessels. Irregular collateral networks in
and occlusion of the
be avoided, if possible, because they may the basal ganglia develop in response
bilateral distal internal
precipitate or exacerbate myopathy in to progressive luminal narrowing.40
carotid arteries and
proximal middle/
MELAS mutation carriers.35,36 Moyamoya disease shares associations
anterior cerebral arteries with other nonatherosclerotic large
accompanied by a
Moyamoya Disease vessel arteriopathies, including cervical
network of abnormal Moyamoya disease is associated with artery dissection, fibromuscular dys-
lenticulostriate collateral recurrent TIA, ischemic stroke, and plasia, and intracranial aneurysms.41 Pro-
vessels that has the hemorrhagic stroke. In children with gressive stenosis and occlusion of the
appearance of a puff moyamoya disease, ischemic stroke and terminal internal carotid arteries and
of smoke. TIA are more prevalent and may be proximal middle/anterior cerebral arter-
triggered by exercise, crying, coughing, ies with irregular collateral networks may
fever, or hyperventilation; in adults, develop secondary to atherosclerosis,
intraparenchymal and intraventricular radiation, sickle cell disease, or other
hemorrhage is more prevalent. causes and has been termed moyamoya
Hemiparesis, aphasia, altered menta- syndrome.
tion, and visual disturbance are common There is a paucity of randomized
stroke sequelae.37 In addition to cere- clinical trials of specific treatment for
brovascular events, epilepsy occurs as an moyamoya disease. In clinical prac-
additional neurologic manifestation. The tice, antiplatelet therapy has been
incidence of moyamoya disease is highest used to prevent recurrent ischemic
in Japanese and other East Asian events. To augment blood flow, sur-
populations. The classic angiographic ap- gical procedures that have been done
pearance of moyamoya is stenosis and include direct revascularization by
occlusion of the bilateral distal internal extracranial-intracranial bypass and
carotid arteries and proximal middle/ante- indirect revascularization by several
rior cerebral arteries accompanied by a techniques, including encephalo-
network of abnormal lenticulostriate col- duroarteriosynangiosis (EDAS) and
lateral vessels that has the appearance of a encephaloduroarteriomyosynangiosis
puff of smoke. Conventional arteriography (EDAMS). EDAS is performed by
is considered the gold standard for diag- attaching the dissected superficial tem-
nosis, but the vascular abnormalities may poral artery to the edges of a linear
be visualized with noninvasive modalities, dural incision, which promotes neovas-
including CT and MR angiography. cularization of the ischemic hemisphere.
Familial forms of moyamoya, which EDAMS is an extension of the EDAS
account for about 15% of cases, have technique that uses the superficial
408 www.ContinuumJournal.com April 2014

Copyright © American Academy of Neurology. Unauthorized reproduction of this article is prohibited.


Case 9-3
A 50-year-old woman was hospitalized with recurrent transient neurologic
symptoms. Four months before presentation, she began having episodes
of slurred speech accompanied by numbness of the left side of her face
and left hand lasting 2 to 3 minutes followed by spontaneous improvement.
On the day of admission, she developed a 5-minute spell of left-sided
paresthesia, facial droop, and slurred speech. Her neurologic examination
was normal. Noncontrasted head CT showed an area of chronic
encephalomalacia in the right parietal lobe. CT perfusion study demonstrated
reduced cerebral blood flow in the right hemisphere, with a prolongation of
the mean transit time in the right middle cerebral artery distribution.
Conventional angiography was performed to further evaluate the cerebral
circulation. The supraclinoid segment of the right internal carotid artery and
proximal middle cerebral and anterior cerebral arteries demonstrated
hemodynamically significant occlusive lesions. Abnormal lenticulostriate
collateral vessels were evident (Figure 9-6), suggestive of moyamoya disease.

FIGURE 9-6 Moyamoya disease. Cerebral angiogram of the right internal carotid
artery with oblique (A) and lateral (B) projections demonstrating severe
tapering stenosis of the right carotid terminus (arrows) as well as severe
stenosis of the M1 segment of the right middle cerebral artery with multiple small
hypertrophied collateral branches extending from the region of stenosis (arrowheads).

Comment. Patients with moyamoya disease are at risk for hemodynamic


TIA and stroke involving the watershed areas of the anterior, middle,
and posterior circulation. This patient continued to have symptoms
despite permissive hypertension and IV normal saline for maintenance of
intravascular volume. A right craniotomy with superficial temporal artery
to middle cerebral artery bypass with encephaloduroarteriosynangiosis
was performed to augment blood flow to the ischemic hemisphere. The
patient remained asymptomatic after the procedure.

temporal artery in addition to the deep CONCLUSION


temporal artery of the temporalis muscle Several monogenic disorders with clin-
and middle meningeal artery to promote ical phenotypes that include ischemic
neovascularization (Case 9-3).42 stroke have been well characterized.
Continuum (Minneap Minn) 2014;20(2):399–411 www.ContinuumJournal.com 409
Copyright © American Academy of Neurology. Unauthorized reproduction of this article is prohibited.
Genetic Stroke Syndromes

Accurate diagnosis has therapeutic im- on transcranial Doppler ultrasonography.


N Engl J Med 1998;339(1):5Y11.
plications in Fabry disease and sickle
cell disease. Recognition of the cerebro- 12. Lee MT, Piomelli S, Granger S, et al. Stroke
Prevention Trial in Sickle Cell Anemia
vascular, neurologic, systemic, and ra- (STOP): extended follow-up and final results.
diographic manifestations of these Blood 2006;108(3):847Y852.
inherited disorders can lead to disease- 13. Hulbert ML, Scothorn DJ, Panepinto JA,
specific diagnostic testing in selected pa- et al. Exchange blood transfusion compared
tients with cryptogenic stroke or a family with simple transfusion for first overt stroke
is associated with a lower risk of subsequent
history of stroke. stroke: a retrospective cohort study of 137
children with sickle cell anemia. J Pediatr
REFERENCES 2006;149(5):710Y712.
1. Flemming KD, Wiebers DO, Brown RD Jr, 14. Ware RE, Helms RW; SWiTCH Investigators.
et al. The natural history of radiographically Stroke With Transfusions Changing to
defined vertebrobasilar nonsaccular Hydroxyurea (SWiTCH). Blood 2012;119(17):
intracranial aneurysms. Cerebrovasc Dis 3925Y3932.
2005;20(4):270Y279.
15. Adams RJ, Brambilla D; Optimizing Primary
2. Fellgiebel A, Muller MJ, Ginsberg L. CNS Stroke Prevention in Sickle Cell Anemia
manifestations of Fabry’s disease. Lancet (STOP 2) Trial Investigators. Discontinuing
Neurol 2006;5(9):791Y795. prophylactic transfusions used to prevent
3. Gomes I, Nora DB, Becker J, et al. Nerve stroke in sickle cell disease. N Engl J Med
conduction studies, electromyography and 2005;353(26):2769Y2778.
sympathetic skin response in Fabry’s disease. 16. Opherk C, Peters N, Herzog J, et al.
J Neurol Sci 2003;214(1Y2):21Y25. Long-term prognosis and causes of death
4. Moore DF, Ye F, Schiffmann R, Butman JA. in CADASIL: a retrospective study in 411
Increased signal intensity in the pulvinar on patients. Brain 2004;127(pt 11):2533Y2539.
T1-weighted images: a pathognomonic MR 17. Vahedi K, Chabriat H, Levy C, et al. Migraine
imaging sign of Fabry disease. AJNR Am J with aura and brain magnetic resonance
Neuroradiol 2003;24(6):1096Y1101. imaging abnormalities in patients with
5. Takanashi J, Barkovich AJ, Dillon WP, et al. CADASIL. Arch Neurol 2004;61(8):1237Y1240.
T1 hyperintensity in the pulvinar: key imaging 18. O’Sullivan M, Jarosz JM, Martin RJ, et al. MRI
feature for diagnosis of Fabry disease. AJNR hyperintensities of the temporal lobe and
Am JNeuroradiol 2003;24(5):916Y921. external capsule in patients with CADASIL.
6. Mehta A, Beck M, Elliott P, et al. Enzyme Neurology 2001;56(5):628Y634.
replacement therapy with agalsidase alfa in 19. Dichgans M, Holtmannspotter M, Herzog J,
patients with Fabry’s disease: an analysis et al. Cerebral microbleeds in CADASIL: a
of registry data. Lancet 2009;374(9706): gradient-echo magnetic resonance imaging
1986Y1996. and autopsy study. Stroke 2002;33(1):67Y71.
7. Rombach SM, Hollak CE, Linthorst GE, 20. Viswanathan A, Godin O, Jouvent E, et al.
Dijkgraaf MG. Cost-effectiveness of enzyme Impact of MRI markers in subcortical
replacement therapy for Fabry disease. vascular dementia: a multi-modal analysis in
Orphanet J Rare Dis 2013;8:29. CADASIL. Neurobiol Aging 2010;31(9):
8. Ohene-Frempong K, Weiner SJ, Sleeper LA, 1629Y1636.
et al. Cerebrovascular accidents in sickle 21. Peters N, Opherk C, Bergmann T, et al.
cell disease: rates and risk factors. Blood Spectrum of mutations in biopsy-proven
1998;91(1):288Y294. CADASIL: implications for diagnostic strategies.
9. Powars D, Wilson B, Imbus C, et al. The Arch Neurol 2005;62(7):1091Y1094.
natural history of stroke in sickle cell disease. 22. Eikermann-Haerter K, Yuzawa I, Dilekoz E,
Am J Med 1978;65(3):461Y471. et al. Cerebral autosomal dominant
10. Bunn HF. Pathogenesis and treatment of arteriopathy with subcortical infarcts and
sickle cell disease. N Engl JMed 1997;337(11): leukoencephalopathy syndrome mutations
762Y769. increase susceptibility to spreading depres-
sion. Ann Neurol 2011;69(2):413Y418.
11. Adams RJ, McKie VC, Hsu L, et al. Prevention
of a first stroke by transfusions in children 23. Hara K, Shiga A, Fukutake T, et al. Association
with sickle cell anemia and abnormal results of HTRA1 mutations and familial ischemic

410 www.ContinuumJournal.com April 2014

Copyright © American Academy of Neurology. Unauthorized reproduction of this article is prohibited.


cerebral small-vessel disease. N Engl J Med 33. Rodriguez MC, MacDonald JR, Mahoney DJ,
2009;360(17):1729Y1739. et al. Beneficial effects of creatine, CoQ10,
and lipoic acid in mitochondrial disorders.
24. Shiga A, Nozaki H, Yokoseki A, et al.
Muscle Nerve 2007;35(2):235Y242.
Cerebral small-vessel disease protein HTRA1
controls the amount of TGF-"1 via cleavage 34. Lam CW, Lau CH, Williams JC, et al.
of proTGF-"1. Hum Mol Genet 2011;20(9): Mitochondrial myopathy, encephalopathy,
1800Y1810. lactic acidosis and stroke-like episodes
(MELAS) triggered by valproate therapy.
25. Mateen FJ, Krecke K, Younge BR, et al. Eur J Pediatr 1997;156(7):562Y564.
Evolution of a tumor-like lesion in
cerebroretinal vasculopathy and TREX1 35. Thomas JE, Lee N, Thompson PD. Statins
mutation. Neurology 2010;75(13):1211Y1213. provoking MELAS syndrome. A case report.
Eur Neurol 2007;57(4):232Y235.
26. Richards A, van den Maagdenberg AM, Jen
JC, et al. C-terminal truncations in human 36. Tay SK, Dimauro S, Pang AY, et al.
3’-5’ DNA exonuclease TREX1 cause autosomal Myotoxicity of lipid-lowering agents in a
dominant retinal vasculopathy with cerebral teenager with MELAS mutation. Pediatr
leukodystrophy. Nat Genet 2007;39(9): Neurol 2008;39(6):426Y428.
1068Y1070. 37. Choi JU, Kim DS, Kim EY, Lee KC. Natural
27. Kernt M, Gschwendtner A, Neubauer AS, history of moyamoya disease: comparison of
et al. Effects of intravitreal bevacizumab activity of daily living in surgery and non
treatment on proliferative retinopathy in a surgery groups. Clin Neurol Neurosurg
1997;99(suppl 2):S11YS18.
patient with cerebroretinal vasculopathy.
J Neurol 2010;257(7):1213Y1214. 38. Kamada F, Aoki Y, Narisawa A, et al. A
genome-wide association study identifies
28. Pavlakis SG, Phillips PC, DiMauro S, et al.
RNF213 as the first Moyamoya disease gene.
Mitochondrial myopathy, encephalopathy,
J Hum Genet 2011;56(1):34Y40.
lactic acidosis, and strokelike episodes: a
distinctive clinical syndrome. Ann Neurol 39. Mineharu Y, Takenaka K, Yamakawa H,
1984;16(4):481Y488. et al. Inheritance pattern of familial
moyamoya disease: autosomal dominant
29. Hirano M, Ricci E, Koenigsberger MR, et al. mode and genomic imprinting. J Neurol
Melas: an original case and clinical criteria
Neurosurg Psychiatry 2006;77(9):1025Y1029.
for diagnosis. Neuromuscul Disord 1992;2(2):
125Y135. 40. Takekawa Y, Umezawa T, Ueno Y, et al.
Pathological and immunohistochemical
30. Dickerson BC, Holtzman D, Grant PE, Tian D. findings of an autopsy case of adult
Case records of the Massachusetts General moyamoya disease. Neuropathology
Hospital. Case 36-2005. A 61-year-old 2004;24(3):236Y242.
woman with seizure, disturbed gait, and
altered mental status. N Engl J Med 41. Southerland AM, Meschia JF, Worrall BB.
2005;353(21):2271Y2280. Shared associations of nonatherosclerotic,
large-vessel, cerebrovascular arteriopathies:
31. Sofou K, Steneryd K, Wiklund LM, et al. MRI considering intracranial aneurysms, cervical
of the brain in childhood-onset mitochondrial artery dissection, moyamoya disease and
disorders with central nervous system fibromuscular dysplasia. Curr Opin Neurol
involvement. Mitochondrion 2013;13(4): 2013;26(1):13Y28.
364Y371.
42. Patel NN, Mangano FT, Klimo P Jr. Indirect
32. Koga Y, Akita Y, Nishioka J, et al. MELAS revascularization techniques for treating
and L-arginine therapy. Mitochondrion moyamoya disease. Neurosurg Clin N Am
2007;7(1Y2):133Y139. 2010;21(3):553Y563.

Continuum (Minneap Minn) 2014;20(2):399–411 www.ContinuumJournal.com 411


Copyright © American Academy of Neurology. Unauthorized reproduction of this article is prohibited.
Review Article

Predicting Outcomes
Address correspondence to
Dr David A. Rempe, 621 South
New Ballas Road, Tower B,
Suite 6005, St Louis, MO 63141,
david.rempe@mercy.net.
Relationship Disclosure:
Dr Rempe reports no disclosure.
After Transient Ischemic
Unlabeled Use of
Products/Investigational
Use Disclosure:
Attack and Stroke
Dr Rempe reports no disclosure.
David A. Rempe, MD, PhD
* 2014, American Academy
of Neurology.

ABSTRACT
Purpose of Review: Predicting functional outcome and mortality after stroke, with
or without thrombolysis, is a critical role of neurologists. This article reviews the
predictors of outcome after ischemic stroke.
Recent Findings: Several scores were recently designed to predict (1) mortality and
poor functional outcome after ischemic stroke, (2) the functional outcome and risk
of symptomatic intracranial hemorrhage (sICH) after thrombolysis, and (3) the risk
of stroke following TIA. Validation of these prediction instruments is ongoing, and
studies will be critical to determine the general applicability of these scores.
Summary: Although several scores were developed to predict mortality and
outcome after stroke, it may be premature to employ these prediction scores to
determine individual patient outcome. Similarly, prediction scores should not be
used to deny patients tissue plasminogen activator (tPA), even if the scores predict
that the patient has a high likelihood of sICH or poor outcome after thrombolysis.

Continuum (Minneap Minn) 2014;20(2):412–428.

INTRODUCTION of these prediction scores can be


After a stroke, providers are often verified, they could be an invaluable
asked to predict both the chance that tool for neurologists and other health
the patient will recover and their care providers.
ultimate quality of life. The impor-
tance of an accurate assessment can- PREDICTING MORTALITY AND
not be overstated. An overly negative OUTCOME IN STROKE PATIENTS
prediction may lead to withdrawal of Recently, investigators employing var-
care in misclassified patients, forming ious patient registries developed sev-
a self-fulfilling prophecy. Traditionally, eral prediction models for stroke
neurologists have relied on their ex- mortality (Table 10-1, Table 10-2,
perience to predict outcome, which Table 10-3, and Figure 10-1), includ-
may be subject to bias, either positive ing the ASTRAL score (age, severity of
or negative. With the greater use of stroke, stroke onset to admission
neurohospitalists, a lack of experience time, range of visual fields, acute
in following the long-term natural glucose, and level of consciousness)1;
history of stroke patients potentially the PLAN score (preadmission comor-
exists and may lead to errant pre- bidity, level of consciousness, age, and
dictions of outcome. For these rea- neurologic deficit) 3; the Ischemic
sons, the development of prediction Stroke Predictive Risk Score
models for stroke patients is a very (IScore)2,7; the Get With the Guide-
active area of research. If the accuracy lines (GWTG) stroke risk model8; the

412 www.ContinuumJournal.com April 2014

Copyright © American Academy of Neurology. Unauthorized reproduction of this article is prohibited.


a
TABLE 10-1 ASTRAL and Ischemic Stroke Predictive Risk Score (IScore) Prediction Instruments

C Statistic in
Prediction What Is Derived
Instruments Predicted? Components Calculating Points Cohort
1
ASTRAL Modified Rankin Age Every 5 years = 1 point 0.85
Scale (mRS)
NIH Stroke Scale score Every point of NIH
score 93 or
Stroke Scale = 1 point
death at 30 days
Time to Present 93 hours
presentation from onset = 2 points
Visual field cut Visual deficit = 2 points
Glucose Glucose 9131 or
G66 mg/dL = 1 point
Level of LOC decreased = 3 points
consciousness (LOC)
IScore 30-day mortality; Age 1 point for each year 0.82Y0.85
(30-day score)2 1-year mortality;
Sex Male = 10 points
30-day death or
mRS score 9 3; Female = 0 points
30-day mortality or
Canadian Neurological 0 = 105 points
institutionalization
Score
at discharge G4 = 65 points
5Y7 = 40 points
98 = 0 points
Stroke subtype Lacunar = 0 points
Nonlacunar = 30 points
Undetermined = 35 points
Vascular risk factors Atrial fibrillation = 10 points
Congestive heart
failure = 10 points
Comorbid conditions Cancer = 10 points
Renal dialysis = 35 points
Preadmission disability Independent = 0 points
Dependent = 15 points
Glucose on admission G135 mg/dL = 0 points
9135 mg/dL = 15 points
ASTRAL = age, severity of stroke, stroke onset to admission time, range of visual fields, acute glucose, and level of consciousness score;
NIH = National Institutes of Health.
a
Data from Ntaios G, et al, Neurology,1 www.neurology.org/content/78/24/1916.short?sid=b01602b5-2e1e-4848-aa33-20a22fdf909b
and Saposnik G, et al, Circulation.2 circ.ahajournals.org/content/123/7/739.long.

six simple variables (SSV) score4,9; and predicting outcome following stroke.
the Bologna Outcome Algorithm for The patient’s age and stroke severity
Stroke (BOAS) score.5 When deriving are the two common independent risk
these prediction models, several inde- factors identified as important predic-
pendent risk factors were identified as tors of stroke outcome (Figure 10-1).

Continuum (Minneap Minn) 2014;20(2):412–428 www.ContinuumJournal.com 413


Copyright © American Academy of Neurology. Unauthorized reproduction of this article is prohibited.
TIA and Stroke Outcomes

TABLE 10-2 PLAN, Bologna Outcome Algorithm for Stroke, and Six Simple Variations
Prediction Instrumentsa

C Statistic
Prediction What Is in Derived
Instruments Predicted? Components Calculating Points Cohort
3
PLAN 30-day mortality; Age 1 point per decade 0.80Y0.89
death or severe
Stroke severity Severe arm
disability at discharge
weakness = 2 points
modified Rankin
Scale (mRS) score 5Y6; Preadmission medical Severe leg weakness = 2 points
1-year mortality; comorbidities
Neglect or aphasia = 1 point
mRS score 0Y2
at discharge Atrial fibrillation = 1 point
Congestive heart failure = 1 point
Cancer = 1.5 points
Dependence = 1.5 points

Level of LOC reduced = 5.0 points


consciousness (LOC)
BOAS5 Predicts mRS score 92 Age 978 = 1 point 0.89
or death at 6 months
NIH Stroke Scale score 910 = 1 point
Persistent upper limb If present = 1 point
weakness
Paralysis at discharge If present = 1 point
from stroke unit
Need for oxygen If present = 1 point
administration
Need for urinary catheter If present = 1 point
4
SSV Alive at 60 days; Age; living alone Not reported 0.84Y0.88
alive and
Independence prestroke
independent
(mRS score e2)
at 6 months
Verbal component of
Glasgow coma scale
Able to lift both arms
to the horizon
Able to walk without help
PLAN = preadmission comorbidity, level of consciousness, age, and neurologic deficit score; BOAS = Bologna Outcome Algorithm for
Stroke; NIH = National Institutes of Health; SSV = six simple variables.
a
Data from O’Donnell MJ, et al, Arch Intern Med,3 archinte.jamanetwork.com/article.aspx?articleid=1391071. Counsell C, et al, Stroke,4
stroke.ahajournals.org/content/33/4/1041.long. and Muscari A, et al, Acta Neurol Scand.5 onlinelibrary.wiley.com/doi/10.1111/j.1600-0404.
2010.01479.x/abstract.

As would be expected, the older the predictors of stroke outcome among


patient, the worse the predicted outcome the various prediction instruments
after stroke. Similarly, patients with (Figure 10-1). For example, blood
more severe strokes have a greater glucose is a predictor of poor outcome
mortality and morbidity. Several other in the ASTRAL score1 and IScore.2,7
independent risk factors are shared as Premorbid disability is incorporated

414 www.ContinuumJournal.com April 2014

Copyright © American Academy of Neurology. Unauthorized reproduction of this article is prohibited.


a
TABLE 10-3 Get With the Guidelines Stroke Risk Model Prediction Instrument

C Statistic
Prediction What Is in Derived
Instruments Predicted? Components Calculating Points Cohort
Get With the In-hospital Age G60 years = 0 points 0.84
Guidelines Stroke mortality
60Y70 years = 1 point
Risk Model (GWTG)6
70Y80 years = 5 points
980 years = 9 points
NIH Stroke Scale 0Y2 = 0 points
score
3Y5 = 10 points
6Y10 = 21 points
11Y15 = 37 points
16Y20 = 48 points
21Y25 = 56 points
925 = 65 points
Mode of arrival Private transport = 0 points
Did not present through emergency
department = 16 points
Ambulance from scene = 12 points
Sex Female = 3 points
Vascular risk If no prior stroke/TIA = 2 points
factors
If history of diabetes mellitus = 2 points
If history of hyperlipidemia = 2 points
If history of atrial fibrillation = 2 points
If history of coronary artery
disease = 5 points
If no hyperlipidemia present = 2 points
NIH = National Institutes of Health; TIA = transient ischemic attack.
a
Data from Smith EE, et al, Circulation.6 circ.ahajournals.org/content/122/15/1496.long.

KEY POINT
into the SSV score,4,9 the PLAN score,3 useful. As discussed elsewhere,10Y13
and IScore. The BOAS score5 is unique in several factors that make a prediction h The patient’s age and
stroke severity are the
that it incorporated the need for oxygen scale valuable include accuracy of the
two common
administration and the use of a urinary prediction, validation in separate independent predictors
catheter in patients with stroke outcome. populations, applicability to the pa- of mortality after stroke.
tient population in whom the rule will
Accuracy of Prediction Scores be applied, ease of use, and its ability
While a detailed discussion of clinical to affect treatment decisions and pre-
prediction rules is beyond the scope vent or delay poor outcomes. A pop-
of this discussion, it is worth briefly ular measure of the accuracy of
considering attributes that make a prediction models is the C statistic,
clinical prediction score reliable and which is a function of the sensitivity
Continuum (Minneap Minn) 2014;20(2):412–428 www.ContinuumJournal.com 415
Copyright © American Academy of Neurology. Unauthorized reproduction of this article is prohibited.
TIA and Stroke Outcomes

patients of different ethnic back-


grounds, and in populations with
divergent medical systems of care.
For example, the GWTG stroke risk
model8 (derived from a US popula-
tion) accurately predicts inpatient
stroke mortality when applied to a
Chinese registry (C statistic = 0.87).14
Similarly, the ASTRAL score (derived
from a Swiss population) predicted a
poor outcome (modified Rankin Scale
[mRS] score 92) at 3 months, 1 year, and
5 years after stroke in a Chinese registry
(C statistic Q0.81).15 Finally, the IScore
predicts poor outcome or death in both
a Chinese registry (C statistic = 0.82)
and a Korean population of patients (C
statistic = 0.81).16,17 At the time of this
FIGURE 10-1 Venn diagram demonstrating the overlap of independent writing, the PLAN and BOAS scores have
predictors for mortality and functional outcome following yet to be validated in diverse geographic
ischemic stroke.
populations. It is uncertain whether
ASTRAL = age, severity of stroke, stroke onset to admission
time, range of visual fields, acute glucose, and level of
some of the independent predictors in
consciousness score; IScore = Ischemic Stroke Predictive Risk the BOAS score (eg, need for oxygen
Score; GWTG = Get With the Guidelines stroke risk model; and urinary catheter), which are depen-
LOC = loss of consciousness; PLAN = preadmission comorbidity,
dent on medical practices, will perform
level of consciousness, age, and neurologic deficit score;
BOAS = Bologna Outcome Algorithm for Stroke score; SSV = well in various medical systems of care.
six simple variables score; GCS = Glasgow coma scale.
Convenience of Use
KEY POINTS and specificity of the prediction model To be useful to busy clinicians, clinical
h Several factors that for discriminating between outcomes. scores must be easily accessible and easy
make a prediction scale The C statistic ranges from 0.5 to 1.0. to use. Prediction scores should include
valuable include accuracy A value of 1.0 would indicate perfect independent variables that are easily
of the prediction, measurable, be readily available to clini-
discrimination, whereas a value of 0.5
validation in separate cians, have an unambiguous scoring
would be no better than chance (no
populations, applicability system, and predict outcome without
to the patient population
better than flipping a coin). A value of
0.8 is considered to have ‘‘very good needing a complicated calculation.
in whom the rule will be
discrimination’’ and a value of 0.9 Regarding the measure of stroke se-
applied, ease of use, and
its ability to affect ‘‘excellent discrimination’’ (Table 10-1, verity, most scales utilize the NIH
treatment decisions and Table 10-2, and Table 10-3). Stroke Scale (NIHSS) (Appendix A) or
prevent or delay poor the Canadian Neurological Scale (CNS)
outcomes. Validation of Prediction Scores to measure stroke severity. The PLAN
in Multiple Populations score was designed to avoid this hurdle
h To be clinically useful, a
stroke outcomes score To be clinically useful, a stroke out- by using the presence of arm weakness,
should be validated in comes score should be validated in leg weakness, aphasia, or neglect in-
several patient several patient populations or regis- stead of a formal stroke scale. This may
populations or registries tries distinct from those used to derive make this scale of particular value to the
distinct from those used the score. Predictive scores are con- nonstroke neurologist.3,12 The availability
to derive the score. sidered more valid if they are predic- of prediction scores on websites or ap-
tive in separate geographic regions, in plications (apps) for personal electronic

416 www.ContinuumJournal.com April 2014

Copyright © American Academy of Neurology. Unauthorized reproduction of this article is prohibited.


KEY POINT
devices make some of these prediction illustrate the agreement (or lack thereof) h Prediction scores should
instruments more accessible and easy to among these scores, we calculated the include independent
use. Currently, the NIHSS and IScore predicted outcome and mortality for a variables that are easily
are available both on apps and 48-year-old man who presented to our measurable, be readily
websites. The predicted stroke out- hospital with a moderately severe stroke available to clinicians,
comes of the ASTRAL score can be (NIHSS score of 15) (Case 10-1). In this have an unambiguous
approximated using Figure 10-21 and example, the predicted outcomes for a scoring system, and
Table 10-1. good outcome (mRS score of 0 to 2) predict outcome
varied by almost fourfold, and the without needing a
Predicting Outcome After predicted 30-day mortality varied by complicated calculation.
Stroke Using a Patient Example more than fivefold. Thus, predicting
As an example of how to calculate individual outcomes with these scores
these stroke prediction scores, and to needs to be done with caution.

FIGURE 10-2 Color chart for the estimation of the predicted probability of unfavorable outcome at 3 months in patients
with acute ischemic stroke using the ASTRAL score.
NIHSS = National Institutes of Health Stroke Scale.
Reprinted with permission from Ntaios G, et al, Neurology.1 B 2012 American Academy of Neurology. www.neurology.org/
content/78/24/1916.short?sid=b01602b5-2e1e-4848-aa33-20a22fdf909b.

Continuum (Minneap Minn) 2014;20(2):412–428 www.ContinuumJournal.com 417


Copyright © American Academy of Neurology. Unauthorized reproduction of this article is prohibited.
TIA and Stroke Outcomes

KEY POINTS
h As the treatment of
stroke evolves and
Case 10-1
A 48-year-old man with no significant past medical history or disability presented
improves, the accuracy
with aphasia and right-sided weakness. He had awakened from sleep with
of prediction scores may
right-sided weakness and aphasia and arrived to the hospital by ambulance.
diminish.
Head CT and CT angiography demonstrated a subacute infarct and left internal
h Prediction scores are carotid artery occlusion. His NIH Stroke Scale score was 15, blood glucose was
unlikely to incorporate normal, and he had no diminished level of consciousness when he arrived. For
all the factors that are the ease of comparison across the many scores, the functional outcome was
important to outcome calculated as the percentage of patients predicted to have an mRS score of 0 to 2
for a particular patient. as well as mortality. For this patient, the IScore was 153 with 17% of patients
h Only one of the predicted to achieve an mRS score of 0 to 2 at 30 days and predicted mortality of
prediction scores has 8% at 30 days. The PLAN score was 7 with 60% of patients predicted to have an
been demonstrated to mRS score of 0 to 2 at discharge and a predicted 30-day mortality of 1.4%. The
improve prediction of ASTRAL score was 28 with 31% to 40% of patients predicted to have an mRS
patient outcome score of 0 to 2 at 3 months (Figure 10-2). The GWTG score was 53, predicting a
compared to an 5.2% in-hospital mortality rate.6
experienced clinician’s Comment. The prediction for an outcome of mRS score of 0 to 2 varies
judgment. The scores from as good as 60% at discharge (PLAN score) to a 17% chance at 30 days
may give a false sense (IScore). When calculating the likelihood of death for our patient, the
of confidence in 30-day mortality rate varies from 1.4% to 8% as calculated by the PLAN
predictions of poor and IScore, respectively. As such, the predicted functional outcome and
outcome, leading to mortality varies severalfold among the scores. Because of this variability,
early withdrawal of care predicting individual outcomes with these scores needs to be done with
in some patients. caution. Comparisons of several scores may help the clinician appreciate
the potential variability of the outcomes for a given patient.

Limitations of Stroke Outcome are linked to poor stroke outcome19 but


Prediction Scores are not included in these models.
These prediction scores have several po- It is important to note that, to date,
tential limitations. First, these models were only one of these prediction scores has
derived from past patient populations. been demonstrated to improve predic-
As the treatment of stroke evolves and tion of patient outcome compared with
improves, the accuracy of prediction an experienced clinician’s judgment.20
scores may diminish. For example, many Thus, additional studies comparing these
of the scales did not include patients that various scores against each other and
were treated with tPA, so these scales may against experienced clinician judgment
be overly pessimistic if trying to predict a are needed. Moreover, these scores
poor outcome in patients given tPA. In may give a false sense of confidence in
fact, the PLAN score was demonstrated to predictions of poor outcome, leading to
be less accurate in predicting outcome in early withdrawal of care in some pa-
patients who had received tPA.3 Addition- tients. Grouping together poor out-
ally, prediction scores were derived from come (mRS score of greater than or
registries that may not have all predictors equal to 3) and mortality, as is done in
that determine outcome in a particular ASTRAL, BOAS, and IScore may in-
patient and are unlikely to incorporate all advertently imply to the patient’s family
the factors that are important for that poor outcome is so severe that it is
predicting that patient’s outcome. For equivalent to death. Yet, patients with
example, intracranial artery occlusion18 moderate disability do not necessarily
and acute respiratory distress syndrome have a poor quality of life. For example,

418 www.ContinuumJournal.com April 2014

Copyright © American Academy of Neurology. Unauthorized reproduction of this article is prohibited.


KEY POINT
most patients (or their caregivers) blood Pressure at presentation, and h Stroke severity and
who had a hemicraniectomy for severe Severity of stroke at presentation) glucose abundance are
stroke indicated that the decision to score.28 Finally, while the IScore was central factors driving
have the surgery was the correct originally developed to predict mortality both functional
choice despite the fact that only 14% after stroke (see earlier discussion), outcome and
of them achieved an mRS score of 2 recent analyses demonstrated that it symptomatic intracranial
or less.21 also predicts outcome and sICH follow- hemorrhage after
ing tPA treatment.29,30 thrombolysis.
PREDICTING FUNCTIONAL Stroke severity and glucose abun-
OUTCOME AND/OR THE RISK OF dance are central factors driving both
SYMPTOMATIC INTRACRANIAL functional outcome and sICH after
HEMORRHAGE IN PATIENTS thrombolysis (Figure 10-3). Age is
TREATED WITH TISSUE also common to all scales, except for
PLASMINOGEN ACTIVATOR the HAT score. Three of these scales
The seminal National Institutes of (DRAGON, SEDAN, and HAT) include
Neurological Disorders and Stroke imaging data as part of the score.
(NINDS) tPA trial revolutionized acute They identified early ischemic changes
stroke care. For the first time, throm- and/or a dense middle cerebral artery
bolysis was demonstrated to improve (MCA) sign as predictors of poor
stroke outcome18; however, the risk stroke outcome or sICH after throm-
of intracerebral hemorrhage, such as bolysis. Finally, history of diabetes
symptomatic intracranial hemorrhage mellitus, higher systolic blood pres-
(sICH), limits the use of tPA. A risk sure, male sex, time to treatment with
score that could accurately predict tPA, low platelet count, race (specifi-
which patients are most likely to cally Asian), and premorbid disability
benefit, or be harmed, with tPA would were found to be predictors of poor
be a welcome tool for clinicians. functional outcome or sICH in some
Several scores are available to predict of the scores.
outcome after tPA and/or the probability
of sICH after tPA treatment (Table 10-4, Accuracy of Thrombolysis
Table 10-5, and Appendix A). Instru- Prediction Scores
ments that predict outcome include: Most of these scores were derived from
the DRAGON (dense middle cerebral patient populations in Europe or North
artery sign or early infarct on CT, base- America, and validation in other cohorts
line modified Rankin Scale score, is an ongoing process. The Stroke-TPI
age, glucose, onset-to-treatment time, score was determined to be accurate in
NIH Stroke Scale) score22; the Stroke predicting outcome with a C statistic of
Thrombolytic Predictive Instrument 0.83 for patients within the 3-hour
(TPI) score23; and the Multimodal window in a patient population in the
Outcome Score for Stroke Thromboly- Netherlands.31 Yet, this study, and a sub-
sis (MOST) score.26 Those developed to sequent study in the UK,32 suggest that
predict sICH after tPA include the the Stroke-TPI slightly overestimates
hemorrhage after thrombolysis (HAT) good outcome and underestimates
score,24 the Multicenter Stroke Survey poor outcome. In its original pub-
(MSS) score,27 the SEDAN (Sugar, Early lication, the SEDAN score was exter-
infarct sign, hyperDense middle cere- nally validated using data from 880
bral artery, Age, Neurologic deficit) patients in three Swiss cohorts with a
score,25 and the GRASPS (Glucose at C statistic of 0.77.25 Yet, in a subse-
presentation, Race, Age, Sex, Systolic quent publication it only had low
Continuum (Minneap Minn) 2014;20(2):412–428 www.ContinuumJournal.com 419
Copyright © American Academy of Neurology. Unauthorized reproduction of this article is prohibited.
TIA and Stroke Outcomes

a
TABLE 10-4 The Stroke-Thrombolytic Predictive Instrument and DRAGON Prediction Instrument

C Statistic
Prediction Calculating in Derived
Instruments What Is Predicted? Components Points Cohort
Stroke-Thrombolytic Good outcome modified Age Not reported 0.77Y0.78
Predictive Instrument23 Rankin Scale (mRS)
Sex
score 0Y1; poor outcome
mRS score 5Y6 NIH Stroke Scale
score
Time to treatment
Glucose
Diabetes mellitus
Systolic blood
pressure
Prior stroke
22
DRAGON Good outcome mRS Dense middle Both = 2 points 0.84
score 0Y2; poor outcome cerebral artery sign
Either = 1 point
mRS score 5Y6 or early infarct on
head CT Neither = 0 points

Baseline mRS Yes = 1 point


score 91

Age 980 years = 2 points


65Y79 years = 1 point
G65 years = 0 points
Glucose 9144 mg/dL = 1 point
Onset-to-treatment 990 minutes = 1 point
time
NIH Stroke Scale 915 = 3 points
score
10Y14 = 2 points
5Y9 = 1 point
0Y4 = 0 points
NIH = National Institutes of Health; DRAGON = dense middle cerebral artery sign or early infarct on CT, baseline Modified Rankin Scale,
age, glucose, onset-to-treatment time, NIH Stroke Scale score.
a
Data from Strbian D, et al, Neurology,22 www.neurology.org/content/78/6/427.short?sid=69cd245d-7a2c-43a0-b108-8a79a5debff9
and Kent DM et al, Stroke.23 stroke.ahajournals.org/cgi/pmidlookup?view=long&pmid=17068305.

(0.60) to moderate (0.66) discrimina- and 0.78 for predicting catastrophic out-
tory capability in two other cohorts.33 come [mRS score of 5 to 6]). Yet, the
The HAT score is reasonably accurate HAT score had limited to moderate
at predicting outcome in the patients discriminatory power in patients from
in the NINDS trial and in a series two clinical trials.34 Similarly, the MSS
of patients from a single institution score had limited discriminatory power
(C statistic of 0.74 for predicting sICH, (C statistic of 0.61) when examined in
0.75 for predicting good outcome, patients from the StrokeYAcute Ischemic

420 www.ContinuumJournal.com April 2014

Copyright © American Academy of Neurology. Unauthorized reproduction of this article is prohibited.


a
TABLE 10-5 SEDAN and HAT Prediction Instruments

C Statistic
Prediction in Derived
Instruments What Is Predicted? Components Calculating Points Cohort
SEDAN Symptomatic Sugar (glucose) 145Y216 mg/dL = 1 point; 0.77
(Sugar, early intracerebral 9216 mg/dL = 2 points
infarct sign, hemorrhage risk
Early infarct sign If present = 1 point
hyperdense
middle cerebral Hyperdense middle If present = 1 point
artery, age, cerebral artery
neurologic (MCA)
deficit)25 Age 975 years = 1 point
Neurologic deficit NIH Stroke Scale
910 = 1 point
HAT Symptomatic Glucose History of diabetes 0.72Y0.78
(Hemorrhage intracerebral mellitus or blood
after hemorrhage risk; glucose 9200 = 1 point
thrombolysis)24 modified
Rankin Scale
(mRS) score NIH Stroke G15 = 0 points;
0Y1 (90 day); Scale score 15Y20 = 1 point;
920 = 2 points
mRS score
0Y2 (90 day);
mRS score Early ischemic If not present = 0 points;
5Y6 (90 day) changes on G One-third of MCA territory = 1 point;
head CT 9 One-third of MCA
territory = 2 points
NIH = National Institutes of Health; CT = computed tomography.
a
Data from Lou M, et al, Neurology,24 www.neurology.org/content/71/18/1417.short?sid=5f3a0dd1-ee88-433c-b43b-2b0dc59a5634
and Strbian D, et al, Ann Neurol.25 onlinelibrary.wiley.com/doi/10.1002/ana.23546/abstract.

NXY Treatment (SAINT) trials.34 In pa- an IScore of greater than 200 points did KEY POINT
tients from the NINDS trial, the GRASPS not appear to benefit from tPA treatment.30 h The ability of scores to
score had a C statistic of 0.68. In predict outcome or
Examples of Treating Patients symptomatic intracranial
summary, the ability of these scores to
With Thrombolysis: Comparing hemorrhage risk following
predict outcome or sICH risk following
the Prediction Instruments tissue plasminogen
tPA administration needs to be viewed activator administration
with caution as their accuracy in valida- For illustrative purposes, we examined needs to be viewed with
tion cohorts is variable and lower than in the predicted functional outcome and caution, as their accuracy
their derivation studies. risk of sICH of two patients with in validation cohorts is
While the IScore was developed as a moderate to severe stroke severity variable and lower than in
predictor of mortality in patients with who were treated with tPA (Case 10-2, their derivation studies.
ischemic stroke, it also predicts functional Case 10-3). To calculate the DRAGON,
outcome and sICH risk in patients treated HAT, IScore, and Stroke-TPI scores,
with tPA.29 Patients with a low (less than commercially available apps were used.
139 points) or medium (140 to 179 points) The GRASPS score is available online at
IScore appeared to benefit from tPA www.strokeassociation.org/STROKEORG/
treatment, while those patients with an General/Get-With-The-Guidelines-Stroke-
IScore of greater than 200 did not benefit. sICH-Calculator_UCM_453748_
Similarly, patients in the NINDS trial with SubHomePage.jsp. The SEDAN score is

Continuum (Minneap Minn) 2014;20(2):412–428 www.ContinuumJournal.com 421


Copyright © American Academy of Neurology. Unauthorized reproduction of this article is prohibited.
TIA and Stroke Outcomes

cohorts were treated with tPA within 3


hours of stroke onset. As such, the
accuracy of these scales within the 3- to
4.5-hour window is likely to be reduced.

RISK OF STROKE FOLLOWING TIA


Two of the most common questions that
neurologists are asked by primary care
providers, medical hospitalists, and
emergency medicine physicians are (1)
Was the patient’s transient neurologic
episode a TIA? and (2) Does the patient
need to be admitted to the hospital for
an evaluation? Approximately 200,000 to
500,000 TIAs occur in the United States
yearly with about 5 million patients in the
United States having had a TIA.35 Histor-
Venn diagram demonstrating the overlap of independent ically, there was a general misconception
FIGURE 10-3
predictors for functional outcome and risk of symptomatic that TIAs were benign; yet TIAs represent
intracerebral hemorrhage when treating patients with
thrombolysis. a transient diminished blood flow to the
DRAGON = dense middle cerebral artery sign or early infarct
brain that shares the same pathophysio-
on CT, baseline modified Rankin scale, age, glucose, onset-to- logic mechanisms as stroke. In addition,
treatment time, NIH Stroke Scale score; SEDAN = sugar, early patients with TIAs have an increased risk
infarct sign, hyperdense middle cerebral artery, age, neurologic
deficit score; MCA = middle cerebral artery; Stroke-TPI = Stroke- of disability and death.36
Thrombolytic Predictive Instrument; early CT $ = early ischemic Several disease processes mimic TIAs
changes on the head CT; HAT = hemorrhage after thrombolysis
score; MSS = Multicenter Stroke Survey; GRASPS = glucose (eg, complicated migraine, focal seizure,
at presentation, race, age, sex, systolic blood pressure at tumor, subdural hematoma, metabolic
presentation, and severity of stroke at presentation.
derangements). Migraine aura, with or
without headache, is commonly encoun-
KEY POINT calculated as detailed in Table 10-5. For tered and can usually be distinguished by
h It is premature to use the SEDAN score, the absolute risk of a history of migraine, or the typical visual
scores from various sICH with thrombolysis is as follows: obscurations that occur with migraine
prediction instruments (scintillating scotomas, fortification spec-
1.4%, 2.9%, 8.5%, 12.2%, 21.7%, and
as a means to exclude
33.3% for score points of 0, 1, 2, 3, 4, tra). TIAs are less likely to be repeated
patients from
and 5, respectively. frequently, to be stereotypic (as opposed
thrombolysis. Similarly,
it would be premature
As can be appreciated by the case to seizure or migraine), or be accompa-
to bypass tissue examples (Table 10-6 and Table 10-7), nied by headache. While sensory symp-
plasminogen activator variation occurs among these prediction toms or dizziness are less specific and less
and perform other instruments to estimate sICH risk and suggestive of TIA, transient language
interventions based on functional outcome in patients receiving problems and/or weakness are more
these scores. tPA for acute strokes. Until these scores suggestive of TIA. TIAs typically last several
are better validated to predict sICH and minutes or 1 hour or more as opposed to
outcome, it is premature to use these a few seconds. Obviously, older patients,
various scores as a means to exclude especially those with vascular disease risk
patients from thrombolysis. Similarly, it factors, are at higher risk of TIA.
would be premature to bypass tPA and
perform other interventions based on Risk of Recurrent Stroke After TIA
these scores. It is also worth noting that To decide whether the patient needs
most of the patients in the derivation to be admitted for evaluation after the
422 www.ContinuumJournal.com April 2014

Copyright © American Academy of Neurology. Unauthorized reproduction of this article is prohibited.


Case 10-2
A 75-year-old man with a past medical history of hyperlipidemia, hypertension, and distant prior
stroke but without premorbid disability presented with left-sided weakness and slurred speech. The
patient had been normal when he awoke but suddenly became weak later that morning. Upon arrival
to the emergency department (via ambulance), he had significant slurred speech, left facial weakness,
left visual field cut, right gaze preference, and left arm weakness, with a NIH Stroke Scale score of 11.
Chronic right cerebellar and bilateral occipital lobe infarcts were identified on head CT, but no
intracranial hemorrhage (ICH), early ischemic changes, or dense MCA sign was found. The international
normalized ratio
(INR) was 1.0 and
blood glucose was
107 mg/dL. His
blood pressure was
151/83 mm Hg. He
was treated with
tPA 1 hour and 14
minutes after
symptom onset.
Brain MRI the next
day demonstrated
a right MCA stroke
primarily involving
the posterior
division of the
MCA (Figure 10-4).
During several days
in the hospital, he
slowly improved Diffusion-weighted MRI sequences (A and B) of the patient in Case 10-2
and upon discharge FIGURE 10-4 demonstrating an ischemic infarct primarily involving the posterior division of
he had an NIHSS the right middle cerebral artery territory.
score of 4.
Comment. The GRASPS points were 73, the SEDAN score was 2, and the IScore was 155.
The predicted risk of symptomatic ICH varied from 2% to 8.5%, but the predicted good outcome
(mRS score of 0 to 2) was in general agreement.

possible TIA, one must consider the (2 points) or 10 to 59 minutes (1 point); KEY POINT
risk of recurrent stroke after TIA. One and diabetes mellitus (1 point).37 The h One of the most
of the most commonly used scores to ABCD2 score can vary from 0 to 7, and commonly used scores
predict stroke risk following TIA is the increasing scores predict increased risk to predict stroke risk
following TIA is the
ABCD2 score, which was derived from of stroke. For example, the risk of stroke
ABCD2 score.
combining the ABCD and California 2 days after TIA is 0% if the ABCD2
score.37 The ABCD2 score is calculated score is only 0 to 1, but the risk
as follows: age greater than or equal to increases to greater than 8% if the
60 years (1 point); blood pressure ABCD2 score is 6 to 7. The ABCD2 score
greater than or equal to 140/90 mm Hg is easy to calculate, and the risk of
upon presentation (1 point); clinical recurrent stroke can be calculated using
features: unilateral weakness (2 points) available apps.
or speech/language impairment The ABCD2 score is widely employed
without weakness (1 point); duration and supported by American Heart Asso-
greater than or equal to 60 minutes ciation and American Stroke Association

Continuum (Minneap Minn) 2014;20(2):412–428 www.ContinuumJournal.com 423


Copyright © American Academy of Neurology. Unauthorized reproduction of this article is prohibited.
TIA and Stroke Outcomes

Case 10-3
An 88-year-old woman with a history of atrial fibrillation, congestive heart
failure, and hypertension was in a nursing home recovering from a
vertebral compression fracture but was scheduled for discharge in 2 days
to live independently. She was not taking warfarin because of a risk of
falling. She had last been seen normal at 6:00 AM and at 6:50 AM was noted
to have dysarthric
speech and no
movement on her
left side. She arrived
to the emergency
department via
ambulance 1 hour
and 20 minutes
after last seen normal.
An initial head CT
showed a dense MCA
sign on the right
(Figure 10-5) with
some minimal patchy
low attenuation of
the basal ganglia
on the right.
No intracranial
hemorrhage (ICH)
was seen on CT, and
the early ischemic
changes were less
than one-third of the
MCA territory. The
INR was 1.1; blood
glucose level was FIGURE 10-5 Head CT of the patient in Case 10-3
demonstrating a dense middle cerebral
131 mg/dL; systolic artery sign.
blood pressure was
182 mm Hg, but this reduced to 155 mm Hg without treatment. The patient was
lethargic, had no movement on the left side, neglected the left side, and was
dysarthric with a NIHSS score of 23. She was treated with tPA 1 hour and 56
minutes after last seen normal. Given the patient’s advanced age and family’s
wishes, no endovascular therapy was pursued. The patient’s symptoms showed no
improvement, and a large stroke evolved in her right hemisphere. A few days
following the stroke the family elected to pursue comfort care measures. No ICH
was identified on follow-up imaging.
Comment. The GRASPS score was 83, the SEDAN score was 4, and the IScore
was 258. The predicted risk of ICH varied widely from 5% to 21.7%, the
predicted good outcome (mRS score of 0 to 2) varied from 0% to 13%, but the
prediction of catastrophic outcome (mRS score of 5 to 6) was in good agreement.

guidelines to triage TIA patients for of the following criteria are present: an
hospitalization. It is reasonable to hos- ABCD2 score greater than or equal to 3,
pitalize TIA patients if they present an ABCD2 score less than or equal to 2
within 72 hours of the event and any and uncertainty that diagnostic workup

424 www.ContinuumJournal.com April 2014

Copyright © American Academy of Neurology. Unauthorized reproduction of this article is prohibited.


TABLE 10-6 Calculating the Outcome and Risk of Symptomatic Intracerebral Hemorrhage
With Tissue Plasminogen Activator for the Patient in Case 10-2

30-Day
Symptomatic Modified Modified Modified Death or
Intracerebral Rankin Rankin Rankin Modified
Prediction Hemorrhage Scale Score Scale Score Scale Score 30-Day Death or Rankin Scale
Instrument Risk 0Y1 0Y2 5Y6 Institutionalization Score 92
DRAGON NA 74% 5%
Stroke-TPI 2% 52% 12.9%
HAT 2% 66% 77% 6%
GRASPS 5%
SEDAN 8.5%
IScore 7.7% 20% 76%
DRAGON = dense middle cerebral artery sign or early infarct on CT, baseline modified Rankin Scale, age, glucose, onset-to-treatment
time, NIH Stroke Scale score; NA = not applicable; Stroke-TPI = Stroke-Thrombolytic Predictive Instrument; HAT = hemorrhage after
thrombolysis score; GRASPS = glucose at presentation, race, age, sex, systolic blood pressure at presentation, and severity of stroke at
presentation]; SEDAN = sugar, early infarct sign, hyperdense middle cerebral artery, age, neurologic deficit score; IScore = Ischemic
Stroke Predictive Risk Score.

can be completed within 2 days as an mendations: neuroimaging should be


outpatient, or an ABCD2 score of 0 to 2 obtained within 24 hours (with MRI the
and other evidence that indicates the preferred modality); noninvasive imag-
patient’s event was caused by focal ing of the cervicocephalic vessels should
ischemia (Class IIa recommendations).35 be obtained; noninvasive evaluation of
Regarding the evaluation of TIA patients, the intracranial vasculature is reasonable
all of the following are Class Ia recom- if likely to alter management; patients

TABLE 10-7 Calculating the Outcome and Risk of Symptomatic Intracerebral Hemorrhage
With Tissue Plasminogen Activator for the Patient in Case 10-3

30-Day
Death or
Symptomatic Modified
Intracerebral Modified Modified Modified Rankin
Prediction Hemorrhage Rankin Scale Rankin Scale Rankin Scale 30-Day Death or Scale
Instrument Risk Score 0Y1 Score 0Y2 Score 5Y6 Institutionalization Score 92
DRAGON NA 0% 70%
StrokeYTPI 5% 5% 64%
HAT 15% 13% 13% 62%
GRASPS 11%
SEDAN 21.7%
IScore 11.2% 89% 98%
DRAGON = dense middle cerebral artery sign or early infarct on CT, baseline modified Rankin Scale, age, glucose, onset-to-treatment
time, NIH Stroke Scale score; NA = not applicable; Stroke-TPI = Stroke-Thrombolytic Predictive Instrument; HAT = hemorrhage after
thrombolysis score; GRASPS = glucose at presentation, race, age, sex, systolic blood pressure at presentation, and severity of stroke at
presentation]; SEDAN = sugar, early infarct sign, hyperdense middle cerebral artery, age, neurologic deficit score; IScore = Ischemic Stroke
Predictive Risk Score.

Continuum (Minneap Minn) 2014;20(2):412–428 www.ContinuumJournal.com 425


Copyright © American Academy of Neurology. Unauthorized reproduction of this article is prohibited.
TIA and Stroke Outcomes

KEY POINTS
with suspected TIA should be evaluated SUMMARY
h While the ABCD2 score
is widely used for as soon as possible after an event. Providing patients and families with
stratifying risk of stroke accurate predictions of stroke function-
recurrence in TIA Estimating the Risk of Stroke al outcome and mortality is a critical job
patients, some studies After TIA of neurologists. If overly pessimistic,
have suggested a limited While the ABCD2 score is widely used these predictions could lead to with-
predictive value of the for stratifying risk of stroke recurrence drawal of care prematurely; however, a
ABCD2. in TIA patients, some studies have prediction that is too optimistic could
h The identification of suggested a limited predictive value of lead to the patient living with unwanted
diffusion-weighted the ABCD2.38 Recurrent strokes are ob- long-term disability. The outcome pre-
imagingYpositive served in some patients with low ABCD2 diction models that have been devel-
strokes in TIA patients scores. Several validation studies of the oped have good accuracy but are still
significantly increases ABCD2 have been performed, yet they being validated in diverse populations.
the risk of recurrent have not demonstrated C statistics of 0.8 In the case of the scores used to predict
stroke.
or greater.39 Because of these limitations outcome and sICH after tPA, it is pre-
of the ABCD2 score, several studies have mature to use the scores to deny patients
examined the ability of imaging to im- tPA. Finally, in terms of predicting stroke
prove its predictive ability. The identifica- risk in patients with TIA, the addition of
tion of diffusion-weighted imaging imaging criteria to the ABCD2 score
(DWI)Ypositive strokes in patients with improves accuracy when attempting to
TIA significantly increases the risk of triage patients for an expedited stroke
recurrent stroke.40Y42 The 7-day risk of evaluation.
recurrent stroke in patients with an
ABCD2 score of 6 to 7 was 12.5% when REFERENCES
DWI-positive lesions were seen on MRI, 1. Ntaios G, Faouzi M, Ferrari J, et al. An
whereas it was only 0.4% in patients integer-based score to predict functional
outcome in acute ischemic stroke: the
without these lesions.40 Similarly, patients
ASTRAL score. Neurology 2012;78(24):1916Y1922.
with a low ABCD2 score but with DWI-
2. Saposnik G, Kapral MK, Liu Y, et al. IScore: a
positive lesions on MRI were at a similar risk score to predict death early after
risk of stroke as those with high ABCD2 hospitalization for an acute ischemic stroke.
scores but without DWI-positive lesions. Circulation 2011;123(7):739Y749.
By including imaging as part of the score 3. O’Donnell MJ, Fang J, D’Uva C, et al.
the C statistic of the ABCD2 score im- The PLAN score: a bedside prediction rule for
death and severe disability following acute
proves from approximately 0.66 to 0.8, ischemic stroke. Arch Intern Med 2012;172(20):
indicating a very good ability to discrim- 1Y9.
inate patients at risk of stroke.40Y42 This 4. Counsell C, Dennis M, McDowall M, Warlow C.
ABCD2 score incorporating stroke imag- Predicting outcome after acute and subacute
ing is referred to as the ABCD2I score. stroke: development and validation of new
prognostic models. Stroke 2002;33(4):
Recent studies have also incorporated 1041Y1047.
dual and multiple events into the predic-
5. Muscari A, Puddu GM, Santoro N, Zoli M. A
tion scores, and/or the presence of simple scoring system for outcome prediction
carotid stenosis.43 Incorporating this addi- of ischemic stroke. Acta Neurol Scand
tional information improves the C statistic 2011;124(5):334Y342.
to greater than 0.8,43 which has been 6. Smith EE, Shobha N, Dai D, et al. Risk score
validated in a Chinese population.44 How- for in-hospital ischemic stroke mortality
derived and validated within the Get With
ever, since MRI is many times not the Guidelines-Stroke Program. Circulation
immediately available prior to making 2010;122(15):1496Y1504.
admission decisions, the utility of this 7. Saposnik G, Raptis S, Kapral MK, et al. The
approach is reduced. iScore predicts poor functional outcomes early

426 www.ContinuumJournal.com April 2014

Copyright © American Academy of Neurology. Unauthorized reproduction of this article is prohibited.


after hospitalization for an acute ischemic prediction of ischemic stroke outcomes.
stroke. Stroke 2011;42(12):3421Y3428. Neurology 2013;81(5):448Y455.
8. Smith EE, Shobha N, Dai D, et al. A risk score 21. Vahedi K, Hofmeijer J, Juettler E, et al.
for in-hospital death in patients admitted Early decompressive surgery in malignant
with ischemic or hemorrhagic stroke. J Am infarction of the middle cerebral artery: a
Heart Assoc 2013;2(1):e005207. pooled analysis of three randomised controlled
9. Counsell C, Dennis M, McDowall M. trials. Lancet Neurol 2007;6(3):215Y222.
Predicting functional outcome in acute 22. Strbian D, Meretoja A, Ahlhelm FJ, et al.
stroke: comparison of a simple six variable Predicting outcome of IV thrombolysis-treated
model with other predictive systems and ischemic stroke patients: the DRAGON score.
informal clinical prediction. J Neurol Neurology 2012;78(6):427Y432.
Neurosurg Psychiatry 2004;75(3):401Y405.
23. Kent DM, Selker HP, Ruthazer R, et al. The
10. Dennis M. Predictions models in acute stroke-thrombolytic predictive instrument: a
stroke: potential uses and limitations. predictive instrument for intravenous
Stroke 2008;39(6):1665Y1666. thrombolysis in acute ischemic stroke. Stroke
2006;37(12):2957Y2962.
11. Toll DB, Janssen KJ, Vergouwe Y, Moons KG.
Validation, updating and impact of clinical 24. Lou M, Safdar A, Mehdiratta M, et al. The
prediction rules: a review. J Clin Epidemiol HAT Score: a simple grading scale for
2008;61(11):1085Y1094. predicting hemorrhage after thrombolysis.
Neurology 2008;71(18):1417Y1423.
12. Katz MH. Good prediction rules are hard to
find: did we succeed? Arch Intern Med 25. Strbian D, Engelter S, Michel P, et al.
Symptomatic intracranial hemorrhage after
2012;172(20):1536.
stroke thrombolysis: the SEDAN score. Ann
13. Grady D, Berkowitz SA. Why is a good Neurol 2012;71(5):634Y641.
clinical prediction rule so hard to find? Arch
26. Molina CA, Alexandrov AV, Demchuk AM,
Intern Med 2011;171(19):1701Y1702.
et al. Improving the predictive accuracy of
14. Zhang N, Liu G, Zhang G, et al. A risk score recanalization on stroke outcome in patients
based on Get With the Guidelines-Stroke treated with tissue plasminogen activator.
program data works in patients with acute Stroke 2004;35(1):151Y156.
ischemic stroke in China. Stroke 2012;43(11): 27. Cucchiara B, Tanne D, Levine SR, et al. A risk
3108Y3109.
score to predict intracranial hemorrhage
15. Liu G, Ntaios G, Zheng H, et al. External after recombinant tissue plasminogen
validation of the ASTRAL score to predict activator for acute ischemic stroke. J Stroke
3- and 12-month functional outcome in the Cerebrovasc Dis 2008;17(6):331Y333.
China National Stroke Registry. Stroke 28. Menon BK, Saver JL, Prabhakaran S, et al.
2013;44(5):1443Y1445. Risk score for intracranial hemorrhage in
16. Zhang N, Liu G, Zhang G, et al. External patients with acute ischemic stroke treated
validation of the iScore for predicting ischemic with intravenous tissue-type plasminogen
stroke mortality in patients in China. Stroke activator. Stroke 2012;43(9):2293Y2299.
2013;44(7):1924Y1929. 29. Saposnik G, Demchuk A, Tu JV, et al. The
17. Park TH, Saposnik G, Bae HJ, et al. The iScore iScore predicts efficacy and risk of bleeding
predicts functional outcome in Korean in the national institute of neurological
patients with ischemic stroke. Stroke disorders and stroke tissue plasminogen
2013;44(5):1440Y1442. activator stroke trial. J Stroke Cerebrovasc
Dis 2012;22(6):876Y882.
18. Tissue plasminogen activator for acute
ischemic stroke. The National Institute of 30. Saposnik G, Fang J, Kapral MK, et al. The
Neurological Disorders and Stroke rt-PA iScore predicts effectiveness of thrombolytic
Stroke Study Group. N Engl J Med therapy for acute ischemic stroke. Stroke
1995;333(24):1581Y1587. 2012;43(5):1315Y1322.

19. Rincon F, Maltenfort M, Dey S, et al. The 31. Uyttenboogaart M, Stewart RE, Vroomen
prevalence and impact of mortality of the PC, et al. Utility of the stroke-thrombolytic
acute respiratory distress syndrome on predictive instrument. J Neurol Neurosurg
admissions of patients with ischemic stroke Psychiatry 2008;79(9):1079Y1081.
in the United States. J Intensive Care Med
2013. [Epub ahead of print]. 32. McMeekin P, et al. Validating the
stroke-thrombolytic predictive instrument in
20. Saposnik G, Cote R, Mamdani M, et al. a population in the United Kingdom. Stroke
JURaSSiC: accuracy of clinician vs risk score 2012;43(12):3378Y3381.

Continuum (Minneap Minn) 2014;20(2):412–428 www.ContinuumJournal.com 427


Copyright © American Academy of Neurology. Unauthorized reproduction of this article is prohibited.
TIA and Stroke Outcomes

33. Mazya MV, Bovi P, Castillo J, et al. External 38. Perry JJ, Sharma M, Sivilotti ML, et al.
validation of the SEDAN score for prediction Prospective validation of the ABCD2 score
of intracerebral hemorrhage in stroke for patients in the emergency department
thrombolysis. Stroke 2013;44(6): with transient ischemic attack. CMAJ
1595Y1600. 2011;183(10):1137Y1145.
34. Cucchiara B, Kasner S, Tanne D, et al. 39. Lemmens R, Smet S, Thijs VN. Clinical scores
Validation assessment of risk scores to predict for predicting recurrence after transient
postthrombolysis intracerebral haemorrhage. ischemic attack or stroke: how good are
Int J Stroke 2011;6(2):109Y111. they? Stroke 2013;44(4):1198Y1203.
35. Easton JD, Saver JL, Albers GW, et al. 40. Giles MF, Albers GW, Amarenco P, et al.
Definition and evaluation of transient Early stroke risk and ABCD2 score performance
ischemic attack: a scientific statement for in tissue- vs time-defined TIA: a multicenter
healthcare professionals from the American study. Neurology 2011;77(13):1222Y1228.
Heart Association/American Stroke Association
41. Giles MF, Albers GW, Amarenco P, et al.
Stroke Council; Council on Cardiovascular
Addition of brain infarction to the ABCD2
Surgery and Anesthesia; Council on Cardiovascular
Score (ABCD2I): a collaborative analysis of
Radiology and Intervention; Council on
unpublished data on 4574 patients. Stroke
Cardiovascular Nursing; and the Interdisciplinary
2010;41(9):1907Y1913.
Council on Peripheral Vascular Disease.
The American Academy of Neurology affirms 42. Ay H, Arsava EM, Johnston SC, et al. Clinical-
the value of this statement as an educational and imaging-based prediction of stroke risk
tool for neurologists. Stroke 2009;40(6): after transient ischemic attack: the CIP
2276Y2293. model. Stroke 2009;40(1):181Y186.
36. Weimar C, Benemann J, Huber R, et al. 43. Merwick A, Albers GW, Amarenco P, et al.
Long-term mortality and risk of stroke after Addition of brain and carotid imaging to the
transient ischemic attack: a hospital-based ABCD2 score to identify patients at early risk
cohort study. J Neurol 2009;256(4): of stroke after transient ischaemic attack: a
639Y644. multicentre observational study. Lancet
Neurol 2010;9(11):1060Y1069.
37. Johnston SC, Rothwell PM, Nguyen-Huynh
MN, et al. Validation and refinement of 44. Song B, Fang H, Zhao L, et al. Validation of
scores to predict very early stroke risk after the ABCD3-I score to predict stroke risk after
transient ischaemic attack. Lancet transient ischemic attack. Stroke 2013;44(5):
2007;369(9558):283Y292. 1244Y1248.

428 www.ContinuumJournal.com April 2014

Copyright © American Academy of Neurology. Unauthorized reproduction of this article is prohibited.


LIFELONG LEARNING IN NEUROLOGY ®
\

Evaluation and Management of Acute


Ischemic Stroke
Khatri, Pooja MD, MSc. Continuum (Minneap Minn). April 2014; 20(2 Stroke): 283Y295.

Abstract
Purpose of Review:
This review provides an overview of emergent evaluation of the stroke patient with an emphasis
on practical issues regarding ischemic stroke treatment.
Recent Findings:
The IV recombinant tissue-type plasminogen activator (rtPA) treatment window has been
expanded from 3 to 4.5 hours from symptom onset. The evidence for better outcomes with more
rapid initiation of reperfusion therapies is very strong. Adjunctive endovascular therapy has
not been shown to benefit all patients with moderate or severe strokes, and investigations are
underway to identify subgroups that may benefit from this approach. Endovascular therapy
should be considered for patients who are ineligible for IV rtPA and can begin treatment
within 6 hours of stroke onset.
Summary:
Effective emergent evaluation of a stroke patient requires well-organized systems that maximize
speed of assessment and administration of appropriate therapies, including IV rtPA and
endovascular therapies.

Key Points
& Rapid evaluation and treatment are critical for the best outcomes.
& A patient should receive IV recombinant tissue-type plasminogen activator within 1 hour
of arrival to an emergency department (and sooner is better).
& IV thrombolysis, specifically using recombinant tissue-type plasminogen activator,
should be considered for patients for whom it can be administered within 4.5 hours of
onset of stroke deficits.
& For patients ineligible for IV recombinant tissue-type plasminogen activator who have
significant stroke deficits (typically, an NIH Stroke Scale score of 8 or greater),
endovascular therapy should be considered if treatment can be initiated within 6 hours.

* 2014, American Academy of Neurology.

Copyright © American Academy of Neurology. Unauthorized reproduction of this article is prohibited.


& Rapidly improving deficits should not be considered a contraindication unless the
remaining deficit is minor.
& Additional exclusion criteria for IV recombinant tissue-type plasminogen activator
within the 3- to 4.5-hour time window include a history of stroke and diabetes mellitus,
NIH Stroke Scale score greater than 25, age greater than 80 years, and warfarin use
(regardless of international normalized ratio value).
& All clinicians who care for stroke patients in the acute setting should be NIH Stroke
Scale certified.
& The risk of intracranial hemorrhage when treating a stroke mimic with IV recombinant
tissue-type plasminogen activator inadvertently is extremely low (less than 1%).
& The combined IV/endovascular approach in all patients with an NIH Stroke Scale score
of 8 or greater is not superior to IV recombinant tissue-type plasminogen activator
alone despite comparable safety.
& One should not delay IV recombinant tissue-type plasminogen activator administration
for any laboratory result other than finger-stick glucose level unless a clinical suspicion
for an abnormality exists.
& Subtle ischemic changes on CT scan are not contraindications to acute reperfusion therapy.
& Standard postYtissue plasminogen activator management for the first 24 hours includes
aggressive blood pressure treatment if systolic blood pressure is greater than 180 mm Hg
or diastolic blood pressure is greater than 105 mm Hg.
& Experts generally recommend permissive hypertension if tolerated (up to 220/120 mm Hg)
in the nonreperfusion therapy setting.
& Current recommendations are to treat hyperglycemia to a level lower less than 180 mg/dL.
& Early aspirin treatment leads to a 1% absolute reduction of stroke over the next 2 weeks.
& Short-term combined aspirin and clopidogrel administration may better prevent early
stroke recurrence in minor strokes or TIAs.

Risk Factor Management for Stroke


Prevention
Prabhakaran, Shyam MD, MS; Chong, Ji Y. MD. Continuum (Minneap Minn). April 2014;
20(2 Stroke): 296Y308.

Abstract
Purpose of Review:
This review focuses on the recommendations for management of hypertension, dyslipidemia,
diabetes mellitus, diet, physical activity, and lifestyle choices commonly encountered in
neurologic practice. Specific studies, including those relevant to lipid targets, blood pressure
targets, and adherence to medications after stroke, are reviewed.
Recent Findings:
In addition to traditional risk factors such as hypertension, dyslipidemia, and diabetes mellitus,
this review discusses sleep apnea, diet, physical activity, and other novel risk factors that are
potentially modifiable. Recent studies confirm that pharmacologic strategies to achieve

* 2014, American Academy of Neurology.

Copyright © American Academy of Neurology. Unauthorized reproduction of this article is prohibited.


aggressive targets for lipid and blood pressure lowering have significant impact on recurrent
stroke risk.
Summary:
Optimal secondary prevention strategies can prevent as much as 80% of all recurrent strokes.
Key Points
& Studies suggest that 90% of strokes can be explained by modifiable risk factors, and 80%
of recurrent strokes can be prevented with optimal risk factor control and/or elimination.
& Goal blood pressure (BP) lowering should strive for a 10Ymm Hg and 5Ymm Hg
reduction in systolic BP and diastolic BP, respectively, from baseline even in
nonhypertensive patients, and less than 140/90 mm Hg in hypertensive patients
(less than 130/80 mm Hg in patients with diabetes mellitus and chronic kidney disease).
& The most recent American Heart Association guidelines recommend strict goals of
low-density lipoprotein (LDL) cholesterol level lowering by 50% or more or an absolute
LDL goal level of less than 70 mg/dL.
& While the risk of intracerebral hemorrhage may not be increased in patients without
prior stroke, patients with prior lobar hemorrhage, who have higher rates of recurrent
spontaneous intracerebral hemorrhage than those with deep hemorrhage, may be
particularly at risk on statin therapy.
& Glycemic control should aim for hemoglobin A1c of less than 7% using diet, exercise, and
medications such as oral hypoglycemic drugs and insulin.
& Despite the lack of benefit in clinical trials, it is reasonable to treat hyperhomocysteinemia
(greater than 15 Hmol/L), if present, with vitamin B6, vitamin B12, and folic acid
replacement in patients with atherosclerotic ischemic stroke.
& Effective means of maintaining medication adherence can be elusive, and few randomized
trials of interventions to improve compliance are available. Patients should receive
education on their medications and have adequate follow-up.
& Cigarette smoking is a well-established risk factor for stroke and has a strong association
with atherothrombotic stroke.
& Meta-analyses of sodium reduction trials have demonstrated that decreased sodium intake to
1800 mg per day was associated with a 2Ymm Hg reduction in systolic BP and 1Ymm Hg
reduction in diastolic BP in nonhypertensive patients and a 5Ymm Hg (systolic BP) and
2.7Ymm Hg (diastolic BP) drop in hypertensive patients.
& Several studies have suggested the Mediterranean diet as beneficial in cardiovascular
health. The Mediterranean diet encourages fresh fruits and vegetables, fish, legumes,
white meat instead of red meat, and wine.
& No completed randomized trials have evaluated weight loss in preventing vascular
events or in secondary stroke prevention, although maintenance of body mass index within
18.5 kg/m2 to 25 kg/m2 is reasonable.
& Adherence to a healthy lifestyle could have a significant effect on global stroke incidence.

Evaluation and Prevention of


Cardioembolic Stroke
Kim, Anthony S. MD, MAS. Continuum (Minneap Minn). April 2014; 20(2 Stroke): 309Y322.

* 2014, American Academy of Neurology.

Copyright © American Academy of Neurology. Unauthorized reproduction of this article is prohibited.


Abstract
Purpose of Review:
The potential for cardioembolic stroke has important implications for clinical management. This
review describes the diagnostic workup and management options for this key stroke subtype.
Recent Findings:
The suspicion for a cardioembolic source for stroke is raised with a large vessel occlusion
or when strokes occur in multiple vascular territories. Diagnostic workup includes ECG,
echocardiography, and cardiac monitoring. Atrial fibrillation is the most common cause of
cardioembolic stroke and typically justifies anticoagulation therapy. New data on other
mechanisms of cardioembolic strokeYYsuch as congestive heart failure, prosthetic valves,
and aortic arch diseaseYYas well as the availability of novel oral anticoagulants have implications
for optimizing stroke prevention.
Summary:
Cardiogenic embolization is an important cause of stroke with important implications for
diagnosis, treatment, and prevention.

Key Points
& Cardioembolic strokes are often more severe than other stroke subtypes and have a high
recurrence risk.
& The suspicion for cardioembolic stroke is raised with large artery occlusions or strokes in
multiple vascular territories.
& Transesophageal echocardiography is helpful for evaluating the aortic arch, aortic valve,
atrial septum, and left atrial appendage.
& The incidence of stroke from atrial fibrillation is expected to surge in the coming decades
as the US population ages.
& Relatively short and asymptomatic periods of atrial fibrillation may be sufficient to form
thrombus and cause stroke.
& Patients with atrial fibrillation and a history of stroke or TIA are in a high-risk category
that typically justifies anticoagulation.
& The absence of symptoms of atrial fibrillation such as reduced exercise tolerance,
shortness of breath, or palpitations, or an initial ECG showing sinus rhythm is not
sufficient to exclude the possibility of paroxysmal atrial fibrillation as a cause of stroke.
& Extended cardiac monitoring increases the sampling period for detecting paroxysmal
atrial fibrillation, but the appropriate candidates for monitoring, the optimal duration and
mode of monitoring, and the clinical significance of fleeting episodes of atrial fibrillation
have not been established.
& Anticoagulation is preferred over antiplatelet agents for secondary stroke prevention in
most patients with atrial fibrillation.
& Medications for rate or rhythm control for atrial fibrillation do not obviate the need for
antithrombotic therapy in patients with atrial fibrillation.
& Potential benefits of novel oral anticoagulants compared to warfarin include noninferior
(rivaroxaban) or superior efficacy (dabigatran and apixaban), lower intracranial bleeding,

* 2014, American Academy of Neurology.

Copyright © American Academy of Neurology. Unauthorized reproduction of this article is prohibited.


fixed dosing, fewer drug-drug interactions, rapid onset of anticoagulant effect, and a
mortality benefit (apixaban).
& Potential drawbacks of novel oral anticoagulants compared to warfarin include the lack of
a validated reversal strategy or laboratory test of anticoagulant effect, uncertainty about
candidacy for acute thrombolytic therapy, and cost.
& A significant proportion of patients with atrial fibrillation who would benefit from
anticoagulation are not on anticoagulation therapy.
& Overestimating the bleeding risk associated with falls and advanced age and
underappreciating the benefits of anticoagulation with age contribute to the underuse of
anticoagulation for atrial fibrillation.
& A transthoracic echocardiogram is not adequate to exclude the possibility of valvular
vegetations when endocarditis is suspected. Rather, a transesophageal echocardiogram
is critical to the diagnostic workup of possible endocarditis.
& Anticoagulation with warfarin is strongly indicated for all patients with mechanical heart
valves.
& For patients with congestive heart failure with sinus rhythm, the choice of anticoagulation
or antiplatelet agent for secondary stroke prevention is uncertain.
& Patients with left ventricular thrombus should generally be treated with anticoagulation
for at least 3 months.

Large Artery Atherosclerosis: Carotid


Stenosis, Vertebral Artery Disease,
and Intracranial Atherosclerosis
Chaturvedi, Seemant MD, FAAN; Bhattacharya, Pratik MD, MPH. Continuum (Minneap Minn).
April 2014; 20(2 Stroke): 323Y334.

Abstract
Purpose of Review:
Large artery atherosclerosis is an important cause of ischemic stroke. Recent randomized clinical
trials have helped clarify the treatment options for conditions such as carotid stenosis and
intracranial atherosclerosis. This review outlines the primary findings of these trials and provides
current recommendations for treatment.

Recent Findings:
Carotid revascularization is preferred in patients with severe symptomatic carotid stenosis.
Carotid endarterectomy achieves lower rates of stroke or death than carotid artery stenting. The
risk of stroke or death with stenting is higher among older patients and women. Intensive
medical therapy achieves low stroke and death rates in asymptomatic stenosis. Medical therapy
and treatment of atherosclerotic risk factors are the mainstay of therapy for intracranial
atherosclerosis, and medical therapy is recommended for patients with vertebral artery
origin atherosclerosis.

* 2014, American Academy of Neurology.

Copyright © American Academy of Neurology. Unauthorized reproduction of this article is prohibited.


Summary:
Contemporary medical therapy is paramount in large artery atherosclerosis. Patient demographics,
comorbidities, and the periprocedural risks of stroke and death should be carefully weighed
while choosing a revascularization procedure for carotid stenosis.

Key Points
& Symptomatic carotid stenosis has a markedly increased risk of stroke compared with
asymptomatic stenosis.
& In the Carotid Revascularization Endarterectomy versus Stenting Trial (CREST) study,
periprocedural stroke was higher with carotid artery stenting, whereas periprocedural
myocardial infarction was higher with carotid endarterectomy.
& In the CREST study, periprocedural stroke had a greater impact on patients’ quality of life
than periprocedural myocardial infarction.
& Carotid revascularization within 2 weeks of a stroke or TIA is preferred over delaying
revascularization for 6 weeks or more.
& Symptomatic patients appear to have a lower complication rate with carotid
endarterectomy than with carotid artery stenting.
& A new clinical trial, CREST-2, will evaluate whether optimal medical therapy alone is the
preferred treatment for asymptomatic carotid stenosis.
& Intensified medical therapy has reduced the stroke rate for patients with asymptomatic
carotid stenosis.
& In CREST, women had a higher complication rate with carotid artery stenting, compared
with carotid endarterectomy.
& Patients older than 70 years have a higher complication rate with carotid artery stenting
than with carotid endarterectomy.
& Ethnic differences exist with regard to the frequency of intracranial atherosclerosis.
& Antiplatelet therapy, rather than anticoagulation, is preferred for intracranial
atherosclerosis.
& Control of hyperlipidemia and blood pressure is critical in patients with intracranial
atherosclerosis.
& In the Stenting and Aggressive Medical Management for Preventing Recurrent Stroke in
Intracranial Stenosis (SAMMPRIS) trial, medical therapy was superior to intracranial
stenting for stroke prevention.
& Procedural strokes following intracranial stenting can be due to diverse mechanisms.
& No high-quality studies have been done to evaluate treatment methods for vertebral artery
origin stenosis.
& Aggressive medical therapy is preferred as initial treatment for vertebral artery
origin stenosis.

Evaluation and Management of Cerebral


Venous Thrombosis
Bushnell, Cheryl MD, MHS; Saposnik, Gustavo MD, MSc, FAHA, FRCP(C). Continuum
(Minneap Minn). April 2014; 20(2 Stroke): 335Y351.

* 2014, American Academy of Neurology.

Copyright © American Academy of Neurology. Unauthorized reproduction of this article is prohibited.


Abstract
Purpose of Review:
Thrombosis of the dural sinus and/or cerebral veins (CVT) is a rare but potentially devastating type
of stroke that tends to occur in young adults, especially women. In this article, the abbreviation CVT
refers to either cerebral venous thrombosis or dural sinus thrombosis. The purpose of this review is to
review the most up-to-date literature on the epidemiology, diagnosis, management, and prognosis of
CVT. In addition, illustrative cases that represent the spectrum of CVT are provided.

Recent Findings:
CVT represents about 0.5% of all strokes and can be challenging to diagnose because headache,
rather than focal neurologic symptoms, is the prominent feature. The diagnosis is confirmed
with MRI and magnetic resonance venogram (MRV). The mainstay of acute management is
anticoagulation, although, in the cases of severe hemorrhagic conversion of a venous infarction,
endovascular mechanical thrombectomy may be potentially lifesaving. The evaluation of
underlying causes from transient triggers, eg, pregnancy, oral contraceptives, or infection, versus
chronic triggers, eg, cancer and thrombophilia, will often influence the duration of anticoagulation.
The outcomes after CVT are generally favorable, and the risk of recurrence is low.

Summary:
CVT is an important diagnosis to keep in mind when evaluating patients with headache in the
emergency department, and it is important that it not be overlooked because it is treatable.

Key Points
& Cerebral venous thrombosis is a cerebrovascular disease of the young, primarily of women.
& Headache is the most common symptom of cerebral venous thrombosis.
& The most common clinical syndromes in cerebral venous thrombosis include (1)
intracranial hypertension, (2) focal neurologic deficits (eg, motor weakness, sensory
deficit, aphasia), (3) encephalopathy, and (4) seizure disorder.
& MRI/magnetic resonance venography is the recommended imaging for the diagnosis
of cerebral venous thrombosis. Specific sequences (eg, T2* susceptibility-weighted
imaging) are useful to assist in the diagnosis of isolated cortical venous thrombosis.
& Transient risk factors for cerebral venous thrombosis include pregnancy/puerperium,
CNS or ear/sinus/mouth/ face infections, exposure to drugs (eg, oral contraceptives,
steroids, cancer treatments), head trauma, or procedures (eg, lumbar puncture, jugular
catheter placement).
& Chronic triggers of cerebral venous thrombosis include hereditary or acquired
thrombophilias that are established causes of venous thromboembolism.
& Anticoagulation is the main treatment for the acute management of cerebral venous
thrombosis.
& All available guidelines recommend anticoagulation therapy for the acute management of
cerebral venous thrombosis.
& Endovascular interventions are reserved for patients with progressive neurologic
deterioration despite intensive medical treatment.
& Seizures are a common manifestation, observed in 30% to 50% of patients with cerebral
venous thrombosis.

* 2014, American Academy of Neurology.

Copyright © American Academy of Neurology. Unauthorized reproduction of this article is prohibited.


& Antiepileptic drug treatment for cerebral venous thrombosis is only recommended for
patients who have had a seizure with or without parenchymal lesions (eg, venous infarction,
intracranial hemorrhage). Prophylactic use of antiepileptic drugs is not recommended.
& Intracranial hypertension may be seen in 15%to 40% of patients with cerebral venous
thrombosis.
& The risk of recurrence after an unprovoked cerebral venous thrombosis is low, but severe
thrombophilias are associated with a high risk of recurrence of cerebral venous
thrombosis as well as systemic venous thromboembolism.
& For provoked cerebral venous thrombosis, anticoagulation consisting of adjusted-dose
warfarin is recommended for 3 to 6 months. An evaluation for thrombophilia will
potentially aid in the decision for the duration of anticoagulation.

Evaluation and Management of Stroke


in Young Adults
Mackey, Jason MD, MS. Continuum (Minneap Minn). April 2014; 20(2 Stroke): 352Y369.

Abstract
Purpose of Review:
This article provides an overview of the evaluation and management of ischemic stroke in
young adults, with an emphasis on cervical artery dissection, patent foramen ovale, and
hypercoagulable states.
Recent Findings:
The incidence of ischemic stroke in young patients is increasing, although the reasons remain
unclear. Patients with ischemic stroke at a young age are more likely to die at an early age than their
peers. Well-established vascular risk factors are prevalent in young stroke patients. Recent studies
have informed the treatment of dissection and patent foramen ovale among others. The utility of
testing for hypercoagulable states in ischemic stroke is unclear.
Summary:
Ischemic stroke in young adults is a major public health problem. A wide range of etiologies
of ischemic stroke is found in this age group. A careful history, thorough examination, and
methodical workup are essential. Specific management is predicated on identification of the
underlying etiology.

Key Points
& The incidence of stroke in young adults appears to be increasing.
& Stroke in young adults is associated with increased risk of early death.
& Standard vascular risk factors are prevalent in young stroke patients.
& CT angiography and magnetic resonance angiography (MRA) have supplanted
conventional angiography in the diagnosis of dissection.
& CTA appears to have some advantages over MRA in the diagnosis of dissection.

* 2014, American Academy of Neurology.

Copyright © American Academy of Neurology. Unauthorized reproduction of this article is prohibited.


& There is no contraindication to tissue plasminogen activator in strokes due to dissection.
& Patent foramen ovale closure trials have yet to demonstrate superiority over medical
management.
& The best long-term antithrombotic agent for stroke patients with patent foramen ovale
remains unclear.
& The thrombophilias with the best evidence for venous thromboembolism are factor V
Leiden, prothrombin gene mutation, protein S deficiency, protein C deficiency, and
antithrombin III deficiency.
& Studies have not demonstrated a convincing association between hypercoagulable states
and ischemic stroke.
& Young adults with stroke and a right-to-left shunt should be evaluated for deep venous
thrombosis with compression ultrasonography of the lower extremities and venography
of the pelvis.

Pediatric Arterial Ischemic Stroke


Moharir, Mahendra MD, MSc, FRACP; deVeber, Gabrielle MD. Continuum (Minneap Minn).
April 2014; 20(2 Stroke): 370Y386.

Abstract
Purpose of Review:
This article aims to provide a broad overview of pediatric arterial ischemic stroke, from
recognition and diagnosis to the short-term and long-term management based on current available
literature.
Recent Findings:
Arterial ischemic stroke in children represents a significant disorder with a concerning high rate
of adverse outcomes, including potentially preventable recurrent stroke. Although awareness of
pediatric stroke is increasing, diagnosis is still commonly delayed or missed altogether,
particularly in younger children. Current vascular imaging techniques have limitations in accurate
diagnosis of arteriopathies that are now recognized as an important cause of childhood stroke.
Significant variability exists in treatment of pediatric stroke. Management is based on published
consensus guidelines; however, individual children require an individualized approach.
Summary:
As pediatric stroke specialists become increasingly available, the collaboration of such experts on
individual management is crucial. Definitive evidence-based treatment for pediatric stroke awaits
the development of randomized controlled trials.

Key Points
& Arterial ischemic stroke is rarer and has a subtler clinical presentation and a wider
differential diagnosis in children than in adults, which results in delayed and sometimes
missed diagnosis.
& Arterial ischemic stroke should be suspected in any newborn with seizures, particularly
when seizures begin more than 12 hours after birth.

* 2014, American Academy of Neurology.

Copyright © American Academy of Neurology. Unauthorized reproduction of this article is prohibited.


& A feature of TIAs unique to children is a history of paroxysmal episodes of neurologic
dysfunction precipitated by hyperventilation, which should raise the suspicion of
moyamoya.
& When considering antithrombotic therapy in any child with arterial ischemic stroke, it is
critical to continually balance the risk (hemorrhagic complication) of antithrombotic therapy
with the risk of nontreatment (extension of preexisting infarction and recurrence of stroke).
& The risk-benefit ratio of tissue plasminogen activator in pediatric arterial ischemic stroke
is unknown.
& After the diagnosis of acute arterial ischemic stroke, antiplatelet and anticoagulant
therapies in children are usually selected based on the perceived mechanism for arterial
ischemic stroke.
& From a practical standpoint, consensus exists among pediatric stroke experts that some
level of antithrombotic therapy is beneficial in preventing acute progression of thrombus
and stroke recurrence.
& Antithrombotic therapy is rarely indicated in most cases of perinatal or neonatal arterial
ischemic stroke because of negligible recurrence risk except in the presence of congenital
heart disease and abnormal prothrombotic workup.
& Because deficits from perinatal or neonatal arterial ischemic stroke may only emerge with
maturation, longitudinal observation is necessary even in apparently healthy infants.
& Although congenital heart disease correction should theoretically eliminate the
cardiogenic stroke risk, recent data demonstrate that recurrence risk remains increased for
extended periods even after full congenital heart disease correction.
& Beyond the newborn period, arteriopathy (vasculopathy) accounts for approximately
60% of all childhood arterial ischemic stroke and predicts stroke recurrence.
& Transient cerebral arteriopathy of childhood is a well-described unilateral focal arteriopathy
of presumed inflammatory origin. Transient cerebral arteriopathy is characterized by
basal ganglia infarction with ipsilateral irregular stenosis involving the carotid T-junction
represented by the distal internal carotid artery, proximal anterior cerebral artery,
and proximal middle cerebral artery.
& Pediatric moyamoya mainly presents with ischemic stroke whereas adult moyamoya
often presents with hemorrhagic stroke.
& Encephaloduroarteriosynangiosis is preferred over superficial temporal arteryYmiddle
cerebral artery bypass in young children because the small size of the arteries makes
surgery challenging.

Unruptured Intracranial Aneurysms:


Screening and Management
Kelly, Adam G. MD. Continuum (Minneap Minn). April 2014; 20(2 Stroke): 387Y398.

Abstract
Purpose of Review:
Unruptured intracranial aneurysms are found commonly in the general public, and more frequently
in certain populations. This article focuses on the epidemiology, screening strategies, and
management options for patients with unruptured aneurysms.

* 2014, American Academy of Neurology.

Copyright © American Academy of Neurology. Unauthorized reproduction of this article is prohibited.


Recent Findings:
Recent epidemiologic studies show the overall prevalence of intracranial aneurysms to be
approximately 3%, with higher rates seen in familial aneurysm syndromes and in certain medical
conditions, such as autosomal dominant polycystic kidney syndrome. Aneurysm treatment may
include surgical or endovascular techniques, with increasing utilization of endovascular strategies
over time. Increased aneurysm diameter, certain locations, and other anatomical considerations may
be associated with higher risks of aneurysm rupture.
Summary:
Given the high morbidity and mortality associated with aneurysm rupture, screening for
unruptured aneurysms is generally recommended for highrisk patients (patients who have at least
two first-degree relatives with aneurysms, and patients with autosomal dominant polycystic
kidney disease). Screening may be considered for other patients (eg, one first-degree relative
with aneurysm) after discussion of the risks and benefits of imaging. Following identification
of an aneurysm, decisions regarding observation or treatment should be based on patient
characteristics, features of the aneurysm, and provider expertise.

Key Points
& The prevalence of aneurysms in the general population is estimated at approximately 3%.
& A family history of aneurysms is a strong predictor of aneurysm development, and familial
aneurysms may have a higher risk of rupture than aneurysms of similar size and location in
nonfamilial syndromes.
& Autosomal dominant polycystic kidney disease is a strong predictor of the presence of
unruptured intracranial aneurysms.
& Most unruptured intracranial aneurysms are discovered when asymptomatic, although
other presenting symptoms include compressive cranial neuropathies, seizures, or
ischemic stroke.
& Patients with two or more first-degree relatives with unruptured intracranial aneurysms
or subarachnoid hemorrhage, or patients with autosomal dominant polycystic kidney
disease, are candidates for intracranial imaging to screen for unruptured intracranial
aneurysms.
& Both CT angiography and magnetic resonance angiography are reasonable options for
noninvasive screening imaging for unruptured intracranial aneurysms, although catheter
angiography remains the gold standard for aneurysm identification.
& The risk of aneurysm rupture appears to increase as aneurysm size increases.
& Data on the risk of aneurysm rupture based on location are inconsistent across studies.
& Certain anatomic features, such as the presence of a daughter sac, may increase the risk
of aneurysm rupture.
& If indicated, interventional options for unruptured intracranial aneurysm management
include endovascular and surgical approaches; large randomized trial data to support one
strategy over the other are lacking.

Genetic Stroke Syndromes


Barrett, Kevin M. MD, MSc; Meschia, James F. MD. Continuum (Minneap Minn). April 2014;
20(2 Stroke): 399Y411.

* 2014, American Academy of Neurology.

Copyright © American Academy of Neurology. Unauthorized reproduction of this article is prohibited.


Abstract
Purpose of Review:
This review describes the clinical and radiographic features, genetic determinants, and treatment
options for the most well-characterized monogenic disorders associated with stroke.
Recent Findings:
Stroke is a phenotype of many clinically important inherited disorders. Recognition of the clinical
manifestations of genetic disorders associated with stroke is important for accurate diagnosis and
prognosis. Genetic studies have led to the discovery of specific mutations associated with the
clinical phenotypes of many inherited stroke syndromes.
Summary:
Several inherited causes of stroke have established and effective therapies, further underscoring
the importance of timely diagnosis.

Key Points
& The cerebrovascular manifestations of Fabry disease include ischemic stroke at an early
age, often involving the vertebrobasilar circulation, dolichoectasia of cerebral vessels,
and white matter abnormalities on brain MRI.
& Fabry disease is caused by genetic mutations that result in deficient activity of the
lysosomal hydrolase >-galactosidase A.
& Recombinant >-galactosidase A is available as enzyme replacement therapy for Fabry
disease and should be considered for affected individuals.
& Sickle cell disease is complicated by stroke in 25% of affected individuals before the age
of 45 years.
& Sickle cell disease is caused by a point mutation that results in substitution of valine for
glutamic acid in position number 6 of the A-polypeptide chain of hemoglobin.
& Children with sickle cell disease should be routinely screened with transcranial Doppler,
and those with mean flow velocities greater than 200 cm/sec should receive chronic
transfusion therapy to reduce the hemoglobin S fraction to less than 30%.
& T2 hyperintensities involving the white matter of the anterior temporal poles (O’Sullivan
sign) are seen in 90% of patients with cerebral autosomal dominant arteriopathy with
subcortical infarcts and leukoencephalopathy.
& Cerebral autosomal dominant arteriopathy with subcortical infarcts and
leukoencephalopathy is associated with mutations in the NOTCH3 gene on chromosome
19p13.2-p13.1, which codes for a large transmembrane protein necessary for vascular
smooth muscle differentiation and development.
& False-negative results can occur with genetic analysis and should prompt skin biopsy to
evaluate for granular osmophilic material in the vascular basal lamina, a finding highly
specific for cerebral autosomal dominant arteriopathy with subcortical infarcts and
leukoencephalopathy.
& Stroke with onset in the third decade, and dementia with onset in the third through fifth
decade, are characteristic of cerebral autosomal recessive arteriopathy with subcortical
infarcts and leukoencephalopathy. Systemic manifestations include premature alopecia
with onset in the teen years and cervical or lumbar spondylosis in the second and third
decades of life.

* 2014, American Academy of Neurology.

Copyright © American Academy of Neurology. Unauthorized reproduction of this article is prohibited.


& Vision and memory loss, seizures, hemiparesis, apraxia, and dysarthria with onset in the
fourth decade of life followed by death 5 to 10 years after clinical presentation is the
typical disease progression in retinal vasculopathy with cerebral leukodystrophy.
& The mitochondrial encephalopathy, lactic acidosis, and strokelike episodes disorder often
presents in childhood after normal development, and the clinical course can be
relapsing-remitting with eventual development of progressive neurologic disability.
& The mitochondrial encephalopathy, lactic acidosis, and strokelike episodes syndrome is a
maternally inherited mitochondrial disorder, with 80% of cases associated with the
A3243G mutation in the gene encoding transfer RNALEU(UUR).
& The classic angiographic appearance of moyamoya is stenosis and occlusion of the
bilateral distal internal carotid arteries and proximal middle/anterior cerebral arteries
accompanied by a network of abnormal lenticulostriate collateral vessels that has the
appearance of a puff of smoke.

Predicting Outcomes After Transient


Ischemic Attack and Stroke
Rempe, David A. MD, PhD. Continuum (Minneap Minn). April 2014; 20(2 Stroke): 412Y428.

Abstract
Purpose of Review:
Predicting functional outcome and mortality after stroke, with or without thrombolysis, is a
critical role of neurologists. This article reviews the predictors of outcome after ischemic stroke.
Recent Findings:
Several scores were recently designed to predict (1) mortality and poor functional outcome after
ischemic stroke, (2) the functional outcome and risk of symptomatic intracranial hemorrhage (sICH)
after thrombolysis, and (3) the risk of stroke following TIA. Validation of these prediction instruments
is ongoing, and studies will be critical to determine the general applicability of these scores.
Summary:
Although several scores were developed to predict mortality and outcome after stroke, it may be
premature to employ these prediction scores to determine individual patient outcome. Similarly,
prediction scores should not be used to deny patients tissue plasminogen activator (tPA),
even if the scores predict that the patient has a high likelihood of sICH or poor outcome
after thrombolysis.

Key Points
& The patient’s age and stroke severity are the two common independent predictors of
mortality after stroke.
& Several factors that make a prediction scale valuable include accuracy of the prediction,
validation in separate populations, applicability to the patient population in whom the rule

* 2014, American Academy of Neurology.

Copyright © American Academy of Neurology. Unauthorized reproduction of this article is prohibited.


will be applied, ease of use, and its ability to affect treatment decisions and prevent or
delay poor outcomes.
& To be clinically useful, a stroke outcomes score should be validated in several patient
populations or registries distinct from those used to derive the score.
& Prediction scores should include independent variables that are easily measurable, be
readily available to clinicians, have an unambiguous scoring system, and predict outcome
without needing a complicated calculation.
& As the treatment of stroke evolves and improves, the accuracy of prediction scores may
diminish.
& Prediction scores are unlikely to incorporate all the factors that are important to outcome
for a particular patient.
& Only one of the prediction scores has been demonstrated to improve prediction of patient
outcome compared to an experienced clinician’s judgment. The scores may give a false
sense of confidence in predictions of poor outcome, leading to early withdrawal of care in
some patients.
& Stroke severity and glucose abundance are central factors driving both functional
outcome and symptomatic intracranial hemorrhage after thrombolysis.
& The ability of scores to predict outcome or symptomatic intracranial hemorrhage risk
following tissue plasminogen activator administration needs to be viewed with caution, as
their accuracy in validation cohorts is variable and lower than in their derivation studies.
& It is premature to use scores from various prediction instruments as a means to exclude
patients from thrombolysis. Similarly, it would be premature to bypass tissue
plasminogen activator and perform other interventions based on these scores.
& One of the most commonly used scores to predict stroke risk following TIA is the
ABCD2 score.
& While the ABCD2 score is widely used for stratifying risk of stroke recurrence in
TIA patients, some studies have suggested a limited predictive value of the ABCD2.
& The identification of diffusion-weighted imagingYpositive strokes in TIA patients
significantly increases the risk of recurrent stroke.

* 2014, American Academy of Neurology.

Copyright © American Academy of Neurology. Unauthorized reproduction of this article is prohibited.


Ethical Perspective

Telephone Consultations
Address correspondence to
Dr Justin A. Sattin, Department
of Neurology, 1685 Highland
Ave, Madison, WI 53711,

for Tissue Plasminogen sattin@neurology.wisc.edu.


Relationship Disclosure:
Dr Sattin reports no disclosure.

Activator Administration Unlabeled Use of


Products/Investigational
Use Disclosure:

in Acute Stroke Dr Sattin reports no disclosure.


* 2014, American Academy
of Neurology.
Justin A. Sattin, MD

ABSTRACT
Effective treatment for acute ischemic stroke has been available for 17 years, but
wide geographic variability remains in timely access to neurologic expertise and
other components of stroke systems of care. Telemedical technology can be used to
improve such access, but it is debatable whether neurologists have an ethical
obligation to provide consultation regarding tissue plasminogen activator use via
the telephone. This article examines whether neurologists are ethically obligated to
provide telephone-mediated acute stroke consultation.

Continuum (Minneap Minn) 2014;20(2):429–435.

Case 1
A neurologist, on call for the primary stroke center at which he’s on staff,
responded to a page from the telecommunications operator. An emergency
medicine physician from a nonaffiliated hospital was on the line requesting
requesting transfer of an acute stroke patient.
The emergency physician presented the case of an 82-year-old man with
vascular risk factors who abruptly developed right hemiparesis 1 hour before the
call. At the time of the call, the patient’s blood pressure was 160/90 mm Hg.
His neurologic examination was described as showing no aphasia, neglect,
or other gross mental status abnormality. The emergency physician
reported the patient to be severely hemiparetic on the right, with
an NIH Stroke Scale score of 12. The patient’s head CT had been read by
the local radiologist and reportedly showed no hemorrhage or early ischemic
change.
The emergency physician wanted to transfer the patient but asked for
advice on whether to administer tissue plasminogen activator (tPA) first.
He had reviewed a checklist of tPA contraindications and found none.
Should the neurologist make such a treatment recommendation?

Case 2
The same neurologist took a call from a family practice resident
moonlighting in a nonaffiliated rural emergency department; he
wanted to transfer an acute stroke patient.
Continued on page 430

Continuum (Minneap Minn) 2014;20(2):429–435 www.ContinuumJournal.com 429


Copyright © American Academy of Neurology. Unauthorized reproduction of this article is prohibited.
Telephone Consultations in Acute Stroke

Continued from page 429


The patient was a 76-year-old woman presenting with altered mental
status. She had a left hemispheric stroke 2 years ago and was known to
have language and cognitive impairments at baseline; she lived with
her daughter. She had reported having a headache earlier in the day
but otherwise was not speaking much. The daughter had gone shopping
briefly and returned home to find her mother severely aphasic and confused,
which prompted her to call 911.
The physician reported that the patient’s blood pressure was 180/100 mm Hg.
He described the patient as obtunded and confused, not following
commands. She preferred to look to the left. The right face was suspected to
be slightly weak. She would not cooperate with motor testing, but spontaneous
movements of the right arm and leg were less than those on the left. Head
CT had been done, but no formal reading was available; the referring physician
stated that he saw no blood.
The physician asked to transfer the patient but also wanted to know whether
to administer tPA first. Should the neurologist make such a recommendation?

DISCUSSION
IV tPA is the best-proven treatment for acute ischemic stroke, and its efficacy
is highly time dependent.1 Communities are developing stroke systems of
care in order to quickly identify stroke patients, determine tPA eligibility,
administer the drug, and provide appropriate postthrombolytic care.2,3
Neurologists serve very important roles in stroke systems of care, but there is
wide geographic variability in access to neurologic expertise, especially on an
on-call basis.4
Telecommunication technologies can help overcome this geographic
variability by allowing remotely located neurologists to interview and examine
stroke patients, review their neuroimaging studies, and make tPA treatment
recommendations to local physicians.5 The use of such telestroke systems has
been shown to result in more accurate recommendations regarding tPA use as
compared to telephone-mediated consultation.6 Presently, however, relatively
few hospitals have developed telestroke programs, and neurologists are often
called on to render treatment advice regarding acute stroke patients via
telephone. Evidence to support this practice is limited.7,8

Legal Considerations
A threshold question is whether the neurologist owes a legal duty of care to the
patient. That is, must the neurologist even take the phone calls? In most cases,
the patient will not have already established care with the neurologist’s practice
and so the neurologist would not have a preexisting legal obligation to consult
on the individual’s case. A duty may arise from contractual agreements among
community and tertiary hospitals and their medical staffs to provide on-call
specialty services and accept patients in transfer. Such agreements are
becoming more common as hospitals seek primary stroke center and acute
stroke-ready designations.9 Even in the absence of contractual agreements,
however, the Emergency Medical Treatment and Active Labor Act (EMTALA)
requires hospitals that are designated regional referral centers or that offer

430 www.ContinuumJournal.com April 2014

Copyright © American Academy of Neurology. Unauthorized reproduction of this article is prohibited.


specialized capabilities to accept in transfer appropriate patients who require
their services.10 Analysis of physicians’ obligations under EMTALA and hospital
bylaws is outside the scope of this article; the following discussion assumes that
the neurologist, being on call for an accredited stroke center, does take the
phone calls in order to at least discuss the merits of transfer.
A variety of other legal issues are pertinent to telemedical consultation.
These involve licensure (if the patient and physician are in different states),
privileging, documentation, billing, and professional liability; they are reviewed
elsewhere.11,12

Ethical Analysis
Once taking the phone calls, the neurologist is faced with two ethical questions:
Is there an obligation to recommend, over the telephone, for or against tPA use?
If not obligatory, is it permissible to make such recommendations?
One argument in support of the neurologist’s providing advice about tPA is
that it is beneficent to do so, meaning that the intent is to help the patient.13
Most neurologists have extensive training and experience in the diagnosis and
treatment of acute stroke. Many emergency physicians will not administer tPA
absent neurologic consultation,14,15 a position supported by some professional
societies.16,17 Most emergency physicians will, however, administer tPA if a
neurologist is at least available to consult via telephone,14,15 and the neurologist
consulting via telephone will usually make the correct decision regarding tPA use.6
As will be discussed further below, the neurologist does have the option of
simply accepting the patients in transfer for further evaluation and treatment.
To the extent that the receiving facility has an organized stroke unit, transfer
alone will confer significant benefits in terms of mortality and functional
outcome.18 However, the transfers will impose treatment delays; even if the
referring facilities are relatively close and the patients are still within the IV tPA
time window upon arrival to the stroke center, the odds of favorable responses
to the drug will have fallen.19 The patients may still qualify for endovascular
treatments, but these should generally not substitute for IV tPA administration
in otherwise eligible patients.1
On the side of the argument against the neurologist’s making tPA
recommendations is the obligation of nonmaleficenceVthe obligation not to
inflict harm, or the risk of harm. Incorrect tPA treatment recommendations
(both over- and under-treatments) are significantly more likely to result from
consultation by telephone as compared to full audio-visual evaluation.6 The risk
of adverse outcomes from incorrect treatment recommendations is relatively
low to the extent that they are due to treatment of patients with stroke
mimics,20,21 but treatment of patients with tPA contraindications may result in a
higher risk of intracerebral hemorrhage.22,23 The evidence on this latter point is
conflicting,24 but the cautious neurologist may wish to err on the side of simply
transferring the patient rather than giving tPA advice.
The lower accuracy of tPA decisions made via telephone highlights the need
for the neurologist to consider not only the facts of the case, as always, but also
to assess the reliability of the facts provided by the referring physician. In Case 1,
the referring doctor is an emergency physician, who has likely managed acute
stroke patients before. The reported history and examination are very clear, and
the scan has been read by a radiologist. In this case, the reported facts sound
Continuum (Minneap Minn) 2014;20(2):429–435 www.ContinuumJournal.com 431
Copyright © American Academy of Neurology. Unauthorized reproduction of this article is prohibited.
Telephone Consultations in Acute Stroke

reliable, and the neurologist can make treatment recommendations with relative
confidence. In Case 2, the referring doctor may not be as familiar with acute stroke
diagnosis and management. The precise time that the patient was last known to be
well is unclearVfurther detailed questioning will be required to ascertain whether
the patient may still be in the tPA window. The prior stroke history confounds the
examination, which is reported to have both focal and diffuse features. The scan
has not been read by a radiologist and is presumably not available to the
neurologist. In this case, the diagnosis may indeed end up being acute ischemic
stroke, but the neurologist cannot make the diagnosis or recommend tPA with
confidence and may wish to confine his consultation to the discussion of transfer.
Even when the clinical decision to recommend tPA is straightforward, it is
difficult for a neurologist to ensure, via telephone, adherence to a tPA mixing,
administration, and treatment-monitoring protocol. The author of this article
has accepted in transfer two patients who were administered tenecteplase
instead of alteplase, one in whom a second alteplase infusion was prepared (but
fortunately not administered) after the first one had ended, and several for
whom the bolus or infusion dosing were incorrect. These errors are easier to
correct when the neurologist is at the bedside, either in person or via
videoconference. They are also less likely to occur in the context of a formal
interfacility transfer agreement with accompanying treatment protocols, which
is an argument for the development of such agreements within stroke systems
of care instead of relying on ad hoc consultation via telephone.
Another consideration is the informed consent process. If the patient has
decisional capacity or a surrogate decision maker is available, informed consent
should be obtained prior to treatment with tPA. This is not to imply that a
signed consent form is requiredVonly that a discussion should take place if
possible.25 Even in person, the discussion of tPA risks and benefits can be
challengingVthe patient will usually have suffered a potentially catastrophic
medical emergency, the emotional and time pressures of which can encumber
the patient’s and family’s comprehension and judgment. Nonetheless, patients
expect that they or their surrogates will be afforded the opportunity to
participate in their medical decision making, and experienced neurologists
excel in the art of situationally sensitive, face-to-face communication and
decision shaping required in such circumstances.26 Consultation by telephone
may severely hinder this process. The emergency medicine physician is on-site,
of course, but may not have as much expertise in managing the tPA consent
process as the neurologist. A variety of decision support tools are available to
aid bedside physicians in communicating to patients and families the risks and
benefits of tPA, but they have not been rigorously developed27 and do not
substitute for a robust face-to-face discussion. Therefore, the neurologist not
only has to judge the reliability of the clinical data provided by the referring
physician, but must also consider whether that physician is able to reliably
communicate the neurologist’s assessment of the patient’s individual risks and
benefits.
A final consideration pertains to distributive justiceVthe fair, equitable, and
appropriate distribution of resources. As noted above,4 access to neurologic
expertise varies geographically. There is also wide variability in hospital tPA use,
with many hospitals never administering the drug and a significant portion of
the US population residing in counties lacking a hospital that regularly does
432 www.ContinuumJournal.com April 2014

Copyright © American Academy of Neurology. Unauthorized reproduction of this article is prohibited.


so.28 As a result of this geographic lottery, some patients have access to
evidence-based neurologic care, including tPA treatment when appropriate,29
whereas many others are underserved. The Institute of Medicine of the National
Academy of Science has recommended the establishment of systems of care to
address disparities such as these,30 and the stroke community is responding
accordingly.2,3,9,31 As these systems become more robust over time, reliance on
stroke consultation by telephone may lessen. In the meantime, some neurologists
may choose to offer such consultations in the interest of social justice.

CONCLUSIONS
Requests for telephone consultation regarding tPA use are both clinically and
ethically challenging. Because the effect of tPA is so time sensitive, patients
stand to benefit if the drug can be given locally instead of waiting until transfer
to a stroke center. Neurologists providing such consultation can also help
address geographic disparities in access to timely stroke care. However,
telephone consultation carries risks that the reported or inferred clinical
information will be inaccurate and lead to an incorrect treatment recommen-
dation, that the consent process will be inadequate, or that thrombolytic
therapy will be executed incorrectly. In this author’s view, the balance of these
concerns does not tilt sufficiently toward individual or societal benefit to justify
an ethical obligation for the neurologist to make tPA recommendations over the
phone.
However, if the neurologist is inclined to offer such advice, the practice can
be supported on the grounds that it is beneficent and socially just. The risk of
making an incorrect treatment recommendation is moderate, but evidence
suggests, at least in cases of stroke mimics, that the consequences thereof are
not grave. It may be possible to lessen the risk of treatment error through
careful attention to the perceived reliability of the information conveyed over
the phone. In a situation like that in Case 1, where the information sounds
reliable, the neurologist may choose to make an affirmative tPA recommenda-
tion prior to transfer. In a situation like that in Case 2, where the clinical
information is ambiguous, the neurologist may instead choose to simply
facilitate the transfer of care.
In any case, having formal transfer agreements and treatment protocols in
place is preferable to emergent, ad hoc, telephone consultation. Wider
implementation of telestroke systems that incorporate two-way audio and
video communication, electronic image transmission, and written documenta-
tion offers the promise of improving access to high-quality acute stroke care.

REFERENCES
1. Jauch EC, Saver JL, Adams HP Jr, et al. Guidelines for the early management of patients with
acute ischemic stroke: a guideline for healthcare professionals from the American Heart
Association/American Stroke Association. Stroke 2013;44(3):870Y947.
2. Schwamm LH, Pancioli A, Acker JE 3rd, et al. Recommendations for the establishment of stroke
systems of care: recommendations from the American Stroke Association’s Task Force on the
Development of Stroke Systems. Stroke 2005;36(3):690Y703.
3. Higashida R, Alberts MJ, Alexander DN, et al. Interactions within stroke systems of care: a policy
statement from the American Heart Association/American Stroke Association. Stroke
2013;44(10):2961Y2984.
4. Freeman WD, Vatz KA. The future of neurology. Neurol Clin 2010;28(2):537Y561.

Continuum (Minneap Minn) 2014;20(2):429–435 www.ContinuumJournal.com 433


Copyright © American Academy of Neurology. Unauthorized reproduction of this article is prohibited.
Telephone Consultations in Acute Stroke

5. Schwamm LH, Holloway RG, Amarenco P, et al. A review of the evidence for the use of
telemedicine within stroke systems of care: a scientific statement from the American Heart
Association/American Stroke Association. Stroke 2009;40(7):2616Y2634.
6. Meyer BC, Raman R, Hemmen T, et al. Efficacy of site-independent telemedicine in the STRokE
DOC trial: a randomised, blinded, prospective study. Lancet Neurol 2008;7(9):787Y795.
7. Frey JL, Jahnke HK, Goslar PW, et al. tPA by telephone: extending the benefits of a
comprehensive stroke center. Neurology 2005;64(1):154Y156.
8. Uchino K, Massaro L, Jovin TG, et al. Protocol adherence and safety of intravenous thrombolysis
after telephone consultation with a stroke center. J Stroke Cerebrovasc Dis 2010;19(6):417Y423.
9. Alberts MJ, Wechsler LR, Jensen ME, et al. Formation and function of acute stroke-ready
hospitals within a stroke system of care. Recommendations from the Brain Attack Coalition.
Stroke 2013;44(12):3382Y3393.
10. Examination and treatment for emergency medical conditions and women in labor. 42 USC § 1395dd(g),
et seq. uscode.house.gov/view.xhtml?req=(title:42%20section:1395dd%20edition:prelim)
#miscellaneous-note. Accessed December 3, 2013.
11. Aita MC, Nguyen K, Bacon R, Capuzzi KM. Obstacles and solutions in the implementation of
telestroke: billing, licensing, and legislation. Stroke 2013;44(12):3602Y3606.
12. Clark PA, Capuzzi K, Harrison J. Telemedicine: medical, legal and ethical perspectives. Med Sci
Monit 2010;16(12):RA261YRA2672.
13. Beauchamp TL, Childress JF. Principles of biomedical ethics. 5th ed. New York: Oxford University
Press; 2001:166.
14. Bobrow BJ, Demaerschalk BM, Wood JP, et al. Views of emergency physicians on thrombolysis
for acute ischemic stroke. J Brain Dis 2009;1:29Y37.
15. Scott PA, Xu Z, Meurer WJ, et al. Attitudes and beliefs of Michigan emergency physicians toward
tissue plasminogen activator use in stroke: baseline survey results from the INcreasing Stroke
Treatment through INteractive behavioral Change Tactic (INSTINCT) trial hospitals. Stroke
2010;41(9):2026Y2032.
16. AAEM Work Group on Thrombolytic Therapy in Stroke. Position statement of the American
Academy Of Emergency Medicine on the use of intravenous thrombolytic therapy in the
treatment of stroke. 2002. www.aaem.org/em-resources/position-statements/2002/thrombolytic-
therapy. Accessed August 29, 2013.
17. Canadian Association of Emergency Physicians Committee on Thrombolytic Therapy for Acute
Ischemic Stroke. Thrombolytic therapy for acute ischemic stroke. CJEM 2001;3(1):8Y12.
18. Stroke Unit Trialists’ Collaboration. Organised inpatient (stroke unit) care for stroke. Cochrane
Database Syst Rev 2013;(9):CD000197.
19. Hacke W, Donnan G, Fieschi C, et al; ATLANTIS Trials Investigators; ECASS Trials Investigators;
NINDS rt-PA Study Group Investigators. Association of outcome with early stroke treatment:
pooled analysis of ATLANTIS, ECASS, and NINDS rt-PA stroke trials. Lancet 2004;363(9411):
768Y774.
20. Zinkstok SM, Engelter ST, Gensicke H, et al. Safety of thrombolysis in stroke mimics: results from
a multicenter cohort study. Stroke 2013;44(4):1080Y1084.
21. Chen Y, Bogosavljevic V, Leys D, et al. Intravenous thrombolytic therapy in patients with stroke
mimics: baseline characteristics and safety profile. Eur J Neurol 2011;18(10):1246Y1250.
22. Buchan AM, Barber PA, Newcommon N, et al. Effectiveness of t-PA in acute ischemic stroke:
outcome relates to appropriateness. Neurology 2000;54(3):679Y684.
23. Lopez-Yunez AM, Bruno A, Williams LS, et al. Protocol violations in community-based rTPA
stroke treatment are associated with symptomatic intracerebral hemorrhage. Stroke
2001;32(1):12Y16.
24. Lyerly MJ, Albright KC, Boehme AK, et al. Safety of protocol violations in acute stroke tPA
administration. J Stroke Cerebrovasc Dis 2013. [Epub ahead of print].
25. American Academy of Neurology policy on consent issues for the administration of IV tPa.
www.aan.com/uploadedFiles/Website_Library_Assets/Documents/6.Public_Policy/1.Stay_Informed/
2.Position_Statements/3.PDFs_of_all_Position_Statements/IV.pdf. Accessed February 28, 2014.

434 www.ContinuumJournal.com April 2014

Copyright © American Academy of Neurology. Unauthorized reproduction of this article is prohibited.


26. Murtagh MJ, Burges Watson DL, Jenkings KN, et al. Situationally-sensitive knowledge translation
and relational decision making in hyperacute stroke: a qualitative study. PLoS One
2012;7(6):e37066.
27. Flynn D, Ford GA, Stobbart L, et al. A review of decision support, risk communication and patient
information tools for thrombolytic treatment in acute stroke: lessons for tool developers. BMC
Health Serv Res 2013;13:225.
28. Kleindorfer D, Xu Y, Moomaw CJ, et al. US geographic distribution of rt-PA utilization by
hospital for acute ischemic stroke. Stroke 2009;40(11):3580Y3584.
29. Mullen MT, Kasner SE, Kallan MJ, et al. Joint commission primary stroke centers utilize more
rt-PA in the nationwide inpatient sample. J Am Heart Assoc 2013;2(2):e000071.
30. Institute of Medicine. Crossing the quality chasm: a new health system for the 21st century.
Washington, DC: National Academies Press, 2001.
31. Schwamm LH, Audebert HJ, Amarenco P, et al. Recommendations for the implementation of
telemedicine within stroke systems of care: a policy statement from the American Heart
Association. Stroke 2009;40(7):2635Y2660.

Continuum (Minneap Minn) 2014;20(2):429–435 www.ContinuumJournal.com 435


Copyright © American Academy of Neurology. Unauthorized reproduction of this article is prohibited.
Practice

Cost and Utility in the


Address correspondence to
Dr James F. Burke, RWJ
Foundation Clinical Scholars
Program, 2800 Plymouth Rd,
NCRC B10-G016, Ann Arbor,
MI 48109-2800,
jamesbur@med.umich.edu.
Diagnostic Evaluation
Relationship Disclosure:
Dr Burke has reviewed case
materials for medical
of Stroke
malpractice defense cases and
receives funding from the James F. Burke, MD
National Institute of
Neurological Disorders and
Stroke.
Unlabeled Use of ABSTRACT
Products/Investigational
Use Disclosure:
The diagnostic evaluation in a patient presenting with acute stroke has several
Dr Burke reports no disclosure. purposes depending on the clinical circumstances. These include identifying stroke
* 2014, American Academy mimics, differentiating ischemic stroke from intracerebral hemorrhage in the acute
of Neurology. setting, clarifying stroke localization, and determining the stroke mechanism to
guide secondary prevention. The neurologist needs to be aware of the cost
implications of different approaches to the diagnostic evaluation.

Continuum (Minneap Minn) 2014;20(2):436–440.

Case
An 85-year-old woman living alone had a history of atrial fibrillation,
hypertension, and hyperlipidemia. She was receiving aspirin but not an
anticoagulant. She presented to the emergency department after her
family found her with impaired speech and right arm and leg weakness.
She was last seen normal 2 days earlier. Examination showed a mild
expressive aphasia, a right upper motor neuron pattern facial weakness,
and a right hemiparesis affecting her arm more than her leg. CT of the
head revealed a wedge-shaped hypodensity in the territory of an anterior
branch of the left middle cerebral artery.

Hypothetical Evaluation 1
MRI of the brain with and without contrast and magnetic resonance
angiography (MRA) of the head and neck with contrast demonstrated a
wedge-shaped area of restricted diffusion in the left middle cerebral
artery territory and less than 50% bilateral stenosis of the extracranial
internal carotid arteries immediately distal to the bifurcations. A
transthoracic echocardiogram revealed a possible left atrial appendage
thrombus, a normal ejection fraction, and mild left atrial enlargement. A
transesophageal echocardiogram was performed to evaluate the possible
thrombus and was normal.

Hypothetical Evaluation 2
Duplex Doppler ultrasound showed less than 50% extracranial internal
carotid artery stenosis bilaterally.

436 www.ContinuumJournal.com April 2014

Copyright © American Academy of Neurology. Unauthorized reproduction of this article is prohibited.


DISCUSSION
Clinical Background
Brain CT or MRI is necessary in all patients with an acute focal neurologic deficit
to help exclude stroke mimics. Neuroimaging is also required to differentiate
ischemic stroke and intracerebral hemorrhage because clinical evaluation alone
is inadequate.1 Either study can effectively identify brain hemorrhage.2,3 MRI,
when obtained within the first 12 hours after stroke onset, is clearly and
substantially more sensitive than CT for the detection of acute ischemic injury.4
It is likely the case that MRI’s sensitivity advantage diminishes as time from
stroke elapses because of increases in the sensitivity of CT.5
Clinical practice guidelines for the management of patients with acute stroke
provide recommendations for neuroimaging that focus on selecting candidates
for thrombolysis.6 MRIs, however, are generally obtained beyond 12 hours from
onset,7 as MRI is not immediately available in most emergency departments,
and approximately 25% of stroke patients present outside of the established
time frame for thrombolytic treatment.8 Guidelines for acute stroke evaluation
and management do not specifically address the optimal strategy for evaluating
the brain parenchyma in stroke patients presenting more than 12 hours after
symptom onset. Guidelines from the American Heart Association recommend
either MRI or CT but do not explicitly address what to do for patients who are
not considered candidates for acute therapy.9,10
American Academy of Neurology guidelines support the superior diagnostic
sensitivity of MRI within 12 hours but also do not address the optimal approach
beyond 12 hours.11 The European Stroke Organization states that either CT or
MRI is an acceptable initial neuroimaging technique regardless of the time of
presentation.12 Guidelines do not strongly argue for a specific approach to
carotid imaging. Although routine noninvasive imaging of the cervical arteries is
universally recommended, guidelines either do not specify a preferred
modality9,12 or offer an expert opinionYbased preference for duplex ultraso-
nography as the initial imaging modality.13
Guidelines do not offer specific recommendations on whether echocardiog-
raphy should be performed when atrial fibrillation is known to be present;
however, a decision analysis has argued against echocardiography in the context
of known atrial fibrillation.14 Guidelines do suggest that treatment with oral
anticoagulation is appropriate given the patient’s atrial fibrillation.2,3,12,15

Policy Background
Health spending per capita is considerably higher in the United States than
anywhere else in the world,1,16 and the use of new technology is the primary
driver of spending growth.2,3,17 Of most relevance to stroke evaluation, the
United States is second only to Japan in the number of MRI and CT scanners per
capita compared to the rest of the world.4,18 Although a high aggregate rate of
spending per capita may reflect a rational societal prioritization of health care
over other potential expenditures, analyses of regional variations in health care
expenditures suggest that as much as 30% of overall health care spending in the
United States may be unnecessary.5,19 Such variation exists in stroke neuroim-
aging; in 2008, 55% of stroke patients received an MRI in Oregon versus 79% in
Arizona. With this variation, a significant degree of inefficiency is likely to

Continuum (Minneap Minn) 2014;20(2):436–440 www.ContinuumJournal.com 437


Copyright © American Academy of Neurology. Unauthorized reproduction of this article is prohibited.
Cost and Utility in Diagnostic Evaluation

occurV92% of patients who receive MRI also receive CT imaging. As MRI use
has increased, no reduction has occurred in the use of CT. Largely as a
consequence of MRI dissemination, neuroimaging was the fastest growing cost
component of inpatient stroke care from 1999 to 2007, growing by 213% after
adjusting for inflation.6,20

Costs of Diagnostic Evaluation in Stroke


Costs, as opposed to payments and charges, refer to the actual economic
valuation (ie, labor and capital) required to provide a service. Although
estimating costs is not trivial, Medicare reimbursement is a commonly used
proxy. Medicare reimburses $582 for MRI with and without contrast, $953 for
head/neck MRA with contrast, $190 for chest wall echocardiography, $285 for
transesophageal echocardiography, and $244 for carotid Doppler studies.7,21
The aggregate cost of the postYemergency department diagnostic evaluation in
Hypothetical Evaluation 1 is $2000 compared to $244 for evaluation in
Hypothetical Evaluation 2. Even though the cost of Hypothetical Evaluation 1
is relatively modest, it represents a substantial proportion of the usual hospital
reimbursement for a typical stroke admissionVabout $5000 for the care of a
patient with an uncomplicated ischemic stroke. The aggregate societal costs of
these differences can be substantial. If all of the patients with ischemic stroke
with atrial fibrillation in the United States in a given year (approximately 20% of
795,0008,22) receive Hypothetical Evaluation 1 instead of Hypothetical
Evaluation 2, the aggregate societal cost would be roughly $300 million
per year. Given that additional costly tests can be obtained as part of the
stroke evaluation, the aggregate costs of the diagnostic evaluation can be
even higher.23

Strategies for Minimizing Diagnostic Evaluation Costs


Thoughtful consideration of the pretest probability, clinical context, and
probability that a test will change management should help clinicians choose
a cost-effective diagnostic strategy for each patient. Considering pretest
probability, the primary advantage of MRI over CT in the diagnosis of stroke
is its increased sensitivity.4,9 When the pretest probability of stroke is low, MRI’s
superior sensitivity means that a negative test will reduce the posttest
probability of stroke considerably more than a negative CT. For example, if
the pretest probability of stroke is 20% (odds 1:4), the posttest probability of
stroke after a negative MRI is about 2% versus about 12% with a negative CT.
Conversely, because the specificity of CT is comparable to that of MRI, a
positive CT in the context of a high pretest probability is highly valuable. In this
case, in which the pretest probability of stroke is relatively high (ie, 80%), the
posttest probability of stroke is 99.7% with MRI and similar with CT.11,24
Therefore, in a clinical context in which a positive CT has increased an already
high pretest probability, there is little marginal value for MRI to increase
diagnostic certainty. The clinical context should also be carefully considered.
While often challenging in a protocol-driven world, minimizing reflexive test
ordering may reduce unnecessary costs. For example, obtaining a
thrombophilia evaluation in an older patient with no history of venous
thromboembolic events is generally not useful.25

438 www.ContinuumJournal.com April 2014

Copyright © American Academy of Neurology. Unauthorized reproduction of this article is prohibited.


Finally, consider how test findings might alter management and how
probable those findings are compared to the possibility of false-positives. In
this case, the patient had known atrial fibrillation, a presumptive stroke
mechanism, and a strong indication for oral anticoagulation. What is the
probability that an echocardiogram will lead to a management strategy other
than oral anticoagulation? A test is of low clinical utility independent of cost
when the probability of changing management is low and the possibility of false-
positive is nontrivial.9,12,26 In this case, both echocardiograms were low-value
tests. It was unlikely that any finding would alter the decision to recommend
oral anticoagulation. The transesophageal echocardiogram was particularly
questionable given the possibility of harm associated with the test.

SUMMARY
Patients who have had a stroke routinely undergo a costly diagnostic evaluation.
Considerable variation in this evaluation exists across the country with
important societal cost implications. By carefully reflecting on the pretest
probability, clinical context, and likelihood of altering management before
ordering a test, aggregate costs of the diagnostic evaluation can likely be
substantially reduced. In doing so, it is important for the neurologist to
document the reasons for choosing a particular diagnostic strategy in the
patient’s medical record.

REFERENCES
1. Goldstein LB, Simel DL. Is this patient having a stroke? JAMA 2005;293(19):2391Y2402.
2. Kidwell CS, Chalela JA, Saver JL, et al. Comparison of MRI and CT for detection of acute
intracerebral hemorrhage. JAMA 2004;292(15):1823Y1830.
3. Chalela JA, Kidwell CS, Nentwich LM, et al. Magnetic resonance imaging and computed
tomography in emergency assessment of patients with suspected acute stroke: a prospective
comparison. Lancet 2007;369(9558):293Y298.
4. Brazzelli M, Sandercock PA, Chappell FM, et al. Magnetic resonance imaging versus computed
tomography for detection of acute vascular lesions in patients presenting with stroke symptoms.
Cochrane Database Syst Rev 2009;(4):CD007424.
5. Mullins ME, Schaefer PW, Sorensen AG, et al. CT and conventional and diffusion-weighted MR
imaging in acute stroke: study in 691 patients at presentation to the emergency department.
Radiology 2002;224(2):353Y360.
6. Latchaw RE, Alberts MJ, Lev MH, et al. Recommendations for imaging of acute ischemic stroke: a
scientific statement from the American Heart Association. Stroke 2009;40(11):3646Y3678.
7. Burke JF, Sussman JB, Morgenstern LB, Kerber KA. Time to stroke magnetic resonance imaging.
J Stroke Cerebrovasc Dis 2013;22(6):784Y791.
8. Reeves MJ, Arora S, Broderick JP, et al. Acute stroke care in the US: results from 4 pilot
prototypes of the Paul Coverdell National Acute Stroke Registry. Stroke 2005;36(6):1232Y1240.
9. Jauch EC, Saver JL, Adams HP, et al. Guidelines for the early management of patients with acute
ischemic stroke: a guideline for healthcare professionals from the American Heart Association/
American Stroke Association. Stroke 2013;44(3):870Y947.
10. Culebras A, Kase CS, Masdeu JC, et al. Practice guidelines for the use of imaging in transient
ischemic attacks and acute stroke. A report of the Stroke Council, American Heart Association.
Stroke 1997;28(7):1480Y1497
11. Schellinger PD, Bryan RN, Caplan LR, et al. Evidence-based guideline: the role of diffusion and
perfusion MRI for the diagnosis of acute ischemic stroke: report of the Therapeutics and
Technology Assessment Subcommittee of the American Academy of Neurology. Neurology
2010;75(2):177Y185.

Continuum (Minneap Minn) 2014;20(2):436–440 www.ContinuumJournal.com 439


Copyright © American Academy of Neurology. Unauthorized reproduction of this article is prohibited.
Cost and Utility in Diagnostic Evaluation

12. European Stroke Organisation (ESO) Executive Committee; ESO Writing Committee. Guidelines
for management of ischaemic stroke and transient ischaemic attack 2008. Cerebrovasc Dis
2008;25(5):457Y507.
13. Brott TG, Halperin JL, Abbara S, et al. 2011 ASA/ACCF/AHA/AANN/AANS/ACR/ASNR/CNS/SAIP/
SCAI/SIR/SNIS/SVM/SVS guideline on the management of patients with extracranial carotid and
vertebral artery disease. Stroke 2011;42(8):e464Ye540.
14. Meenan RT, Saha S, Chou R, et al. Effectiveness and cost-effectiveness of echocardiography and
carotid imaging in the management of stroke. Evid Rep Technol Assess (Summ) 2002;49:1Y10.
15. Furie KL, Kasner SE, Adams RJ, et al. Guidelines for the prevention of stroke in patients with
stroke or transient ischemic attack: a guideline for healthcare professionals from the American
Heart Association/American Stroke Association. Stroke 2011;42(1):227Y276.
16. Kaiser Family Foundation, editor. Snapshots: Health Care Spending in the United States &
Selected OECD Countries [Internet]. Kaiser Family Foundation. http://kff.org/health-costs/issue-
brief/snapshots-health-care-spending-in-the-united-states-selected-oecd-countries/. Accessed
December 10, 2013.
17. Orszag P. Growth in Health Care Costs [Internet]. Office CB, editor. CBO.gov. Washington, DC;
2008. http://www.cbo.gov/ftpdocs/89xx/doc8948/01-31-HealthTestimony.pdf. Accessed December
10, 2013.
18. Organization for Economic Cooperation and Development, editor. OECD Indicators [Internet].
http://www.oecd-ilibrary.org/sites/health_glance-2011-en/04/02/index.html?contentType=/ns/
StatisticalPublication,/ns/Chapter&containerItemId=/content/serial/19991312&itemId=/content/
chapter/health_glance-2011-30-en&mimeType=text/html. Accessed December 10, 2013.
19. Fisher E, Bynum J, Skinner J. Slowing the growth of health care costsVlessons from regional
variation. N Engl J Med 2009;360(9):849Y852.
20. Burke JF, Kerber KA, Iwashyna TJ, Morgenstern LB. Wide variation and rising utilization of stroke
magnetic resonance imaging: data from 11 states. Ann Neurol 2012;71(2):179Y185.
21. Center for Medicare and Medicaid Services, editor. Physician Fee Schedule Search [Internet].
http://www.cms.gov/apps/physician-fee-schedule/search/search-criteria.aspx. Accessed December
10, 2013.
22. Go AS, Mozaffarian D, Roger VL, et al. Heart disease and stroke statisticsV2013 update: a report
from the American Heart Association. Circulation 2013;127(1):e6Ye245.
23. Ning M, Gonzalez RG. Case records of the Massachusetts General Hospital. Case 34-2013. A
69-year-old man with dizziness and vomiting. N Engl J Med 2013;369(18):1736Y1748.
24. Haynes R, Sackett D. The architecture of diagnostic research. BMJ 2002;324(7336):539Y541.
25. Bushnell CD, Goldstein LB. Screening for hypercoagulable syndromes following stroke. Curr
Atheroscler Rep 2003;5(4):291Y298
26. Kaymaz C, Ozdemir N, Kirma C, et al. Location, size and morphological characteristics of left
atrial thrombi as assessed by echocardiography in patients with rheumatic mitral valve disease.
Eur J Echocardiogr 2001;2(4):270Y276.

440 www.ContinuumJournal.com April 2014

Copyright © American Academy of Neurology. Unauthorized reproduction of this article is prohibited.


Practice

Coding for Telestroke


Address correspondence to
Dr Timothy Ingall, Mayo Clinic
Hospital, 5E, 5777 East Mayo
Boulevard, Phoenix, AZ 85054,
Timothy J. Ingall, MB BS, PhD; Bart M. Demaerschalk, MD, MSc, FRCP(C) tingall@mayo.edu.
Relationship Disclosure:
Drs Ingall and Demaerschalk
report no disclosures.
Unlabeled Use of
Accurate coding is an important function of neurologic practice. This contribution Products/Investigational
to is part of an ongoing series that presents helpful coding Use Disclosure:
Drs Ingall and Demaerschalk
information along with examples related to the issue topic. Tips for diagnosis coding, report no disclosures.
Evaluation and Management coding, procedure coding, or a combination are * 2014, American Academy
presented, depending on which is most useful for the subject area of the issue. of Neurology.

TELESTROKE OVERVIEW
For many years, physicians caring for acute stroke patients have called specialist
physicians in larger hospitals to obtain advice regarding patient management.
These ‘‘curbside consults’’ have been a mainstay of patient care for decades, but
physicians providing advice have received no reimbursement. Although
telemedicine became a tool for providing care remotely over 20 years ago, it
was only after 2000 that improvements in both camera technology and the
infrastructure of the Internet allowed physicians to effectively evaluate patients
in real time in remote locations. The past decade has seen an explosion in the
use of video and audio functionality provided over Internet connections to
allow neurologists to perform virtual face-to-face consultations with acute stroke
patients at remote locations and provide management recommendations to the
remote staff involved with the care of the patient. While telestroke services have
expanded dramatically, changes to how physicians and hospitals are reimbursed
have lagged behind.

CODING, BILLING, AND REIMBURSEMENT CONSIDERATIONS


OF TELESTROKE
In spite of a robust and growing evidence base supporting the use of
telemedicine in general and telestroke in particular, a host of considerations
remain incompletely resolved and thus constitute barriers to more widespread
implementation. Among them are disparate licensing, credentialing, and
privileging requirements between each state and the nation. Furthermore,
current means of coding telemedical care and arbitrary restrictions on eligibility
for billing and reimbursement serve as financial disincentives to establish a
telestroke network.

Licensure
The essence of telemedicine is to disseminate medical expertise to patients and
local providers regardless of geographic boundaries. Currently, medical
licensure and hospital credentialing processes run counter to that principle, as
they are predicated almost entirely on geography. In the United States, medical
licensure is under the purview of individual states. Furthermore, in most states,
a physician must be licensed in the state where a patient seeks care. Therefore,
a telemedicine physician must undergo the rigorous licensure process in nearly

Continuum (Minneap Minn) 2014;20(2):441–443 www.ContinuumJournal.com 441


Copyright © American Academy of Neurology. Unauthorized reproduction of this article is prohibited.
Coding for Telestroke

each and every state and US territory in order to provide telemedical care to
patients in those locations. The exceptions, which have a mechanism to grant a
telemedicine license for practitioners licensed in another state, include Alabama
(ALA.CODE § 34-24-502), Louisiana (LA.REV. STAT.ANN. §1276.1), Minnesota
(MINN. STAT. § 147.032[1]), Montana (MONT.ADMIN.R. 24.156.802[5]), Nevada
(NRS § 630.261[e]), New Mexico (NM STAT.ANN. 1978 § 61-6-6), Ohio (OH.
REV.CODE ANN. § 4731.296[C]), Oregon (OR.REV. STAT.ANN. § 677.139),
Tennessee (TCA § 63-6-209[b]), Texas (22 TEX.ADMIN.CODE § 174.12) and
Guam (10 G.C.A. § 12202). The Federation of State Medical Boards proposed
the Model Act in 1995, which would afford a licensed physician in any state the
privilege to practice telemedicine across state lines, limiting in-person medical
care to the primary state of licensure. This act has not been formally accepted
by any state to date, although the aforementioned states that grant telemedicine
licensure based on a medical license in good standing elsewhere in the United
States have enacted its basic tenet. A recent piece of federal legislation (42 CFR
§§ 482.12 and 482.22) helped to streamline the process of being credentialed
for a telemedicine site by allowing the credentialing process of the hub site to
effectively transfer, so as to better avoid onerous, duplicative administrative
barriers.

Reimbursement
Reimbursement mechanisms for telemedical care have not kept pace with the
expanded clinical use of telemedicine. The Centers for Medicare & Medicaid
Services (CMS), the most prominent payer in the US healthcare system,
requires that concurrent care by more than one provider be medically necessary
(42 USC § 1395y[a]) and that the consultation originate within arbitrary
geographic constraints designated as ‘‘rural’’ for reimbursement of service.
Although these stipulations are ostensibly reasonable, in practice only very few
payments are provided for telemedical care of Medicare beneficiaries. In
addition, the current federal definition of ‘‘rural’’ does not encompass all
underserved populations; thus, a provider is given a financial disincentive to
practice telemedicine in other, nonrural, underserved areas. Nineteen states
have enacted provisions that compel private insurers to cover a telemedical
consultation, but the lack of a clear federal standard (Medicare payments are
considered a benchmark for most medical services) leads to general ambiva-
lence regarding how telemedical services should be reimbursed, which may
impede investment of resources by physicians and industry. The lack of
collaboration between government authorities, both state and federal, and
third-party payers is a major hurdle to the implementation of appropriate
reimbursement for telemedical services to physicians and hospitals. Payment to
hospitals for acute stroke patients treated with IV thrombolysis at one hospital
and then transferred immediately to a stroke center for further care (‘‘drip and
ship’’) has been acknowledged by CMS as an issue that may warrant
modification to the reimbursement system. In order to collect data that could
help determine whether a change in reimbursement is necessary, CMS created
a diagnostic code, V45.88, in 2008 that can be used by the receiving hospital to
indicate that they treated a patient who had tPA administered in a different
facility within the past 24 hours before admission to the receiving hospital.

442 www.ContinuumJournal.com April 2014

Copyright © American Academy of Neurology. Unauthorized reproduction of this article is prohibited.


Case
A 73-year-old man living in a rural area was witnessed by his family to have
the sudden onset of dysarthria and left-sided weakness, which persisted.
The patient was transported by emergency medical services to his local
community hospital within 60 minutes of the onset of symptoms, where
he was evaluated by an emergency department (ED) physician. The
hospital had a telestroke service agreement with a tertiary care hospital;
the ED physician initiated the system, and a telestroke consultation was provided
by a vascular neurologist at the tertiary care hospital. A video-audio link
was established over the Internet, and a virtual face-to-face consult was performed
by the neurologist with the assistance of an ED nurse who obtained the history
and performed the NIH Stroke Scale examination under the supervision of the
neurologist. After performing a 30-minute evaluation, the vascular
neurologist conferred with the ED physician, and they determined that the
patient was a candidate for IV thrombolytic treatment, which was initiated in
the ED. The patient was then transferred to the tertiary care hospital for
ongoing care. The neurologist dictated a full consultation note, which was
available in the community hospital’s medical record system.

DISCUSSION
Coding
CMS has written a guideline that provides information for health care
professionals providing telehealth services.1 Previous articles
have addressed coding issues related to stroke prevention2 and critical care
coding for stroke patients.3
The receiving hospital diagnosis coding should include the ‘‘drip-and-ship’’
International Classification of Diseases, Ninth Revision, Clinical Modification
(ICD-9-CM) code V45.884 (after October 1, 2014, the International Classifica-
tion of Diseases, Tenth Revision, Clinical Modification (ICD-10-CM) code is
Z92.825). For reimbursement, the health care provider has a contract with
either the hospital or the entity providing the telemedicine service and is
reimbursed per the specifics of the contract.

REFERENCES
1. Department of Health and Human ServicesVCenters for Medicare & Medicaid Services. Rural
Health Fact Sheet Series: Telehealth Services (ICN 901705). www.cms.gov/Outreach-and-Education/
Medicare-Learning-Network-MLN/MLNProducts/downloads/TelehealthSrvcsfctsht.pdf. ICN 901705.
Published December 2012. Accessed December 26, 2013.
2. Powers LB. Coding issues: coding for stroke prevention. Continuum (Minneap Minn) 2011;17(6
2ndary Stroke Prevention):1344Y1348.
3. Levine SR, ed. Appendix: stroke coding guide for critical care coding. Continuum (Minneap Minn)
2008;14(6):133Y136.
4. 2014 ICD-9-CM Diagnosis Code V45.88 (Final Revision). ICD9Data.com. www.icd9data.com/2014/
Volume1/V01-V91/V40-V49/V45/V45.88.htm. Accessed December 26, 2013.
5. 2014 ICD-10-CM Diagnosis Code Z92.82. ICD10Data.com. www.icd10data.com/ICD10CM/Codes/
Z00-Z99/Z77-Z99/Z92-/Z92.82. Accessed December 26, 2013.

Continuum (Minneap Minn) 2014;20(2):441–443 www.ContinuumJournal.com 443


Copyright © American Academy of Neurology. Unauthorized reproduction of this article is prohibited.
CME

Postreading CME Test—


Preferred Responses
Following are the preferred responses for the Postreading CME Test in this
issue. The questions and answer options are repeated, and
the preferred response is given, followed by an explanation and a reference
with which you may seek more specific information. You are encouraged to
review the responses and explanations carefully to evaluate your general
understanding of the course material. The comments and references included
with each question are intended to encourage independent study.

b 1. A 2-year-old girl with known sickle cell disease undergoes screening for her
stroke risk with transcranial Doppler. Her mean flow velocities remain greater
than 200 cm/sec on repeat screening done a month after an initial abnormal
scan. She has never had a stroke. Which of the following therapies is most
appropriate for stroke prevention in this patient?
A. bone marrow transplant
B. exchange transfusion
C. phlebotomy
D. phlebotomy and hydroxyurea
E. warfarin
The correct answer is B (exchange transfusion). Children with known sickle
cell disease should be screened for increased cerebral blood flow velocities
from about 2 years of age and those with mean flow velocities greater than
200 cm/sec should be treated with exchange transfusions to maintain the
hemoglobin S fraction at less than 30%. Chelation therapy is recommended when
iron overload accompanies repeated transfusions. For more information, refer
to page 403 of the article ‘‘Genetic Stroke Syndromes.’’

b 2. Which of the following is the most appropriate timing for cerebrovascular


imaging to screen for unruptured intracranial aneurysms in asymptomatic
patients belonging to high-risk subpopulations?
A. at 5 years of age; if negative, every 5 to 10 years
B. at 5 years of age; if negative, no need to repeat
C. at 20 years of age; if negative, every 5 to 10 years
D. at 20 years of age; if negative, no need to repeat
E. never, as long as the patient remains asymptomatic
The correct answer is C (at 20 years of age; if negative, every 5 to 10 years).
Even in high-risk subpopulations, aneurysms are uncommon in patients younger
than 20 years. Because aneurysms slowly enlarge over time, a negative imaging
test does not exclude the possibility of subsequent aneurysm formation. Therefore,
a reasonable strategy in high-risk subpopulations is to perform initial noninvasive
imaging at age 20 years and repeat imaging every 5 to 10 years thereafter.
For more information, refer to pages 392 and 393 of the article
‘‘Unruptured Intracranial Aneurysms: Screening and Management.’’

Continuum (Minneap Minn) 2014;20(2):459–475 www.ContinuumJournal.com 459


Copyright © American Academy of Neurology. Unauthorized reproduction of this article is prohibited.
Postreading CME Test—Preferred Responses

b 3. Which of the following clinical factors is weighted most heavily when


calculating the ABCD2 scale?
A. age greater than or equal to 60 years
B. blood pressure greater than or equal to 140/90 mm Hg
C. diabetes mellitus
D. speech/language impairment without weakness
E. unilateral weakness
The correct answer is E (unilateral weakness). The ABCD2 scale is widely
employed to predict stroke risk after TIA. The ABCD2 score is calculated
as follows: age Q60 years (1 point); blood pressure Q140/90 mm Hg
(1 point); clinical features: unilateral weakness (2 points) or speech/language
impairment without weakness (1 point); duration Q60 minutes (2 points)
or 10 to 59 minutes (1 point); and diabetes mellitus (1 point). The ABCD2 score
can vary from 0 to 7, and increasing scores predict increased risk of stroke.
For more information, refer to pages 422 and 423 of the article
‘‘Predicting Outcomes After Transient Ischemic Attack and Stroke.’’

b 4. In a 27-year-old woman with a right middle cerebral artery stroke, which of


the following features would provide the most compelling reason to test for the
factor V Leiden mutation?
A. fibromuscular dysplasia
B. history of five prior pregnancies, all carried to term
C. history of recent chiropractic manipulation
D. nonwhite ethnic background
E. right-to-left cardiac shunt
The correct answer is E (right-to-left cardiac shunt). The factor V Leiden
mutation is the most common genetic risk factor for thrombophilia, and it is
associated with venous thrombosis, especially in homozygotes. It has not been
convincingly linked to arterial thrombosis, however, so patients with arterial
stroke usually warrant testing for the factor V Leiden mutation only when they
have a right-to-left shunt. A history of recent chiropractic manipulation or
fibromuscular dysplasia would increase the likelihood of arterial dissection as a
cause of stroke, so these conditions would make the factor V Leiden mutation less
relevant. Testing for the factor V Leiden mutation is of lower yield in nonwhite
patients than in white patients because of the difference in prevalence of the
mutation in the two populations. For more information, refer to pages 359 and 360 of
the article ‘‘Evaluation and Management of Stroke in Young Adults.’’

460 www.ContinuumJournal.com April 2014

Copyright © American Academy of Neurology. Unauthorized reproduction of this article is prohibited.


b 5. Which of the following statements best summarizes the available evidence
regarding the relative efficacy of endovascular and surgical management of
unruptured intracranial aneurysms?
A. endovascular management is equivalent to surgical management, based
on large randomized controlled trials
B. endovascular management is inferior to surgical management, based on
large randomized controlled trials
C. endovascular management is superior to surgical management, based on
large randomized controlled trials
D. large randomized controlled trials have yielded contradictory results
E. no large randomized controlled trials have been conducted
The correct answer is E (no large randomized controlled trials have been
conducted). The International Subarachnoid Aneurysm Trial (ISAT) was a
large randomized controlled trial comparing endovascular to surgical
management in patients with ruptured aneurysms. No large randomized
controlled trials have compared endovascular to surgical management
in patients with unruptured intracranial aneurysms. For more information,
refer to pages 395 and 396 of the article ‘‘Unruptured Intracranial
Aneurysms: Screening and Management.’’

b 6. A 72-year-old woman had an episode of numbness and weakness of the left


arm and leg, which resolved spontaneously within 3 minutes. Her neurologic
examination is normal in the emergency department 2 hours later. She has
a history of hypertension, well controlled on hydrochlorothiazide and lisinopril,
and her blood pressure in the emergency department is 106/68 mm Hg.
She has the following lipid levels: total cholesterol, 210 mg/dL; low-density
lipoprotein (LDL), 140 mg/dL; high-density lipoprotein (HDL), 50 mg/dL;
triglycerides, 422 mg/dL; total cholesterol/HDL ratio, 4.2. To reduce the risk of
subsequent stroke, the evidence is most compelling for which of the following
treatment targets?
A. HDL cholesterol level greater than 60 mg/dL
B. LDL cholesterol level less than 70 mg/dL
C. total cholesterol level less than 160 mg/dL
D. total cholesterol level/HDL ratio less than 3.0
E. triglyceride level less than 120 mg/dL
The correct answer is B (LDL cholesterol level less than 70 mg/dL). In the
Stroke Prevention by Aggressive Reduction in Cholesterol Levels (SPARCL)
trial, patients who achieved at least a 50% reduction in LDL cholesterol level
and patients who achieved an LDL cholesterol level less than 70 mg/dL had
the greatest reduction in stroke risk, so these are the goals recommended
in the most recent American Heart Association guidelines for secondary
stroke prevention. For more information, refer to pages 299 and 300 of the
article ‘‘Risk Factor Management for Stroke Prevention.’’

Continuum (Minneap Minn) 2014;20(2):459–475 www.ContinuumJournal.com 461


Copyright © American Academy of Neurology. Unauthorized reproduction of this article is prohibited.
Postreading CME Test—Preferred Responses

b 7. In the Carotid Revascularization Endarterectomy versus Stenting Trial


(CREST), the results were better (ie, a significantly lower rate of the primary
outcome) for patients in the stenting group than they were for patients in the
endarterectomy group when the analysis was restricted to patients belonging to
which of the following subgroups?
A. female patients
B. male patients
C. patients older than 70 years
D. patients younger than 70 years
E. patients with asymptomatic carotid stenosis
The correct answer is D (patients younger than 70 years). In CREST,
patients in the stenting group did significantly better with respect to the
primary outcome than patients in the endarterectomy group when the
analysis was restricted to patients younger than 70 years. When the analysis
was restricted to those older than 70 years, patients in the endarterectomy
group did better than patients in the stenting group. No significant difference
was evident in the primary outcome between the endarterectomy group
and the stenting group when all patients were analyzed, or when the
analysis was restricted to male patients, female patients, patients with
symptomatic stenosis, or patients with asymptomatic stenosis. For more
information, refer to page 328 of the article ‘‘Large Artery
Atherosclerosis: Carotid Stenosis, Vertebral Artery Disease, and Intracranial
Atherosclerosis.’’

b 8. An 86-year-old woman with hypertension, hyperlipidemia, and diabetes


mellitus was found to have 80% stenosis of the proximal left internal carotid
artery on a CT angiogram (ordered by her primary physician to evaluate
prominent calcification in several intracranial vessels on a head CT scan,
which had been done as part of an evaluation for mild problems with
memory). The CT scan showed no evidence of ischemia or atrophy. In
addition to aggressive management of risk factors, which of the following is
the most appropriate management of this patient?
A. antiplatelet therapy alone
B. left carotid angioplasty and stent plus antiplatelet therapy
C. left carotid angioplasty plus antiplatelet therapy but no stent
D. left carotid endarterectomy plus antiplatelet therapy
E. warfarin alone
The correct answer is A (antiplatelet therapy alone). Extremely low rates
of stroke can be achieved with aggressive medical therapy alone in patients
with asymptomatic carotid stenosis. Because of the uncertain benefit of carotid
endarterectomy in asymptomatic, elderly patients, the American Academy
of Neurology guidelines recommend that endarterectomy for asymptomatic
carotid stenosis be considered only in patients younger than 75 years.
No evidence supports the use of warfarin in patients with asymptomatic
carotid stenosis. For more information, refer to pages 326 to 328 of the
article ‘‘Large Artery Atherosclerosis: Carotid Stenosis, Vertebral
Artery Disease, and Intracranial Atherosclerosis.’’

462 www.ContinuumJournal.com April 2014

Copyright © American Academy of Neurology. Unauthorized reproduction of this article is prohibited.


b 9. An increased risk of atherosclerotic disease is associated with elevated
levels of which of the following?
A. cyanocobalamin
B. homocysteine
C. niacin
D. riboflavin
E. vitamin A
The correct answer is B (homocysteine). Hyperhomocysteinemia has
been consistently associated with atherosclerotic disease. Homocysteine
levels can be lowered by administering folic acid, pyridoxine (vitamin B6),
or cyanocobalamin (vitamin B12), but this has not been convincingly shown
to reduce stroke risk. For more information, refer to page 302 of the
article ‘‘Risk Factor Management for Stroke Prevention.’’

b 10. A 68-year-old woman is being evaluated in the emergency department for


left hemiparesis. She says that she awoke 3 hours ago, and she fell when she
tried to get out of bed. In trying to stand, she realized that she couldn’t move
her left leg. Her examination is notable for severe weakness and diminished
sensation of the left lower extremity, with mild weakness and diminished
sensation of the left upper extremity. She says that her left arm weakness has
definitely become more pronounced over the past hour. Her health was last
known to be normal 10 hours ago. Which of the following treatments should
be administered?
A. aspirin
B. intra-arterial rtPA
C. IV heparin
D. IV recombinant tissue-type plasminogen activator (rtPA)
E. warfarin
The correct answer is A (aspirin). Because she was last known normal 10 hours
ago, she is outside the time window for administration of either IV or intra-arterial
rtPA. Acute treatment with aspirin in patients with ischemic stroke reduces
the risk of recurrent stroke; short-term treatment with a combination of aspirin
and clopidogrel was more effective than aspirin alone in one recent trial,
but these results must still be confirmed. Acute anticoagulation has not been
shown to improve clinical outcomes or reduce the risk of recurrent stroke.
For more information, refer to page 293 of the article
‘‘Evaluation and Management of Acute Ischemic Stroke.’’

b 11. Which of the following therapies is recommended as the initial step in


treating cerebral venous thrombosis (CVT) in the presence of hemorrhagic
transformation of a venous infarction?
A. aspirin
B. balloon venoplasty
C. low-molecular-weight heparin
D. mechanical clot retrieval
E. tissue plasminogen activator
The correct answer is C (low-molecular-weight heparin). Despite the limited
evidence from large randomized clinical trials, initial anticoagulation (typically
heparin followed by warfarin) is the standard treatment for acute CVT, even

Continuum (Minneap Minn) 2014;20(2):459–475 www.ContinuumJournal.com 463


Copyright © American Academy of Neurology. Unauthorized reproduction of this article is prohibited.
Postreading CME Test—Preferred Responses

in the presence of intracerebral hemorrhage or hemorrhagic transformation.


Some trials have shown an advantage of low-molecular-weight heparin over
standard unfractionated heparin because of a better safety and efficacy profile.
For more information, refer to pages 341 and 342 of the article
‘‘Cerebral Venous Thrombosis.’’

b 12. A 35-year-old woman presents to the office because of severe, recurrent


migraines that are often associated with visual aura. Family history reveals an
aunt with strokes in her 50s and a sister with recently diagnosed multiple
sclerosis. MRI of the patient’s brain shows multiple areas of white matter
abnormality. White matter lesions in which of the following locations would
be most supportive of the diagnosis of cerebral autosomal dominant
arteriopathy with subcortical infarcts and leukoencephalopathy (CADASIL) in
this patient?
A. anterior temporal pole
B. centrum semiovale
C. corpus callosum
D. periventricular
E. thalamus
The correct answer is A (anterior temporal pole). Patients with cerebral
autosomal dominant arteriopathy with subcortical infarcts and leukoencephalopathy
(CADASIL) may present with typical migraine syndromes in the second decade
of life or later, often with aura. Because of the frequent finding of multiple
white matter lesions in young patients, some of which may involve the corpus
callosum, multiple sclerosis is often entertained as a potential diagnosis.
The pattern of white matter involvement in CADASIL is often distinctive,
however, with involvement of the anterior temporal poles being a classic
radiologic sign. For more information, refer to page 404 of the
article ‘‘Genetic Stroke Syndromes.’’

b 13. A 33-year-old man who has had an ischemic stroke in the territory of the
right middle cerebral artery is most likely to have which of the following risk
factors?
A. dyslipidemia
B. fibromuscular dysplasia
C. Kawasaki disease
D. prothrombin gene (G20210A) mutation
E. pulmonary arteriovenous malformation
The correct answer is A (dyslipidemia). Although most of the conditions that
predispose to atherosclerotic disease become much more prevalent as
people age, these risk factors are also important in younger stroke patients.
In one study of stroke patients who were between 15 and 49 years of age,
39% had hypertension, 60% had dyslipidemia, and 44% had a history of
cigarette smoking. The prothrombin gene mutation is a risk factor for
venous thrombosis, but it has not been demonstrated to be a risk factor
for arterial thrombosis. The other conditions listed are risk factors for
arterial stroke, but they are extremely rare. For more information, refer
to page 352 of the article ‘‘Evaluation and Management
of Stroke in Young Adults.’’

464 www.ContinuumJournal.com April 2014

Copyright © American Academy of Neurology. Unauthorized reproduction of this article is prohibited.


b 14. The US Food and Drug Administration (FDA) has approved the use of
three novel oral anticoagulants for nonvalvular atrial fibrillation: dabigatran,
rivaroxaban, and apixaban. Which of the following is a disadvantage of these
therapies compared to warfarin?
A. greater need for monitoring
B. increased incidence of intracranial hemorrhage
C. more complicated dosing regimens
D. more frequent drug-drug interactions
E. no validated reversal strategy
The correct answer is E (no validated reversal strategy). Newly approved
anticoagulants offer some advantages over warfarin, including fewer
drug-drug interactions, little monitoring, fixed dose, rapid onset of action,
a reported lower incidence of intracranial hemorrhage, and noninferior
(rivaroxaban) or superior efficacy (dabigatran and apixaban). However, some
of the disadvantages include a much higher cost and the absence of a validated
reversal strategy. For more information, refer to pages 314 and 315 of the
article ‘‘Evaluation and Prevention of Cardioembolic Stroke.’’

b 15. Which of the following symptoms and signs in late infancy or childhood is
most suggestive of a previously undiagnosed perinatal stroke?
A. dysphagia
B. early hand dominance
C. failure to thrive
D. headache
E. visual impairment
The correct answer is B (early hand dominance). Sometimes acute neonatal
arterial ischemic stroke is not diagnosed until later infancy or childhood,
when hemiparesis becomes gradually evident with maturation. In such children,
imaging shows remote arterial ischemic stroke, which is presumed to
have occurred in the prenatal or perinatal period. The typical history includes
motor delay, early hand dominance (before the first birthday), or focal seizures
evolving into a classical hemiplegic cerebral palsy. For more information, refer
to page 371 of the article ‘‘Pediatric Arterial Ischemic Stroke.’’

b 16. In patients being evaluated for cervicocephalic arterial dissection, magnetic


resonance angiography combined with axial T1-weighted cervical MRI with fat
suppression is superior to CT angiography for identifying which of the following
abnormalities?
A. high-grade stenoses
B. intimal flaps
C. pseudoaneurysms
D. small intramural hematomas
E. vertebral artery dissections
The correct answer is D (small intramural hematomas). Magnetic resonance
angiography (MRA) combined with axial T1-weighted cervical MRI with
fat suppression is superior to CT angiography (CTA) for identifying small
intramural hematomas. CTA might be superior to MRA for diagnosing
intimal flaps, pseudoaneurysms, high-grade stenoses, and vertebral artery

Continuum (Minneap Minn) 2014;20(2):459–475 www.ContinuumJournal.com 465


Copyright © American Academy of Neurology. Unauthorized reproduction of this article is prohibited.
Postreading CME Test—Preferred Responses

dissections. For more information, refer to pages 356 and 357 of the
article ‘‘Evaluation and Management of Stroke in Young Adults.’’

b 17. An 83-year-old woman in an acute rehabilitation facility 4 weeks after a


left hip replacement surgery abruptly develops aphasia and right hemiparesis
during a physical therapy session. She has a history of chronic obstructive
pulmonary disease (and had an arterial blood gas performed 1 day ago), and
had a right occipital stroke 2 months ago. A head CT scan shows the prior
stroke, but no acute changes. Which of the following would be the most
compelling contraindication to the use of IV recombinant tissue-type
plasminogen activator (rtPA) in this patient?
A. her age
B. her hip surgery 4 weeks ago
C. her stroke 2 months ago
D. history of upper gastrointestinal bleeding 6 months ago
E. radial artery puncture 24 hours ago
The correct answer is C (her stroke 2 months ago). A history of stroke or
significant head trauma within the past 3 months is a contraindication to the
use of IV rtPA. Major surgery within the past 14 days and gastrointestinal
or urinary tract hemorrhage within the past 21 days are contraindications,
but this patient’s hip surgery was 4 weeks ago and her gastrointestinal
hemorrhage was 6 months ago. Arterial puncture at a noncompressible site
within the past 7 days is another contraindication, but the radial artery is
a compressible site. The patient’s age is not a contraindication to IV rtPA.
For more information, refer to page 289 of the article
‘‘Evaluation and Management of Acute Ischemic Stroke.’’

b 18. Which of the following skeletal abnormalities is most commonly


associated with cerebral autosomal recessive arteriopathy with subcortical
infarcts and leukoencephalopathy (CARASIL)?
A. ankylosing spondylitis
B. atlantoaxial subluxation
C. osteogenesis imperfecta
D. spina bifida
E. spondylosis
The correct answer is E (spondylosis). CARASIL is associated with stroke
with onset in the third decade and dementia with onset in the third through
fifth decade. Systemic manifestations of CARASIL include premature
alopecia with onset in the teen years and spondylosis in the second and
third decades of life. For more information, refer to page 406 of the
article ‘‘Genetic Stroke Syndromes.’’

466 www.ContinuumJournal.com April 2014

Copyright © American Academy of Neurology. Unauthorized reproduction of this article is prohibited.


b 19. An 80-year-old man who had a left pontine stroke two days ago is found
to have 90% stenosis of the proximal left vertebral artery. In addition to
aggressive management of risk factors, which of the following is the most
appropriate management of this patient?
A. angioplasty and stenting of the left vertebral artery plus antiplatelet therapy
B. angioplasty and stenting of the left vertebral artery plus warfarin
C. antiplatelet therapy alone
D. transposition of the left vertebral artery to the left common carotid artery
plus antiplatelet therapy
E. warfarin therapy alone
The correct answer is C (antiplatelet therapy alone). No evidence from
prospective, controlled trials suggests that revascularization methods or
warfarin are superior to aggressive risk factor management and antiplatelet
therapy in patients with vertebral artery disease. For more information, refer to
pages 331 and 332 of the article ‘‘Large Artery Atherosclerosis:
Carotid Stenosis, Vertebral Artery Disease, and Intracranial Atherosclerosis.’’

b 20. A 4-year-old boy with normal development is seen for three paroxysmal
episodes of speech arrest and alternating left- and right-sided weakness. He
is investigated for epilepsy, with negative results. His parents say that all three
spells occurred in the setting of crying excessively. Which of the following
vascular abnormalities is most consistent with this presentation?
A. carotid dissection
B. cerebral vasculitis
C. cerebral venous thrombosis
D. moyamoya disease
E. multiple cavernous malformations
The correct answer is D (moyamoya disease). Moyamoya is a progressive
occlusion of arteries of the circle of Willis associated with the presence of
hypertrophied, friable collateral vessels in the basal ganglia region (‘‘puff
of smoke’’), detected as prominent flow voids on T1-MRI and the ‘‘ivy sign’’
on fluid-attenuated inversion recovery (FLAIR) images, which represent
prominent leptomeningeal collaterals. Moyamoya in children may be idiopathic
(moyamoya disease) or occur in the presence of conditions such as sickle
cell disease, neurofibromatosis, and other systemic medical and genetic
conditions (moyamoya syndrome). A stereotypic presentation in children
is the occurrence of transient neurologic episodes, which may be focal and
can be provoked by hyperventilation. The substrate for these episodes is
believed to be vasoconstriction in a bed of already stenosed or irregular vessels.
For more information, refer to pages 381, 382, and 384 of the
article ‘‘Pediatric Arterial Ischemic Stroke.’’

Continuum (Minneap Minn) 2014;20(2):459–475 www.ContinuumJournal.com 467


Copyright © American Academy of Neurology. Unauthorized reproduction of this article is prohibited.
Postreading CME Test—Preferred Responses

b 21. Clinical monitoring alone, without cerebrovascular imaging to evaluate for


aneurysm, would be most appropriate for a patient with which of the following?
A. autosomal dominant polycystic kidney disease
B. headache associated with transient loss of consciousness
C. intermittent unilateral headache, alternating sides
D. two first-degree relatives with unruptured intracranial aneurysms
E. unilateral oculomotor nerve palsy
The correct answer is C (intermittent unilateral headache, alternating sides).
Patients with autosomal dominant polycystic kidney disease have a
substantially higher risk of aneurysm development than the general population,
and so do patients who have two or more first-degree relatives with
intracranial aneurysms. The possibility of ruptured aneurysm must be considered
in a patient who develops a headache associated with loss of consciousness,
and aneurysms (whether ruptured or unruptured) must also be considered
in patients with an oculomotor nerve palsy. In contrast, unruptured aneurysms
are unlikely to cause headache, and ruptured aneurysms are unlikely to result in
a history of intermittent headaches on alternate sides of the head. For more
information, refer to pages 388 and 389 of the article ‘‘Unruptured
Intracranial Aneurysms: Screening and Management.’’

b 22. In the Carotid Revascularization Endarterectomy versus Stenting Trial


(CREST), patients in the endarterectomy group had which of the following
patterns of perioperative complications (relative to patients in the stenting group)?
A. fewer perioperative myocardial infarctions, fewer perioperative strokes
B. fewer perioperative myocardial infarctions, more perioperative strokes
C. more perioperative myocardial infarctions, fewer perioperative strokes
D. more perioperative myocardial infarctions, more perioperative strokes
E. the same rate of both perioperative myocardial infarctions and perioperative
strokes
The correct answer is C (more perioperative myocardial infarctions, fewer
perioperative strokes). In CREST, patients in the endarterectomy group
had more perioperative myocardial infarctions but fewer perioperative
strokes (and, in fact, fewer subsequent strokes) than patients in the
stenting group. For more information, refer to page 324 of the
article ‘‘Large Artery Atherosclerosis: Carotid Stenosis, Vertebral Artery
Disease, and Intracranial Atherosclerosis.’’

468 www.ContinuumJournal.com April 2014

Copyright © American Academy of Neurology. Unauthorized reproduction of this article is prohibited.


b 23. A 76-year-old woman presents to the emergency department because of
three 5-minute episodes of weakness and numbness of the right face and
arm, associated with a right visual field deficit and difficulty with language.
Vascular imaging with CT angiography reveals 40% stenosis of the left internal
carotid artery just distal to its origin and 90% stenosis of the left middle
cerebral artery. In addition to aggressive management of risk factors, which of
the following is the most appropriate management of this patient?
A. angioplasty and stenting of the left middle cerebral artery, plus
antiplatelet therapy
B. angioplasty and stenting of the left middle cerebral artery, plus warfarin
C. antiplatelet therapy alone
D. angioplasty of the left middle cerebral artery, plus antiplatelet therapy, but
no stenting
E. warfarin alone
The correct answer is C (antiplatelet therapy alone). A study comparing
warfarin to aspirin in patients with stroke attributed to stenosis of an
intracranial artery was stopped early because the warfarin group had a
significantly higher rate of death, major hemorrhage, and myocardial infarction.
A study comparing aggressive medical management alone to aggressive
medical management plus percutaneous transluminal angioplasty and stenting
was stopped early because of a lack of benefit and concerns about safety in
the angioplasty/stenting group. A retrospective study suggested that balloon
angioplasty alone, without stenting, might result in fewer complications than
the combined procedure, but no randomized controlled trial results are
available, and the long-term durability of angioplasty remains unknown.
For more information, refer to the Intracranial Atherosclerosis section of the
article ‘‘Large Artery Atherosclerosis: Carotid Stenosis, Vertebral
Artery Disease, and Intracranial Atherosclerosis.’’

b 24. An 85-year-old man who has a history of atrial fibrillation but is otherwise
healthy is seen for an assessment of stroke risk. He has hypertension but is
not diabetic and besides his atrial fibrillation has no other known heart
disease. He has never had a reported neurologic event. According to current
data, what is the most appropriate stroke-prevention treatment in this patient?
A. anticoagulation
B. anticoagulation plus 81 mg aspirin
C. aspirin 81 mg daily
D. aspirin 81 mg daily plus clopidogrel
E. no additional therapy is needed
The correct answer is A (anticoagulation). This patient’s CHADS2 (congestive
heart failure, hypertension, age equal to or greater than 75 years, diabetes
mellitus, stroke) scale score is 2, calculated by his age being greater than
75 years and his hypertension. Anticoagulation is therefore indicated in this
patient, unless there is a clear-cut contraindication such as the presence of
a significant bleeding or fall risk. For more information, refer to page 312 of
the article ‘‘Evaluation and Prevention of Cardioembolic Stroke.’’

Continuum (Minneap Minn) 2014;20(2):459–475 www.ContinuumJournal.com 469


Copyright © American Academy of Neurology. Unauthorized reproduction of this article is prohibited.
Postreading CME Test—Preferred Responses

b 25. Which of the following laboratory tests may be a useful adjunctive test in
making a diagnosis of acute cerebral venous thrombosis?
A. D-dimer
B. erythrocyte sedimentation rate
C. international normalized ratio
D. prothrombin time
E. partial thromboplastin time
The correct answer is A (D-dimer). An elevated D-dimer level, as a marker
of fibrin degradation products, may be useful as an adjunctive test in
diagnosing an acute cerebral venous thrombosis. For more information, refer
to pages 336 and 337 of the article ‘‘Cerebral Venous Thrombosis.’’

b 26. In patients known to have unruptured intracranial aneurysms, which of


the following features is associated with a reduced risk of rupture?
A. alcohol abuse
B. aneurysm diameter of less than 3 mm
C. familial aneurysm syndrome
D. hypertension
E. tobacco use
The correct answer is B (aneurysm diameter of less than 3 mm). Risk of
aneurysmal rupture increases as aneurysm size increases; aneurysms less
than 3 mm in diameter are unlikely to rupture. Alcohol abuse, tobacco use,
hypertension, and a familial aneurysm syndrome all increase the risk of
aneurysmal rupture. For more information, refer to pages 390 and 391 of the
article ‘‘Unruptured Intracranial Aneurysms: Screening and
Management.’’

b 27. Which of the following is the most common clinical presenting sign or
symptom of cerebral venous thrombosis?
A. headache
B. hemiparesis
C. seizure
D. syncope
E. visual changes
The correct answer is A (headache). Most patients with cerebral venous
thrombosis (CVT) present with headache, and headache affects approximately
90% of patients with CVT; isolated headaches are not common, however,
and may constitute a clinical challenge. For more information, refer to
page 336 of the article ‘‘Cerebral Venous Thrombosis.’’

470 www.ContinuumJournal.com April 2014

Copyright © American Academy of Neurology. Unauthorized reproduction of this article is prohibited.


b 28. In light of recent clinical trials, which of the following is the standard
treatment for a patient with an acute ischemic stroke who comes to medical
attention 2 hours after the onset of deficits?
A. intra-arterial recombinant tissue-type plasminogen activator (rtPA)
B. IV rtPA
C. IV rtPA followed by intra-arterial rtPA
D. IV rtPA followed by mechanical endovascular clot retrieval
E. mechanical endovascular clot retrieval
The correct answer is B (IV rtPA). The benefit of IV rtPA has been clearly
established in patients with ischemic stroke who come to medical attention
within the first 4.5 hours of the onset of deficits, and especially in patients
who come to medical attention within the first 3 hours. Although some
evidence supports the use of intra-arterial rtPA outside the time window
in which IV rtPA is known to be beneficial, no clinical evidence is available
to support the substitution of intra-arterial rtPA or mechanical endovascular
clot retrieval in patients who are eligible for IV rtPA. Recent studies failed
to demonstrate any improvement in outcome when IV rtPA was followed by
intra-arterial rtPA or mechanical endovascular clot retrieval. Subgroups
of patients for whom such sequential treatment is beneficial have yet to
be identified. For more information, refer to page 286 of the
article ‘‘Evaluation and Management of Acute Ischemic Stroke.’’

b 29. Which of the following statements best characterizes the results of each
of the three randomized controlled trials that have been published to date
comparing medical therapy to closure in patients with stroke and patent
foramen ovale?
A. there were fewer recurrent strokes with closure, and the results were
statistically significant
B. there were fewer recurrent strokes with closure, but the results were not
statistically significant
C. there were fewer recurrent strokes with medical therapy, and the results
were statistically significant
D. there were fewer recurrent strokes with medical therapy, but the results
were not statistically significant
E. there were no consistent trends in favor of either approach
The correct answer is B (there were fewer recurrent strokes with closure, but
the results were not statistically significant). Three randomized controlled
trials have now been published comparing medical therapy to closure in
patients with stroke and patent foramen ovale. The trials differed with respect
to the specific closure device, patient inclusion criteria, and primary end
point, but in each study the patients treated with a closure device had fewer
recurrent strokes than the patients treated with medication alone. This result
was not statistically significant in any of the studies, although a meta-analysis
of the three trials suggested potential benefit. At this point, optimal
management of cryptogenic stroke in a patient with patent foramen ovale
remains unclear. For more information, refer to pages 358 and 359 of the
article ‘‘Evaluation and Management of Stroke in Young Adults.’’

Continuum (Minneap Minn) 2014;20(2):459–475 www.ContinuumJournal.com 471


Copyright © American Academy of Neurology. Unauthorized reproduction of this article is prohibited.
Postreading CME Test—Preferred Responses

b 30. In patients who stop smoking after having smoked more than 40 cigarettes
a day for 10 years, at what point does the risk of stroke return to nonsmoker
levels?
A. 6 months after stopping smoking
B. 5 years after stopping smoking
C. 15 years after stopping smoking
D. 30 years after stopping smoking
E. never
The correct answer is B (5 years after stopping smoking). In the
Framingham study, 5 years after smoking cessation, stroke risk returned to
nonsmoker levels. For more information, refer to pages 302 and 303 of the
article ‘‘Risk Factor Management for Stroke Prevention.’’

b 31. For which of the following conditions is the evidence strongest that the
risk of stroke can be reduced by aggressively treating the condition?
A. hypernatremia
B. hypertension
C. insulin resistance
D. obesity
E. physical inactivity
The correct answer is B (hypertension). Studies have consistently shown
that antihypertensive therapy is effective for both primary and secondary
stroke prevention. It has not been convincingly demonstrated that stroke
risk can be reduced by treating any of the other conditions listed. For
more information, refer to pages 296 and 297 of the article
‘‘Risk Factor Management for Stroke Prevention.’’

b 32. Which of the following disorders affecting the coagulation cascade is most
closely associated with arterial (as opposed to venous) thrombosis?
A. antiphospholipid antibody syndrome
B. antithrombin III deficiency
C. factor V Leiden mutation
D. protein S deficiency
E. prothrombin gene (G20210A) mutation
The correct answer is A (antiphospholipid antibody syndrome).
Antiphospholipid antibody syndrome and homocystinuria are associated with
arterial thrombosis. Most other thrombophilias, including antithrombin III
deficiency, factor V Leiden mutation, protein S deficiency, and prothrombin
gene mutation, are associated with venous thrombosis. For more information,
refer to page 359 of the article ‘‘Evaluation and Management
of Stroke in Young Adults.’’

472 www.ContinuumJournal.com April 2014

Copyright © American Academy of Neurology. Unauthorized reproduction of this article is prohibited.


b 33. A 66-year-old woman with a history of headaches since adolescence
volunteered as a healthy control for a cerebrovascular imaging study, and was
found to have an aneurysm of the anterior communicating artery. Which of
the following historical features would be most likely to affect management?
A. heterozygosity for factor V Leiden
B. history of nausea and vomiting accompanying her headaches
C. history of visual aura
D. recent cataract surgery
E. recent headache associated with severe neck stiffness
The correct answer is E (recent headache associated with severe neck
stiffness). In a patient with an incidentally discovered cerebral aneurysm, a
recent headache associated with severe neck stiffness would suggest the
likelihood of recent aneurysmal rupture, which could change management.
For more information, refer to page 390 of the article
‘‘Unruptured Intracranial Aneurysms: Screening and Management.’’

b 34. Which of the following factors has the best established correlation with
the likelihood of benefit from IV recombinant tissue-type plasminogen
activator (rtPA) in patients with acute ischemic stroke?
A. age of patient
B. degree of diffusion/perfusion mismatch on imaging studies
C. severity of neurologic deficits at the time treatment is initated
D. time since patient was last known well
E. weight of patient
The correct answer is D (time since patient was last known well). Studies
consistently show that the outcome of IV rtPA treatment in patients with
acute ischemic stroke strongly correlates with the amount of time that has
elapsed since the patient was last known well. Although it seems plausible
that the degree of diffusion/perfusion mismatch identified on imaging studies
should correlate with outcome, this has not been established in clinical trials. Patient
age, weight, and stroke severity do not clearly correlate with the likelihood
of benefit from IV rtPA. For more information, refer to pages 283 and 284 of the
article ‘‘Evaluation and Management of Acute Ischemic Stroke.’’

b 35. Magnetic resonance angiography (MRA) and CT angiography (CTA) have


greater than 90% sensitivity for detecting unruptured intracranial aneurysms
as long as the aneurysm diameter exceeds what cutoff?
A. 3 mm for CTA and 3 mm for MRA
B. 3 mm for CTA and 10 mm for MRA
C. 10 mm for CTA and 3 mm for MRA
D. 10 mm for CTA and 10 mm for MRA
E. no cutoff: sensitivity is greater than 90% for both modalities regardless
of aneurysm diameter
The correct answer is A (3 mm for CTA and 3 mm for MRA). Both CTA
and MRA have greater than 90% sensitivity for detecting aneurysms larger
than 3 mm in diameter, but for aneurysms smaller than 3 mm in diameter,
CTA has a sensitivity of 61% and MRA has a sensitivity of 38%. For more
information, refer to pages 392 and 393 of the article ‘‘Unruptured
Intracranial Aneurysms: Screening and Management.’’

Continuum (Minneap Minn) 2014;20(2):459–475 www.ContinuumJournal.com 473


Copyright © American Academy of Neurology. Unauthorized reproduction of this article is prohibited.
Postreading CME Test—Preferred Responses

b 36. Which of the following levels of daily alcohol intake is associated with the
lowest relative risk of ischemic stroke?
A. zero drinks
B. one to two drinks
C. three to four drinks
D. five to six drinks
E. seven or more drinks
The correct answer is B (one to two drinks). The relationship between alcohol
consumption and ischemic stroke risk is represented by a J-shaped curve,
with the relative risk being lowest for patients with moderate consumption
(up to two drinks per day), higher for nondrinkers, and highest for people with
heavy consumption (more than five drinks per day). For more information,
refer to page 304 of the article ‘‘Risk Factor Management for
Stroke Prevention.’’

b 37. Fabry disease is most commonly associated with strokes in which of the
following arterial distributions?
A. anterior cerebral artery
B. carotid artery
C. lenticulostriate arteries
D. middle cerebral artery
E. posterior circulation
The correct answer is E (posterior circulation). Fabry disease is an X-linked
genetic disorder associated with deficiency of the lysosomal hydrolase
!-galactosidase A. Fabry disease produces strokes in young male patients
that are typically posterior circulation ischemic events. Other CNS vascular
manifestations include dolichoectasia of cerebral arteries and white matter
disease. For more information, refer to page 399 of the article
‘‘Genetic Stroke Syndromes.’’

b 38. For patients with diabetes mellitus, the evidence is strongest that
aggressive control of which of the following serum levels is effective for
secondary stroke prevention?
A. fasting glucose
B. hemoglobin A1c
C. low-density lipoprotein
D. random glucose
E. triglyceride
The correct answer is C (low-density lipoprotein). Although diabetes
mellitus is a risk factor for recurrent stroke, trials of aggressive glycemic
control have failed to show a benefit with respect to secondary stroke
prevention, whereas aggressive lipid lowering with statins has been shown to
lower stroke risk. The evidence for an association with stroke risk is greater
for low-density lipoprotein (LDL) cholesterol levels than for triglyceride
levels. For more information, refer to page 301 of the article
‘‘Risk Factor Management for Stroke Prevention.’’

474 www.ContinuumJournal.com April 2014

Copyright © American Academy of Neurology. Unauthorized reproduction of this article is prohibited.


b 39. A 54-year-old man with no known medical problems is donating blood when
he suddenly develops a left hemiparesis. He is immediately brought to the
emergency department, where his CT scan is normal. His weakness continues to
progress. Which of the following blood test results must be reviewed before
administering IV recombinant tissue-type plasminogen activator (rtPA) to this
patient?
A. cholesterol
B. creatinine
C. glucose
D. international normalized ratio (INR)
E. platelet count
The correct answer is C (glucose). The serum glucose level must be reviewed
before administering IV rtPA to be sure that hypoglycemia or hyperglycemia
is not responsible for the clinical presentation (although in cases where the
clinical presentation is thought to be due to concurrent ischemia, neither
hypoglycemia nor hyperglycemia precludes IV rtPA). Although it is desirable
to also obtain the results of other blood tests, especially partial thromboplastin
time, international normalized ratio, and platelet count, the administration of
IV rtPA should not be delayed for any laboratory result other than finger-stick
glucose level unless a clinical basis for suspecting an abnormality exists. In
a patient with no known medical problems, the likelihood of a coagulation
disorder is very low. For more information, refer to page 291 of the
article ‘‘Evaluation and Management of Acute Ischemic Stroke.’’

b 40. A 72-year-old man presents to the emergency department after awakening


with severe right leg weakness. CT of the brain obtained on admission shows
a small acute ischemic stroke in the left anterior cerebral artery territory.
By the next day, he has improved to the point where he has only mild
weakness of right ankle dorsiflexion, with no sensory deficits. Carotid
ultrasound and CT angiography both show 70% to 80% stenosis of the left
internal carotid artery just distal to its origin. When should endarterectomy
be performed in this patient?
A. at least 12 weeks after the stroke
B. 6 to 12 weeks after the stroke
C. 2 to 4 weeks after the stroke
D. within 2 weeks of the stroke
E. neverVendarterectomy is not indicated
The correct answer is D (within 2 weeks of the stroke). The benefit of
carotid revascularization declines with time from the initial ischemic event,
so the American Heart Association recommends that endarterectomy be
performed within 2 weeks of the event rather than delaying surgery.
For more information, refer to page 325 of the article
‘‘Large Artery Atherosclerosis: Carotid Stenosis, Vertebral Artery Disease,
and Intracranial Atherosclerosis.’’

Continuum (Minneap Minn) 2014;20(2):459–475 www.ContinuumJournal.com 475


Copyright © American Academy of Neurology. Unauthorized reproduction of this article is prohibited.
CME

Patient Management
Address correspondence to
Dr Adam Kelly, University of
Rochester, 601 Elmwood
Avenue, PO Box 673, Rochester,
NY 14642-0001,
Adam_kelly@urmc.rochester.edu.
Relationship Disclosure:
Problem—Preferred
Dr Kelly has received research
support from the Donald W.
Reynolds Foundation and
Responses
compensation from the
American Academy of Adam G. Kelly, MD
Neurology as a question writer
for Continuum.
Unlabeled Use of
Products/Investigational
Use Disclosure: Following are the preferred responses for the Patient Management Problem
Dr Kelly reports no disclosure. in this issue. The case, questions, and answer options are
* 2014, American Academy
of Neurology. repeated, and the preferred response is given, followed by an explanation
and a reference with which you may seek more specific information. You are
encouraged to review the responses and explanations carefully to evaluate
your general understanding of the material. The comment and references
included with each question are intended to encourage independent study.

Learning Objectives
Upon completion of this activity, the participant will be able to:
& Describe the early management of patients with acute ischemic stroke,
including the use of IV thrombolytic therapy
& Select appropriate diagnostic tests to be used in the initial evaluation of
patients with ischemic stroke
& Discuss evidence-based strategies for secondary stroke prevention after
stroke or transient ischemic attack

Case
A 64-year-old man is brought to the emergency department by ambulance
after developing the acute onset of left-sided weakness and left-sided
neglect. Because of his neglect, he has difficulty acknowledging his
symptoms and is unable to state the exact time of their onset. He reports
that his medical history is notable for hypertension and hyperlipidemia and
states that he is not currently taking any medications. He reports no recent
hospitalizations, bleeding events, or other relevant history.
On initial evaluation, his blood pressure (BP) is 177/80 mm Hg, pulse is
80 beats/min and regular, and he is afebrile. He is awake but inattentive.
Language is intact but speech is mildly dysarthric as a result of weakness of
the left lower face. Eye movements and visual fields are full, but he has
extinction to double simultaneous visual stimulation on his left side. He is
able to briefly lift his left arm and leg, but both drift down to the bed
within seconds. He notes decreased sensation on his left side compared to
the right and has extinction to double simultaneous tactile stimulation on
the left side. The NIH Stroke Scale (NIHSS) score is 10.
Head CT is completed and shows no signs of intracranial hemorrhage or
early ischemic changes. Finger-stick blood glucose is 103 mg/dL.

482 www.ContinuumJournal.com April 2014

Copyright © American Academy of Neurology. Unauthorized reproduction of this article is prohibited.


b 1. Which of the following clinical or diagnostic features is most critical to
determine whether IV thrombolytic therapy may be indicated in this patient?
A. assessing results of coagulation studies (eg, plasma thromboplastin, partial
thromboplastin time)
B. evaluating response of BP to labetalol 10 mg intravenously
C. identifying a clear time of symptom onset
D. inquiring about a possible prior history of ischemic stroke
E. obtaining a CT angiogram of the brain to evaluate for intracranial vascular
occlusion

The preferred response is C (identifying a clear time of symptom onset).


One of the most important aspects of the initial evaluation of a patient with
suspected stroke is identifying the exact time of symptom onset, if possible,
or identifying the time at which the patient was last known to be at his or her
neurologic baseline. Many stroke patients, especially those with neglect
syndromes, have difficulty providing this information, so other informants
(eg, family, friends, coworkers) may need to be asked. Especially since his CT
does not show any early ischemic changes, this patient may still be within the
3-hour time window for which IV thrombolysis (tissue plasminogen activator
[tPA]) is approved by the US Food and Drug Administration (FDA), or within
the 3- to 4.5-hour window during which treatment is supported by American
Stroke Association guidelines.
Except in cases in which anticoagulant use or liver disease is suspected,
thrombolytic treatment does not need to be delayed for results of coagulation
studies. This patient’s BP, although slightly elevated, is still below the threshold
where thrombolytic treatment must be withheld (9185/105 mm Hg). Noninvasive
evaluation of the intracranial circulation is often part of decision-making regarding
intra-arterial stroke treatments (eg, embolectomy) but is not necessary for
decisions regarding IV tPA. Finally, although a history of prior intracranial
hemorrhage is an absolute contraindication to tPA, a history of prior ischemic
stroke (unless within the previous 3 months) does not preclude treatment.1,2

1. Jauch EC, Saver JL, Adams HP Jr, et al. Guidelines for the early management of patients with
acute ischemic stroke: a guideline for healthcare professionals from the American Heart
Association/American Stroke Association. Stroke 2013;44(3):870Y947.
2. Lansberg MG, O’Donnell MJ, Khatri P, et al. Antithrombotic and thrombolytic therapy for
ischemic stroke: Antithrombotic Therapy and Prevention of Thrombosis, 9th ed: American
College of Chest Physicians Evidence-Based Clinical Practice Guidelines. Chest 2012;
141(2 suppl):e601SYe636S.

The patient’s family arrives in the emergency department and reports that
he was in his usual good neurologic health until 90 minutes ago, at which
point he had the witnessed onset of the left-sided weakness and other
symptoms now seen on examination. His family confirms that he has
had no recent prior stroke symptoms, head trauma, bleeding events,
seizure activity at the start of his symptoms, nor history of intracranial
hemorrhage. His examination has not significantly changed since arrival at
the hospital, and his BP is now 169/84 mm Hg.

Continuum (Minneap Minn) 2014;20(2):482–493 www.ContinuumJournal.com 483


Copyright © American Academy of Neurology. Unauthorized reproduction of this article is prohibited.
PMP—Preferred Responses

b 2. Which of the following therapeutic or diagnostic options is the best next


step in management of this patient?
A. administration of alteplase
B. administration of aspirin 325 mg and clopidogrel 75 mg
C. monitoring for 30 to 60 minutes to allow for possible symptom improvement
D. MRI of the brain with diffusion-weighted imaging
E. referral for mechanical thrombectomy

The preferred response is A (administration of alteplase). Now that the time


of symptom onset is much clearer (and within a 3-hour window) and other
possible contraindications/warnings for tPA treatment have been more definitively
ruled out, this patient appears to be a good candidate for treatment with tPA
(alteplase).1 This is the only FDA-approved treatment for acute stroke, and
even though his examination is suggestive of a possible right middle cerebral
artery occlusion, bypassing IV tPA and proceeding directly to mechanical
thrombectomy/embolectomy is not recommended. Combinations of aspirin and
clopidogrel are used for stroke prevention in some settings (eg, intracranial
atherosclerosis) and are being studied for use in the acute phase following TIA
or mild stroke. However, these medications should not be administered in
patients who are potential candidates for tPA treatment. Since the odds of good
outcomes following tPA treatment decrease with time, treatment should not be
withheld to allow for any symptom improvement.2 Similarly, because the clinical
picture is very consistent with stroke and there are no obvious reasons not to
proceed with tPA treatment, performing an MRI at this point is unlikely to
significantly change management and will probably delay tPA administration.
1. American College of Emergency Physicians; American Academy of Neurology. Clinical policy:
use of intravenous tPA for the management of acute ischemic stroke in the emergency
department. Ann Emerg Med 2013;61(2):225Y243.
2. Del Zoppo GJ, Saver JL, Jauch EC, et al. Expansion of the time window for treatment of acute
ischemic stroke with intravenous tissue plasminogen activator: a science advisory from the
American Heart Association/American Stroke Association. Stroke 2009;40(8):2945Y2948.

The risks and benefits of tissue plasminogen activator (tPA) treatment are
discussed with the patient and his family, and they agree to treatment; the
tPA bolus is administered at 2 hours and 15 minutes after symptom onset. At
the end of the tPA infusion, his examination has shown some improvement.
His neglect is significantly better, and the left-sided weakness, while still
present, is also improved. The NIHSS score is now 4. He is transferred to the
intensive care unit (ICU) for close neurologic monitoring.

b 3. Which of the following is part of the recommended management of this


patient after treatment with IV tPA?
A. immediate administration of aspirin 81 mg
B. placement of a Foley catheter
C. referral for mechanical thrombectomy
D. repeat brain imaging at 24 hours after treatment
E. subcutaneous heparin for venous thromboembolism prophylaxis

484 www.ContinuumJournal.com April 2014

Copyright © American Academy of Neurology. Unauthorized reproduction of this article is prohibited.


The preferred response is D (repeat brain imaging at 24 hours after treatment).
Patients treated with tPA need to be admitted to an ICU or other unit where close
monitoring of vital signs and neurologic status can be performed. This includes
assessments every 15 minutes for the first 2 hours during and the first 2 hours after
treatment, every 30 minutes for the next 6 hours, and then hourly until 24 hours
after treatment. At 24 hours, patients should undergo brain imaging with either
CT or MRI to evaluate for any hemorrhagic transformation of the infarct and
establish the size and location of the stroke. If no hemorrhage is evident on the
24-hour scan, then antithrombotic therapy can be administered. Aspirin, other
antiplatelet agents, and anticoagulant medications are not prescribed in the first
24 hours following tPA treatment, in order to minimize bleeding complications.1,2
The recently reported Interventional Management of Stroke 3 (IMS-3) clinical trial3
failed to show benefit of following IV tPA with an endovascular treatment, such
as thrombectomy. While some subgroups of patients may benefit from this combined
approach, it is not indicated in this patient who has experienced improvement after
tPA and whose current NIHSS is only 4. Any unnecessary blood draws, catheter
placements, or other invasive procedures should be deferred after patients have
received tPA, in order to lower the risk of systemic bleeding. In this patient, who does
not have a history of urinary or bladder dysfunction, catheter placement is not needed.

1. Jauch EC, Saver JL, Adams HP Jr, et al. Guidelines for the early management of patients with
acute ischemic stroke: a guideline for healthcare professionals from the American Heart
Association/American Stroke Association. Stroke 2013;44(3):870Y947.
2. Lansberg MG, O’Donnell MJ, Khatri P, et al. Antithrombotic and thrombolytic therapy for ischemic
stroke: Antithrombotic Therapy and Prevention of Thrombosis, 9th ed: American College of Chest
Physicians Evidence-Based Clinical Practice Guidelines. Chest 2012;141(2 suppl):e601SYe636S.
3. Broderick JP, Palesch YY, Demchuk AD, et al. Endovascular therapy after intravenous t-PA
versus t-PA alone for Stroke. New Engl J Med 2013;368(10):893Y903.

The patient spends 24 hours in the ICU with some continued improvement in
his symptoms. An MRI scan performed 1 day after treatment shows a
wedge-shaped area of acute infarction affecting the right frontal lobe and no
signs of intracranial hemorrhage. His examination is now notable only for the
left facial droop and dysarthria; the NIHSS score is 2. His systolic BP has ranged
between 150 mm Hg and 170 mm Hg since he was admitted to the hospital.

b 4. Which of the following is the best next step in management?


A. begin aspirin 325 mg/d
B. begin IV unfractionated heparin infusion
C. continue ICU monitoring for another 24 hours
D. lower BP to less than 140/90 mm Hg
E. refer for cerebral angiogram

The preferred response is A (begin aspirin 325 mg/d). Because his follow-up
imaging shows no sign of hemorrhage and the MRI confirms the clinical
suspicion of acute ischemic stroke, this patient should be started on antiplatelet
therapy 24 hours after tPA treatment. Available options include aspirin (at doses
greater than 50 mg/d), clopidogrel, or combination aspirin and extended-release
dipyridamole. All are considered reasonable options for secondary stroke
prevention; in addition to long-term stroke prevention, aspirin has also been

Continuum (Minneap Minn) 2014;20(2):482–493 www.ContinuumJournal.com 485


Copyright © American Academy of Neurology. Unauthorized reproduction of this article is prohibited.
PMP—Preferred Responses

shown to decrease the risk of early recurrent stroke when given within the first
48 hours after stroke. Whether this early benefit extends to other antiplatelet
options such as clopidogrel or aspirin plus dipyridamole is uncertain because
limited studies of these options have been performed in the acute stroke setting.
High BP is a critically important risk factor for recurrent stroke, and BP
lowering is a key aspect of secondary stroke prevention in many cases. However,
in the short-term setting immediately after stroke, it is recommended to not
abruptly lower BP and instead allow for a period of permissive hypertension.
First, patients with chronic hypertension can have altered autoregulation of
cerebral blood flow compared to normotensive patients, and this regulation can
be further disrupted by stroke. As a result, lowering of BPs can lead to significant
reductions in blood flow. Second, adjacent to the infarcted tissue seen on
imaging, there may be areas of penumbra that are perfused through collateral
blood vessels. These collateral supplies may be more sensitive to BP reductions.
No role has been established for the routine use of heparin infusions after
ischemic stroke in most patients. Any benefit in terms of early stroke
prevention is outweighed by an increase in hemorrhagic transformation. At
this point in the case, there does not appear to be a clear rationale for
continued ICU monitoring or a cerebral angiogram.1

1. Jauch EC, Saver JL, Adams HP Jr, et al. Guidelines for the early management of patients with
acute ischemic stroke: a guideline for healthcare professionals from the American Heart
Association/American Stroke Association. Stroke 2013;44(3):870Y947.

The patient is transferred to the stroke unit for continued care. The
following diagnostic test results are obtained.
& Magnetic resonance (MR) angiogram of the neck: no evidence of
carotid or vertebral artery stenosis
& MR angiogram of the head: no intracranial stenosis or occlusion
& Transthoracic echocardiogram: left ventricle ejection fraction 55%;
no regional wall motion abnormalities; no sign of intracardiac
thrombus; no significant valvular disease
& 48 hours of continuous ECG monitoring: normal sinus rhythm, no
significant dysrhythmia
& Fasting lipids: Total cholesterol 210 mg/dL; triglycerides 223 mg/dL;
high-density lipoprotein (HDL) 34 mg/dL; low-density lipoprotein
(LDL) 136 mg/dL

b 5. Which of the following diagnostic tests is the best next step in management?
A. carotid ultrasound
B. genetic testing for factor V Leiden
C. MR venogram of the pelvis
D. transcranial Doppler ultrasound
E. transesophageal echocardiogram

The preferred response is E (transesophageal echocardiogram). The description


of this patient’s infarctionVa wedge-shaped area and a hemispheric locationVis

486 www.ContinuumJournal.com April 2014

Copyright © American Academy of Neurology. Unauthorized reproduction of this article is prohibited.


suggestive of an embolic process. Since the initial set of diagnostic testing was
unremarkable in identifying a specific etiology for his stroke, performing a
transesophageal echocardiogram seems appropriate for several reasons. First, no
comment was made on the echocardiogram result regarding the presence or
absence of a patent foramen ovale, which may be a possible contributor to his
stroke. Second, some potential sources of cardioembolism might be identified
on a transesophageal echocardiogram but not on a transthoracic study, such as
aortic arch atheromatous disease1 and left atrial appendage thrombus.2
MR angiography has high sensitivity but relatively lower specificity for the
identification of a vessel stenosis, either carotid or intracranial in location. Since an
initial, noninvasive study was reassuring, there does not appear to be a clear
reason to proceed with a second imaging modality, such as either carotid or
transcranial ultrasonography.3 At this point, an obvious history of an underlying
hypercoagulable state such as factor V Leiden (eg, prior deep vein thrombosis or
pulmonary embolism) is lacking. As a result, routine investigation for
hypercoagulability is not indicated in this case. Finally, without yet having
evidence of a right-to-left shunt that could serve as a source of paradoxical
embolism (such as via a patent foramen ovale), it is premature to look for
evidence of deep vein thrombosis in the veins of the leg or pelvis.

1. Di Tullio MR, Russo C, Jin Z, et al. Aortic arch plaques and risk of recurrent stroke and death.
Circulation 2009;119(17):2376Y2382.
2. Young KC, Benesch CB. Transesophageal echocardiography screening in subjects with a first
cerebrovascular ischemic event. J Stroke Cerebrovasc Dis 2011;20(6):503Y509.
3. Latchaw RE, Alberts MJ, Lev MH, et al. Recommendations for imaging of acute ischemic stroke:
a scientific statement from the American Heart Association. Stroke 2009;40(11):3646Y3678.

b 6. Which of the following is the best option for management regarding his
fasting lipids?
A. adherence to a low-fat diet
B. atorvastatin
C. ezetimibe
D. fenofibrate
E. niacin

The preferred response is B (atorvastatin). While the association between


elevated cholesterol and stroke risk is not as strong as the association between
elevated cholesterol and coronary heart disease, lipid-lowering therapy still has a
role in stroke prevention. National guidelines suggest using statin medications
for lowering lipids after stroke,1 primarily based on the Stroke Prevention by
Aggressive Reduction in Cholesterol Levels (SPARCL) study. In this study,
patients with stroke but no other established indication for statin therapy were
randomized to receive atorvastatin 80 mg/d or placebo. The risk of stroke was
lower in the statin-treated group, although the effect was relatively modest
(absolute risk reduction of 2.2% after 5 years of treatment).2
Most guidelines for statin therapy after stroke use the LDL as the primary target of
treatment. For example, guidelines from the American Heart Association and
American Stroke Association recommend lipid-lowering therapy with statins for
patients with LDL values of 100 mg/dL or higher; in patients with an atherosclerotic
etiology to their stroke or TIA, targeting a goal LDL of 70 mg/dL or lower, or a

Continuum (Minneap Minn) 2014;20(2):482–493 www.ContinuumJournal.com 487


Copyright © American Academy of Neurology. Unauthorized reproduction of this article is prohibited.
PMP—Preferred Responses

50% reduction in LDL values, is recommended. Evidence suggests that intensive


lipid-lowering treatments (higher-potency statins at higher doses) may further lower
stroke risk compared to less intensive strategies,3 although concerns exist regarding
small but significant increased risks of hemorrhagic stroke with this approach.
While adherence to a low-fat diet should be considered part of an overall approach
to hyperlipidemia in most patients, it should not be used as the sole treatment option
in this case. Niacin and fibrates have been shown to lower triglyceride levels and
increase HDL levels in patients with dyslipidemia, although it has not been shown
that these strategies are associated with decreased stroke risk. Similarly, limited data
exist for the use of ezetimibe for lipid management following stroke.

1. Furie KL, Kasner SE, Adams RJ, et al. Guidelines for the prevention of stroke in patients with
stroke or transient ischemic attack: a guideline for healthcare professionals from the american
heart association/american stroke association. Stroke 2011;42(1):227Y276.
2. Amarenco P, Bogousslavsky J, Callahan A 3rd, et al; Stroke Prevention by Aggressive Reduction
in Cholesterol Levels (SPARCL) Investigators. High-dose atorvastatin after stroke or transient
ischemic attack. N Engl J Med 2006;355(6):549Y559.
3. The Cholesterol Treatment Trialists Collaboration, Baigent C, Blackwell L, Emberson J, et al.
Efficacy and safety of more intensive lowering of LDL cholesterol: a meta-analysis of data from
170,000 participants in 26 randomised trials. Lancet 2010;376(9753):1670Y1681.

His transesophageal echocardiogram is unremarkable for any source of


cardioembolism. He is discharged from the hospital on aspirin 325 mg/d
and atorvastatin 40 mg/d.
The patient is seen in the neurology clinic 3 weeks after his stroke. He has
continued to make additional progress on his recovery; he now notes only some
minimal trouble articulating his speech, usually when he is tired or fatigued. He
reports no new symptoms concerning for recurrent stroke or TIA and is
tolerating his aspirin without any adverse effects. His BP today is 155/90 mm Hg.
While he does not check his BP regularly outside of a physician’s office, he says
the reading was similar when he saw his primary care physician 1 week ago.

b 7. Which of the following is the best approach to BP management in this patient?


A. begin low-salt diet alone
B. continue to allow for permissive hypertension for 1 to 2 months poststroke
C. order renal ultrasound to assess for renal artery stenosis
D. prescribe hydrochlorothiazide 12.5 mg/d
E. recheck BP in 1 week

The preferred response is D (prescribe hydrochlorothiazide 12.5 mg/d).


This patient has had persistent hypertension both while in the hospital and
since his discharge, and he reported a history of hypertension on initial
presentation to the hospital despite not being treated with antihypertensive
medications at that point. Since his readings have been persistently elevated,
this seems unlikely to be related to ‘‘white coat hypertension’’ alone, and
given the strong association between hypertension and stroke, treatment
should be initiated. While permissive hypertension has a role in the acute
phase immediately following stroke, no clear benefit to permissive hypertension
remains several weeks after stroke. Lifestyle modifications should be considered

488 www.ContinuumJournal.com April 2014

Copyright © American Academy of Neurology. Unauthorized reproduction of this article is prohibited.


in all cases in which BP is elevated, including a low-salt or Dietary Approaches
to Stop Hypertension (DASH) diet as well as weight loss and regular aerobic
exercise. However, given that these interventions are likely to take considerable
time before having an effect on BP and that this patient’s BP is significantly above
target, an antihypertensive medication should be started1 No clear evidence
has shown that one class of antihypertensive medications is superior for stroke
prevention purposes, although thiazide diuretics and angiotensin-converting
enzyme (ACE) inhibitors are both considered reasonable options. At this point,
there is no indication that his hypertension is secondary to other medical
comorbidities, so screening for conditions such as renal artery stenosis,
hyperthyroidism, or pheochromocytoma is not indicated.
1. Davis SM, Donnan GA. Clinical practice. Secondary prevention after ischemic stroke of transient
ischemic attack. N Engl J Med 2012;366(20):1914Y1922.

b 8. Which of the following is the best long-term BP goal in this patient?


A. diastolic BP lower than 80 mm Hg
B. mean arterial pressure lower than 100 mm Hg
C. systolic BP between 120 mm Hg and 140 mm Hg
D. systolic BP lower than 120 mm Hg
E. systolic BP lower than 150 mm Hg

The preferred response is C (systolic BP between 120 mm Hg and 140 mm Hg).


Hypertension is the most important modifiable risk factor for stroke and is
associated with both ischemic and hemorrhagic cerebrovascular disease.1,2 As
a result, BP lowering should be considered in all patients with stroke. Targets
for BP treatment should be tailored to the individual patient, but in general,
systolic BPs between 120 mm Hg and 140 mm Hg and diastolic BPs between
80 mm Hg and 90 mm Hg are reasonable goals. Blood pressures that remain
above these levels may still expose the patient to considerable risk for
recurrent stroke. However, some studies show that the risk of recurrent stroke
and other adverse effects may be increased in patients with BP reductions
below 120/80 mm Hg. This may especially be the case with older patients.3
1. Davis SM, Donnan GA. Clinical practice. Secondary prevention after ischemic stroke of transient
ischemic attack. N Engl J Med 2012;366(20):1914Y1922.
2. SPS3 Study Group, Benavente OR, Coffey CS, Conwit R, et al. Blood pressure targets in patients
with recent lacunar stroke: the SPS3 randomised trial. Lancet 2013;382(9891):507Y515.
3. Rothwell PM, Algra A, Amarenco P. Medical treatment in acute and long-term stroke
prevention after transient ischemic attack and ischaemic stroke. Lancet 2011;377(9778):1681Y1692.

Three months later, he returns to clinic with concerns about ongoing fatigue. He
notes that this was a problem before his stroke but that it has been considerably
worse over the past few months. Despite getting an average of 8 hours of sleep
per night, he does not feel particularly refreshed when he awakens in the
morning, and he notes a tendency to doze while he is sitting at his desk
attempting to work. His BP has continued to be elevated despite an increase
in his hydrochlorothiazide and the addition of a low dose of lisinopril; his BP is
141/93 mm Hg in clinic today. Examination is unchanged from his prior visits.

Continuum (Minneap Minn) 2014;20(2):482–493 www.ContinuumJournal.com 489


Copyright © American Academy of Neurology. Unauthorized reproduction of this article is prohibited.
PMP—Preferred Responses

b 9. Which of the following is the most likely cause of his fatigue?


A. adverse effects of hydrochlorothiazide
B. hypothyroidism
C. obstructive sleep apnea
D. poststroke depression
E. recurrent stroke affecting the left caudate nucleus

The preferred response is C (obstructive sleep apnea). This patient’s


symptoms of excessive daytime sleepiness and significant fatigue despite
sleeping an adequate amount of time at night are both suggestive of
obstructive sleep apnea (OSA). OSA can be seen as a residual effect after
stroke, but it is also an independent risk factor for both stroke and sudden
death.1 It is unclear whether treatment with continuous positive airway
pressure or other modalities can lower the risk of stroke conferred by OSA.
However, it would be reasonable to consider evaluation for OSA with
polysomnography and treatment if indicated, since he has considerable
daytime sleepiness and impairment of his daily activities. His persistent
hypertension that is relatively refractory to treatment could also be a
consequence of OSA, since it can be associated with increased adrenergic tone.2,3

1. Yaggi HK, Concato J, Kernan WN, et al. Obstructive sleep apnea as a risk factor for stroke and
death. N Engl J Med 2005;353(19):2034Y2041.
2. Dyken ME, Im KB. Obstructive sleep apnea and stroke. Chest 2009;136(6):1668Y1677.
3. Hermann DM, Bassetti CL. Sleep-related breathing and sleep-wake disturbances in ischemic
stroke. Neurology 2009;73(16):1313Y1322.

He is referred to a local sleep center, where he receives an overnight


polysomnogram and is found to have moderate to severe obstructive sleep
apnea, with an apnea-hypopnea index of 31 events per hour. He is
started on continuous positive airway pressure (CPAP). He phones the
office 1 month after initiation of treatment and notes that his fatigue is
considerably improved. He has also been checking his BP on a consistent
basis at home and reports that his typical readings have been between
130 mm Hg and 135 mm Hg systolic and 80 mm Hg and 85 mm Hg diastolic.
One year later, the patient is admitted to the hospital after awakening
with the onset of right upper extremity weakness and expressive aphasia.
An MRI shows a small area of infarction in the left frontal lobe. Vascular
imaging shows no evidence of carotid or intracranial stenosis. His
admission electrocardiogram shows an irregularly irregular rhythm
consistent with atrial fibrillation, and he reports several episodes of
palpitations over the past several weeks.

b 10. Which of the following is the best next step in management?


A. add clopidogrel to aspirin
B. change to combination aspirin/extended-release dipyridamole
C. change to dabigatran
D. refer for cardioversion; if successful, stop antithrombotic therapy
E. refer for left atrial appendage occlusion

490 www.ContinuumJournal.com April 2014

Copyright © American Academy of Neurology. Unauthorized reproduction of this article is prohibited.


The preferred response is C (change to dabigatran). This patient is presenting
with recurrent stroke in the setting of a new diagnosis of atrial fibrillation
(AF). Because his initial event 1 year ago also had an embolic appearance
(wedge-shaped), it is possible that his first stroke could have been attributable
to AF as well. The yield of continuous electrocardiographic monitoring increases
with the duration of time that patients are monitored; therefore, a brief hospital
stay may not be long enough to detect paroxysmal episodes of AF. This is the
basis for using prolonged outpatient monitors, such as implantable loop
recorders, in patients for whom there is a high suspicion for paroxysmal AF.
In patients with AF who have experienced a stroke or TIA, anticoagulation is
the preferred option for secondary stroke prevention (assuming no strong
contraindications are present). Since the presence of stroke or TIA confers two
points on the CHADS2 (congestive heart failure, hypertension, age equal to
or greater than 75 years, diabetes mellitus, stroke) risk score, patients in this
setting will always have scores of 2 or higher; this is a range in which
anticoagulation is recommended over antiplatelet therapy. Available options for
oral anticoagulation at this time include warfarin (goal international normalized
ratio 2 to 3), dabigatran, rivaroxaban, and apixaban; all are considered reasonable
options in a setting like this.1,2
Combination aspirin and clopidogrel has been shown to be more effective
than aspirin alone in patients with AF who cannot be treated with anticoagulation,
although this may come at the risk of increased bleeding complications. It would
not be the preferred option here, when no contraindications to anticoagulation
are apparent. Aspirin plus extended-release dipyridamole is an option for patients
with noncardioembolic stroke but would not be the preferred option in the
setting of known AF. Cardioversion may be an option for this patient, but it would
not remove the obligation of some form of antithrombotic therapy. While there
have been some studies of left atrial appendage occlusion in patients with AF, this
is not a standard therapy at this time.3
1. Adam SS, McDuffie JR, Ortel TL, Williams JW Jr. Comparative effectiveness of warfarin and new
oral anticoagulants for the management of atrial fibrillation and venous thromboembolism: a
systematic review. Ann Intern Med 2012;157(11):796Y807.
2. Lip GY, Tse HF, Lane DA. Atrial fibrillation. Lancet 2012;379(9816):648Y661.
3. Furie KL, Goldstein LB, Albers GW, et al. Oral antithrombotic agents for the prevention of
stroke in nonvalvular atrial fibrillation: a science advisory for healthcare professionals from the
American Heart Association/American Stroke Association. Stroke 2012;43(12):3442Y3453.

After 4 days on the stroke unit for additional testing and monitoring, he is
discharged to acute rehabilitation for ongoing management of his right-sided
weakness and functional limitations. Before discharge, the various options for
stroke prevention in the setting of atrial fibrillation are discussed with the
patient, and dabigatran (150 mg twice daily) is prescribed.

Continuum (Minneap Minn) 2014;20(2):482–493 www.ContinuumJournal.com 491


Copyright © American Academy of Neurology. Unauthorized reproduction of this article is prohibited.
PMP—Preferred Responses

b 11. Early administration of which of the following medications, in addition to


physiotherapy, has been associated with improved motor recovery following stroke?
A. amantadine
B. baclofen
C. donepezil
D. fluoxetine
E. memantine

The preferred response is D (fluoxetine). The Fluoxetine for Motor Recovery


After Acute Ischaemic Stroke (FLAME) study, published in 2011, randomized
patients with hemiplegia or hemiparesis after a stroke to receive either fluoxetine
20 mg/d or placebo within 5 to 10 days of their stroke.1 In this study, patients
who received active treatment had more significant recovery of their motor
function at 90 days poststroke, as measured by the Fugl-Meyer motor scale. This
change in motor function was believed to be independent of fluoxetine’s effect
as an antidepressant, although, not surprisingly, poststroke depression was seen
less commonly in patients receiving this medication. The improvement in the
Fugl-Meyer score appeared to translate into better functional outcomes, since the
distribution of scores on the modified Rankin Scale also favored treatment with
fluoxetine. Specifically, 34% of patients in the active arm achieved a modified
Rankin score of 2 or lower, compared with 11% in the placebo group (P=.021).
The exact mechanism by which fluoxetine may facilitate improved motor
recovery after stroke remains uncertain. Decisions regarding the use of fluoxetine
can be based on the degree of hemiparesis, the presence of underlying
depression, possible side effects, and patient preferences for treatment.
Donepezil and memantine have not been definitively shown to improve
motor recovery after stroke. Baclofen is an option for treatment of poststroke
spasticity, although there is no indication that this patient’s functional
limitations are related to spasticity at this time. Amantadine is occasionally
used to improve attention and alertness after brain injuries (most commonly
traumatic brain injury), but there is no evidence that this translates into
improved motor recovery.2

1. Chollet F, Tardy J, Albucher JF, et al. Fluoxetine for motor recovery after acute ischaemic stroke
(FLAME): a randomised placebo-controlled trial. Lancet Neurol 2011;10(2):123Y130.
2. Langhorne P, Bernhardt J, Kwakkel G. Stroke rehabilitation. Lancet 2011;377(9778):1693Y1702.

Six weeks after discharge from the acute rehabilitation unit, he is seen in
clinic and his right-sided strength has improved considerably. He still has
difficulty with activities that require significant dexterity of the right hand,
such as buttoning his shirt, but has returned to nearly all of his previous
activities. His expressive aphasia has completely resolved, and he denies
any residual language dysfunction. He is tolerating his dabigatran without
any bruising, bleeding, or other side effects.
His family asks about dietary recommendations for stroke prevention,
specifically inquiring about the evidence for a Mediterranean-type diet.

492 www.ContinuumJournal.com April 2014

Copyright © American Academy of Neurology. Unauthorized reproduction of this article is prohibited.


b 12 Which of the following best explains the current evidence for an
association between adherence to a Mediterranean diet and stroke risk?
A. even low-to-moderate diet adherence is associated with a lower risk of stroke
B. high diet adherence is associated with a lower risk of dementia but not stroke
C. high diet adherence is associated with a lower risk of stroke
D. high diet adherence is associated with an increased risk of stroke
E. high diet adherence is not associated with a lower risk of stroke

The preferred response is C (high diet adherence is associated with a lower


risk of stroke). Several different lifestyle habits can be associated with a lower
risk of stroke and vascular disease. Examples include abstinence from
tobacco, restriction of alcohol intake to no more than one alcoholic beverage
per day, maintenance of a healthy body weight, and regular physical activity.
A number of different dietary routines have been investigated for their
possible protective effects on stroke risk. The Mediterranean diet typically
consists of relatively higher intake of fruits, vegetables, and nuts; moderate
intake of dairy products, fish, and poultry; limited intake of red meats; and
regular mild-to-moderate consumption of alcohol. It has been studied in a
variety of populations, and results were recently summarized in a
meta-analysis; this analysis showed a 29% relative risk reduction in stroke
with high adherence to a Mediterranean diet (compared to low adherence).1
Moderate adherence was associated with a non-significant 10% relative risk
reduction in stroke, although moderate adherence was still associated with
significant reductions in the risk of depression and cognitive impairment. It is
important to note that the studies included in this meta-analysis were largely
focused on primary stroke prevention, so it may not be possible to extrapolate
these results to a secondary stroke prevention strategy. However, given that
minimal risk is associated with incorporating dietary changes such as these, they
may still be reasonable to recommend to patients after a first stroke or TIA.2
1. Psaltopoulou T, Sergentanis TN, Panagiotakos DB, et al. Mediterranean diet, stroke, cognitive
impairment, and depression: a meta-analysis. Ann Neurol 2013. [Epub ahead of print].
2. Estruch R, Ros E, Salas-Salvadó J, et al. Primary prevention of cardiovascular disease with a
Mediterranean diet. N Engl J Med 2013;368(14):1279Y1290.

Six months later, the patient is seen in clinic. He has remained free of any
new symptoms suggestive of cerebral ischemia and continues to tolerate
his anticoagulation without adverse effects. His daytime fatigue is
considerably better with compliance with CPAP therapy. Blood pressure in
the clinic is 127/80 mm Hg, and recent lipids drawn by his primary care
provider show his LDL to be 78 mg/dL.

Continuum (Minneap Minn) 2014;20(2):482–493 www.ContinuumJournal.com 493


Copyright © American Academy of Neurology. Unauthorized reproduction of this article is prohibited.

You might also like